FCI Placement Paper

FCI Placement Paper Food Corporation of India Previous Year Solved Placement Paper-General Awareness: 1. Which of the following organization is known as Market Regulator in India? (A) IBA (B) SEBI (C) AMFI (D) NSDL (E) None of these 2. Which of the following phenomenon occurred /surfaced recently associated with the economy of USA has put a catastrophic influence on the economy of the World? (A) Increase in the oil prices (B) High level of inflation (C) Huge and mounting expenditure due to attack on Iraq by USA and allied forces. (D) Sub-prime Crisis (E) None of these 3. The Reserve Bank of India keeps on changing various ratio/rates frequently. Why this is done? (1) To keep inflation under control (2) To ensure that Indian Rupee does not loose its market value (3) To ensure that Banks do not earn huge profits at the cost of public money. (A) Only 1 (B) Only 2 (C) Only 3 (D) All 1,2 & 3 (E) None of these 4. Which of the following is true about the National Rural Employment Guarantee Act? (1) It is applicable only in 100 rural districts. (2) It gives a guaranteed employment of 100 days to all those who are eligible for the same and are also willing to take it. (3) This Act is applicable only for men between the age of 21-65. Women do not get the benefit of the same. (A) Only 1 (B) Only 2 (C) Only 3 (D) All 1 , 2 & 3 (E) None of these 5. Many times we here about SEZ in newspapers/magazines. What is the full form of SEZ? (A) Small Economic Zone (B) Social Economic Zone (C) Special Enforcewment Zone (D) Service & Economic Zone (E) None of these 6. How many languages are there in the Eighth Schedule of the Constitution of India ? (A) 16 (B) 18 (C) 20 (D) 22 7. Article 340 of the Indian Constitution deals with? (A) Backward Classes Commission (B) Election Commission (C) Union Public Service Commission (D) Finance Commission 8. In which year ?Advocate?s Welfare Fund Act? was enacted by Parliament of India ? (A) 1999 (B) 2001 (C) 2003 (D) 2008 9. In which one of the following cases it has been held that prohibition on sale of eggs within municipal area of Rishikesh is not violative of Article 19(1)g of the Constitution ? (A) B. R. Enterprise Vs. State of U.P. (B) Sreeniwas General Traders Vs. State of Uttarakhand (C) Om Prakash Vs. State of U.P. (D) C. K. Jain Vs. State of Uttaranchal 10. ?Right to Information? is defined under? (A) Section 2(f) of the Right to Information Act, 2005 (B) Section 2(j) of the Right to Information Act, 2005 (C) Section 4 of the Right to Information Act, 2005 (D) Section 2(b) of the Right to Information Act, 2005 11. In Uttarakhand Legislative Assembly one member is nominated from? (A) Christian Community (B) Muslim Community (C) Anglo-Indian Community (D) Parsi Community 12. A Supreme Court?s Judge retires at an age of? (A) 58 years (B) 70 years (C) 62 years (D) 65 years 13. Who is the head of the ?State-Executive? ? (A) The State Legislature Assembly (B) The State Cabinet (C) The Chief Minister (D) The Governor 14. Case of I. R. Coelho V. State of Tamil Nadu is related with? (A) Judicial review of Article 356 of the Constitution (B) Judicial review of Article 226 of the Constitution (C) Judicial review of Ninth Schedule Laws (D) Judicial review of action taken by Speaker under Article 105 of the Constitution 15. Humanisation and decrimanisation of attempts to suicide was recommended by? (A) 210th Report of Law Commission (B) 212th Report of Law Commission (C) 216th Report of Law Commission (D) 215th Report of Law Commission 16. Jaya Bachchan V. Union of India, A.I.R. 2006 S.C. 2119 is related with? (A) Article 102(1)(a) of the Constitution (B) Article 109 of the Constitution (C) Article 190 of the Constitution (D) Article 226 of the Constitution 17. Joint Meeting of both Houses of Parliament is chaired by? (A) President of India (B) Chairman of Rajya Sabha (C) Speaker of Lok Sabha (D) None of the above 18. Who among the following was the Chairman of the Sixth Pay Commission ? (A) Justice A. K. Majumdar (B) Justice B. N. Srikrishna (C) Justice A. R. Lakshmanan (D) Justice R. C. Lahoti 19. ?www? on the internet stands for? (A) Words Words Words (B) Wide Word Words (C) World Wide Web (D) When Where Why 20. To which of the following companies, the Delhi High Court has ordered to remove its ?Logo? ? (A) Pepsico India (B) TISCO (C) Reliance Petrochemicals (D) None of the above Reasoning: 1. If the following numbers are rewritten by interchanging the digits in ten?s place and hundred?s place and then arranging them in the descending order. What will be the second digit of the newly formed fifth number from your right ? 479, 736, 895, 978, 389, 675 (A) 3 (B) 4 (C) 5 (D) 6 2. P is 60 m South-East of Q. R is 60 m North-East of Q. Then R is in which direction of P ? (A) North (B) North-East (C) South (D) South-East Directions?(Q. 3?5) Read the following information for answering the questions that follow? On a playing ground A, B, C, D and E are standing as described below facing the North. (i) B is 50 metres to the right of D. (ii) A is 60 metres to the South of B (iii) C is 40 metres to the West of D. (iv) E is 80 metres to the North of A. 3. If a boy walks from C, meets D followed by B, A and then E, how many metres has he walked if he has travelled the straight distance all through ? (A) 120 (B) 150 (C) 170 (D) 230 4. What is the minimum distance (in metre approximately) between C and E? (A) 53 (B) 78 (C) 92 (D) 120 5. Who is to the South-East of the person who is to the left of D ? (A) A (B) B (C) C (D) E 6. A man was walking in the evening just before the sun set. His wife said that, his shadow fell on his right. If the wife was walking in the opposite direction of the man, then which direction the wife was facing ? (A) North (B) West (C) South (D) East Directions?(Q. 7?11) In each of the following questions choose the set of numbers from the four alternative sets that is similar to the given set. 7. Given set : (4, 9, 18) (A) (8, 14, 22) (B) (10, 15, 25) (C) (6, 12, 23) (D) (12, 17, 26) 8. Given set : (10, 14, 17) (A) (4, 11, 14) (B) (9, 12, 15) (C) (8, 13, 18) (D) (6, 9, 12) 9. Given set : (7, 27, 55) (A) (21, 35, 52) (B) (18, 42, 65) (C) (16, 40, 72) (D) (13, 30, 58) 10. Given set : (39, 28, 19) (A) (84, 67, 52) (B) (52, 25, 17) (C) (70, 49, 36) (D) (65, 45, 21) 11. Given set : (246, 257, 358) (A) (233, 343, 345) (B) (273, 365, 367) (C) (143, 226, 237) (D) (145, 235, 325) Directions?(Q. 12?16) Each question contains six or seven statements followed by four sets of combinations of three. Choose the set in which the statements are logically related. 12. (1) All books are having pages. (2) All kings are having pages. (3) All kings are books. (4) Some heavy things are having pages. (5) Some heavy things are books. (6) Some books are heavy. (7) Some heavy things are having pages. (A) 1, 2, 3 (B) 6, 1, 4 (C) 4, 6, 1 (D) 1, 5, 7 13. (1) No athletes are vegetarians. (2) All cricket players are athletes. (3) Some athletes play cricket. (4) Some cricket players are vegetarians. (5) No cricket player is a vegetarian. (6) All athletes are vegetarians. (A) 1, 2, 5 (B) 3, 4, 1 (C) 1, 5, 2 (D) 2, 5, 6 14. (1) All grandmothers cook well. (2) No man is a grandmother. (3) Some men do not cook well. (4) All those who cook well are men. (5) No one who cooks well is a man. (6) All those who cook well are grandmothers. (7) Some men are not grandmothers. (A) 2, 6, 5 (B) 2, 5, 6 (C) 1, 4, 2 (D) 6, 4, 7 15. (1) Looting is a crime. (2) Some crooked people are criminals. (3) All those involved in looting are criminals. (4) Some crooked people are involved in looting. (5) All criminals are looked down in society. (6) Some crooked people are not criminals. (A) 1, 4, 6 (B) 3, 6, 2 (C) 1, 2, 6 (D) 3, 4, 2 16. (1) Some women are those who are successful in life. (2) Some men are those who have patience. (3) No man is a woman. (4) All those who have patience are successful in life. (5) Some who are successful in life are men. (6) Some men are not those are successful in life. (A) 1, 3, 6 (B) 4, 2, 6 (C) 1, 5, 3 (D) 2, 4, 5 Directions?(Q. 17?21) Each of the questions below consists of a question and two statements numbered (I) and (II). You have to decide whether the data provided in the statements are sufficient to answer the question. Give answers? (A) If the data in statement (I) alone are sufficient to answer the question, while the data in statement (II) alone are not sufficient to answer the question; (B) If the data in statement (II) alone are sufficient to answer the question, while the data in statement (I) alone are not sufficient to answer the questions; (C) If the data even in both statements (I) and (II) together are not sufficient to answer the question; (D) If the data in both statement (I) and (II) together are necessary to answer the question. 17. In which direction is Mahatmaji?s statue facing ? I. The statue is towards the northern end of the city. II. The statue?s shadow falls towards East at 5 O?clock in the evening. Ans : (C) 18. What is the total number of pupils in the final year class ? I. The number of boys in the final year class is twice as much as the number of girls in that class. II. The sum of the ages of all the pupils in the class is 399 years and their average age is 19 years. Ans : (B) 19. Who is the tallest among A, B, C and D ? I. A is taller than C. II. B is taller than C and D. Ans : (C) 20. How many Sundays are there in a particular month of a particular year? I. The month begins on Monday. II. The month ends on Wednesday. Ans : (D) Quantitative Aptitude: 1. In a division sum, the divisor is 10 times the quotient and 5 times the remainder. If the remainder is 46, the dividend is: (1) 4236 (2) 4306 (3) 4336 (4) 5336 2. If 1.5 x= 0.04 y, then the value of (y-x) (y+x) is: (1) 730/77 (2) 73/77 (3) 7.3/77 (4) 703/77 3. An employee may claim Rs. 7.00 for each km when he travels by taxi and Rs. 6.00 for each km if he drives his own car. If in one week he claimed Rs. 595 for traveling km. How many kms did he travel by taxi? (1) 55 (2) 65 (3) 62 (4) 70 4. The square root of 3 + ?5 is : (1) ?3 /2 + 1/?2 (2) ?3 /2 - 1/?2 (3) ?5 /2 - 1/?2 (4) ?(5/2) + ?(1/2) 5. The mean temperature of Monday to Wednesday was 370C and of Tuesday to Thursday was 340C , if the temperature on Thursday was 4/5th that of Monday, then what was the temperature on Thursday? (1) 36.50C (2) 360C (3) 35.50C (4) 340C 6. A certain number of two digits is three times the sum of its digits.If 45 be added to it, the digits are reversed. The number is: (1) 72 (2) 32 (3) 27 (4) 23 7. Three years ago the average age of A and B was 18 years. While C joining them now, the average becomes 22 years. How old (in years) is C now? (1) 24 (2) 27 (3) 28 (4) 30 8. If 2^(2x-1) = 8^(3-x), then the value of x is: (1) -1 (2) -2 (3) 2 (4) 3 9. A man?s basic pay for a 40 hours? week is Rs. 200. Overtimes is paid at 25% above the basic rate. In a certain week, he worked overtime and his total was Rs. 300. He therefore, worked for a total of (in hours): (1) 52 (2) 56 (3) 58 (4) 62 10. On a Rs. 10, 000 payment order, a person has choice between 3 successive discounts of 10%, 10% and 30% and 3 successive discounts of 40%, 5% and 5%. By choosing the better one he can save (in Rupees): (1) 200 (2) 255 (3) 400 (4) 433 11. Rs. 600 are divided among A, B, C so that Rs. 40 more than 2/5 th of A?s share, Rs. 20 more that 2/7 th of B?s share and Rs. 10 more than 9/17 th of C?s may all be equal. What is A?s share (in Rupees)? (1) 150 (2) 170 (3) 200 (4) 280 12. A, B, C started a business with their investment in the ratio 1 : 3 :5. After 4 months, A invested the same amount as before and B as well as C withdrew half of their investments. The ratio of their profits at the end of the year was: (1) 5 : 6 : 10 (2) 6 : 5 :10 (3) 10 : 5 : 6 (4) 4 : 3 : 5 13. If 9 men working 71/2 hours a day can finish a piece of work in 20 days, then how many days will be taken by 12 men, working 6 hours a day to finish the work? It is being given that 2 men of latter type work as much as 3 men of the former type? (1) 91/2 (2) 11 (3) 121/2 (4) 13 14. Three pipes A, B and C can fill a cistern in 6 hours. After working at it together for 2 hours, C is closed and A and B can fill the remaining part in 7 hours. The number of hours taken by C alone to fill the cistern is: (1) 12 (2) 14 (3) 16 (4) 18 15. A train B speeding with 120 kmph crosses another train C running in the same direction, in 2 minutes. If the lengths of the trains B and C be 100 m and 200 m respectively, what is the speed (in kmph) of the train C? (1) 111 (2) 123 (3) 127 (4) 129 16. River is running at 2 kmph. It took a man twice as long to row up as to row down the river. The rate (in km/hr) of the man in still water is: (1) 8 (2) 10 (3) 4 (4) 6 17. A merchant has 1000 kg of sugar, part of which he sells at 8% profit and the rest at 18% profit. He gains 14% on the whole. The quantity (in kg.) sold at 18% profit is: (1) 560 (2) 600 (3) 400 (4) 640 18. A sum of money becomes Rs. 13, 380 after 3 years and Rs. 20, 070 after 6 years on compound interest. The sum (in Rupees) is: (1) 8800 (2) 8890 (3) 8920 (4) 9040 19. A well with 14 m inside diameter is dug 10 m deep. Earth taken out of it, has been evenly spread all around it to a width of 21m to form an embankment. The height (in metres) of the embankment is: (1) 1/2 (2) 2/3 (3) 3/4 (4) 3/5 20. A rectangular carpet has an area of 120 sq metres and a perimeter of 46 metres. The length of its diagonal (in metres) is: (1) 11 (2) 13 (3) 15 (4) 17 FCI Paper For FCI exams selection of the elegible candidates will be done through the written examination. Written examination will comprise question from GS General Study, Latest Current affairs and current events , reasoning , Maths , English, etc . This paper contains questions for Food Corporation of India Assistant Grade 3 AG III recruitment Examination for General Awareness. FCI Previous year Question Paper 1. Kinetic energy is converted into electrical energy in (A) Electric Motor (B) Dynamo (C) Electromagnet (D) Ammeter Ans : B 2. Floppy disk is an example of (A) Read Only Memory (B) Random Access Memory (C) Cache Memory (D) Secondary Storage Memory Ans : D 3. Which of the following compounds are sprayed over clouds to cause artificial rain? (A)Ice (B) NaOH (C)AgI (D) NH4CL Ans : C 4. Iodine deficiency in diet is know to cause (A)rickets (B) obesity (C)scurvy (D) goitre Ans : D 5. The refrigeration of the food articles keeps them fresh because (A) water forms crystals (B) chemical reaction is slowed down (C) chemical reaction take place faster (D) bacteria are killed Ans : D 6. Which substance is called the ?liquid gold?? (A) Gold as a liquid form (B) Sodium as a liquid form (C) Mustard oil (D) Petroleum oil Ans : D 7. Percentage of silver in German silver is (A)25°/o (B) 0% (C)75°/o (D) 80% 8. Which type of compounds are sugar and common salt? (A) Both are organic compounds (B) Both are inorganic compounds (C) Sugar is an organic compound and common salt is an inorganic compound (D) Sugar is inorganic compound and common salt is an organic compound Ans : C 9. The chemical name of Vitamin ?E? is (A) Ascorbic acid (B) Retinol (C) Tocopherol (D) Thiamine Ans : C 10. Which of the following gases has bleaching property (A) Carbon dioxide (B) Carbon Monoxide (C) Chlorine (D) Hydrogen Ans : C 11. The heaviest body of our Solar system is (A)Sun (B) Uranus (C)Jupiter (D) Saturn Ans : A 12. From amongst the following; Name famous Indian ornithologist (A) Dr. Salim Ali (B) Dr. Gopalasamudram N. Ramachandran (C) Dr. J.B.S. Haldane (D) Dt H.G. Khorana Ans : a 13. Who performed the longest space journey in the year 2007 (A). Venus Williams (B) Serena Willlams (C) Sunita William (D) Sir John William Ans : C 14. ?The Lost Child? was written by (A) Nirad C. Chowdlltiury (B) Mulk Raj Anand (C) Khushwant Singh (D) Annie Besant Ans : B 15. Dr. MS. Swaminathan has distinguished himself in which of the following fields ? (A) Nuclear Physics (B) Agriculture (C) Astrophysics (D) Medicine Ans : B 16. The Headquarters of UNESCO at (A)Rome (B) Geneva (C)Paris (D) New York Ans : C 17. January 15 is celebrated as the (A) Army Day (B) Martyr?s Day (C) Independence Day (D)) Ugadhi Ans : A 18. Bijapur is known for its (A) Heavy rainfall (B) Rock Temple (C) Gol Gumbaj (D) Statue of Gomateshwara Ans : C 19. The Americans are also known as (A) Kiwis (B) Yankees (C) Tories (D) None of the above Ans : B 20. India lifted the ONGC Nehru Cup, 2009, after defeating in the final (A)Syria (B) Pakistan (C)Sri Lanka (D) Bangladesh Ans : A 21. An element which does not react with oxygen is (A)Chlorine (B) Iodine (C)Helium (D) Nitrogen Ans : C 22. During sleep, man?s blood pressure (A) increases (B) decreases (C) fluctuates (D) remains constant Ans : B 23. Which Indian fllrnstar was recently conferred Honorary Doctorate by Bedfordshire University (U.K.) ? (A) Amitabh Bachchan (B) Shah Rukh Khan (C) Orn Puri (D) Aamir Khan Ans : B 24. Williams Sisters won the U.S. Open Women?s Doubles Title 2009 after defeating in the final (A) Leizal Huber and Cara Black (B) Kim Clijsters and Anna Kournikova (C) Caroline Woaniacki and Dinara Safina (D) Nathalie Deshy and Sania Mirza Ans : A 25. Which one of the following is a military alliance ? (A) ASEAN (B) SAARC (C)NATO (D) NAFTA Ans : C 26. The recipient of the 42nd Jnanpeeth Award is (A) Manohar Shastri (B) Harish Pandya (C) Satya Vrat Shastri (D) K Kamal Kumar Ans : C 27. World Development Report is an annual publication of (A)UNICEF (B) UNDP (C)WTO (D) World Bank Ans : D 28. Which one of the following is correct ? Player--- Sport (A) Jeev Milkha?TennisSingh (B) Jhulan?Cricket Goswami (C) Baichung Bhutia?Hockey (D) Pankaj Advani?Badminton Ans : B 29. Which one of the following industrialists was declared ?The Business Person of the Year 2008? by the Times of India Survey ? (A) Anil Ambani (B) Rahul Bajaj (C) Ratan Tata (D) Gautam Adani Ans : C 30. Naina Devi peak forms a part of (A) Himalayan range located in Sikkim (B) Himalayan range located in Kumaon region (C) Himalayan range located in Nepal (D) Himalayan range located in Jammu & Kashmir Ans : B FCI Placement Paper FCI Last Year Paper Numerical Aptitude Questions: 1 (95.6x 910.3) ÷ 92.56256 = 9? (A) 13.14 (B) 12.96 (C) 12.43 (D) 13.34 (E) None of these 2. (4 86%of 6500) ÷ 36 =? (A) 867.8 (B) 792.31 (C) 877.5 (D) 799.83 (E) None of these 3. (12.11)2 + (?)2 = 732.2921 (A)20.2 (B) 24.2 (C)23.1 (D) 19.2 (E) None of these 4.576÷ ? x114=8208 (A)8 (B)7 (C)6 (D)9 (E) None of these 5. (1024?263?233)÷(986?764? 156) =? (A)9 (B)6 (C)7 (D)8 (E) None of these (D) 6878 (E) None of these 6. 22 x4+(?)2=(13)2 (A) 81 (C) 27 (B) 9 (D) 64 (E) None of these 7. 432+2170+35=? (A)494 (B) 475 (C)481 (D) 469 (E) None of these 8. Three numbers are in the ratio of 3: 4 :5 respectively. If the sum of the first and third numbers is more than the second number by 52, then which will be the largest number? (A) 65 (B) 52 (C) 79 (D) 63 (E) None of these 9. The compound interest on a certain amount for 2 years at the rate of 8 p.c.p.a. is Rs.312. What will be the simple interest on the same amount and at the same rate and same time? (A)Rs. 349.92 (B) Rs. 300 (C)Rs. 358.92 (D) Rs. 400 (E) None of these 10. The length of a rectangle exceeds its breadth by 7 ems. If the length is decreased by 4 cm. and the breadth is increased by 3 cms., then the area of the new rectangle will be the same as the area of the original rectangle. What will be the perimeter of the original rectangle? (A)45 cms. (B)40 cems. (C)50 cms. (D)55 cms. (E)None of these 11. The sum of the digits of a two digit number is 12. If the new number formed by reversing the digits is greater than the original number by 54,then what will be the original number? (A)28 (B) 48 (C)39 (D) 93 (E) None of these 12. In a fraction, twice the numerator is two more than the denominator.If 3 is added to the numerator and the denominator each, then the resultant fraction will be 2/3 .What was the original fraction ? (A)5/18 (B)6/13 (C)13/6 (D)7/12 (E) None of these 13. In a fraction, twice the numerator is two more than the denominator. If 3 is added to the numerator and the denominator each, then the resultant fraction will be 2/3 .What was the original fraction ? (A)5/18 (B)6/13 (C)13/6 (D)7/12 (E) None of these 14. Four-fifth of a number is 10 more than two-third of the same number.What is the number? (A) 70 (B) 75 (C) 69 (D) 85 (E) None of these 15. A shopkeeper purchased 200 bulbs for Rs. 10 each. However, 5 bulbs were fused and had to be thrown away. The remaining were sold at Rs. 12 each. What will be the percentage profit? (A) 25 (B) 15 (C) 13 (D) 17 (E) None of these 16. What should come in the place of question mark (?) in the number series given below? 25, 34, 52,79, 115,? (A)160 (B) 140 (C)153 (D) 190 (E)None of these 17. What number should replace both the question marks (?) in the following question ? ?/144= 49 /? (A)95 (B) 76 (C)82 (D) 84 (E)None of these 18. The sum of three consecutive even numbers is 252. What is the sum of the smallest and the largest numbers? (A)158 (B) 148 (C)168 (D) 198 (E) None of these 19. In how many different ways, can the letters of the word ?CRISIS? be arranged ? (A) 150 (B) 240 (C) 120 (D) 200 (E) None of these 20. At each corner of a square park with side equal to 40 m, there is a flower bed in the form of a sector of radius 14 m. What is the area of the remaining part of the park? (A)984 Square m (B) 789 Square m (C) 1014 Square m (D) 1024 Square m (E) None of these Reasoning: 1. six members of a familyare travelling these are a,b,c,d,e, and f. b is the son of c but c is not mothers of b. a and c are a married couple, e is the brother of c. d is the daughter of a. f is the brother of b.who is the wife of e: (a) a (b)f (c)b (d) cant be determined 2.six members of a familyare travelling these are a,b,c,d,e, and f. b is the son of c but c is not mothers of b. a and c are a married couple, e is the brother of c. d is the daughter of a. f is the brother of b.which of following is pair of female? (a)ae (b)bd (c)df (d)ad 3. six members of a familyare travelling these are a,b,c,d,e, and f. b is the son of c but c is not mothers of b. a and c are a married couple, e is the brother of c. d is the daughter of a. f is the brother of b.how is e related to d: (a) father (b)brother (c)uncle (d)none of these 4.there is a group of five teachers a,b,c,d and e - b and c teaches maths and geography - a and c teach maths and history - b and d teach political science and geofraphy - d and e tech political science and biology - e teaches biology, history, and political science who teaches political science, geography and biology (a)e (b)d (c)c (d)b 5. there os a group of five teachers a,b,c,d and e - b and c teaches maths and geography - a and c teach maths and history - b and d teach political science and geofraphy - d and e tech political science and biology - e teaches biology, history, and political science who teaches in mathematics, political science and geography: (a) a (b)b (c)d (d)e 6. there os a group of five teachers a,b,c,d and e - b and c teaches maths and geography - a and c teach maths and history - b and d teach political science and geofraphy - d and e tech political science and biology - e teaches biology, history, and political science who teaches maths, geography and history (a) c (b)e (c)a (d)b 7. there os a group of five teachers a,b,c,d and e - b and c teaches maths and geography - a and c teach maths and history - b and d teach political science and geofraphy - d and e tech political science and biology - e teaches biology, history, and political science who teaches maths, geography and history (a) e (b)a (c)d (d)c 8.there os a group of five teachers a,b,c,d and e - b and c teaches maths and geography - a and c teach maths and history - b and d teach political science and geofraphy - d and e tech political science and biology - e teaches biology, history, and political science who teaches political science, history and biology (a)a (b)b (c)c (d)e 9.1.gita, ravi and suresh are children of. mr. and mrs. khanna 2. renu,raja and sumit are children of mr. and mrs. chopra 3. sumit and gita are married and ashok and sanjay their children 4. garima and ram are children of mr. and mrs. chawla/ 5 garima is married to suresh and has three children rata,sonu,raju how is ravi related to raju (a)brother (b)uncle (c)cousin (d)maternal uncle 10.1.gita, ravi and suresh are children of. mr. and mrs. khanna 2. renu,raja and sumit are children of mr. and mrs. chopra 3. sumit and gita are married and ashok and sanjay their children 4. garima and ram are children of mr. and mrs. chawla/ 5 garima is married to suresh and has three children rata,sonu,raju how ravi related to ashok (a)maternal uncle (b)cousin (c)brother in law (d)father in law 11.1.gita, ravi and suresh are children of. mr. and mrs. khanna 2. renu,raja and sumit are children of mr. and mrs. chopra 3. sumit and gita are married and ashok and sanjay their children 4. garima and ram are children of mr. and mrs. chawla/ 5 garima is married to suresh and has three children rata,sonu,raju how is ram related to suresh: (a)brother (b)cousin (c)uncle (d)brother in law 12. 1.gita, ravi and suresh are children of. mr. and mrs. khanna 2. renu,raja and sumit are children of mr. and mrs. chopra 3. sumit and gita are married and ashok and sanjay their children 4. garima and ram are children of mr. and mrs. chawla/ 5 garima is married to suresh and has three children rata,sonu,raju what is the surname of sanjay: (a) chawla (b) chopra (c) khanna (d) suresh 13. 1.gita, ravi and suresh are children of. mr. and mrs. khanna 2. renu,raja and sumit are children of mr. and mrs. chopra 3. sumit and gita are married and ashok and sanjay their children 4. garima and ram are children of mr. and mrs. chawla/ 5 garima is married to suresh and has three children rata,sonu,raju how is ram related to rita: (a) brother (b) uncle (c) maternal uncle (d) cousin 14. 1.gita, ravi and suresh are children of. mr. and mrs. khanna 2. renu,raja and sumit are children of mr. and mrs. chopra 3. sumit and gita are married and ashok and sanjay their children 4. garima and ram are children of mr. and mrs. chawla/ 5 garima is married to suresh and has three children rata,sonu,raju renu is sanjay's ---------- (a) sister (b)sister in law (c)aunty (d)cousin 15. 1.gita, ravi and suresh are children of. mr. and mrs. khanna 2. renu,raja and sumit are children of mr. and mrs. chopra 3. sumit and gita are married and ashok and sanjay their children 4. garima and ram are children of mr. and mrs. chawla/ 5 garima is married to suresh and has three children rata,sonu,raju raju's sur name is: (a) chawla (b)khanna (c)chopra (d)none of these 16. 1.gita, ravi and suresh are children of. mr. and mrs. khanna 2. renu,raja and sumit are children of mr. and mrs. chopra 3. sumit and gita are married and ashok and sanjay their children 4. garima and ram are children of mr. and mrs. chawla/ 5 garima is married to suresh and has three children rata,sonu,raju sumit and suresh are related as: (a) brother in law (b)cousin (c)brother (d)uncle 17. 1.gita, ravi and suresh are children of. mr. and mrs. khanna 2. renu,raja and sumit are children of mr. and mrs. chopra 3. sumit and gita are married and ashok and sanjay their children 4. garima and ram are children of mr. and mrs. chawla/ 5 garima is married to suresh and has three children rata,sonu,raju how sonu related to ravi. (a) brother (b)brother in law (c)nephew (d)father 18.1.gita, ravi and suresh are children of. mr. and mrs. khanna 2. renu,raja and sumit are children of mr. and mrs. chopra 3. sumit and gita are married and ashok and sanjay their children 4. garima and ram are children of mr. and mrs. chawla/ 5 garima is married to suresh and has three children rata,sonu,raju how garima is related to mr. & mrs. khanna: (a)daughter (b)neice (c)sister in law (d)daughter in law 19. 1.gita, ravi and suresh are children of. mr. and mrs. khanna 2. renu,raja and sumit are children of mr. and mrs. chopra 3. sumit and gita are married and ashok and sanjay their children 4. garima and ram are children of mr. and mrs. chawla/ 5 garima is married to suresh and has three children rata,sonu,raju how ashok and sanjay are lated to mr. mrs. copra: (a)sons (b)nephews (c)grand son (d)none of these 20. in the office amongst staff a,b,c,d ,e of g. h there is only computer. a programe has to be made and can be made by only one person at a time. the office has 3 shifts, the morning shift from 6 am-12 am has 3 staff, afternoon shift from 12am. to 6 pm. has 3 staff, evening shift from 6 pm to 12 pm has 2 staff one of the person did not used a printer: 1 b is the last person who use the computer 2 h use the computer after a and came iwth a 3 d used the computer after h and come with h but before e 4 c used the computer before b but arrived after e 5 a use the computer before f and arrived before g if g used a computer justbefore c who use 4: (a) a (b)b (c)e (d)d General Awareness: 1. The foreign trade policy announced in the year 2004 was announced for a period of ____ (A) two years (B) three years (C) four years (D) five years (E) ten years Ans (D) 2. As per reports in the newspapers the Indian Rupee is appreciating these days. What does it really mean it ? (1) The value of the Rupee has gone up. It is now 110 paise and not 100 paise. (2) The exchange rate of Rupee has gone up. (3) Now we can purchase more in one Rupee which was not possible earlier. (A) Only 1 (B) Only 2 (C) Only 3 (D) Both 1 & 2 (E) None of these Ans (B) 3. As per newspaper reports the inflation in India and China was at very high level . In such a situation the central Banks of these countries are required to follow _____ (A) a more liberal credit policy (B) a very tight credit policy (C) create an atmosphere of easy liquidity in the market. (D) raise the limits of personal and corporate income taxes. (E) None of these Ans (B) 4. The US Federal Reserve had recently reduced the interest rates by 150 basis points in the last few months. This was done because ____ (A) US was going into recession. (B) Inflation was high in countries like India and China hence a correction in economic policies by all the countries was needed. (C) Euro had become more attractive currency, hence US wanted to save dollars from further depreciation. (D) Iraq had bacome a very lucrative destination for foreign investment . US does not want it to happen. (E) None of these Ans (A) 5. The Stock market index of London Stock Market is referred as _____ (A) Sensex (B) Footsie (FTSE) (C) NIFTY (D) Bullish (E) None of these Ans (B) 6. As we all know Ministry of Finance every year prepare Union Budget and present it to the parliament . Which of the following is / are the elements of the Union Budget ? (1) Estimates of revenue and capital receipts. (2) Ways and MeAns to raise the revenue. (3) Estimates of expenditure. (A) Only 1 (B) Only 2 (C) Only 3 (D) All 1,2 & 3 (E) None of these Ans (D) 7. Which of the following is NOT used as raw material for production of nuclear power? (A) Uranium (B) Barylium (C) Zirconium (D) Sodium (E) All these are used Ans (A) 8. As per figure released recently , the GDP growth in the last two years has been in the range of ____ (A) 6%-7% (B) 7%-8% (C) 8%-9% (D) 9%-9.5% (E) 10%-11% Ans (D) 9. Sri Lankan Monitoring Mission (SLMM) has Nordic Monitors from five nations. Which of the following is NOT one of them? (A) India (B) Sweden (C) Norway (D) Denmark (E) Iceland Ans (A) 10.Many a times we read a term in financial newspaper GDR. What is the full form of the same? (A) Gross Domestic Revenue (B) Global Domestic Ratio (C) Global Depository Receipts (D) Gross Depository Revenue (E) None of these Ans (C) 11. The Govt. of India has fixed some targets for various programmes for the entire plan period/or a part of it. Which of the following is/are TRUE about the Bharat Nirman Programme? (1) The plan envisages irrigation of 10 million hectares additional land. (2) It is planned that about six hundred km. of new roads will be developed every month. (3) About 1000 new telephone connections will be given to rural areas every month so that by the end of the plan all villages in India are brought under telephone services network. (A) Only (1) is true (B) Only (2) is true (C) Only (3) is true (D) All (1), (2) & (3) are true (E) None is true Ans (A) 12. Who played the role of the Hockey coach in the Film ?Chak De India? for which he was given the award for Best Actor? (A) Shahrukh Khan (B) Abhishek Bachchan (C) Aaamir Khan (D) Salman Khan (E) None of these Ans (A) 13. India has signed a Memorandum of Understanding (MOU) with which of the following countries which says they will create a ?Multi Polar Order? based on the rule of law, sovereign equality and non-interference in internal affairs of States? (A) USA (B) Nepal (C) China (D) Britain (E) Russia Ans (E) 14. Recently one Indian cricket player has become the sixth batsman to earn 10000 runs in Test Crickets. The name of the player is ____ (A) Virendra Sehwag (B) Sachin Tendulkar (C) Sourav Ganguly (D) Anil Kumble (E) Rahul Dravid Ans (E) 15. Yousuf Raza Gillani who was elected the Prime Minister of Pakistan is from which political parties? (A) Jamiat-e-Ulema Islami (B) Pakistan People?s Party (C) Awami National Party (D) Muslim League (E) None of these Ans (B) 16. As per the Govt. Notifications / Advertisements given in various newspapers the wheat procurement this year was of a record of 210 lakh MTs. Which of the following was/were the contributory factors in such a record making procurement? (1) Minimum support price was keep as Rs. 1,500/- per quintal. (2) Wheat was also procured from non-traditional State like Gujrat and Maharashtra (3) State Govt. were also involved actively in the Scheme (A) Only 1 (B) Only 2 (C) Only 3 (D) Both 1 & 2 (E) None of these Ans (D) 17. Very often we read in newspapers that some sports persons have to go for a Dope Test after their performance in the field. Why Dope Test is conducted ? (1) To know the general health of the sports person (2) To know the sugar level of their blood (3) To detect whether they have taken any performance inhancing drugs (A) Only 1 (B) Only 2 (C) Only 3 (D) All 1 , 2 & 3 (E) None of these Ans (B) 18. Santosh Trophy is associated with the game of ____ (A) Cricket (B) Badminton (C) Tennis (D) Chess (E) None of these Ans (E) Santosh Trophy is associated with the game of Football. 19. As per the newspaper reports some economically developed states onlt hold about 60% of the total ?Demat Accounts in India. A Demat Account is ____ (A) an acount which is opened by the people of the lower income groups of the society. (B) an account in which trading of the shares is done. (C) an account which can be opened only by minors. (D) an account which can be operated by big corporate houses and are mainly business accounts like current accounts. (E) None of these Ans (B) 20. Federal Reserve is the Financial Organization of the ___ (A) USA (B) Britain (C) France (D) Japan (E) Germany Ans (A) FCI Placement Paper FCI Management Trainee (General/Depot) Question Paper Reasoning 1. What should come in the place of (?) in the given series? ACE, FGH, ?, PON (A) KKK (B) JKI (C) HJH (D) IKL Ans. (A) 2. Typist : Typewriter : : Writer: ? (A) Script (B) Pen (C) Paper (D) Book Ans. (B) 3. Paint: Artist : : Wood: ? (A) Furniture (B) Forest (C) Fire (D) Carpenter Ans. (D) 4. acme : mace :: alga: ? (A) glaa (B) gaal (C) laga (D) gala Ans. (D) 5. EIGHTY : GIEYTH : : OUTPUT:? (A) UTOPTU (B) UOTUPT (C) TUOUTP (D) TUOTUP Ans. (D) 6. ?Medicine? is related to ?Patient? in the same way as ?Education? is related to? (A) Teacher (B) School (C) Student (D) Tuition Ans. (C) 7. Fill in the missing letter in the following series? S, V, Y, B, ? (A) C (B) D (C) E (D)G Ans. (C) 8. What should come in the place of question mark in the following series? 3, 8, 6, 14, ?, 20 (A) 11 (B) 10 (C) 8 (D) 9 Ans. (D) 9. Select the correct option in place of the question mark. AOP, CQR, EST, GUV, ? (A) IYZ (B) HWX (C) IWX (D) JWX Ans. (C) 10. What should come in the place of question mark in the following series? 1, 4, 9, 25, 36, ? (A) 48 (C) 52 (B) 49 (D) 56 Ans. (B) Directions?(Q. 11 to 14): Select the one which is different from the other three. 11. (A) Bokaro (B) Jamshedpur (C) Bhilai (D) Agra Ans. (D) 12. (A) January (B) February (C) July (D) December Ans. (B) 13. (A) Bible (B) Panchsheel (C) Geeta (D) Quran Ans. (B) 14. (A) Star (B) Sun (C) Sky (D) Moon Ans. (C) Directions?(Q. 15 to 17): based on alphabets. 15. If the sequence of the alphabets is reversed which of the following would be the 14th letter from your left? (A) N (B) L (C) O (D) None of these Ans. (D) 16. Which letter is the 8th letter to the right of the letter, which is 12th from the left? (A) V (B) T (C) W (D) Y Ans. (B) 17. Which letter is the 8th letter to the right of the letter which is 10th to the left of the last but one letter from the right? (A) V (B) X (C) W (D) I Ans. (C) Directions?(Q. 18 to 23) Three of the following four are alike in a certain way and so form a group. Which is the one that does not belong to that group? 18. (A) Green (B) Red (C) Colour (D) Orange Ans. (C) 19. (A) Rabbit (B) Crocodile (C) Earthworm (D) Snail Ans. (A) 20. (A) Polo (B) Chess (C) Ludo (D) Carrom Ans. (A) 21. (A) Sun (B) Universe (C) Moon (D) Star Ans. (B) 22. (A) Cheese (B) Milk (C) Curd (D) Ghee Ans. (B) 23. (A) Carrot (B) Radish (C) Potato (D) Brinjal Ans. (D) 24. In a certain code ?CONTRIBUTOR? is written as ?RTNOCIROTUB?. How is ?prohibition? written in that code? (A) NOITIBIHORP (B) IHORPBITION (C) ITIONBIHOTP (D) IHORPBNOITI Ans. (D) 25. If ?CAT? and ?BOAT? are written as XZG and ?YLZG? respectively in a code language how is ?EGG? to be written in the same language? (A) VSS (B) URR (C) VTT (D) UTF Ans. (C) 26. In a code language SINGER is written as AIBCED then GINGER will be written in the same code as? (A) CBIECD (B) CIBCED (C) CBICED (D) CIBECD Ans. (B) 27. If BAT is coded as 283, CAT is coded as 383 and ARE is coded as 801, then the code for BETTER is? (A) 213310 (B) 213301 (C) 123301 (D) 012334 Ans. (A) 28. If water is called black, black is called tree, tree is called blue, blue is called rain, rain is called pink and pink is called fish in a certain language then what is the colour of sky called in that language? (A) Blue (B) Fish (C) Rain (D) Pink Ans. (C) 29. A man walks 3 km northwards and then turns left and goes 2 km. He again turns left and goes 3 km. He turns right and walks straight. In which direction he is walking now? (A) East (B) West (C) North (D) South Ans. (B) 30. One morning after sunrise Vikram and Shailesh were standing in a lawn with their back towards each other. Vikram?s shadow fell exactly towards left-hand side. Which direction Shailesh was facing? (A) East (B) West (C) North (D) South Ans. (D) 31. Nageena is taller than Pushpa but not as tall as Manish. Rama is taller than Namita but not as tall as Pushpa. Who among them is the tallest? (A) Manish (B) Pushpa (C) Namita (D) Nageena Ans. (A) 32. In an examination Raj got more marks than Moti but not as many as Meena. Meena got more marks than Ganesh and Rupali. Ganesh got less marks than Moti but his marks are not the lowest in the group. Who is second in the descending order of marks? (A) Meena (B) Rupali (C) Raj (D) None of these Ans. (C) 33. Pointing to a photograph of a girl, Rajan said ?She has no sister or daughter but her mother is the only daughter of my mother.? How is the girl in the photograph related with Rajan?s mother? (A) Sister in law (B) Grand daughter (C) Daughter in law (D) None of these Ans. (B) 34. If Amit? s father is Billoo? s father?s only son and Billoo has neither a brother nor a daughter. What is the relationship between Amit and Billoo? (A) Uncle?Nephew (B) Father?Daughter (C) Father?Son (D) Grandfather?Grandson Ans. (C) 35. An application was received by inward clerk in the afternoon of a weekday. Next day he forwarded it to the table of the senior clerk, who was on leave that day. The senior clerk next day evening put up the application to the desk officer. Desk officer studied the application and disposed off the matter on the same day, i.e., Friday. Which day the application was received by the inward clerk? (A) Tuesday (B) Earlier week?s Saturday (C) Wednesday (D) Monday Ans. (C) 36. Flight to Mumbai leaves every 5 hours. At the information counter I learnt that the flight took off 25 minutes before. If the time now is 10 : 45 a.m., what is the time for the next flight? (A) 2 : 20 a.m. (B) 3 : 30 a.m. (C) 3 : 55 p.m. (D) 3 : 20 p.m. Ans. (D) 37. Babloo ranked 16th from the top and 29th from the bottom among those who passed an examination. 6 boys did not participate in the competition and 5 failed in the examination. How many boys were there in the class? (A) 44 (B) 40 (C) 50 (D) 55 Ans. (D) 38. Indra is 7th from the left and Jaya is 5th from the right. When they interchange their position Jaya becomes 19th from the right. What is Indra?s position from the left? (A) 21st (B) 19th (C) 23rd (D) 20th Ans. (D) 39. How many 5?s are in the following sequence of numbers which are immediately preceded by 7? 8 9 5 3 2 5 3 8 5 5 6 8 7 3 3 5 7 7 5 3 6 5 3 3 5 7 3 8 (A) One (B) Two (C) Three (D) Four Ans. (A) 40. How many 8?s are there in the following sequence which are immediately preceded by 6 but not immediately followed by 5? 6 8 5 7 8 5 4 3 6 8 1 9 8 5 4 6 8 2 9 6 8 1 3 6 8 5 3 6 (A) One (B) Two (C) Three (D) Four Ans. (C) 41. If EARTHQUAKE is coded as MOGPENJOSM then EQUATE will be coded as? (A) MENOPM (B) MENOMP (C) NJOGPM (D) MNJOPM Ans. (D) 42. If COUNTRY is coded in certain way as EMWLVPA, ELECTORATE will be coded in the same manner as? (A) CJCEFQPYWC (B) CJGERQTYVG (C) CNCERQPCRG (D) GJGAVMTYVC Ans. (D) 43. ?Air? is to ?Bird? as ?Water? is to ??.. (A) Drink (B) Fish (C) Wash (D) Swim Ans. (B) 44. ?Pencil? is to ?Write? as ?Knife? is to ??? (A) Injure (B) Peel (C) Prick (D) Attack Ans. (B) 45. Mohan is 18th from either end of a row of boys ? How many boys are there in that row? (A) 26 (B) 32 (C) 24 (D) 35 Ans. (D) 46. In a class of 60 where boys are twice that of girls, Ramya ranked 17th from the top. If there are 9 boys ahead of Ramya, how many girls are after her in the rank? (A) 26 (B) 12 (C) 10 (D) 33 Ans. (B) 47. ?Soldier? is related to ?Army? in the same way as ?Pupil? is related to ??. (A) Education (B) Teacher (C) Student (D) Class Ans. (D) 48. ?Kilogram? is related to ?Quintal? in the same way as ?Paisa? is related to??? (A) Coin (B) Money (C) Cheque (D) Rupee Ans. (D) 49. ?Stammering? is to ?Speech? as Deafness is to ???? (A) Ear (B) Hearing (C) Noise (D) Commotion Ans. ( = 813.08 3.5-{(-824X-288.4)/X} = 813.08 (3.5X+824X-288.4)/X = 813.08 827.5X-288.4 / X = 813.08 827.5X-288.4 = 813.08X 827.5X-813.08X = 288.4 14.42X = 288.4 X = 288.4 / 14.42 X = 20 FCI Aptitude-Reasoning To find the number of factors of a given number, express the number as a product of powers of prime numbers. In this case, 48 can be written as 16 * 3 = (24 * 3) Now, increment the power of each of the prime numbers by 1 and multiply the result. In this case it will be (4 + 1)*(1 + 1) = 5 * 2 = 10 (the power of 2 is 4 and the power of 3 is 1) Therefore, there will 10 factors including 1 and 48. Excluding, these two numbers, you will have 10 – 2 = 8 factors. Sum of n natural numbers -> The sum of first n natural numbers = n (n+1)/2 -> The sum of squares of first n natural numbers is n (n+1)(2n+1)/6 -> The sum of first n even numbers= n (n+1) -> The sum of first n odd numbers= n^2 Finding Squares of numbers To find the squares of numbers near numbers of which squares are known To find 41^2 , Add 40+41 to 1600 =1681 To find 59^2 , Subtract 60^2-(60+59) =3481 Finding number of Positive Roots If an equation (i:e f(x)=0 ) contains all positive co-efficient of any powers of x , it has no positive roots then. Eg: x^4+3x^2+2x+6=0 has no positive roots . Finding number of Imaginary Roots For an equation f(x)=0 , the maximum number of positive roots it can have is the number of sign changes in f(x) ; and the maximum number of negative roots it can have is the number of sign changes in f(-x) . Hence the remaining are the minimum number of imaginary roots of the equation(Since we also know that the index of the maximum power of x is the number of roots of an equation.) Reciprocal Roots The equation whose roots are the reciprocal of the roots of the equation ax^2+bx+c is cx^2+bx+a Roots Roots of x^2+x+1=0 are 1,w,w^2 where 1+w+w^2=0 and w^3=1 Finding Sum of the rootsFor a cubic equation ax^3+bx^2+cx+d=o sum of the roots = - b/a sum of the product of the roots taken two at a time = c/a product of the roots = -d/a For a biquadratic equation ax^4+bx^3+cx^2+dx+e = 0 sum of the roots = - b/a sum of the product of the roots taken three at a time = c/a sum of the product of the roots taken two at a time = -d/a product of the roots = e/a Maximum/Minimum -> If for two numbers x+y=k(=constant), then their PRODUCT is MAXIMUM if x=y(=k/2). The maximum product is then (k^2)/4 -> If for two numbers x*y=k(=constant), then their SUM is MINIMUM if x=y(=root(k)). The minimum sum is then 2*root(k) . Inequalties -> x + y >= x+y ( stands for absolute value or modulus ) (Useful in solving some inequations) -> a+b=a+b if a*b>=0 else a+b >= a+b -> 2<= (1+1/n)^n <=3 -> (1+x)^n ~ (1+nx) if x<<<1> When you multiply each side of the inequality by -1, you have to reverse the direction of the inequality. Product Vs HCF-LCM Product of any two numbers = Product of their HCF and LCM . Hence product of two numbers = LCM of the numbers if they are prime to each other AM GM HM For any 2 numbers a>b a>AM>GM>HM>b (where AM, GM ,HM stand for arithmetic, geometric , harmonic menasa respectively) (GM)^2 = AM * HM Sum of Exterior Angles For any regular polygon , the sum of the exterior angles is equal to 360 degrees hence measure of any external angle is equal to 360/n. ( where n is the number of sides) For any regular polygon , the sum of interior angles =(n-2)180 degrees So measure of one angle in Square-----=90 Pentagon--=108 Hexagon---=120 Heptagon--=128.5 Octagon---=135 Nonagon--=140 Decagon--=144 Problems on clocks Problems on clocks can be tackled as assuming two runners going round a circle , one 12 times as fast as the other . That is , the minute hand describes 6 degrees /minute the hour hand describes 1/2 degrees /minute . Thus the minute hand describes 5(1/2) degrees more than the hour hand per minute . The hour and the minute hand meet each other after every 65(5/11) minutes after being together at midnight. (This can be derived from the above) . Co-ordinates Given the coordinates (a,b) (c,d) (e,f) (g,h) of a parallelogram , the coordinates of the meeting point of the diagonals can be found out by solving for [(a+e)/2,(b+f)/2] =[ (c+g)/2 , (d+h)/2] Ratio If a1/b1 = a2/b2 = a3/b3 = .............. , then each ratio is equal to (k1*a1+ k2*a2+k3*a3+..............) / (k1*b1+ k2*b2+k3*b3+..............) , which is also equal to (a1+a2+a3+............./b1+b2+b3+..........) Finding multiples x^n -a^n = (x-a)(x^(n-1) + x^(n-2) + .......+ a^(n-1) ) ......Very useful for finding multiples .For example (17-14=3 will be a multiple of 17^3 - 14^3) Exponents e^x = 1 + (x)/1! + (x^2)/2! + (x^3)/3! + ........to infinity 2 <>GP -> In a GP the product of any two terms equidistant from a term is always constant . -> The sum of an infinite GP = a/(1-r) , where a and r are resp. the first term and common ratio of the GP . Mixtures If Q be the volume of a vessel q qty of a mixture of water and wine be removed each time from a mixture n be the number of times this operation be done and A be the final qty of wine in the mixture then , A/Q = (1-q/Q)^n Some Pythagorean triplets: 3,4,5----------(3^2=4+5) 5,12,13--------(5^2=12+13) 7,24,25--------(7^2=24+25) 8,15,17--------(8^2 / 2 = 15+17 ) 9,40,41--------(9^2=40+41) 11,60,61-------(11^2=60+61) 12,35,37-------(12^2 / 2 = 35+37) 16,63,65-------(16^2 /2 = 63+65) 20,21,29-------(EXCEPTION) Appolonius theorem Appolonius theorem could be applied to the 4 triangles formed in a parallelogram. Function Any function of the type y=f(x)=(ax-b)/(bx-a) is always of the form x=f(y) . Finding Squares To find the squares of numbers from 50 to 59 For 5X^2 , use the formulae (5X)^2 = 5^2 +X / X^2 Eg ; (55^2) = 25+5 /25 =3025 (56)^2 = 25+6/36 =3136 (59)^2 = 25+9/81 =3481 Successive Discounts Formula for successive discounts a+b+(ab/100) This is used for succesive discounts types of sums.like 1999 population increses by 10% and then in 2000 by 5% so the population in 2000 now is 10+5+(50/100)=+15.5% more that was in 1999 and if there is a decrease then it will be preceeded by a -ve sign and likewise. Rules of Logarithms: -> loga(M)=y if and only if M=ay -> loga(MN)=loga(M)+loga(N) -> loga(M/N)=loga(M)-loga(N) -> loga(Mp)=p*loga(M) -> loga(1)=0-> loga(ap)=p -> log(1+x) = x - (x^2)/2 + (x^3)/3 - (x^4)/4 .........to infinity [ Note the alternating sign . .Also note that the ogarithm is with respect to base e ] 1. In India, other than ensuring that public funds are used efficiently and for intended purpose, what is the importance of the office of the Comptroller and Auditor General (CAG)? 1. CAG exercises exchequer control on behalf of the Parliament when the President of India declares national emergency/financial emergency. 2. CAG reports on the execution of projects or programmes by the ministries are discussed by the Public Accounts Committee. 3. Information from CAG reports can be used by investigating agencies to press charges against those who have violated the law while managing public finances. 4. While dealing with the audit and accounting of government companies, CAG has certain judicial powers for prosecuting those who violate the law. Which of the statements given above is/are correct? (a) 1, 3 and 4 only (b) 2 only (c) 2 and 3 only (d) 1, 2, 3 and 4 ANSWER: (c) 2. The endeavour of 'Janani Suraksha Yojana' Programme is 1. to promote institutional deliveries 2. to provide monetary assistance to the mother to meet the cost of delivery 3. to provide for wage loss due to pregnancy and confinement Which of the statements given above is /are correct? (a) 1 and 2 only (b) 2 only (c) 3 only (d) 1, 2 and 3 ANSWER: (d) 3. The Prime Minister of India, at the time of his/her appointment (a) need not necessarily be a member of one of the Houses of the Parliament but must become a member of one of the" Houses within six months (b) need not necessarily be a member of one of the Houses of the Parliament but must become a member of the Lok Sabha within six months (c) must be a member of one of the Houses of the Parliament (d) must be a member of the Lok Sabha ANSWER: (a) 4. With reference to the Delimitation Commission, consider the following statements : 1. The orders of the Delimitation Commission cannot be challenged in a Court of Law. 2. When the orders of the Delimitation Commission are laid before the Lok Sabha or State Legislative Assembly, they cannot effect any modifications in the orders. Which of the statements given above is/are correct? (a) 1 only (b) 2 only (c) Both 1 and 2 (d) Neither 1 nor 2 ANSWER: (c) 5. Consider the following: 1. Hotels and restaurants 2. Motor transport undertakings 3. Newspaper establishments 4. Private medical institutions The employees of which of the above can have the 'Social Security' coverage under Employees' State Insurance Scheme? (a) 1, 2 and 3 only (b) 4 only (c) 1, 3 and 4 only (d) 1, 2, 3 and 4 ANSWER: (d) 6. According to the Constitution of India, it is the duty of the President of India to cause to be laid before the Parliament which of the following? 1. The Recommendations of the Union Finance Commission 2. The Report of the Public Accounts Committee 3. The Report of the Comptroller and Auditor General 4. The Report of the National Commission for Scheduled Castes Select the correct answer using the codes given below : (a) 1 only (b) 2 and 4 only (c) 1, 3 and 4 only (d) 1, 2, 3 and 4 ANSWER: (c) 7. A deadlock between the Lok Sabha and the Rajya Sabha calls for a joint sitting of the Parliament during the passage of 1. Ordinary Legislation 2. Money Bill 3. Constitution Amendment Bill Select the correct answer using the codes given below : (a) 1 only (b) 2 and 3 only (c) 1 and 3 only (d) 1, 2 and 3 ANSWER: (a) 8. How do District Rural Development Agencies (DRDAs) help in the reduction of rural poverty in India? 1. DRDAs act as Panchayati Raj Institutions in certain specified backward regions of the country. 2. DRDAs undertake area-specific scientific study of the causes of poverty and malnutrition and prepare detailed remedial measures. 3. DRDAs secure inter-sectoral and inter-departmental coordination and cooperation for effective implementation of anti-poverty programmes. 4. DRDAs watch over and ensure effective utilization of the funds intended for anti-poverty programmes. Which of the statements given above is/are correct? (a) 1, 2 and 3 only (b) 3 and 4 only (c) 4 only (d) 1, 2, 3 and 4 ANSWER: (b) 9. Which of the following is/are among the Fundamental Duties of citizens laid down in the Indian Constitution? 1. To preserve the rich heritage of our composite culture 2. To protect the weaker sections from social injustice 3. To develop the scientific temper and spirit of inquiry 4. To strive towards excellence in all spheres of individual and collective activity Select the correct answer using the codes given below : (a) 1 and 2 only (b) 2 only (c) 1, 3 and 4 only (d) 1, 2, 3 and 4 ANSWER: (c) 10. What is the provision to safeguard the autonomy of the Supreme Court of India? 1. While appointing the Supreme Court Judges, the President of India has to consult the Chief Justice of India. 2. The Supreme Court Judges can be removed by the Chief Justice of India only. 3. The salaries of the Judges are charged on the Consolidated Fund of India to which the legislature does not have to vote. 4. All appointments of officers and staffs of the Supreme Court of India are made by the Government only after consulting the Chief Justice of India. Which of the statements given above is/are correct? (a) 1 and 3 only (b) 3 and 4 only (c) 4 only (d) 1, 2, 3 and 4 ANSWER: (a) 11. To meet its rapidly growing energy demand, some opine that India should pursue research and development on thorium as the future fuel of nuclear energy. In this context, what advantage does thorium hold over uranium? 1. Thorium is far more abundant in nature than uranium. 2. On the basis of per unit mass of mined mineral, thorium can generate more energy compared to natural uranium. 3. Thorium produces less harmful waste compared to uranium. Which of the statements given above is/are correct? (a) 1 only (b) 2 and 3 only (c) 1 and 3 only (d) 1, 2 and 3 ANSWER: (d) 12. The increasing amount of carbon dioxide in the air is slowly raising the temperature of the atmosphere, because it absorbs (a) the water vapour of the air and retains its heat (b) the ultraviolet part of the solar radiation (c) all the solar radiations (d) the infrared part of the solar radiation ANSWER: (d) 13. Which one of the following sets of elements was primarily responsible for the origin of life on the Earth? (a) Hydrogen, Oxygen, Sodium (b) Carbon, Hydrogen, Nitrogen (c) Oxygen, Calcium, Phosphorus (d) Carbon, Hydrogen, Potassium ANSWER: (b) 14. What are the reasons for the people's resistance to the introduction of Bt brinjal in India? 1. Bt brinjal has been created by inserting a gene from a soil fungus into its genome. 2. The seeds of Bt brinjal are terminator seeds and therefore, the farmers have to buy the seeds before every season from the seed companies. 3. There is an apprehension that the consumption of Bt brinjal may have adverse impact on health. 4. There is some concern that the introduction of Bt brinjal may have adverse effect on the biodiversity. Select the correct answer using the codes given below : (a) 1, 2 and 3 only (b) 2 and 3 only (c) 3 and 4 only (d) 1, 2, 3 and 4 ANSWER: (c) 15. Other than resistance to pests, what are the prospects for which genetically engineered plants have been created? 1. To enable them to withstand drought 2. To increase the nutritive value of the produce 3. To enable them to grow and do photosynthesis in spaceships and space stations 4. To increase their shelf life Select the correct answer using the codes given below : (a) 1 and 2 only (b) 3 and 4 only (c) 1, 2 and 4 only (d) 1, 2, 3 and 4 ANSWER: (c) 16. Consider the following statements : The most effective contribution made by Dadabhai Naoroji to the cause of Indian National Movement was that he 1. exposed the economic exploitation of India by the British 2. interpreted the ancient Indian texts and restored the self-confidence of Indians 3. stressed the need for eradication of all the social evils before anything else Which of the statements given above is/are correct? (a) 1 only (b) 2 and 3 only (c) 1 and 3 only (d) 1, 2 and 3 ANSWER: (a) 17. With reference to Dhrupad, one of the major traditions of India that has been kept alive for centuries, which of the following statements are correct? 1. Dhrupad originated and developed in the Rajput kingdoms during the Mughal period. 2. Dhrupad is primarily a devotional and spiritual music. 3. Dhrupad Alap uses Sanskrit syllables from Mantras. Select the correct answer using the codes given below : (a) 1 and 2 only (b) 2 and 3 only (c) 1, 2 and 3 (d) None of the above is correct ANSWER: (b) 18. How do you distinguish between Kuchipudi and Bharatanatyam dances? 1. Dancers occasionally speaking dialogues is found in Kuchipudi dance but not in Bharatanatyam. 2. Dancing on the brass plate by keeping the feet on its edges is a feature of Bharatanatyam but Kuchipudi dance does not have such a form of movements. Which of the statements given above is/are correct? (a) 1 only (b) 2 only (c) Both 1 and 2 (d) Neither 1 nor 2 ANSWER: (c) 19. With reference to the religious history of medieval India, the Sufi mystics were known to pursue which of the following practices? 1. Meditation and control of breath 2. Severe ascetic exercises in a lonely place 3. Recitation of holy songs to arouse a state of ecstasy in their audience Select the correct answer using the codes given below : (a) 1 and 2 only (b) 2 and 3 only (c) 3 only (d) 1, 2 and 3 ANSWER: (d) 20. The Rowlatt Act aimed at (a) compulsory economic support to war efforts (b) imprisonment without trial and summary procedures for trial (c) suppression of the Khilafat Movement (d) imposition of restrictions on freedom of the press ANSWER: (b) 21. The Lahore Session of the Indian National Congress (1929) is very important in history, because 1. the Congress passed a resolution demanding complete independence 2. the rift between the extremists and moderates was resolved in that Session 3. a resolution was passed rejecting the two-nation theory in that Session Which of the statements given above is/are correct? (a) I only (b) 2 and 3 (c) 1 and 3 (d) None of the above ANSWER: (a) 22. Lord Buddha's image is sometimes shown with the hand gesture called 'Bhumisparsha Mudra'. It symbolizes (a) Buddha's calling of the Earth to watch over Mara and to prevent Mara from disturbing his meditation (b) Buddha's calling of the Earth to witness his purity and chastity despite the temptations of Mara (c) Buddha's reminder to his followers that they all arise from the Earth and finally dissolve into the Earth, and thus this life is transitory (d) Both the statements (a) and (b) are correct in this context ANSWER: (b) 23. The religion of early Vedic Aryans was primarily of (a) Bhakti (b) image worship and Yajnas (c) worship of nature and Yajnas (d) worship of nature and Bhakti ANSWER: (c) 24. Which of the following statements is/are correct regarding Brahmo Samaj? 1. It opposed idolatry. 2. It denied the need for a priestly class for interpreting the religious texts. 3. It popularized the doctrine that the Vedas are infallible. Select the correct answer using the codes given below : (a) 1 only (b) 1 and 2 only (c) 3 only (d) 1, 2 and 3 ANSWER: (b) 25. The Reserve Bank of India (RBI) acts as a bankers' bank. This would imply which of the following? 1. Other banks retain their deposits with the RBI. 2. The RBI lends funds to the commercial banks in times of need. 3. The RBI advises the commercial banks on monetary matters. Select the correct answer using the codes given below : (a) 2 and 3 only (b) 1 and 2 only (c) 1 and 3 only (d) 1, 2 and 3 ANSWER: (d) 26. Under which of the following circumstances may 'capital gains' arise? 1. When there is an increase in the sales of a product 2. When there is a. natural increase in the value of the property owned 3. When you purchase a painting and there is a growth in its value due to increase in its popularity Select the correct answer using the codes given below : (a) 1 only (b) 2 and 3 only (c) 2 only (d) 1, 2 and 3 ANSWER: (b) 27. Which of the following measures would result in an increase in the money supply in the economy? 1. Purchase of government securities from the public by the Central Bank 2. Deposit of currency in commercial banks by the public 3. Borrowing by the government from the Central Bank 4. Sale of government securities to the public by the Central Bank Select the correct answer using the codes given below : (a) 1 only (b) 2 and 4 only (c) 1 and 3 (d) 2, 3 and 4 ANSWER: (c) 28. Which of the following would include Foreign Direct Investment in India? 1. Subsidiaries of companies in India 2. Majority foreign equity holding in Indian companies 3. Companies exclusively financed by foreign companies 4. Portfolio investment Select the correct answer using the codes given below : (a) 1, 2, 3 and 4 (b) 2 and 4 only (c) 1 and 3 only (d) 1, 2 and 3 only ANSWER: (d) 29. Consider the following statements: The price of any currency in international market is decided by the 1. World Bank 2. demand for goods/services provided by the country concerned 3. stability of the government of the concerned country 4. economic potential of the country in question Which of the statements given above are correct? (a) 1, 2, 3 and 4 (b) 2 and 3 only (c) 3 and 4 only (d) 1 and 4 only ANSWER: (b) 30. The basic aim of Lead Bank Scheme is that (a) big banks should try to open offices in each district (b) there should be stiff competition among the various nationalized banks (c) individual banks should adopt particular districts for intensive development (d) all the banks should make intensive efforts to mobilize deposits ANSWER: (c) 31. Consider the following : 1. Assessment of land revenue on the basis of nature of the soil and the quality of crops 2. Use of mobile cannons in warfare 3. Cultivation of tobacco and red chillies Which of the above was/were introduced into India by the English? (a) 1 only (b) 1 and 2 (c) 2 and 3 (d) None ANSWER: (d) 32. With reference to the guilds (Shreni) of ancient India that played a very important role in the country's economy, which of the following statements is /are correct? 1. Every guild was registered with the central authority of the State and the king was the chief administrative authority on them. 2. The wages, rules of work, standards and prices were fixed by the guild. 3. The guild had judicial powers over its own members. Select the correct answer using the codes given below : (a) 1 and 2 only (b) 3 only (c) 2 and 3 only (d) 1, 2 and 3 ANSWER: (d) 33. The distribution of powers between the Centre and the States in the Indian Constitution is based on the scheme provided in the (a) Morley-Minto Reforms, 1909 (b) Montagu-Chelmsford Act, 1919 (c) Government of India Act, 1935 (d) Indian Independence Act, 1947 ANSWER: (c) 34. Despite having large reserves of coal, why does India import millions of tonnes of coal? 1. It is the policy of India to save its own coal reserves for future, and import it from other countries for the present use. 2. Most of the power plants in India are coal-based and they are not able to get sufficient supplies of coal from within the country. 3. Steel companies need large quantity of coking coal which has to be imported. Which of the statements given above is/are correct? (a) 1 only (b) 2 and 3 only (c) 1 and 3 only (d) 1, 2 and 3 ANSWER: (b) 35. A person stood alone in a desert on a dark night and wanted to reach his village which was situated 5 km east of the point where he was standing. He had no instruments to find the direction but he located the polestar. The most convenient way now to reach his village is to walk in the (a) direction facing the polestar (b) direction opposite to the polestar (c) direction keeping the polestar to his left (d) direction keeping the polestar to his right ANSWER: (c) 36. Recently, there has been a concern over the short supply of a group of elements called 'rare earth metals'. Why? 1. China, which is the largest producer of these elements, has imposed some restrictions on their export. 2. Other than China, Australia, Canada and Chile, these elements are not found in any country. 3. Rare earth metals are essential for the manufacture of various kinds of electronic items and there is a growing demand for these elements. Which of the statements given above is/are correct? (a) 1 only (b) 2 and 3 only (c) 1 and 3 only (d) 1, 2 and 3 ANSWER: (c) 37. Consider the following areas: 1. Bandipur 2. Bhitarkanika 3. Manas 4. Sunderbans Which of the above are Tiger Reserves? (a) 1 and 2 only (b) 1, 3 and 4 only (c) 2, 3 and 4 only (d) l, 2, 3 and 4 ANSWER: (b) 38. Consider the following statements : 1. The duration of the monsoon decreases from southern India to northern India. 2. The amount of annual rainfall in the northern plains of India decreases from east to west. Which of the statements given above is/are correct? (a) 1 only (b) 2 only (c) Both 1 and 2 (d) Neither 1 nor 2 ANSWER: (c) 39. Which one of the following is the characteristic climate of the Tropical Savannah Region? (a) Rainfall throughout the year (b) Rainfall in winter only (c) An extremely short dry season (d) A definite dry and wet season ANSWER: (d) 40. In which one among the following categories of protected areas in India are local people not allowed to collect and use the biomass? (a) Biosphere Reserves (b) National Parks (c) Wetlands declared under Ramsar Convention (d) Wildlife Sanctuaries ANSWER: (b) 41. Consider the following kinds of organisms : 1. Bat 2. Bee 3. Bird Which of the above is/are pollinating agent/agents? (a) 1 and 2 only (b) 2 only (c) 1 and 3 only (d) 1, 2 and 3 ANSWER: (d) 42. Which one of the following groups of animals belongs to the category of endangered species? (a) Great Indian Bustard, Musk Deer, Red Panda and Asiatic Wild Ass (b) Kashmir Stag, Cheetal, Blue Bull and Great Indian Bustard (c) Snow Leopard, Swamp Deer, Rhesus Monkey and Saras (Crane) (d) Lion-tailed Macaque, Blue Bull, Hanuman Langur and Cheetal ANSWER: (a) 43. Consider the following statements : If there were no phenomenon of capillarity 1. it would be difficult to use a kerosene lamp 2. one would not be able to use a straw to consume a soft drink 3. the blotting paper would fail to function 4. the big trees that we see around would not have grown on the Earth Which of the statements given above are correct? (a) 1, 2 and 3 only (b) 1, 3 and 4 only (c) 2 and 4 only (d) 1, 2, 3 and 4 ANSWER: (b) 44. The Millennium Ecosystem Assessment describes the following major categories of ecosystem services-provisioning, supporting, regulating, preserving and cultural. Which one of the following is supporting service? (a) Production of food and water (b) Control of climate and disease (c) Nutrient cycling and crop pollination (d) Maintenance of diversity ANSWER: (d) 45. What is the difference between the antelopes Oryx and Chiru? (a) Oryx is adapted to live in hot and arid areas whereas Chiru is adapted to live in steppes and semi-desert areas of cold high mountains (b) Oryx is poached for its antlers whereas Chiru is poached for its musk (c) Oryx exists in western India only whereas Chiru exists in north-east India only (d) None of the statements (a), (b) and (c) given above is correct ANSWER: (a) 46. Which of the following can be threats to the biodiversity of a geographical area? 1. Global warming 2. Fragmentation of habitat 3. Invasion of alien species 4. Promotion of vegetarianism Select the correct answer using the codes given below : (a) 1, 2 and 3 only (b) 2 and 3 only (c) 1 and 4 only (d) 1, 2, 3 and 4 ANSWER: (a) 47. Consider the following : 1. Black-necked crane 2. Cheetah 3. Flying squirrel 4. Snow leopard Which of the above are naturally found in India? (a) 1, 2 and 3 only (b) 1, 3 and 4 only (c) 2 and 4 only (d) 1, 2, 3 and 4 ANSWER: (b) 48. Consider the following agricultural practices : 1. Contour bunding 2. Relay cropping 3. Zero tillage In the context of global climate change, which of the above helps/help in carbon seques-tration/storage in the soil? (a) 1 and 2 only (b) 3 only (c) 1, 2 and 3 (d) None of them ANSWER: (c) 49. What would happen if phytoplankton of an ocean is completely destroyed for some reason? 1. The ocean as a carbon sink would be adversely affected. 2. The food chains in the ocean would be adversely affected. 3. The density of ocean water would drastically decrease. Select the correct answer using the codes given below : (a) 1 and 2 only (b) 2 only (c) 3 only (d) 1, 2 and 3 ANSWER: (d) 50. Vultures which used to be very common in Indian countryside some years ago are rarely seen nowadays. This is attributed to (a) the destruction of their nesting sites by new invasive species (b) a drug used by cattle owners for treating their diseased cattle (c) scarcity of food available to them (d) a widespread, persistent and fatal disease among them ANSWER: (b) 51. In the areas covered under the Panchayat (Extension to the Scheduled Areas) Act, 1996, what is the role/power of Gram Sabha? 1. Gram Sabha has the power to prevent alienation of land in the Scheduled Areas. 2. Gram Sabha has the ownership of minor forest produce. 3. Recommendation of Gram Sabha is required for granting prospecting licence or mining lease for any mineral in the Scheduled Areas. Which of the statements given above is/are correct? (a) 1 only (b) 1 and 2 only (c) 2 and 3 only (d) 1, 2 and 3 ANSWER: (b) 52. In the Parliament of India, the purpose of an adjournment motion is (a) to allow a discussion on a definite matter of urgent public importance (b) to let opposition members collect information from the ministers (c) to allow a reduction of specific amount in demand for grant (d) to postpone the proceedings to check the inappropriate or violent behaviour on the part of some members ANSWER: (a) 53. How does National Biodiversity Authority (NBA) help in protecting the Indian agriculture? 1. NBA checks the biopiracy and protects the indigenous and traditional genetic resources. 2. NBA directly monitors and supervises the scientific research on genetic modifi-cation of crop plants. 3. Application for Intellectual Property Rights related to genetic/biological resources cannot be made without the approval of NBA. Which of the statements given above is /are correct? (a) 1 -only (b) 2 and 3 only (c) 1 and 3 only (d) 1, 2 and 3 ANSWER: (d) 54. The National Green Tribunal Act, 2010 was enacted in consonance with which of the following provisions of the Constitution of India? 1. Right to healthy environment, construed as a part of Right to life under Article 21 2. Provision of grants for raising the level of administration in the Scheduled Areas for the welfare of Scheduled Tribes under Article 275(1) 3. Powers and functions of Gram Sabha as mentioned under Article 243(A) Select the correct answer using the codes given below : (a) 1 only (b) 2 and 3 only (c) 1 and 3 only (d) 1, 2 and 3 ANSWER: (a) 55. If National Water Mission is properly and completely implemented, how will it impact the country? 1. Part of the water needs of urban areas will be met through recycling of wastewater. 2. The water requirements of coastal cities with inadequate alternative sources of water will be met by adopting appropriate technologies that allow for the use of ocean water. 3. All the rivers of Himalayan origin will be linked to the rivers of peninsular India, 4. The expenses incurred by farmers for digging bore-wells and for installing motors and pump-sets to draw groundwater will be completely reimbursed by the Government. Select the correct answer using the codes given below : (a) 1 only (b) 1 and 2 only (c) 3 and 4 only (d) 1, 2, 3 and 4 ANSWER: (a) 56. Consider the following provisions under the Directive Principles of State Policy as enshrined in the Constitution of India : 1. Securing for citizens of India a uniform civil code 2. Organizing village Panchayats 3. Promoting cottage industries in rural areas 4. Securing for all the workers reasonable leisure and cultural opportunities Which of the above are the Gandhian Principles that are reflected in the Directive Principles of State Policy? (a) 1, 2 and 4 only (b) 2 and 3 only (c) 1, 3 and 4 only (d) 1, 2, 3 and 4 ANSWER: (b) 57. Consider the following statements: 1. Union Territories are not represented in the Rajya Sabha. 2. It is within the purview of the Chief Election Commissioner to adjudicate the election disputes. 3. According to the Constitution of India, the Parliament consists of the Lok Sabha and the Rajya Sabha only. Which of the statements given above is/are correct? (a) 1 only (b) 2 and 3 (c) 1 and 3 (d) None ANSWER: (d) 58. With reference to consumers' rights/ privileges under the provisions of law in India, which of the following statements is/are correct? 1. Consumers are empowered to take samples for food testing. 2. When a consumer files a complaint in any consumer forum, no fee is required to be paid. 3. In case of death of a consumer, his/her legal heir can file a complaint in the consumer forum on his/her behalf. Select the correct answer using the codes given below: (a) 1 only (b) 2 and 3 only (c) 1 and 3 only (d) 1, 2 and 3 ANSWER: (c) 59. Regarding the office of the Lok Sabha Speaker, consider the following statements: 1. He/She holds the office during the pleasure of the President. 2. He/She need not be a member of the House at the time of his/her election but has to become a member of the House within six months from the date of his/her election. 3. If he/she intends to resign, the letter of his/her resignation has to be addressed to the Deputy Speaker. Which of the statements given above is /are correct? (a) 1 and 2 only (b) 3 only (c) 1, 2 and 3 (d) None ANSWER: (b) 60. Which of the following are included in the original jurisdiction of the Supreme Court? 1. A dispute between the Government of India and one or more States 2. A dispute regarding elections to either House of the Parliament or that of Legislature of a State 3. A dispute between the Government of India and a Union Territory 4. A dispute between two or more States Select the correct answer using the codes given below : (a) 1 and 2 (b) 2 and 3 (c) 1 and 4 (d) 3 and 4 ANSWER: (c) 61. Consider the following kinds of organisms : 1. Bacteria 2. Fungi 3. Flowering plants Some species of which of the above kinds of organisms are employed as biopesticides? (a) 1 only (b) 2 and 3 only (c) 1 and 3 only (d) 1, 2 and 3 ANSWER: (d) 62. Biomass gasification is considered to be one of the sustainable solutions to the power crisis in India. In this context, which of the following statements is/are correct? 1. Coconut shells, groundnut shells and rice husk can be used in biomass gasification. 2. The combustible gases generated from biomass gasification consist of hydrogen and carbon dioxide only. 3. The combustible gases generated from biomass gasification can be used for direct heat generation but not in internal combustion engines. Select the correct answer using the codes given below : (a) 1 only (b) 2 and 3 only (c) 1 and 3 onlly (d) 1, 2 and 3 ANSWER: (a) 63. What is the role of ultraviolet (UV) radiation in the water purification systems? 1. It inactivates /kills the harmful microorganisms in water. 2. It removes all the undesirable odours from the water. 3. It quickens the sedimentation of solid particles, removes turbidity and improves the clarity of water. Which of the statements given above is/are correct? (a) 1 only (b) 2 and 3 only (c) 1 and 3 only (d) 1, 2 and 3 ANSWER: (a) 64. Graphene is frequently in news recently. What is its importance? 1. It is a two-dimensional material and has good electrical conductivity. 2. It is one of the thinnest but strongest materials tested so far. 3. It is entirely made of silicon and has high optical transparency. 4. It can be used as 'conducting electrodes' required for touch screens, LCDs and organic LEDs. Which of the statements given above are correct? (a) 1 and 2 only (b) 3 and 4 only (c) 1, 2 and 4 only (d) 1, 2, 3 and 4 ANSWER: (c) 65. Lead, ingested or inhaled, is a health hazard. After the addition of lead to petrol has been banned, what still are the sources of lead poisoning? 1. Smelting units 2. Pens and pencils 3. Paints 4. Hair oils and cosmetics Select the correct answer using the codes given below : (a) 1, 2 and 3 only (b) 1 and 3 only (c) 2 and 4 only (d) 1, 2, 3 and 4 ANSWER: (b) 66. With reference to 'stem cells', frequently in the news, which of the following statements is/are correct? 1. Stem cells can be derived from mammals only. 2. Stem cells can be used for screening new drugs. 3. Stem cells can be used for medical therapies. Select the correct answer using the codes given below : (a) 1 and 2 only (b) 2 and 3 only (c) 3 only (d) 1, 2 and 3 ANSWER: (b) 67. Consider the following statements : Chlorofluorocarbons, known as ozone-depleting substances, are used 1. in the production of plastic foam 2. in the production of tubeless tyres 3. in cleaning certain electronic components 4. as pressurizing agents in aerosol cans Which of the statements given above is/are correct? (a) 1, 2 and 3 only (b) 4 only (c) 1, 3 and 4 only (d) 1, 2, 3 and 4 ANSWER: (c) 68. A team of scientists at Brookhaven National Laboratory including those from India created the heaviest anti-matter (anti-helium nucleus). What is/are the implication/ implications of the creation of anti-matter? 1. It will make mineral prospecting and oil exploration easier and cheaper. 2. It will help probe the possibility of the existence of stars and galaxies made of anti-matter. 3. It will help understand the evolution of the universe. Select the correct answer using the codes given below : (a) 1 only (b) 2 and 3 only (c) 3 only (d) 1, 2 and 3 ANSWER: (b) 69. Which of the following is /are cited by the scientists as evidence/ evidences for the continued expansion of universe? 1. Detection of microwaves in space 2. Observation of redshift phenomenon in space 3. Movement of asteroids in space 4. Occurrence of supernova explosions in space Select the correct answer using the codes given below : (a) 1 and 2 (b) 2 only (c) 1, 3 and 4 (d) None of the above can be cited as evidence ANSWER: (b) 70. Electrically charged particles from space travelling at speeds of several hundred km/sec can severely harm living beings if they reach the surface of the Earth. What prevents them from reaching the surface of the Earth? (a) The Earth's magnetic field diverts them towards its poles (b) Ozone layer around the Earth reflects them back to outer space (c) Moisture in the upper layers of atmosphere prevents them from reaching the surface of the Earth (d) None of the statements (a), (b) and (c) given above is correct ANSWER: (a) 71. With reference to the scientific progress of ancient India, which of the statements given below are correct? 1. Different kinds of specialized surgical instruments were in common use by 1st century AD. 2. Transplant of internal organs in the human body had begun by the beginning of 3rd century AD. 3. The concept of sine of an angle was known in 5th century AD. 4. The concept of cyclic quadrilaterals was known in 7th century AD. Select the correct answer using the codes given below : (a) 1 and 2 only (b) 3 and 4 only (c) 1, 3 and 4 only (d) 1, 2, 3 and 4 ANSWER: (d) 72. With reference to the history. of ancient India, which of the following was/were common to both Buddhism and Jainism? 1. Avoidance of extremities of penance and enjoyment 2. Indifference to the authority of the Vedas 3. Denial of efficacy of rituals Select the correct answer using the codes given below : (a) 1 only (b) 2 and 3 only (c) 1 and 3 only (d) 1, 2 and 3 ANSWER: (b) 73. Which of the following can be said to be essentially the parts of Inclusive Governance? 1. Permitting the Non-Banking Financial Companies to do banking 2. Establishing effective District Planning Committees in all the districts 3. Increasing the government spending on public health 4. Strengthening the Mid-day Meal Scheme Select the correct answer using the codes given below : (a)1 and 2 only (b) 3 and 4 only (c) 2, 3 and 4 only (d) 1, 2, 3 and 4 ANSWER: (c) 74. The Nagara, the Dravida and the Vesara are the (a) three main racial groups of the Indian subcontinent (b) three main linguistic divisions into which the languages of India can be classified (c) three main styles of Indian temple architecture (d) three main musical Gharanas prevalent in India ANSWER: (c) 75. The Congress ministries resigned in the seven provinces in 1939, because (a) the Congress could not form ministries in the other four provinces(b) emergence of a left wing' in the Congress made the working of the ministries impossible (c) there were widespread communal disturbances in their provinces (d) None of the statements (a), (b) and (c) given above is correct ANSWER: (d) 76. With reference to National Rural Health Mission, which of the following are the jobs of 'ASHA', a trained community health worker? 1. Accompanying women to the health facility for antenatal care checkup 2. Using pregnancy test kits for early detection of pregnancy 3. Providing information on nutrition and immunization 4. Conducting the delivery of baby Select the correct answer using the codes given below: (a) 1, 2 and 3 Only (b) 2 and 4 Only (c) 1 and 3 Only (d) 1, 2, 3 and 4 ANSWER: (c) 77. Which of the following is/are the principal feature(s) of the Government of India Act, 1919? 1. Introduction of dyarchy in the executive government of the provinces 2. Introduction of separate communal electorates for Muslims 3. Devolution of legislative authority by the centre to the provinces Select the correct answer using the codes given below : (a) 1 only (b) 2 and 3 only (c) 1 and 3 only (d) 1, 2 and 3 ANSWER: (c) 78. During Indian freedom struggle, the National Social Conference was formed. What was the reason for its formation? (a) Different social reform groups or organizations of Bengal region united to form a single body to discuss the issues of larger interest and to prepare appropriate petitions/representations to the government. (b) Indian National Congress did not want to include social reforms in its deliberations and decided to form a separate body for such a purpose (c) Behramji Malabari and M. G. Ranade decided to bring together all the social reform groups of the country under one organization (d) None of the statements (a), (b) and (c) given above is correct in this context ANSWER: (d) 79. Which of the following parties were established by Dr. B. R. Ambedkar? 1. The Peasants and Workers Party of India 2. All India Scheduled Castes Federation 3. The Independent Labour Party Select the correct answer using the codes given below : (a) 1 and 2 only (b) 2 and 3 only (c) 1 and 3 only (d) 1, 2 and 3 ANSWER: (b) 80. Which of the following special powers have been conferred on the Rajya Sabha by the Constitution of India? (a) To change the existing territory of a State and to change the name of a State (b) To pass a resolution empowering the Parliament to make laws in the State List and to create one or more All India Services (c) To amend the election procedure of the President and to determine the pension of the President after his/her retirement (d) To determine the functions of the Election Commission and to determine the number of Election Commissioners ANSWER: (b) 81. How does the National Rural Livelihood Mission seek to improve livelihood options of rural poor? 1. By setting up a large number of new manufacturing industries and agribusiness centres in rural areas 2. By strengthening 'self-help groups' and providing skill development 3. By supplying seeds, fertilizers, diesel pump-sets and micro-irrigation equipment free of cost to farmers Select the correct answer using the codes given below : (a) 1 and 2 only (b) 2 only (c) 1 and 3 only (d) 1, 2 and 3 ANSWER: (b) 82. The Multi-dimensional Poverty Index developed by Oxford Poverty and Human Development Initiative with UNDP support covers which of the following? 1. Deprivation of education, health, assets and services at household level 2. Purchasing power parity at national level 3. Extent of budget deficit and GDP growth rate at national level Select the correct answer using the codes given below : (a) 1 only (b) 2 and 3 only (c) 1 and 3 only (d) 1, 2 and 3 ANSWER: (a) 83. Which of the following is /are among the noticeable features of the recommendations of the Thirteenth Finance Commission? 1. A design for the Goods and Services Tax, and a compensation package linked to adherence to the proposed design 2. A design for the creation of lakhs of jobs in the next ten years in consonance with India's demographic dividend 3. Devolution of a specified share of central taxes to local bodies as grants Select the correct answer using the codes given below : (a) 1 only (b) 2 and 3 only (c) 1 and 3 only (d) 1, 2 and 3 ANSWER: (a) 84. What is/are the recent policy initiative(s) of Government of India to promote the growth of manufacturing sector? 1. Setting up of National Investment and Manufacturing Zones 2. Providing the benefit of 'single window clearance' 3. Establishing the Technology Acquisition and Development Fund Select the correct answer using the codes given below : (a) 1 only (b) 2 and 3 only (c) 1 and 3 only (d) 1, 2 and 3 ANSWER: (d) 85. Which of the following are the methods of Parliamentary control over public finance in India? 1. Placing Annual Financial Statement before the Parliament 2. Withdrawal of moneys from Consolidated Fund of India only after passing the Appropriation Bill 3. Provisions of supplementary grants and vote-on-account 4. A periodic or at least a mid-year review of programme of the Government against macroeconomic forecasts and expenditure by a Parliamentary Budget Office 5. Introducing Finance Bill in the Parliament Select the correct answer using the codes given below : (a) 1, 2, 3 and 5 only (b) 1, 2 and 4 only (c) 3, 4 and 5 only (d) 1, 2, 3, 4 and 5 ANSWER: (a) 86. Mahatma Gandhi undertook fast unto death in 1932, mainly because (a) Round Table Conference failed to satisfy Indian political aspirations (b) Congress and Muslim League had differences of opinion (c) Ramsay Macdonald announced the Communal Award (d) None of the statements (a), (b) and (c) given above is correct in this context ANSWER: (c) 87. With reference to Ryotwari Settlement, consider the following statements : 1. The rent was paid directly by the peasants to the Government. 2. The Government gave Pattas to the Ryots. 3. The lands were surveyed and assessed before being taxed. Which of the statements given above is/are correct? (a) 1 only (b) 1 and 2 only (c) 1, 2 and 3 (d) None ANSWER: (c) 88. Consider the following specific stages of demographic transition associated with economic development : 1. Low birthrate with low death rate 2. High birthrate with high death rate 3. High birthrate with low death rate Select the correct order of the above stages using the codes given below : (a) 1, 2, 3 (b) 2, 1, 3 (c) 2, 3, 1 (d) 3, 2, 1 ANSWER: (c) 89. In India, in the overall Index of Industrial Production, the Indices of Eight Core Industries have a combined weight of 37-90%. Which of the following are among those Eight Core Industries? 1. Cement 2. Fertilizers 3. Natural gas 4. Refinery products 5. Textiles Select the correct answer using the codes given below : (a) 1 and 5 only (b) 2, 3 and 4 only (c) 1, 2, 3 and 4 only (d) 1, 2, 3, 4 and 5 ANSWER: (c) 90. Which of the following provisions of the Constitution of India have a bearing on Education? 1. Directive Principles of State Policy 2. Rural and Urban Local Bodies 3. Fifth Schedule 4. Sixth Schedule 5. Seventh Schedule Select the correct answer using the codes given below : (a) 1 and 2 only (b) 3, 4 and 5 only (c) 1, 2 and 5 only (d) 1, 2, 3, 4 and 5 ANSWER: (c) 91. Government of India encourages the cultivation of 'sea buckthorn'. What is the importance of this plant? 1. It helps in controlling soil erosion and in preventing desertification. 2. It is a rich source of biodiesel. 3. It has nutritional value and is well-adapted to live in cold areas of high altitudes. 4. Its timber is of great commercial value. Which of the statements given above is /are correct? (a) 1 only (b) 2, 3 and 4 only (c) 1 and 3 only (d) 1, 2, 3 and 4 ANSWER: (c) 92. Which of the following is the chief characteristic of 'mixed farming'? (a) Cultivation of both cash crops and food crops (b) Cultivation of two or more crops in the same field (c) Rearing of animals and cultivation of crops together (d) None of the above ANSWER: (c) 93. A particular State in India has the following characteristics : 1. It is located on the same latitude which passes through northern Rajasthan. 2. It has over 80% of its area under forest cover. 3. Over 12% of forest cover constitutes Protected Area Network in this State. Which one among the following States has all the above characteristics? (a) Arunachal Pradesh (b) Assam (c) Himachal Pradesh (d) Uttarakhand ANSWER: (a) 94. Consider the following crops of India : 1. Cowpea 2. Green gram 3. Pigeon pea Which of the above is/are used as pulse, fodder and green manure? (a) 1 and 2 only (b) 2 only (c) 1 and 3 only (d) 1, 2 and 3 ANSWER: (c) 95. Consider the following factors: 1. Rotation of the Earth 2. Air pressure and wind 3. Density of ocean water 4. Revolution of the Earth Which of the above factors influence the ocean currents? (a) 1 and 2 only (b) 1, 2 and 3 (c) 1 and 4 (d) 2, 3 and 4 ANSWER: (b) 96. With reference to the wetlands of India, consider the following statements : 1. The country's total geographical area under the category of wetlands is recorded more in Gujarat as compared to other States. 2. In India, the total geographical area of coastal wetlands larger than that of wetlands. Which of the statements given above is/are correct? (a) 1 only (b) 2 only (c) Both 1 and 2 (d) Neither 1 nor 2 ANSWER: (a) 97. Consider the following crops of India : 1. Groundnut 2. Sesamum 3. Pearl millet Which of the above is/are predomi-nantly rainfed crop/crops? (a) 1 and 2 only (b) 2 and 3 only (c) 3 only (d) 1, 2 and 3 ANSWER: (d) 98. When you travel in Himalayas, you will see the following : 1. Deep gorges 2. U-turn river courses 3. Parallel mountain ranges 4. Steep gradients causing land-sliding Which of the above can be said to be the evidences for Himalayas being young fold mountains? (a) 1 and 2 only (b) 1, 2 and 4 only (c) 3 and 4 only (d) 1, 2, 3 and 4 ANSWER: (b) 99. Normally, the temperature decreases with the increase in height from the Earth's surface, because 1. the atmosphere can be heated upwards only from the Earth's surface 2. there is more moisture in the upper atmosphere 3. the air is less dense in the upper atmosphere Select the correct answer using the codes given below : (a) 1 only (b) 2 and 3 only (c) 1 and 3 only (d) 1, 2 and 3 ANSWER: (c) 100. The acidification of oceans is increasing. Why is this phenomenon a cause of concern? 1. The growth and survival of calcareous phytoplankton will be adversely affected. 2. The growth and survival of coral reefs will be adversely affected. 3. The survival of some animals that have phytoplanktonic larvae will be adversely affected. 4. The cloud seeding and formation of clouds will be adversely affected. Which of the statements given above is /are correct? (a) 1, 2 and 3 only (b) 2 only (c) 1 and 3 only (d) 1, 2, 3 and 4 ANSWER: (a) FCI General Intelligence and aptitude FCI Assisstatn Grade III questions with answers for practice FCI Arithmetical Ability 1. The product of 2 numbers is 1575 and their quotient is 9/7. Then the sum of the numbers is – a. 74 b. 78 c. 80 d. 90 Ans : Let the numbers be x and y . ? xy = 1575 And x/y = 9/7 ? xy/x/y = 1575/9/7 ? y2 = 1225 ? y = 35 and x = 45 ? The sum of the numbers = 45+35 = 80 2. The value of (81)3.6 * (9)2.7/ (81)4.2 * (3) is __ a. 3 b. 6 c. 9 d. 8.2 Ans : (81)3.6 * (9)2.7/(81)4.2 * (3) = (3)14.4 * (3)5.4/(3)16.8 * (3) = 314.4+5.4-16.8-1 = 32 = 9 3. v6+v6+v6+… is equal to – a. 2 b. 5 c. 4 d. 3 Ans : Let v6+v6+v6+…. be x. ? x =v6+x ? x2 = 6+x ? x2 + x – 6 = 0 ? (x-3) (x+2) = 0 ? x = 3 4. The sum of the squares of two natural consecutive odd numbers is 394. The sum of the numbers is – a. 24 b. 32 c. 40 d. 28 Ans : Let the consecutive odd numbers be x and (x+2) ? x2 + (x + 2)2 = 394 ? x2 + x2 + 4x + 4 = 394 ? 2 x2 +4x – 390 = 0 ? x2 + 2x – 195 = 0 ? (x +15) (x-13) = 0 ? x = 13 ? Required sum = 13 +15 =28 5. When (6767 +67) is divided by 68, the remainder is- a. 1 b. 63 c. 66 d. 67 Ans: (6767+ 67) = 67(6766 + 166) As 66 is an even number ? 6766 is an even number ? (6766 + 1) is perfectly divisible by (67 + 1) i.e. 68 ? The remainder = 67 6. In a division sum, the divisor is 4 times the quotient and twice the remainder. If a and b are respectively the divisor and the dividend then- a. 4b-a2 /a =3 b. 4b-2a /a2 =2 c. (a+1)2 = 4b d. A(a+2)/b = 4 Ans : As divisor is a, and dividend is b. ? Quotient = a/4 And Remainder = a/2 ? b = a * a/4 + a/2 ? 4b = a2 + 2a ? a(a+2)/b = 4 7. If 738 A6A is divisible by 11, then the value of A is- a. 6 b. 3 c. 9 d. 1 Ans : As 738A6 A is divisible by 11. ? A + A + 3 = 6 + 8 + 7 ? A = 9 8. The east positive integer that should be subtracted from 3011 * 3012 so that the difference is perfect square is- a. 3009 b. 3010 c. 3011 d. 3012 Ans : ? 3011 * 3012 = 3011 (3011 + 1) = (3011)2 + 3011 ? Required least number = 3011 9. P, Q, R are employed to do a work for Rs. 5750. P and Q together finished 19/23 of work and Q and R together finished 8/23 of work. Wage of Q, in rupees, is- a. 2850 b. 3750 c. 2750 d. 1000 Ans : Work done by Q = 19/23 + 8/23 – 1 4/23 ? Wage of Q = 4/23 * 5750 = Rs. 1000 10. A can do a piece of work in 24 day, B in 32 days and C in 64 days. All begin to do it together, but A leaves after 6 days and B leaves 6 days before the completion of the work. How many days did the work last? a. 15 b. 20 c. 18 d. 30 Ans : Work done by A = 6/24 = ¼ Work done by B = (x-6)/32 (where x is no. of days in which work is competed) ? ¼ + x – 6/32 + x/64 = 1 ? 16 + 24 – 12 + x /64 = 1 ? 3x + 4 = 64 ? x = 60/3 = 20 days 11. The square root of (0.75)3 /1-0.75 + [0.75 + 90.75)2 +1] is- a. 1 b. 2 c. 3 d. 4 Ans : The square root of [(0.75)3/1 – 0.75 + {0.75 + (0.75)2 + 1}] = v1.6875 + 2.3125 =v4 = 2 12. Given that v4096 = 64, the value of v4096 + v40.96 +v0.004096 is- a. 70.4 b. 70.464 c. 71.104 d. 71.4 Ans : Given Exp. = v4096 + v40.96 +v0.004096 = 64 + 6.4 + 0.064 = 70.464 13. By selling an article at 3/4th of the marked price, there is a gain of 25%. The ratio of the marked price and the cost price is- a. 5 : 3 b. 3 : 5 c. 3 : 4 d. 4 : 3 Ans : Let of M.P. be Rs. x. ? S.P. = Rs.3x/4 and C.P. = 3x/4 * 100/125 = Rs.3x/5 ? required ratio = x: 3x/5 = 5:3 14. A and B earn in the ratio 2:1. They spend in the ratio 5:3 and save in the ratio 4:1. If the total monthly savings of both A and B are Rs.5,000, the monthly income of B is- a. Rs.7,000 b. Rs.14,000 c. Rs.5,000 d. Rs.10,000 Ans : Let the monthly income of B be Rs. x. ? Monthly income of A = Rs. 2x and Saving of A =5000 * 4/(4 + 1) = Rs. 4000 Saving of B = Rs.1000 ? 2x – 4000/x-1000 = 5/3 ? 6x – 12000 = 5x – 5000 ? x = Rs.7000 15. The ratio of the sum of two numbers and their difference is 5:1. The ratio of the greater number to the smaller number is- a. 2 : 3 b. 3 : 2 c. 5 : 1 d. 1 : 5 Ans : Let the numbers be x and y. ? x + y/ x – y = 5/1 ? 5x – 5y = x + y ? 4x = 6y ? x/y = 6/4 x : y = 3 : 2 16. A cistern has 3 pipes A, B and C. A and B can fill it in 3 and 4 hours respectively, and C can empty it in 1 hour. If the pipes are opened at 3 p.m., 4 p.m. and 5 p.m. respectively on the same day, the cistern will be empty at- a. 7.12 p.m. b. 7.15 p.m. c. 7.10 p.m. d. 7.18 p.m. Ans : Let the cistern be emptied at x p.m. ? x -3/3 + x – 4/4 = x - 5/1 &r FCI written test FCI written test In Kerala KochiFCI written test FCI written test In Kerala Kochi Part -d English Language Directions : In Question Nos .151 to 155,some parts of the sentences have errors and some have none.  find out which part of a sentence has an error and blacken the oval [] corresponding to the appropriate letter (A,B,C).  If a sentence is free from error, blacken the oval corresponding to (D) 151    While we love nature in its peaceful and pleasant moments(A) /we find it hardly(B)-Answer/to love its furies and wildness(C)/No error(D) 152     Umbrella is(A)/ of no avail(B) -Answer against a thunderstorm (C)/No error(D) 153    I shall return the book(A)/when you will arrive(B)/here(C)-Answer/No error(D) 154    The old man saw (A)/that the bird (B)/is circling again (C)/No error(D)-Answer 155    Our manager doesn,t mind(A)/to pay   handsome salary (B)-Answer/as long as the employees are competent enough to (C)/meet the deadlines/No error (D) Directions : In Question Nos 156 to 160 sentences are given with blanks to be filled in with an appropriate word(s). four alternatives are suggested for each question.  Choose the correct alternatives  are suggested for each question.  Choose the correct alternative out of the four and indicate it by  blackening the appropriate oval [] in the Answer Sheet 156     what are you going to do school today? a.from b.until c.after -Answer d.before 157    That small dictionary is all right,but a big one would be ------- a.important b.easy-Answer c.expensive d.useful 158    ----large city Mumbai has become in last twenty years a.What b.what a -Answer c.How d.How a 159    All his statements duly tesify -----his truthfulness and honesty a.about-Answer b.of c.to d.in 160    The accused ----having committed the theft a. disagreed b.declined c.denied-Answer d.refused Directions In Question Nos 161 to 165 out of the four alternatives, choose the one which best express the meaning of the given word 161 flimsy a.filmy b.weak-Answer c.firm d.fly 162    mingle a.blend-Answer b.jingle c.join d.diminish 163commence a commit b.start-Answer c.convince d.programmes 164    endeavours a.plans b.activities-Answer c efforts .d programmes 165    motive a.design b.reason-Answer c.impulse d.urge Directions : In question Nos 166 to 170 choose the word opposite in meaning to the given word 166    refinement a.rudeness-Answer b.coarseness-Answer c.anger d.foolishness 167    belated a.premature-Answer b outdated c.delayed d deferred 168    reverence a.contempt b.astonishment c.firmness d.displeasure-Answerr 169    quieten a.to soothe b.to settle c.to rouse d.to lull 170    impulsive a.cautious-answer bhasty c.reckless .spontaneous Directions : In question Nos 171 to 175 four alternatives are given for the Idiom/phrase underlined in the sentence choose the alternative which best express the meaning of the Idiom/Phrase 171    I have a feeling that she is taking you for a ride a.taking you in the car b.trying to push you c.pulling you along d.trying to trick you 172    It is a far cry from Delhi to Athnes a.a long way off b.an emotional journey c.a broing journey d.not too long way 173    Their opinions in the meeting fell flat a.did not inspire others b.did not produce the desired effect-Answer c.were not  goal oriented d.left everyone awestruck 174    At the end of the argumentation ,he got the better of me a. he thought I was good b.he got defeated-answer c.I Understood him better d.he overcame me 175    Please look through this chapter before the examinations a.turn the pages of b.study c.omit d.get an explanation of Directions : In Question Nos. 176 to 180 a part of the sentence is underlined Below are given alternatives to the underlined part at (A),(B) and (C) which may improve the sentence  Choose the correct alterative In case no improvement is needed your answer is (D) 176    He was rich by sheer accident  of birth a.chance b.coincidence c.incidence-Answer d.No improvement 177     John recollects me of a boy I used to know a recalls b. reminds  -Answer c. rememebers d.No improvement 178     The streeet is infested by rats a.to b. from-Answer c.with d.No improvement 179    Rememebr that examinations never start late,they always start in time a.on time-Answer b.by the clock c.on the nick of time d.No Improvement 180    If I were you, I will buy this book a.might b.shall c.would-Answer d.No improvement Directions: In Questions Nos 181 to 185 out of the four alternatives choose the one which can be substituted for the given words/sentence 181    Enter a coutry as an enemy a. defeat b. spy c. invade-Answer d. overcome 182    Plants of a region a. fauna b. flora -Answer c. nursery d.forest 183    Expressions no longer in current use a. artistic b. archaic  -Answer c. ancient d. modern 184    A talk between two person a prologue b. monologue c. dialogue d. speech 185     A person who cannot be corrected a.illegible b.impossible c.incorrigble -Answer invulnerable Directions : In question Nos 186 to 190 there are four words out of which one is correctly spelt find the correctly spelt word and indicate it by blackening the appropriate oval [] 186    a.allaince b.alliance-answer c.allianse d.allianns 187     labirinthine\ b.labyrinthine   answer c.labirrinthine d.labyrynthine 188     a. substansial b.substancial c.substantialanswer d.substancal 189    exagerate b.exaggerateanswer c.exhaggerate d.exaggirate 190     a. accomodate b.acomodate c.accommodate answer d.accommodete Directions : In Question Nos 191-200 you have a passage with 10 questions following the passage read the passage carefully and choose the best answer to each question out of the four alternatives Animals do not know worry.  what bird could raise a family if it worried about the problems to be overcome, the impossible number of feeding trips in day to keep those clamouring mouths stilled with food? that is not the way birds or animals respond to life.  Nature says Feed them! and the mother bird goes ahead and does it,  Between dawn and sunset a tiny wren must make hundreds of such round trips to feed her brood An animal doesn't know what brotherhood means but when it hears the call ""Help It answers instinctively  If a prairie doe is shot, the other in the prairies dog village come tumbling out, not worried about gunfire and pull their wounded have seen elephants, disregarding danger, lift a wounded elephant to his feet with their tusks and by supporting him with one member of the herd on each side, help hi, walk to the forest depths 191    A bird makes innumerable trips to collect food to a.feed its mate b.feed itself and its mate-answer c.feed its young in the nest d.store food for the winter 192    the mother bird feeds the brood a.when it sees them hungry b.when her instinct tells her to do so  -answer c.when they cry d.after it feeds itself and its mate 193when an animal hears the call for help a.it doesnt bother b.it rushes to the spot c.it gets scared and runs away d.it responds instinctively-answer 194    If a prairie dog is shot,the others a.go undergroudn b.chase away their enemy c.pull the wounded dog to safety-answer d.start barking together 195    elephants lift a wounded fellow elephant to his feet a.with their tusks b.with their trunks c.with their feet d.by pusing on his side 196    which of the following statement is true in the context of the passage? a. animals worry about raising a family b.Animals often behave sensibly. c.animals do not know what brotherhood means. d. animals make several feeding trips in a day to collect food for their young ones 197     As used in the passage,the word 'stilled'means a.calmed b.supplied c.provided d.filled 198    The authors tone in the passage can best be described as a.sentimental b. biased c.critical d.informative--answer 199. The author is primarily concerned with a. expressing the beauty of birds and animals b. discussing how birds and animals evolved c. explaining that birds and animals are free from worry d. explaining  the birds and animals respond to life instinctively-answer 200    The expression tumbling out'in one of the sentences in the passage means a.reacting aggressively b. flowing out hurriedly and confusedly c.moving out steadily d.attacking the enemy colectively-answer FCI FCI 11 th Nov 2012 general awareness solved question papers, FCI Part a- general intelligence questions with answekeys,FCI 2012 Nov 11th English language free solved question papers, FCI VG 2012 Part - B questions with detailed explanations and answers General Awareness 51. New Economic Policy was introduced in India in (A) 1971 (B) 1981 (C) 1991-Answer (D) 2001 52. Which of the following is NOT an instrument of credit control used by the Central Bank? (A) Bank rate (B) Open market operations (C) Selective credit controls (D) Foreign exchange controls-Answer 53. In which type of economy National Income and Domestic Income is equal? (A) Closed Economy (B) Open Economy (C) Developed Economy-Answer (D) Developing Economy 54. The term Capital Consumption Allowance (CCA) means (A) A part of income used for consumption. (B) Increase in the value of capital assets. (C) Depreciation value of capital goods. (D) A fund used to purchase machinery for the production of consumption goods.-Answer 55. In the following, who is a self employed person? (A) Teacher teaching in a school (B) Doctor appointed in hospital (C) Railway employee-Answer (D) Businessman-Answer 56. Who rules the State in the event of declaration of emergency under Article 356? (A) Prime Minister (B) Chief Minister (C) Governor (D) Chief Justice of the High Court 57. National Commission for Backward Classes was set up in (A) 1991 (B) 1992 (C) 1993-Answer (D) 1994 58. Official language of the Indian Union is (A) Hindi in Devanagari script-Answer (B) Hindi and English (C) English (D) Hindi, English and Urdu 59. Panchayatraj form of rural government was first adopted by (A) Rajasthan & Madhya Pradesh-Answer (B) Rajasthan & West Bengal (C) Rajasthan & Andhra Pradesh (D) Rajasthan & Uttar Pradesh 60. The Constitution of India provides for citizenship by (A) Birth (B) Residence (C) Naturalization-Answer (D) All of the above 61. Who assumed the title Muhammad-bin-Tuglak and became the ruler of Delhi in 1325 A.D ? (A) Juna Khan (B) Nasiruddin Shah (C) Chengiz Khan (D) Iltutmish 62. The Sikh Guru put to death by Aurangzeb was (A) Arjun Singh (B) Har Gobind (C) Tegh Bahadur-Answer (D) Har Kishan 63. The Home Rule Movement in India was started by (A) S.N. Banerjee and W.C. Banerjee (B) Annie Besant and Tilak-Answer (C) Mahatma Gandhi and Motilal Nehru (D) Annie Besant and Gokhale 64. Who among the following was responsible for the revival of Hinduism in 19th Century? (A) Swami Dayanand (B) Swami Vivekanand (C) Guru Shankaracharya (D) Raja Ram Mohan Rai 65. "Give me blood, I will give you freedom". These words are attributed to (A) Khudiram Bose (B) Bhagat Singh (C) Subhash Chandra Bose-Answer (D) Veer Savarkar 66. India's northern most latitude is (A) 36 0 6' N (B) 37 0 8' S (C) 37 0 6' N (D) 8 0 4' N 67. The Bhabar plains have been developed by (A) glacial deposit (B) wind deposit (C) river deposit (D) deposition in the lakes 68. Subsistence type of farming is practised in (A) Delta (B) Plateau-Answer (C) Hill (D) Coast 69. Africa is separated from Europe by _______. (A) Red Sea (B) Mediterranean Sea-Answer (C) Atlantic Ocean (D) Persian Gulf 70. Which one of the following rivers is recharged by subsoil water? (A) Godavari (B) Damodar (C) Narmada (D) Krishna 71. The dolphin is (A) Fish (B) Reptile (C) Mammal-Answer (D) Turtle 72. Animals which have a well marked digestive cavity are put under (A) Metazoa (B) Bryozoa (C) Parazoa (D) Enterozoa 73. Mushroom are (A) Fungi-Answer (B) Viruses (C) Bacteria (D) Protozoans 74. Which one of the following is an endocrine as well as an exocrine gland ? (A) Pituitary (B) Thyroid (C) Pancreas (D) Parathyroid 75. Which one of the following is associated with kidney disorder ? (A) Ventilator (B) Dialysis-Answer (C) Pacemaker (D) Baropacing 76. Most important function of the food is to (A) get energy-Answer (B) satisfy hunger (C) help growth of the body (D) relish the taste 77. A racing car is designed to have a broad base and low height such that its centre of gravity is (A) very low (B) in the middle (C) raised (D) outside the car 78. Energy transformation in a microphone is from (A) sound to electrical-Answer (B) electrical to sound (C) sound to mechanical (D) mechanical to sound 79. The time-period of a simple pendulum is independent of its (A) length (B) mass-Answer (C) location on the Earth (D) amplitude of vibration 80. What happens when some charge is placed on a soap bubble? (A) Its radius decreases (B) Its radius increases (C) The bubble collapses (D) The bubble expands 81. The last three letters of the domain name describes the type of (A) organization-Answer (B) connectivity (C) server (D) protocol 82. A RAM (A) is a random access memory.-Answer (B) is a volatile memory. (C) is either static or dynamic memory. (D) has all of the above characteristics. 83. Naturally occurring heaviest element is (A) aluminium (B) iron (C) silicon (D) uranium-Answer 84. Which of the following has the highest density? (A) Charcoal (B) Coke (C) Diamond (D) Graphite 85. When a gas expands to a region of low pressure, its temperature (A) increases (B) decreases (C) remains same (D) first increases, then decreases 86. Most of the explosions in mines occur due to the mixing of (A) hydrogen with oxygen (B) oxygen with acetylene (C) methane with air (D) carbon dioxide with ethane 87. Which of the following affects biodiversity ? (A) Environmental pollution (B) Ocean acidification (C) Climate change (D) All of the above 88. What was Montreal Protocol concerned with? (A) Checking global warming (B) Checking ozone depletion (C) Both (A) and (B) (D) (A), (B)  and protecting biodiversity 89. Formation of acid rain is due to (A) Water pollution (B) Noise pollution (C) Land pollution (D) Air pollution 90. Enormous population expansion has led to the problem of (A) Rising levels of atmospheric CO2 (B) Global warming (C) Increases in pollution level (D) All the above 91. Which of the following festivals is celebrated on Prophet Mohammed's birthday? (A) Id-ul-Zuha (B) Id-e-Milad-Answer (C) Id-ul-Fitr (D) Muharram 92. The Nobel Peace Prize is awarded in which city ? (A) Brussels (B) Geneva (C) Oslo-Answer (D) Stockholm 93. Which one of the following pairs of cities have two Headquarters each of Indian Railway Zones? (A) Kolkata and Chennai (B) Kolkata and Mumbai (C) Mumbai and Secunderabad (D) Mumbai and Chennai 94. Which one of the following has recently conferred the Honorary Degree of Doctor of Letters (D.Lit.) on U.N. Secretary General, Ban-Ki-Moon? (A) Jamia Millia Islamia University (B) Lal Bahadur Shastri Sanskrit University (C) G.G.S. Indraprastha University (D) I.I.T., Delhi 95. Who amongst the following directed the award winning film "Tare Zamin Par"? (A) Madhur Bhandarkar (B) Salman Khan (C) Aamir Khan (D) Sanjay Leela Bhansali 96. The hill famous for treating mentally depressed persons in Tamil Nadu  is (A) Nilgiri (B) Kutralam (C) Javadi (D) Anaimalai 97. Who among the following was the first Indian to receive the Nobel Prize? (A) Dr. C.V. Raman (B) V.S. Naipal (C) Mother Teresa (D) Rabindranath Tagore-Answer 98. Which of the following Twin city is correct? (A) Cochin-Ernakulam--Answer (B) Bangalore - Mysore (C) Mumbai - Pune (D) Chennai - Chingleput 99. Who said, "Inflation is unjust but deflation is inexpedient. Of the two, deflation is worse"? (A) J.M. Keynes-Answer (B) Amartya Sen (C) J.G. Gurlay (D) J.N. Bhagwati 100. The civilian award bestowed by the President of India to persons for rendering exceptional and distinguished service in any field is (A) Bharat Ratna (B) Padma Shri (C) Padma Vibhushan (D) Padma Bhushan FCI general awareness questions to practice ,General awareness/current affairs previously asked in different examinations in ssc upsc etc,   1. Which Indian filmstars was recently conferred Honorary Doctorate by Bedfordshire University, (U.K.) ? (A) Amitabh Bachhan (B) Shah Rukh Khan (C) Om Puri (D) Amir Khan Ans : (B)   2. Williams Sisters won the U.S. Open Women’s Doubles Title 2009 after defeating in the final— (A) Leizal Herber and Cara Blank (B) Kim Clijsters and Anna Kournikova (C) Coroline Wozniacki and Dinara Safina (D) Nathalie Deshy and Sania Mirza Ans : (A)   3. Which one of the following is a military alliance ? (A) ASEAN (B) SAARC (C) NATO (D) NAFTA Ans : (C)   4. The recipient of the 42nd Jnan Peeth Award is— (A) Manohar Shastri (B) Harish Pandya (C) Satya Vrat Shastri (D) K. Kamal Kumar Ans : (C)   5. World Development Report is an annual publication of— (A) UNICEF (B) UNDP (C) WTO (D) World Bank Ans : (D)   6. Which one of the following is correct ? Player — Sport (A) Jeev Milkha Singh —Tennis (B) Jhulan Goswami —Cricket (C) Baichung Bhutia —Hockey (D) Pankaj Advani —Badminton Ans : (B)   7. Which one of the following industrialists was declared ‘The  Business Person of the year 2008’ by the Times of India Survey ? (A) Anil Ambani (B) Rahul Bajaj (C) Ratan Tata (D) Gautam Adani Ans : (C)   8. Naina Devi peak forms a part of— (A) Himalayan range located in Sikkim (B) Himalayan range located in Kumaon region (C) Himalayan range located in Nepal (D) Himalayan range located in Jammu & Kashmir Ans : (B)   9. The aim of the ISRO’s OCEANS AT-2 satellite is— (A) To provide ocean scientists knowledge about mineral resources  under the sea (B) To aid fishermen in identifying fishing zones (C) To aid weathermen to forecast cyclones and weather conditions (D) All the above Ans : (D)   10. Who amongst the following is not the recipient of the Rajiv Gandhi Khel Ratna Award ? (A) Vijendra Singh (B) Sushil Kumar (C) M. C. Marykom (D) Abhinav Bindra Ans : (D)   11. Nilgiri, Himgiri and ‘Beas’ are— (A) Aircraft Carrier ships (B) Frigates (C) Nuclear submarines (D) Oil tankers of ONGC Ans : (B)   12. The age of a tree can be calculated by— (A) Number of branches (B) Its height (C) Girth of its trunk (D) Number of its annual rings Ans : (D)   13. Which one of the statements below explains the external  economies of scale ? (A) Starting a computer firm in a Techno Park to avail the expertise (B) Expanding firms employing specialist accountants, lawyers and  managers (C) A manufacturer spreading the research and development cost over the output (D) A major supermarket gaining bulk discounts on direct purchase Ans : (C)   14. Which one of the following types of unemployment is common in Indian agriculture ? (A) Frictional (B) Structural (C) Disguised (D) Seasonal Ans : (D)   15. A rolling plan refers to a plan which— (A) Does not change its targets every year (B) Changes its allocations every year (C) Changes its allocations and targets every year (D) Changes only its targets every year Ans : (C)   16. An instrument of qualitative credit control in India is— (A) Open market operations (B) Credit rationing (C) Change in reserve ratio (D) Bank rate policy Ans : (B)   17. Laissez-faire is a feature of— (A) Socialism (B) Communism (C) Capitalism (D) Mixed Economy Ans : (C)   18. Which one of the following is not a source of the tax revenue in  Indian States ? (A) Land Revenue (B) Motor Vehicle Tax (C) Entertainment Tax (D) Corporate Tax Ans : (D)   19. CSO has changed its base year for National Income estimation.   The new base year is— (A) 1990-91 (B) 1993-94 (C) 1994-95 (D) 1995-96 Ans : (B)   20. Which is the biggest enterprise of the Government of India ? (A) Postal and Telegraph (B) Railways (C) Banking (D) Shipping Ans : (B)   21. In which type of competition does Marginal Revenue Curve  coincide with Average Revenue Curve ? (A) Monopoly (B) Imperfect Competition (C) Perfect Competition (D) Monopolistic Competition Ans : (C)   22. According to J. A. Schumpeter, entrepreneurs are entitled to  enjoy the profit for their……… activities. (A) Innovative (B) Risk taking (C) Risk averting (D) Hard work Ans : (A)   23. Demonstration effect means— (A) Effect of advertisement (B) Imitating effect of consumption (C) Effect of entertainment (D) Effect of an experiment Ans : (B)   24. Homogeneous product is a feature of— (A) Imperfect market (B) Monopoly (C) Oligopoly (D) Perfect market Ans : (D)   25. Which part of the Indian Constitution declares the Ideals of  Welfare State ? (A) Fundamental Rights (B) Fundamental Duties (C) Preamble (D) Directive Principles of State Policy Ans : (D)   26. Who said, “Parliamentary System provides a daily as well as a  periodic assessment of the Government” ? (A) Dr. B. R. Ambedkar (B) Shri B. N. Rao (C) Shri Jawahar Lal Nehru (D) Dr. Rajendra Prasad Ans : (A)   27. The legal advisor to the State Government is known as— (A) Advocate-General (B) Attorney-General (C) Solicitor-General (D) State Public Prosecutor Ans : (A)   28. The maximum duration of Zero Hour in Lok Sabha is— (A) 30 minutes (B) One hour (C) Two hours (D) Unspecified Ans : (B)   29. Which agency acts as co-ordinator between Union Government,Planning Commission and State Governments ? (A) National Integration Council (B) Finance Commission (C) National Development Council (D) Inter-State Council Ans : (C)   30. When will demand become a grant ? (A) When a demand is proposed (B) After the discussion on demand is over (C) After the demand is granted (D) When the Budget Session is closed Ans : (C)   31. What makes the Judiciary the guardian of the Constitution ? (A) Independence (B) Service Conditions (C) Salary (D) Judicial Review Ans : (D)   32. What is the name of Judicial organ of UNO ? (A) Supreme Court (B) Court of International Justice (C) Judicial Forum (D) International Court of Justice Ans : (D)   33. The Election disputes regarding the Election of President and Vice-President can be heard by— (A) Parliament (B) Central Election Commission (C) Supreme Court (D) Attorney-General of India Ans : (C)   34. Balwant Rai Mehta Committee suggested that the structure of Panchayati Raj should consist of— (A) The village, the block and the district levels (B) The mandal and the district levels (C) The village, the district and the State levels (D) The village, the mandal, the district and the State levels Ans : (A)   35. Which of the following has banned ‘floor crossing’ by the members elected on a party ticket to the legislature ? (A) 52nd Constitution Amendment (B) Representation of People Act (C) National Security Act (D) Maintenance of Internal Security Act Ans : (A)   36. The expression ‘Creamy Layer’ used in the judgement of the Supreme Court relating to the case regarding reservations refers to— (A) Those sections of society which pay Income-tax (B) Those sections of socially and educationally backward classes of the society that are economically advanced (C) Those sections of the society that are considered advanced according to Karpuri Thakur formula (D) All sections of upper castes of the society Ans : (B)   37. When was the Panchtantra written ? (A) Maurya Period (B) Kanishka Period (C) Gupta Period (D) Harsha Period Ans : (C)   38. Which one among the following is regarded as the ‘Magna Carta’of Indian education ? (A) Wood’s Despatch (B) Hunter’s Commission (C) Sadler Commission (D) Wardha Scheme Ans : (A)   39. The battle of Mahabharata is believed to have been fought at Kurukshetra for— (A) 14 days (B) 16 days (C) 18 days (D) 20 days Ans : (C)   40. The Mukteswara Temple is located at— (A) Puri (B) Belur (C) Konark (D) Bhubaneswar Ans : (D)   41. Which king of the Gupta Dynasty was called the ‘Napoleon of India’ ? (A) Samudragupta (B) Chandragupta Vikramaditya (C) Sri Gupta (D) Chandragupta-I Ans : (A)   42. Between which two rulers was the First Battle of Panipat fought ? (A) Akbar and Bahlol Lodi (B) Babur and Ibrahim Lodi (C) Bairam Khan and Sikandar Lodi (D) Shah Jahan and Daulat Khan Lodi Ans : (B)   43. Who is the author of Ain-i-Akbari ? (A) Abul Fazal (B) Abdus Samad (C) Bairam Khan (D) Raja Todarmal Ans : (A)   44. Match List-I with List-II and select the correct answer by using the codes given below the lists— List-I — List-II (A) Peshwas 1. Nagpur (B) Gaekwads 2. Pune (C) Bhonsles 3. Indore (D) Holkers 4. Baroda Codes : (A) (B) (C) (D) (A) 2 4 1 3 (B) 1 3 2 4 (C) 2 4 3 1 (D) 4 3 2 1 Ans : (A)   45. Which reformer from Maharashtra was known as Lokhitavadi ? (A) Pandit Ramabai (B) Gopal Krishna Gokhale (C) M. G. Ranade (D) Gopal Hari Deshmukh Ans : (D)   46. ‘Din-e-Ilahi’ of Akbar was not a success because— (A) After Akbar, it was not patronized (B) The Muslims did not accept other religious practices (C) It was not suitably projected to the masses (D) All the above Ans : (D)   47. Who was the author of the book ‘My Experiments with Truth’ ? (A) Aurobindo Ghosh (B) Bal Gangadhar Tilak (C) M. K. Gandhi (D) Vinoba Bhave Ans : (C)   48. Give the correct chronological order of the following events— 1. Formation of Muslim League 2. Formation of All India Untouchability League 3. Formation of All India Trade Union Congress 4. Formation of Indian National Congress (A) 2, 4, 1, 3 (B) 3, 4, 1, 2 (C) 4, 3, 1, 2 (D) 4, 1, 3, 2 Ans : (D)   49. 80% of the coal in India comes from— (A) Jharia and Raniganj (B) Kantapalli and Singareni (C) Singrauli and Korba (D) Neyveli Ans : (A)   50. Dry zone agriculture in India contributes nearly 40% of the total— (A) Commercial crops (B) Fodder crops (C) Food crops (D) Plantation products Ans : (D)     51. Who was the first Indian woman to scale the Mount Everest ? (A) Bachhendri Pal (B) Phew Dorajee (C) Onn Saang Su Kayi (D) Yoko Ono Ans : (A)   52. Name of S. Chandrashekhar is associated with which of the   following subjects ? (A) Cosmology (B) Chemistry (C) Fluid Mechanics (D) Astrophysics Ans : (D)   53. Who was the only Indian woman to be elected as the President of   U.N. General Assembly ? (A) Vijayalakshmi Pandit (B) V. K. Krishna Menon (C) Jawaharlal Nehru (D) Rajeshwar Dayal Ans : (A)   54. ‘RAF’ is the abbreviated form of which of the following ? (A) Ready Action Force (B) Rapid Action Force (C) Reverse Action Force (D) Repeat Action Force Ans : (B)   55. Tehri dam is built on which of the following rivers ? (A) Ganga (B) Brahmputra (C) Bhagirathi (D) Yamuna Ans : (C)   56. Who lost the AFC (Asian Football Confederation Final, 2008 to India) ? (A) Myanmar (B) Sri Lanka (C) Pakistan (D) Tajikistan Ans : (D)   57. After the terrorist attack of 26th November, 2008 in Mumbai, who was appointed as the Home Minister of India ? (A) Sh. Shivraj Patil (B) Sh. Pranab Mukherjee (C) Sh. P. Chidambaram (D) Sh. A.K. Anthony Ans : (C)   58. Name the winner of the Sanjay Chopra National Award for Bravery 2008— (A) Saumika Mishra (B) Santosh Sen (C) Amit Kumar (D) Pappu Ans : (A)   59. The concept of the Directive Principles of State Policy was borrowed by our Constitution from the Constitution of— (A) U.S.A. (B) Canada (C) U.S.S.R. (D) Ireland Ans : (D)   60. The three medals that IndiAns won in the Beijing Olympics were in— (A) Shooting, Boxing and Wrestling (B) Shooting, Boxing and Archery (C) Shooting, Wrestling and Tennis (D) Shooting, Boxing and Hockey Ans : (A)   61 There is no provision in the Constitution for the impeachment of the— (A) Chief Justice of India (B) Chief Justice of a High Court (C) Governor (D) Vice President Ans : (C)   62. The train service—‘Thar Express’ between India and Pakistan, originates in India from— (A) Jaisalmer (B) Jodhpur (C) Bikaner (D) Barmer Ans : (D)   63. Garba is a dance form of— (A) Gujarat (B) Rajasthan (C) Orissa (D) Assam Ans : (A)   64. Who is named as the Flying Sikh of India ? (A) Mohinder Singh (B) Ajit Pal Singh (C) Joginder Singh (D) Milkha Singh Ans : (D)   65. Fundamental Rights are not given to— (A) Bankrupt persons (B) Aliens (C) Persons suffering from incurable disease (D) Political sufferers Ans : (B)   66. Which of the following, according to Mahatma Gandhi, is the strongest force in the world ? (A) Non-violence of the brave (B) Non-violence of the weak (C) Non-violence of the coward (D) Non-violence of the downtrodden Ans : (A)   67. What is the tenure of the Prime Minister of India ? (A) Conterminous with the tenure of the Lok Sabha (B) Conterminous with the tenure of the President (C) As long as he enjoys the support of a majority in the Lok Sabha (D) Five years Ans : (C)   68. Consent of the people meAns consent of— (A) A few people (B) All people (C) Majority of the people (D) Leader of the people Ans : (C)   69. Which of the following is not a Panchayati Raj Institution ? (A) Gram Sabha (B) Gram Panchayat (C) Gram Cooperative Society (D) Nyaya Panchayat Ans : (C)   70. Match the following—Union Territory (A) Puducherry (B) Andaman and Nicobar Islands (C) Lakshadweep (D) Daman and Diu Jurisdiction (High Court) 1. Kerala 2. Bombay 3. Madras 4. Calcutta (A) (B) (C) (D) (A) 3 4 1 2 (B) 1 3 4 2 (C) 1 2 3 4 (D) 1 4 3 2 Ans : (A)   71. Which of the following is true regarding ‘No Confidence Motion’in the Parliament ? 1. There is no mention of it in the Constitution. 2. A period of six months must lapse between the introduction of one ‘No Confidence Motion’ and another. 3. Atleast 100 persons must support such a motion before it is introduced in the House. 4. It can be introduced in the Lok Sabha only. (A) 2 and 4 (B) 1, 2, 3 and 4 (C) 1, 2 and 3 (D) 1 and 4 Ans : (D)   72. The toxicity of which of the following heavy metals leads toliver cirrhosis ? (A) Copper (B) Lead (C) Mercury (D) Zinc Ans : (A)   73. Cell or tissue death within a living body is called as— (A) Neutrophilia (B) Nephrosis (C) Necrosis (D) Neoplasia Ans : (A)   74. Typhoid is caused by— (A) Pseudomonas sp. (B) Staphylococcus (C) Bacillus (D) Salmonella typhi Ans : (D)   75 Besides carbohydrates, a major source of energy in our food is constituted by— (A) Proteins (B) Fats (C) Minerals (D) Vitamins Ans : (B)   76. The process of removing calcium and magnesium from hard water is  known as— (A) Sedimentation (B) Filtration (C) Flocculation (D) Water softening Ans : (D)   77. Leprosy bacillus was discovered by— (A) Koch (B) HAnsen (C) Fleming (D) Harvey Ans : (B)   78. Who, of the following, was awarded ‘Ashok Chakra’ on 26th January, 2009 ? 1. Hemant Karkare 2. M. C. Sharma 3. Gajendra Singh 4. Vijay Salaskar (A) 1 and 2 (B) 1, 2 and 4 (C) 1, 2 and 3 (D) All the above Ans : (D)   79. The British introduced the railways in India in order to— (A) Promote heavy industries in India (B) Facilitate British commerce and administrative control (C) Move foodstuff in case of famine (D) Enable IndiAns to move freely within the country Ans : (B)   80. Which religious reformer of Western India was known as ‘Lokhitwadi’ ? (A) Gopal Hari Deshmukh (B) R. G. Bhandarkar (C) Mahadev Govind Ranade (D) B. G. Tilak Ans : (A)   81. In which of the following states was President’s Rule imposed during the month of January, 2009 ? (A) Uttarakhand (B) Meghalaya (C) Mizoram (D) Jharkhand Ans : (D)   82. In which state was the Nalanda University located in India ? (A) Bengal (B) Bihar (C) Orissa (D) Uttar Pradesh Ans : (B)   83. Which of the following are correctly matched ? Persons—Events 1. Sultan Mahmud—Sack of Somnath 2. Muhammad Ghori—Conquest of Sindh 3. Alauddin Khilji—Revolt in Bengal 4. Muhammad Bin Tughlaq—Changiz Khan’s invasion (A) 1 and 3 (B) 2 only (C) 1 only (D) 2 and 4 Ans : ()   84. Where did Babar die ? (A) Agra (B) Kabul (C) Lahore (D) Delhi Ans : (A)   85. A situation where we have people whose level of income is not sufficient to meet the minimum consumption expenditure is considered as— (A) Absolute Poverty (B) Relative Poverty (C) Urban Poverty (D) Rural Poverty Ans : (A)   86. India is called a mixed economy because of the existence of— 1. Public Sector 2. Private Sector 3. Joint Sector 4. Cooperative Sector (A) 1, 4 (B) 1, 2 (C) 3, 4 (D) 2, 4 Ans : (B)   87. Which is the largest state of India, populationwise, according to 2001 census ? (A) Maharashtra (B) Bihar (C) Uttar Pradesh (D) West Bengal Ans : (C)   88. Revealed Preference Theory was propounded by— (A) Adam Smith (B) Marshall (C) P. A. Samuelson (D) J. S. Mill Ans : (C)   89. An exceptional demand curve is one that moves— (A) Upward to the right (B) Downward to the right (C) Horizontally (D) Vertically Ans : (A)   90. Who has been appointed the Governor of RBI after the retirement  of Shri Y. V. Reddy ? (A) Dr. Indra Rangarajan (B) Dr. Dilip Sanghvi (C) Dr. Vijay L. Kelkar (D) Shri D. Subbarao Ans : (D)   91. In Economics the terms ‘Utility’ and ‘Usefulness’ have— (A) Same meaning (B) Different meaning (C) Opposite meaning (D) None of the above Ans : (B)   92. Among the following the celestial body farthest from the Earthis— (A) Saturn (B) Uranus (C) Neptune (D) Pluto Ans : (D)   93. The instrument used to see the distant objects on the Earth is— (A) Terrestrial telescope (B) Astronomical telescope (C) Compound microscope (D) Simple microscope Ans : (A)   94. The fuse in our domestic electric circuit melts when there is a high rise in— (A) Inductance (B) Current (C) Resistance (D) Capacitance Ans : (B)   95. It is difficult to cook rice— (A) At the top of a mountain (B) At the sea level (C) Under a mine (D) Same anywhere Ans : (A)   96. X-rays were discovered by— (A) Becquerel (B) Roentgen (C) Marie Curie (D) Van Lue Ans : (B)   97. The National Chemical Laboratory (India) is located in— (A) Mumbai (B) Bengaluru (C) Hyderabad (D) Pune Ans : (D)   98. The chemicals released by one species of animals in order to attract the other members of the same species are— (A) Hormones (B) Nucleic acids (C) Pheromones (D) Steroids Ans : (C)   99. Match the source in Column B with the product of Column A.Column A (Product) (A) Formic acid (B) Citric acid (C) Tartaric acid Column B (Source) 1. Lemon 2. Tamarind 3. Ants (A) (B) (C)   (A) 3 2 1 (B) 3 1 2 (C) 2 3 1 (D) 2 1 3 Ans : (B)   100. An emulsifier is an agent which— (A) Stabilises an emulsion (B) Aids the flocculation of an emulsion (C) Accelerates the dispersion (D) Homogenises an emulsion Ans : (A)   101“ Sensex” is related to BSE-Anwer NSE RBI SEBI   102 It is an index comprising of share price of companies. Who among following was the longest serving Arab Leader ? King Abdullah Muammar Gaddafi-AnwerHe ruled Libya for 42 Years. Hosni Mubarak Bashar-al-Assad     103 MS Dhoni is the _______ captain of Indian Cricket Team 41th 35th-Anwer 31th 34th   104 Wal-Mart in India is associated with Big Bazaar Spencer's Retail Easy Day Stores-Anwer Spur Retail Bharti Enrerprises & Wal-Mart created a joint venture to run retail business in India. Wal-Mart provides only backend operation support to the business while Bharti looks after fron-end & Management operations   105 Name of the Nokia’s mobile phone operating system Android Windows Symbian-Anwer JAVA Symbian is Nokia's own developed mobile phone OS   106 Which was the India’s first scheduled airline ? Air India Tata Airlines-Anwer Air Birla Indian Airlines   107 JRD TATA formed Tata Airlines in the year 1932, was initially used to carry mails from Karachi to Bombay. In the year 1946 , it was renamed as "Air India".Later, in the year 1948 , the airlines was forcefully acquired    by Government of India as flag carrier.Which political party does Sri Ajit Singh represents ? Rashtriya Lok Dal-Anwer Samta Party Samajwadi Party Jan Lok Dal   108 “Mahatma Gandhi National Rural Employment Guarantee Act” provides employment guarantee for how many days per year 120 100-Anwer 180 90 The Mahatma Gandhi National Rural Employment Guarantee Act (MGNREGA) is an Indian job guarantee scheme, enacted by legislation on August 25, 2005. The scheme provides a legal guarantee for one hundred days of    employment in every financial year to adult members of any rural household   109 What is the current FDI upper cap in telecom in India ? 51%-Anwer 74% 49% No Cap   110 Who among following ordered “Jallian Wala Bagh Massacre” ? Lord Mountbatten John Simon Reginald Dyer-Anwer N.D.K MacEven FCI general intelligence and aptitude and reasoning questions and answers, FCI questions for practice FCI free solved sample placement papers 1. What approximate value should come in place of the question mark (?) in the following equation? 158.25 x 4.6 + 21% of 847+?= 950.93 50 45 * 35 * 40 * 25 * 53.25 * Answer: 45 2. The average of the first and the second of three numbers is 15 more than the average of the second and the third of these numbers. What is the difference between the first and the third of these three numbers? * 15 * 45 * 60 * Data inadequate * Answer: None of these Difference is 30 – The difference between the simple and the compound interest compounded every six rnonths at the rate of 10 %.p.a. at the end of two years is Rs. 124.05. What is the sum? * Rs 10,000 * Rs 6,000 * Rs 12,000 * Answer: Rs 8,000 * None of these 4. What approximate value should come in place of the question mark (?) in the following equation? 85.147 + 34.912 x 6.2 + ? = 802.293 * 400 * 450 * 550 * 600 * Answer: 500 5. What should come in place of the question mark (?) in the following equation? 9548+7314=8362+? * 8230 * Answer: 8500 * 8410 * 8600 * None of these 6. What approximate value should come in place of the question mark (?) in the following equation? 248.251 ÷ 12.62 x 20.52 =?* Answer: 400 * 450 * 600 * 375 * 350 7. When a number is added to another number the total becomes 333.333 per cent of the second number. What is the ratio between the first and the second number? * 3:7 * 7:4 * Answer: 7:3 * Data inadequate * None of these 8. There are 11 members in a family out of which there are 4 males and remaining females. The family has hired three cars for a trip to zoo. The members are to be seated in the cars in such a way that there are not more than four members in one car and there is at least one male in each car. How many different ways can the members travel? * 610 * 126 * 140 * 532 * Answer: None of these 9. 657 Ways 9:- The sum of four numbers is 64.If you add 3 to the first number, 3 is subtracted from second number, the third is multiplied by 3 and the sum is divided by 3, then all the results are equal. What is the difference between the largest and the smallest of the original numbers? * Answer: 32 * 27 * 21 * Cannot be determined * None of these 10. The Numbers are 9,15,4,36 10. In a recent Survey 40% homes contained two or more People. Of those houses containing only one person 25% were having only a male. What is the percentage of all houses which contain exactly one female and no males? * 75 * 40 * 15 * Cannot be determined * Answer: None of these 11. 45 percent 11. Sumitra has an average of 56% on her first 7 examinations. How much should she make on her eighth examination to obtain an average of 60% on 8 examinations? * 88% * 78% * 98% * Answer: Cannot be determined * None of these 12. Since Total marks are not given So Can not be Determined 12. A classroom has equal number of boys and girls. Eight girls left to play Kho-kho, leaving twice as many boys as girls in the classroom. What was the total number of girls boys present initially? - * Cannot be Determined * 16 * 24 * Answer: 32 * None of these 13. 16 boys 16 Girls 13. An amount of money is to be distributed among P, Q and R in the ratio of 6:19:7 respectively. If R gives Rs. 200/- of his share to Q, the ratio among P, Q and R becomes 3:10:3 respectively .What was the total amount? * Answer: Rs. 6,400/- * Rs 12800/- * Rs 3,200/- * Data inadequate * None of these 14. A man in his will distributed his money in such a way that half of it is for his wife, two-thirds of the remaining equally to three of his sons and the remaining amount equally to four of his daughters. If each of the daughters receives Rs. 20,000/-, how much money will each of his sons receive? * Rs. 50,333.33 * Rs 48333.33 * Answer: Rs 53,333.33 * Data inadequate * None of these 15. Arun thinks his weight is more than 65 kg but less than 72 . His brother does not agree with him and thinks Arun’s Weight is more than 60 kg but less than 70 kg. Arun ‘s Mother thinks Arun’s weight cannot be more than 68kg. If all of them are correct in their estimation, what is the average of the probable values of Arun’s weight? * 69 kg * Answer: 67 kg * 68kg * Data inadequate * None of these 16. The length and the breadth of the floor of a room is 20 ft. and 10 ft respectively. Square tiles of 2 ft. dimension having three different colours are placed on the floor. The first row of tiles on all sides is of black colour, out of the remaining one-third is of white colour and the remaining are of blue colour. How many blue Coloured tiles are there? * Answer: 16 * 32 * 48 * 24 * None of these 17. If the arithmetic mean of 6, 8, 10. x, 7 is 8.the value of x will be * 7 * Answer: 9 * 10 * 8 * none of these 18. A train is moving with an uniform speed. It crosses a railway platform 120 metres long in 12 seconds and another platform 170 meires long in 16 seconds. The speed of the train per second is— * Answer: 12.5 m/sec * 10 m/sec * 10.22 m/sec * 14 m/sec * None of these 19.The median of the data 30. 25, 27, 25-8, 29, 35,38. 28 is * Answer: 28.5 * 29.5 * 28 * 29 * None of these 20. If the sum of the two radii of two circles is 7 cm and the difference of their circumference is 8 cm, the two circumferences will be— * 34cm and 26cm * 28cm and 20cm * Answer: 26 cm and 18 cm * 20cm and 12cm * None of these (1-5)What should come in Place of ( ? ) in the following Questions ? 1) 58% of 842 + ? = 1200 1) 874.54 2) 711.64 3) 674.74 4) 543.84 5) None of these Ans 2 2) 52% of ? = 3387.28 1) 6980 2) 6342 3) 6718 4) 6539 5) None of these Ans 5 3) Sq.root of 12321 1) 111 2) 121 3) 81 4) 91 5) None of these Ans 1 4) 72% of 654 - 41% of 312 = ? 1) 276.60 2) 296.72 3) 336.84 4) 342.96 5) None of these Ans 4 5) 48 % of 840 = 36% of ? 1) 1080 2) 1320 3) 1240 4) 720 5) None of these Ans 5 6) If the Product of two successive positive integers is 6162, which is the smallest integer ? 1) 78 2) 72 3) 76 4) 74 5) None of these Ans 1 7) The Simple interest accured on an amount of Rs 10,530/- at the end of 5 years is 6318/- what is the rate of interest of p.c.p.a ? 1) 8 2) 14 3) 10 4) 12 5) None of these Ans 4 8) A plot of 640 sq.ft is available at the rate of Rs 4600/- per sq.ft. If 30% of the total cost of the plot is to be paid for booking the plot, how much is the booking amount ? 1) Rs 938100 /- 2) Rs 883200 /- 3) Rs 983200 /- 4) Rs 838100 /- 5) None of these Ans 2 9) A single person take 6 minutes to fill a bottle from 11 am to 12:30 pm. 1845 bottles are to be filled. How many persons should be employed on this Job ? 1) 121 2) 127 3) 135 4) 139 5) None of these Ans 5 10) The Cost of 12 Notebook and 16 pens is Rs 852/-. What is the Cost of 9 Note Books and 12 Pens ? 1) Rs 743 2) Rs 639 3) Rs 567 4) Cannot be determined 5) None of these Ans 4 11) If (93) Square is to be added to the Square of a number the answer so obtained is 16441.What is the Number ?1) 67 2) 73 3) 89 4) 91 5) None of these Ans 5 12) The Average Age of a Man and his Son is 18 Years.The ratio of their Ages is 5:1 resp. What will be the ratio of their ages after 6 years? 1) 10:3 2) 5:2 3) 4:3 4) 3:1 5) None of these Ans 4 13) 42 Percent of first no is the 56 Percent of the second no.What is the Respective ratio of the first no to the Second no ? 1) 4:5 2) 31:42 3) 4:3 4) Cannot be determined 5) None of these Ans 3 14) one-fifth of a number is 48.What is the 62% of that no? 1) 164.66 2) 148.8 3) 178.22 4) 182.44 5) None of these Ans 2 15) If an Amount of Rs 85,470 /- is distributed equally amongst 35 Children. How much amount would each child get ? 1) Rs 2552 /- 2) Rs 2452 /-3) Rs 2542 /- 4) Rs 2442 /- 5) None of these Ans 4 1) 92.5% of 550 = ? a) 506.45 b) 521.65 c) 518.55 d) 508.75 e) None of these Ans d 2) 12 power 4 * 12 power 3 = ? a) 12 power 7 b) 12 power 39 c) 12 power 17 d) 12 power -7 e) None of these Ans c 3)12.22 + 22.21 + 221.12 = ? a) 250.55 b) 255.50 c) 250.05 d) 255.05 e) None of these Ans e 4) 464 / (16 * 2.32 ) = ? a) 12.5 b) 14.5 c) 10.5 d) 8.5 e) None of these Ans a 5) 78 / 5 / 0.5 = ? a) 15.6 b) 31.2 c) 7.8 d) 20.4 e) None of these Ans b 6) A bus Covers a distance of 2,924 Km in 43 Hours.What is the Speed of Bus ? a) 72 Kms./Hr. b) 60 Kms./Hr. c) 68 Kms./Hr. d) cannot be Determined e) None of these Ans c 7) If 9 cube 3 is subtracted from the square of the a number , the answer so obtained is 567. What is the number ? a) 36 b) 28 c) 42 d) 48 e) None of these Ans a 8) What should be the Simple Interest obtained on an amount of Rs 5760 at the rate of 6% per annum after 3 Years ? a) Rs 1036.80 b) Rs 1666.80 c) Rs 1336.80 d) Rs 1063.80 e) None of these Ans a 9) What is 333 times 131 ? a) 46323 b) 43623 c) 43290 d) 42957 e) None of these Ans b 10) The Product of two successive no is 8556.what is the Smallest no ? a) 89 b) 94 c) 90 d) 92 e) None of these Ans d 11) The Owner of an electronics shop charges his customer 22% more than the cost price.If a Customer paid Rs 10980 for a DVD Player then what was the cost price of DVD Player ? a) Rs 8000 b) Rs 8800 c) Rs 9500 d) Rs 9200 e) None of these Ans e 12) What would be the Compound Interest obtained on an amount of Rs 3000 at the rate of p.c.p.a after 2 years ?a) Rs 501.50 b) Rs 499.20c) Rs 495 d) Rs 510 e) None of theseAns b 13) What is the least no to be added to 4321 to make ti a perfect square ? a) 32 b) 34 c) 36 d) 38 e) None of these Ans e 14) 45% of a number is 255.6.What is the 25 % of that number ? a) 162 b) 132 c) 152 d) 142 e) None of these Ans d 15) Find the Average of the following set of Scores: 221, 231, 441, 359, , 665, 525a) 399 b) 428 c) 407 d) 415 e) None of these Ans c 16) In an Examination it is required to get 40% marks of the aggregate marks to pass.A Student gets 261 marks and it is declared failed by 4% marks.What are the maximum aggregate marks a student can get ? a) 700 b) 730 c) 745 d) 765 e) None of these Ans e 17) Pinku, Rinku , Tinku divide an amount of Rs 4200 amongst themselves in the ratio of 7:8:6 resp. If an amount os Rs 200 is added to each of their shares, What will be the new respective ratio of their share of amount ?a) 8:9:6 b) 7:9:5 c) 7:8:6 d) 8:9:7 Ans d 18) The Average of 5 Consecutive even no A,B,C,D,E is 52. What is the Product of B & E ? a) 2912 b) 2688 c) 3024 d) 2800 e) None of these Ans d 19) The Difference between 42 % of a Number and 28% of Same Number is 210. What is the 59% of that Number ? a) 630 b) 885 c) 420 d) 900 e) None of theseAns b 20) A Canteen require 112 kgs of Wheat for a week. How many kgs of Wheat will it require for 69 days ? a) 1204 Kgs b) 1401 Kgs c) 1114 Kgs d) 1014 Kgs e) None of these Ans c 21) If an amount of Rs 41910 is distributed equally amongst 22 Persons. How much amount would each person get ? a) Rs 1905 b) Rs 2000 c) Rs 1885 d) Rs 2105 e) None of these Ans a 22) The Cost of 4 Cell-phone and 7 Digital cameras is Rs 125627 /- .What is the Cost of 8 Cell-Phones and 14 digital cameras ?a) Rs 2,51,254 /- b) Rs 2,52,627 /- c) Rs 2,25,524 /- d) Cannot be determinede) None of these Ans d 23) What is the total Cost of Girl enrolled in painting from all the Institutions together ? a) 1150 b) 1200 c) 1275 d) 1100 e) None of these Ans c 24) What is the respective ratio of total no of girls enrolled in painting, Stitching, Dancing from all the Institution together? a) 44:48:47 b) 43:47:48c) 44:47:48 d) 47:48:44 e) None of these Ans b 25) What is the respective ratio of total no of girls enrolled in painting in the Institutes A & C together to those Enrolled in Stitching in the in the Institutes D & E together?a) 5 : 4 b) 5 : 7 c) 16 : 23 d) 9 : 8 e) None of these Ans 1) Defence Ministry on March 5,2012 blacklisted six vendors for their alleged involvement in the ordnance Factory Board scam in 2009 including Singapore technologies Kinetics Ltd., Israeli Military Industries Rheinmetall Air Defence Zurich and 1) Corporation Defence,Russia 2) TS Kisan,Delhi 3) RK Machine Tool,Punjab 4) All of the above 5) None of these 2) Which of the following statements are correct about the results of Assembly elections announced on March 6,2012 ? 1) Samajwadi party of Mulayam Singh Yadav emerged the single largest party in U.P 2) The shiromani Akali Dal-Led Govt. in Punjab made history ,becoming the first since state was created in 1996 to win two consecutive elections. 3) In the Goa,the BJP won 21 seats on 40member Assembly 4) Allof the above 5) None of these 3) India and china on march 6,2012 put into operation a new mechanism to diffuse any tensions that may arise along the disputed----- Km border. 1) 3,488 2) 3,256 3) 3,466 4) 3,268 5) None of these 4) The Bhoomi Pujan Angkor Wat Ram Mandir in Hajipur was held on March 5,2012.The Proposed temple is a replica of Cambodia's 12th century Angkor Wat-the largest Hindu temple complex in the World built by Cambodian King 1) Suryavarman I 2) Suryarman II 3) Narsimhavarman I 4) Narsimhavarman II 5) None of these 5) The finance Minister of Haryana Harmohinder Singh Chatha Presented the Budget 2012-13 on March 5,2012.What is the budget estimate for a revenue deficit during 2012-13 1) Rs 2,455.55 Crore 2) Rs 2,845.64 Crore 3) Rs 2,372.76 Crore 4) Rs 2,561.58 Crore 5) None of these 6) The Prime minister of India Dr. Manmohan Singh inaugurated the 50th anniversary celebrations of the Afro-Asian Rural Development organization on March 5,2012 in New Delhi.How many African Countries are its members? 1) 14 2) 15 3) 16 4) 17 5) None of these 7) Russian Prime minister Vladimir Putin won a third term as president securing 63.6 % votes in the election held on March 3,2012.who among the following came second securing 17.9 % percent votes? 1) Gennady Zyuganow 2) Sergel Mironov 3) Mikhail Prokhorov 4) Vladimir Zhrinovsky 5) None of these 8) The Directorate General of Foreign issued a notification on March 5,2012 prohibiting export of cotton with immediate effect.What is main reason for this ban? 1) Low quality of cotton 2) Low production and high consumption 4) Lower price in foreign markets 5) All of the above 5) None of these 9) Four private and Public Financial institutions -ICICI Bank,LIC,Citi Financial and Bank of Baroda -on march 5,2012 joined hands to set up India's first $ 2 billion infrastructure Debt. Fund(IDF).Which institution has the largest stake in the joint venture?1) Life Insurance Company 2) Bank of Baroda 3) Citi Financial 4) ICICI Bank 5) None of these 10) Tata Power in March 2012 joined hands with south Africa based Exxaro to form a joint Venture.What is the generation capacity of TATA Power from all sources of energy? 1) 4678 MW 2) 4798 MW 3) 4772 MW 4) 4776 MW 5) None of these 11) Which of the following automaker companies on March 6,2012 launched small car B-MAX at the Geneva Motor Show ? 1) Volkswagen 2) Hyundai 3) Tata Motors 4) Ford 5) None of these 12) Tha National Association of street Vendors of India(NASVI) and Nidan were honored with the Skoll Award on March 29,2012.In which year this award was instituted? 1) 1997 2) 1998 3) 1999 4) 2000 5) None of these 13) Who among the following became the first person with more than 20 million followers on Twitter in march 2012? 1) Katy Perry 2) Lady Gaga 3) Barack Obama 4) Shakira 5) None of these 14) Who among the following were honored at the 67th National Grassroots Innovations Award on March 9,2012 in New Delhi? 1) Archana Konwar 2) Shalin Kumar 3) Abhishek Bhagat 4) All of the above 5) None of these 15) Who among the following countries won the first women Kabaddi world cup in Patna March 4,2012? 1) Japan 2) India 3) Iran 4) Thailand 5) None of these 16) Who among the following won the ATP Tour Delray Beach International Tennis title? 1) Jarkko Niemi 2) Kevin Peter 3) Stephen kiem 4) Kevin Anderson 5) None of these 17) The AIIMS, Delhi celebrated International childhood cancer day on March 2,2012 on the theme 1) Love Cures Cancer 2) Fight Cancer 3) Beat Cancer with Love 4) Healthy Life, No Cancer 5) None of these 18) Who of the following may be considered as the objectives of the Union Budget 2012-13 presented in the Parliament on March 16,20121) Focus on domestic demand driven growth recovery. 2) Intervene decisively to address the problem of malnutrition 3) Address and improve supply side of economy4) creating conditions for rapid revival of high growth in Private investment 5) All of the above Model Question Paper for General English Direction(1-5):Read the following passage Carefully and answer the questions given below it.Certain words are printed in bold to help you to locate them while answering some of the questions. The window offering anew view of the house opposite.The two families dinot not speak to eachother because of the properity dispute.oneday Ruchira's textbook lay untouched as the young girl's gazer on the happenings in the house opposite.There were two new faces in the neighbouring house-hold of an elderly window and a girl,aged sixteen.sometimes the elderly lady would sit by the window doing young girl's hair.on other days she was absent. The New young neighbours daily routine could be seen through the window-she cleared the paddy:split nuts, put the cushions in the sun to air them.In the afternoons while the men were all at way some of the wome slept and other played cards.The girl sat on the terrace and read.Sometime they wrote.One day there was a hinderance.She was writing when the elderly woman snatched the unfinished letter from her hands.There after the girl was not to be seen on the terrace.Sometime during the Sounds came from the house indicating that a massive argument was going on inside. A few days passed.One evening Ruchira noticed the girl standing on the terrace in tears.evening prayer was in progress.As she did daily, the girl bowed several time in prayer.Then she went down stairs that night Ruchira wrote a letter.She went down and posted it that very instant.But as she laid on the bed that night.She prayed fervently. that her offer of friendship wouldn't reach its destination.Ruchira then left for Madhupur and returned when it was time for college to start.She found the house opposite in darkness,locked.They had left. When she stepped into her Room she found the desk piled with letters one had a local stamp it with her name and address in unfamiliar handwriting.She quickly read it.they Continued to Write each other for the next twenty years. 1) Why did Ruchira Write a letter to her new neighbour ? 1) She wanted to offer her,her help 2) She wanted to be friends with her. 3) To apologize for her family's behavior towards her family. 4) To encourage her to Continue learning to read and write 5) None of these 2) Which of the following can be said about Ruchira ? a) She used to spy on her neighbors because she didnot trust them. b) She was at home because she was studying. c) She didn't Speak to her neighbors because they didn't own property. 1) None 2) Only B 3) Both A and B 4) Only C 5) Both A and C 3) How did the new young neighbour Spend her days? 1) She was busy Writing letters to Ruchira. 2) She used to day dream about her Past experience. 3) She would attend to the need of widow. 4) She spent her time learning to read and write 5) None of these 4) Why was the young neighbour prevented from sitting on the Terrace? 1) She used to while away her time instead of working. 2) The old women could not longer keep an eye on her. 3) She had not finished writing the letter she was asked to 4) She had been writing a letter which she was not supposed to. 5) As a Punishment for being disrespectful and arguing with her elders. 5) Which was the major argument in the House about ? 1) There was too many People living there, which resulted in arguments 2) The Young girl was insisting on attendind college. 3) The Young girl had been wasting her time instead of working. 4) The old woman didnot guard the young girl closely 5) None of these. Direction(6-7): Choose the word which is most similar in meaning to the word printed in bold as used in the passage. 6) Spend 1) Pay 2) Bought 3) Devote 4) Settle 5) Empty Ans 3 7) Lying 1) Sleeping 2) Dishonest 3) Relaxing 4) Remaining 5) Untruthful Ans 4 Direction(8-9): Choose the word which is most opposite in meaning to the word printed in bold as used in the passage. 8) Dismay 1) Joy 2) Interest 3) Desire 4) Humour 5) Luck Ans 1 9) Tended1) Negligible 2) Watched 3) inclined 4) Ignored 5) Spoil Ans 4 Direction(10-14): Which of the phrases 1, 2, 3 and 4 given below each sentence should replace the phrase Printed in Bold letter to make it Grammatically Correct ? If the sentence is correct as it is given and no Correction is required, mark 5 as the answer. 10) Starting out my own business at this time would affect the financial Stability of my family. 1) Staring up my 2) For Starting With 3) To Start out mine 4) By Starting my 5) No Correction required Ans 5 11) Use a Tactice for mixing the inferior with good quality rice is dishonest and you will pose your liencse.1) Using Tactics as 2) Using a Tactics like 3) To use Tactics 4) Used to Tactics like 5) No Correction required Ans 1 12) The company will invest more six hundred crores in the next five Years to expand its operations in Britian.1) will further invest 2) have invested more than3) have invested above 4) will be invested above 5) No Correction required Ans 1 13) Several of our Projects have delayed because the equipment we ordered was delivered late.1) have been delayed then 2) delayed because of 3) are delayed since 4) were delayed with 5) No Correction required Ans 3 14) The committee has ruled out the Possible raising taxes for this financial year.1) Possible raised 2) Possible rise of 3) Possibility to raise 4) Possibility of raising 5) No Correction required Ans 4 FCI general awareness, current affairs questions with answers, FCI general ability. General intelligence questions with answers,FCI free solved sample placement papers, FCI written test examination questions with answers, FCI previous years solved question for practice, FCI sample questions for practice FCI ,2011,2012 current affairs and general awareness questions with answers FCI model questions for practice 1) The Fifth Edition of the World Future Energy Summit was held in a) Abu Dhabi b) New Delhi c) Tehran d) Dhaka Ans a 2) Planning Commission of India on January 18,2012 approved Rudrasagar development scheme.The Scheme is related to a) Tripura b) Assam c) Manipur d) Sikkim Ans a 3) The Board of approval (BoA) under the Commerce ministry on January 24,2012 approved the proposal of this company to set up a special economics zone( SEZ) at Indore in Madhya Pradesh, Identify the Company ? a) Reliance Industry b) Tata Consultancy services( TCS) c) Hero Motors d) Infoysys Ans b 4) Scientists identified a new gene in maize Plants called ? a) Meg 1 b) Meg 2 c) Meg 3 4) Meg 4 Ans a 5) Researches discovered new earthworm species in Port Blair. What is the name of the new Species ? a) Moniligaster ivaniosi b) Annelida c) Nematoda d) Onychophora Ans a 6) Scientists recently developed a new wireless device to detect the Presence of termites. Consider the following statement- 1) when the new device detects the presence of termites,it sends an SMS or email to a pest Control firm. 2) The Device is made of tiny sensor, even smaller than a fingernail Choose the Right option a) Both 1 and 2 Correct b) Only 1 is Correct c) Only 2 is Correct d) Neither 1 not 2 is Correct Ans a 7) According to the draft Sports Policy of this State Govt released on Jan 11,2012, a Special life-time achievement award in memory of G.V.Raja, the Founder-President of the State Sports Council is to be instituted.Which State is being referred to ? a) karnataka b) kerala c) Tamil Nadu d) Andhra Pradesh Ans b 8) Which team defeated Uttarakhand in the Men's category to win the National title for the fourth time, in the 60th Senior national Volleyball Championship at Raipur on Jan 11, 2012 ? a) Kerala b) Andhra Pradesh c) West Bengal d) Tamil Nadu Ans a 9) Which team extended their hegemony by defeating Kerala 25-12, 25-14, 25-16 in a one sided final in the women category at the 60th Senior National Volley-ball Championship ? a) Punjab b) Services c) Railways d) Delhi Ans c 10) This Footballer was awarded the 2011 FIFA Ballon d'Oran January 9, 2012 in Zurich. He Won the award for the third Consecutive year for the world's outstanding footballer.Identify the Footballer ? a) Cristiano Ronaldo b) Xavi c) Sergio Ramos d) Lionel Messi Ans d 11) State-run Northern Coalfields(NCL) agreed to surrender land required to mine two blocks attached to the Sasan ultra mega power plant to which of the following Companies giving the Company the benefit over Rs 6000 crore over the life of the Venture ? 1) Reliance Power 2) Adani Power 3) Birla Power Solutions 4) DLF Power Ans a 12) Which car took home the Crown of Indian car of the Year 2012 in Jan 2012 1) Toyota Etios liva 2) Maruti Suzuki Swift 3) Hyundai Verna 4) Hyundai Eon Ans b 13) As per the Wholesale Price Index, Published by the govt on Jan 16,2012.Reduced food Prices Caused the inflation to dip to a year low at what percent in December 2011 ? a) 7.47% b) 7.65% c) 6.33% d) 5.41% Ans c 14) Maldives Authorities allowed male international airport development and insurance charges from passengers,nearly a month after a local court barred the Bangalore-based company from doing so.Name the operator? a) GMR Infrastructure b) International Air Transport c) Fraport AG d) DIAL Ans a 15) Which French Construction major entered the Indian market by buying out 100 percent stake in NAPC, a Chennai-based construction Company? a) Sinopec Engineering b) Uhde c) Vinci d) Snamprogetti Ans c 16) India Signed MOU (Memorandum of Understanding ) on Tourism with ASEAN on January12,2012.The agreement took place in- a) Manado b) Jakarta c) Malaysia d) Manila Ans a 17) Pravasi Bhartiya Divas-2012 was held in Jaipur.Jaipur is the Capital of which one of the following states? a) Rajasthan b) Bihar c) UP d) Madhya Pradesh Ans a 18) India and Japan on Jan 12,2012 agreed to enhance their cooperation in the ................. Sector. a) Infrastructure b) Tourism c) Road and Transport d) Coal and Mining Ans a 19) He was Conferred the first S.K Singh Award for excellence in the Indian Foreign services in Jan 6,2012. Identify the diplomat a) Ranjan Mathai b) Shivshankar Menon c) D.Bala venkatesh Verma d) K.Raghunath Ans c 8) Name the auto making company which launched three new variants of the Eon-D-Lite Plus,Era Plus and Magna Plus in the Indian market on jan 2012 ? a) Tata Motors b) Hyundai Motors India c) Volkswagen d) Maruti Suzuki Ans b 20) Who was presented the Mahatma Gandhi International Awards for Peace and Reconciliation on Jan 4,2012 the fourth day of the tenth day Kalachakra initiations ? a) Tibetan Spiritual leader, Dalai Lama b) South African leader Nelson Mandela c) Tanzania's Julius Nyerere d) Burmese Pro-democracy leader Aung san suu kyi Ans a 21) European Union agreed in Principle to ban import of Iranian Crude oil.Consider the following statement: 1) The European Union(EU) is one of Iran's biggest markets for oil. 2) The EU is a Political and economical union of 27 member states Choose the Right option a) Both 1 and 2 are correct b) Only 1 is Correct c) Only 2 is Correct d) Neither 1 nor 2 is correct Ans a 22) The Israel-Palestine talks, in the first week of Jan 2012 were held in Amman.Amman is the Capital of a) Jordan b) Syria c) Iraq d) Iran Ans a 23) Maldives on Jan 4,2012 lifted ban on Spas in the upmarket tourist destination .Consider the following statement: 1) The Tourism industry is an important foreign exchange earner in the Maldives. 2) Maldives in 2011 received more than 850000 tourist Choose the Right Option a) Both 1 and 2 are correct b) Only 1 is Correct c) Only 2 is Correct d) Neither 1 nor 2 is correct Ans a 24) India and which country on March 1,2012,decided to open a new chapter in bilateral cooperation by agreeing to undertake joint operations against pirates and sharing technological know how on seabed research? 1) China 2) Japan 3) Russia 4) New Zealand Ans 1 25) A meeting at the prime minister office on Feb 29,2012, decided to increase the total expenditure on health to what percent of the GDP from the current 1.4 percent by the end of the coming 12th plan period? 1) 2.0 Percent 2) 2.5 Percent 3) 3.0 Percent 4) 4.0 Percent Ans 2 26) India and which country recently announced to have resolved their longstanding issues of payments for export shipment? 1) Iran 2) Syria 3) Lebanon 4) Pakistan Ans 1 27) The indigenously development advanced light-weight torpedo and the Akash missile system was on March 3,2012,handed over to the Navy and the Indian Air force respectively at a function in 1) Visakhapatnam 2) Hyderabad 3) Pune 4) Kolkata Ans 2 28) The golden Jubilee celebrations of Afro-Asian Rural development organization were held in New Delhi in March 2012.The AARDO president hails from 1) India 2) Egypt 3) Morocco 4) South Africa Ans 2 29) The National Association of Street vendors of India( NASVI ) and Nidan,an organization that supports unorganized workers,were on March 29,2012 honored with the skoll Award for social entrepreneurship by the skoll foundation in 1) Toronto 2) oxford 3) Paris 4) Frankfurt Ans 2 30) Union minister for Rural development, Drinking Water and sanitation Jairam Ramesh in March 2012 announced a national award for sanitation and water in the name of Maharashtrian saint sant 1) Romesh Hatangadi 2) Gadge Baba 3) Suresh Bhagwan 4) Shiv sagar Baba Ans 2 31) Who among the following, on march 11,2012,said he will be "Very Sad" if India boycotts the London Olympics following the row over sponsorship of the event by Dow Chemicals, linked to the Bhopal gas tragedy? 1) Barack Obama 2) David Cameron 3) Manmohan singh 4) Hillary Clinton Ans 2 32) The high level safety review committee recently submitted its report to the Ministry of railways with a Number of recommendations,who is the chairman of this committee ? 1) C.Rangarajan 2) Lalu Yadav 3) Anil Kakodkar 4) Harish Rawat Ans 3 33) In an important development on March 12,2012, the supreme court issued notices to nsix ministers and eight Indian Administrative services officers from 1) Andhar Pradesh 2) Karnataka 3) Kerala 4) Tamil Nadu Ans 1 34) Which of the following was pointed by the union Finanace minister Pranab Mukherjee in March 2012 as one of the major challenges facing public sector banks(PSB) in emerging as potential global tenders? 1) Human Capital deficit 2) Human Capital deficit 3) Innovation deficit 4) Technology deficit Ans 1 35) The Centre told the Supreme Court in March 2012 that it was Probing into the allegations that the former Chief Justice of India and his relatives amassed assets disproportionate to their Known sources of income.Who is the chief Justice in reference here? 1) K.G.Balakrishna 2) S.H.Kapadia 3) Altamas Kabir 4) Dalveer Bhandari Ans 1 36) The Functioning of both houses of Parliament was disrupted on March 13,2012,When members demanded that India support the resolution to be moved by------ against sri lanka at the U.N Human Rights Council. 1) U.S.A 2) France 3) Norway 4) All of the above Ans 4 37) Noted critic Namvar Singh in March 2012 presented the Jnanpith Award to eminent hindi author Amar Kant at a Special ceremony in 1) New Delhi 2) Lucknow 3) Allahabad 4) Patna Ans 3 38) Increasing the fare for the first time in eight years, the railway budget for 2012-13, presented before the parliament on March 14,2012,envisages how many new trains? 1) 90 2) 113 3) 145 4) 166 Ans 2 39) Okram Ibobi Singh was sworn as chief minister for the third Consecutive term of which state in March 2012? 1) Arunachal Pradesh 2) Nagaland 3) Manipur 4) Tripura Ans 3 17) Governor Shivraj V.Patil in March 2012 administered the oath of office and secrecy to Parkash Singh Badal,Who took over as the chief minister of Punjab for a record- time 1) Fifth 2) Third 3) Fourth 4) Seventh Ans 1 40) The finance ministry, on March 15,201, gave it's nod for the labour and employment ministry to fix the rate of interest payable to EEPF sub-subscribers at which percentage for2011-12? 1) 8.25% 2) 8.75% 3) 9.25% 4) 9.50% Ans 1 41) The Cabinet of which state in march 2012 decided to give a grant of rS. 30,000 to high school pass Muslims girls for perusing their education and marriage? 1) Uttar Pradesh 2) West Bengal 3) Andhra pradesh 4) Delhi Ans 1 42) Which of the following Chief ministers of India figures on the cover of the Time magazine in March 2012 which prasied him in an article for the development of the State? 1) Mamta Banarjee 2) Nitish Kumar 3) Ashok Gehlot 4) Narendra Modi. Ans 4 FCI aptitude and reasoning questions for practice Q. 1 - 5. What should come in place of question Mark (?) in the following questions? 1. 92.5% of 550 = ? (1) 506.45 (2) 521.65 (3) 518.55 (4) 508.75 (Ans) (5) None of these 2. 124 * 1213 = ? (1) 127 (2) 1239 (3) 1217 (Ans) (4) 12-7 (5) None of these 3. 12.22 + 22.21 + 221.12? (1) 250.55 (2) 255.50 (3) 250.05 (4) 255.05 (5) None of these (Ans) Explanation : 255.55 4. 464 ÷ (16 * 2.32) = ? (1) 12.5 (Ans) (2) 14.5 (3) 10.5 (4) 8.5 (5) None of these 5. 78 ÷ 5 ÷ 0.5 = ? (1) 15.6 (2) 31.2 (Ans) (3) 7.8 (4) 20.4 (5) None of these 6. A bus covers a distance of 2,924 kms. in 43 hours. What is the speed of the bus? (1) 72 kms/hr (2) 60 kms/hr (3) 68 kms/hr (Ans) (4) Cannot be determined (5) None of these Explanation : Speed = D/t 7. If (9)3 is subtracted from the square of a number, the answer so obtained is 567. What is the number? (1) 36 (Ans) (2) 28 (3) 42 (4) 48 (5) None of these Explanation : x2 - 93 = 567 ? x = 36 8. What would be the simple interest obtained on an amount of Rs. 5,760 at the rate of 6 p.c.p.a. after 3 years? (1) Rs. 1,036.80 (Ans) (2) Rs. 1,666.80 (3) Rs. 1,336.80 (4) Rs. 1,063.80 (5) None of these Explanation : S. I. = 5760 * 6 * 3/100 = Rs. 1036.80 9. What is 333 times 131? (1) 46,323 (2) 43,623 (Ans) (3) 43,290 (4) 42,957 (5) None of these 10. The product of two successive numbers is 8556. What is the smaller number? (1) 89 (2) 94 (3) 90 (4) 92 (Ans) (5) None of these Explanation : x(x +1) = 8556 ? x = 92 11. The owner of an electronics shop charges his customer 22% more than the the cost price. If a customer paid Rs. 10,980 for a DVD Player, then what was the cost price of the DVD Player? (1) Rs. 8,000 (2) Rs. 8,800 (3) Rs. 9,500 (4) Rs. 9,200 (5) None of these (Ans) Explanation : 122/100 x = 10980 ? x = Rs. 9000 12. What would be the compound interest obtained on an amount of Rs. 3,000 at the rate of 8 p.c.p.a after 2 years? (1) Rs. 501.50 (2) Rs. 499.20 (Ans) (3) Rs. 495 (4) Rs. 510 (5) None of these Explanation : C.I = P |(1+ R/100)n - 1| = 3000 | (1 + 8/100)2- 1| = Rs. 499.20 13. What is the least number of be added to 4321 to make it a perfect square? (1) 32 (2) 34 (3) 36 (4) 38 (5) None of these (Ans) Explanation : 652 < 4321 < 662 Reqd. No. = 662 - 4321 = 35 14. 45% of a number is 255.6. What is 25% of that number? (1) 162 (2) 132 (3) 152 (4) 142 (Ans) (5) None of these Explanation : 45/100 of x = 255.6 ? x = 255.6 x 100/45 ? 25/100 x 255.6 x 100/45 = 142 15. Find the average of the following Set of Scores : 221, 231, 441, 359, 665, 525 (1) 399 (2) 428 (3) 407 (Ans) (4) 415 (5) None of these 16. If (78)2 is subtracted from the square of the number, the answer so obtained is 6,460. What is the number? (1) 109 (2) 111 (3) 113 (4) 115 (5) None of these (Ans) Explanation : x2 - 782 = 6460 ? x =112 17. In an examination it is required to get 40% of the aggregate marks to pass. A student gets 261 marks and is declared failed by 4% marks. What are the maximum aggregate marks a student can get? (1) 700 (2) 730 (3) 745 (4) 765 (5) None of these (Ans) Explanation : 40/100 x = 261 + 4/100 x ? x = 725 18. Pinku, Rinku and Tinku divide an amount of Rs. 4,200 amongst themselves in the ratio of 7 : 8 : 6 respectively. If an amount of Rs. 200 is added to each of their shares, what will be the new respective ratio of their shares of amount? (1) 8 : 9 : 6 (2) 7 : 9 : 5 (3) 7 : 8 : 6 (4) 8 : 9 : 7 (Ans) (5) None of these Explanation : Share of Pinku, Rinku and Tinku in Rs. 4200 are 7/ 7 + 8 + 6 x 4200, 8/ 21 x 4200, 6/ 21 x 4200 i.e. 1400, Rs. 1600, Rs. 1200 Reqd. ratio = (1400 + 200) : (1600 + 200) : (1200 + 200) = 8 : 9 : 7 19. Ms Suchi deposits an amount of Rs. 24,000 to obtain a simple interest at the rate of 14 p.c.p.a. for 8 years. What total amount will Ms Suchi get at the end of 8 years? (1) Rs. 52,080 (2) Rs. 28,000 (3) Rs. 50,880 (Ans) (4) Rs. 26,880 (5) None of these Explanation : Total Amount = Rs. 24000 + 24000 x 14 x 8/100 = Rs. 50880 20. The average of 5 consecutive even numbers A, B, C, D and E is 52. What is the product of B and E? (1) 2912 (2) 2688 (3) 3024 (4) 2800 (Ans) (5) None of these Explanation : A + B + C + D + E x + (x + 2) + (x + 4) + (x + 6) + (x + 8) = 5 x 52 ? x = 48 = A, B = 50 ? E = 48 + 8 = 56 ? BE = 50 x 56 = 2800 21. The difference between 42% of a number and 28% of the same number is 210. What is 59% of that number? (1) 630 (2) 885 (Ans) (3) 420 (4) 900 (5) None of these Explanation : (42 - 28)% of x = 210 ? x = 210 x 100/14 = 1500 ? 59/100 * 15 = 885 22. What approximate value should come in place of the question mark (?) in the following question? 4275 496 x (21)2 = ? (1) 3795 (2) 3800 (Ans) (3) 3810 (4) 3875 (5) 3995 Explanation : Use BODMAS 23. A canteen requires 112 kgs of wheat for a week. How many kgs of wheat will it require for 69 days? (1) 1,204 kgs (2) 1,401 kgs (3) 1,104 kgs (Ans) (4) 1,014 kgs (5) None of these Explanation : 112/7 x 69 = 1104 kg = Reqd. quantity of wheat 24. If an amount of Rs 41,910 is distributed equally amongst 22 persons. How much amount would each person get? (1) Rs. 1,905 (Ans) (2) Rs. 2,000 (3) Rs. 1,885 (4) Rs. 2,105 (5) None of these 25. The cost of 4 Cell-phones and 7 Digital cameras is Rs 1,25,627. What is the cost of 8 Cell-phones and 14 Digital cameras? (1) Rs. 2,51,254 (Ans) (2) Rs. 2,52,627 (3) Rs. 2,25,524 (4) Cannot be determined (5) None of these Explanation : 4x + 7y = 125627] x 2 ? 8x + 14y = 251254 Q. 26-30. Each of the question below consists of a equation and two statement numbered I and II are given below it. You have to decide whether the data provided in the statements are sufficient to answer the question. Read both the statements and give answer: (1) if the data in Statement I alone are sufficeint to answer the question. while the data in Statement II alone are not sufficient to answer the question. (2) if the data in Statement II alone are sufficient to answer the question, while the data in Statement I alone are not sufficient to answer the question. (3) if the data in Statement I alone or in Statement II alone are sufficient to answer the question. (4) if the data in both the Statement I and Statement II are not sufficient to answer the question. (5) if the data in both the Statements I and II together are necessary to answer the question. 26. What is the area of the circle? I. Perimeter of the circle is 88 cms. II. Diameter of the circle is 28 cms. (Ans : 3) Explanation : 2pr = 88 ? r = 88/2pr r = D/2 = 28/2 = 14 Either (i) or (ii) is reqd. A = pr2 27. What is the rate of interest? I. Simple interest accrued on an amount of Rs. 25,000 in two years is less than the compound interest for the same period by Rs. 250. II. Simple interest accrued in 10 years is equal to the principal. (Ans : 3) Explanation : 250 = 25000 [(1 + R)2 -1] - 25000 x R x 2 or R = x * 100/ x * 10 = 10% From either of statement we can find R 28. What is the number of trees planted in the field in rows and columns? I. Number of columns is more than the number of rows by 4. II. Number of trees in each column is an even number. (Ans : 4) 29. What is the area of the right-angled triangle? I. Height of the triangle is three-fourth of the base. II. Diagonal of the triangle is 5 metres. (Ans : 5) Explanation : x2 + [3/4 x]2 = 52 ? x = 4, h = 3/4 * 4 = 3 Area = 1/2 (x * 3/4 x) = 1/2 (4 *3) = 6 sq. units Both (i) and (ii) statement are reqd. 30. What is the father's present age? I. Father's present age is five times the son's present age. II. Five years ago the father's age was fifteen times the son's age that time. (Ans : 5) Explanation : Let son's present age be x ? Father's present age = 5x ATS 5x - 5 = 15 (x - 5) ? x = 7 ? Father's present age = 35 yrs Q. 31-35. Study the following graph carefully to answer these questions: 31. What is the ratio between the profit earned by Company A in 2004 and the profit earned by Company B in 2003 respectively? (1) 4 : 3 (2) 3 : 2 (3) 3 :4 (4) 2 :3 (5) None of these (Ans) Explanation : 40/40 = 1/1 32. What is the difference (in Crore Rs.) between the total profit earned by Companies E, F and G together in 2003 and the total profit earned by these companies in 2004? (1) 70 (2) 75 (3) 78 (4) 82 (5) None of these (Ans) Explanation : Diff = (50 + 80 + 60) - (40 + 20 + 50) = 80 33. What is the ratio between the total profit earned by Company C in 2003 and 2004 together and the total profit earned by Company E in these two years respectively? (1) 11 : 9 (Ans) (2) 9 : 10 (3) 10 : 11 (4) 11 : 10 (5) None of these Explanation : Reqd ratio = 50 + 60 / 40 + 50 = 11/9 34. What was the average profit earned by all the companies in 2003? (In Crore Rs Rounded Off to two digits after decimal.) (1) 52.75 (2) 53.86 (3) 52.86 (Ans) (4) 53.75 (5) None of these 35. Profit earned by Company B in 2004 is what per cent of the profit earned by the same company in 2003? (1) 133.33 (2) 75 (Ans) (3) 67.66 (4) 75.25 (5) None of these Explanation : 30 = x% of 40 ? x = 75 Q. 36-40. Study the following table carefully to answer these questions: TABLE GIVING PERCENTAGE OF UNEMPLOYED MALE AND FEMALE YOUTH AND THE TOTAL POPULATION FOR DIFFERENT STATES IN 2005 AND 2006 STATE 2005 2006 M F T M F T A 12 15 32 7 8 35 B 8 7 18 10 9 20 C 9 10 28 10 12 34 D 10 6 24 8 8 30 E 6 8 30 7 6 32 F 7 5 28 8 7 35 M = Percentage of unemployed Male youth over total population F = Percentage of unemployed Female youth over total population T = Total population of the State in lakhs 36. What was the total number of unemployed youth in State A in 2006? (1) 2,20,000 (2) 3,25,000 (3) 5,20,000 (4) 5,25,000 (Ans) (5) None of these 37. How many female youth were unemployed in State D in 2005? (1) 14,400 (2) 1,44,000 (Ans) (3) 1,40,000 (4) 14,000 (5) None of these 38. Number of unemployed male youth in State A in 2005 was what per cent of the number of unemployed female youth in State E in 2006? (1) 66 (2) 50 (3) 200 (Ans) (4) 133 (5) None of these 39. What was the difference between the number of unemployed male youth in State F in 2005 and the number of unemployed male youth in State A in 2006? (1) 70,000 (2) 45,000 (3) 68,000 (4) 65,000 (5) None of these (Ans) Explanation : Ans 49000 40. What was the respective ratio between unemployed male youth in State D in 2005 and the unemployed male youth in State D in 2006? (1) 1 : 1 (Ans) (2) 2 : 3 (3) 3 : 2 (4) 4 : 5 (5) None of these Explanation : 10 * 24 / 100 / 8 * 30/100 = 1 : 1 Placement Paper FCI aptitude,general ability,general awareness and reasoning questions with answers and detailed explanations, FCI previous years solved question papers. FCI largest collection solved free placement papers FCI model questions for practice all topics,FCI free on line mock tests,Latest general awareness 2012,currents questions with answers, descriptive and one line multiple choice question answers Practice Question Set on General Awareness for all IT Non IT (psu) and banks ibps po clerical general awareness questions with answers Stock Exchange Related Questions 1. Which is the oldest Stock Exchange in Asia ? Answer: Bombay Stock Exchange 2. What is the full-form of SENSEX ? Answer: Sensitive Index 3. In which street did Bombay Stock Exchange situate ? Answer: Dalal Street 4. When did National Stock Exchange of India (NSE) founded? Answer: 1992 (in 1993 it become stock exchange) 5. When did SEBI (Securities and Exchange Board of India) formed ? Answer: 1988 (Officialy started in 1992) 6. Which is the Worlds First Electronic Stock Exchange ? Answer: NASDAQ (National Association of Securities Dealers Automated Quotations) 7. Which Stock Exchange has the nick name "Big Board" ? Answer: New York Stock Exchange (NYSE) 8. Which Stock Exchange is situated in Wall Street ? Answer: New York Stock Exchange (NYSE) FCI latest 2012 general awareness and current affairs questions with answers 1. The Govt of India has decided to declare which of the following rivers a National River? (A) Brahmaputra (B) Yamuna (C) Ganga -Answer (D) Kaveri (E) None of these 2. Who amongst the following economists gave the concept of "economies of scale", which says "many goods and services can be produced more cheaply in long series"? (A) Edward C Prescott (B) Amartya Sen (C) Gary S Becker (D) Edmund S Phelps (E) Paul Krugman-Answer 3. Very often we read in newspapers/magazines about "Sovereign Wealth Funds". Which of the following is/ are the correct description of the same? (A) These are the funds or the reserves of a government or central bank of a country which are invested further to earn profitable returns. (B) These are the funds which were accumulated by some people over the years but were not put in active circulation as they retain them as Black Money for several years. (C) The funds which are created to be used as relief funds or bailout packages are known as sovereign funds. (A) Both B & C only (B) Only B (C) Only C (D) Only A-Answer (E) None of these 4 . Very often we read about Special economic Zones (SEZs) in newspapers. What is the purpose of promoting SEZs in India? (A) They are established to promote exports. (B) They are established to attract investments from foreign countries. (C) They are established to help the poorest of the poor in India as the activities of these zones are reserved only for the poors and those living below poverty line. (A) Only A (B) Only B (C) Only C (D) Both A & B only-Answer (E) None of these 5 . The ratio of the Cash Reserves that the banks are required to keep with the RBI is known as (A) Liquidity Ratio (B) SLR (C) CRR -Answer (D) Net Demand & Time Liability (E) None of these 6. The-Govt of India in order to provide some relief to the exporters announced a relief-package of Rs (A) 1000 cr (B) 2500 cr (C) 4000 cr (D) 5000 cr-Answer (E) 7000 cr 7. The availability of cash and other cash-like marketable instruments that are useful in purchases and investments are commonly known as (A) Cash crunch (B) Liquidity-Answer (C) Credit (D) Marketability (E) None of these 8. The White Tiger is a book written by (A) Amitav Ghosh (B) Arundhati Roy (C) VS Naipaul (D) Kiran Desai (E) Aravind Adiga-Answer 9. Besides USA and France, India has a Civil Nuclear Deal with (A) Britain (B) Russia-Answer (C) Germany (D) Canada (E) Australia 10. Jnanpith Award is given for excellence in the field of (A) Music (B) Literature -Answer (C) Sports (D) Films (E) Social Services 11. As per newspaper reports, India's trade gap has been showing a negative growth for the last few months. "Trade Gap" means what? (A) Gap between total GDP and total consumption (B) Gap between total imports and total exports-Answer (C) Gap between available liquidity and expected demand in next five months (D) Gap between budgeted revenue collection and actual collection of the same (E) None of these 12. 'Equity schemes managed strong NAV gains, which, boost their assets' was a news in some financial newspapers. What is the full form of the term NAV as used in above head lines? (A) Nil Accounting Variation (B) Net Accounting Venture (C) Net Asset Value-Answer (D) New Asset Venture (E) None of these 13. As per reports published in various newspapers, mutual fund companies showed 94 per cent growth in their total profits during 2009-10. This means the profits earned by these companies were 94 per cent (A) in their total investments they made collectively during the year (B) more than their profits during previous year-Answer (C) of the total capital of the company (D) less than the total expenditure of the companies (E) None of these 14. As per the reports published in some major news papers "ADAG companies" made good profits during the year. ADAG companies are popularly known as (A) Companies owned by Tata Group (B) Aditya Birla Companies (C) Reliance Companies-Answer (D) Companies owned by Aptech (E) None of these 15. "Mutual Funds reported exceptional performance in 2009-10" was the news in major financial news papers recently. What is a mutual fund? A. A type of collective investment scheme that pools money from many investors and invests it in stocks, bonds or other money market instruments.-Answer B. It is a subsidiary of a bank or financial company created specially to raise money to be invested in a particular industry, ie housing or insurance etc. The money raised thus cannot be invested anywhere else. C. When several banks and financial companies come together and create a common pool of money to fund mega infrastructural project like bridges, roads, power plants etc, the common pool is known as Mutual Fund. (A) Only A-Answer (B) Only B (C) Only C (D) All A, B & C (E) None of these 16. We very frequently read about Europe's sovereign debt crisis these days. Which of the following statements/is/are true about the same? A. In early 2010 the Euro crisis developed in some countries like Greece, Spain and Portugal. B. This created a credit default swap between the countries of the European Union. C. SAARC countries have offered some assistance to some of the severely affected countries like Portugal, Spain and Greece. (A) Only A-Answer (B) Only B (C) Only A & B (D) All A, B & C (E) None of these 17. As per the announcement made by the RBI, some Stock Exchanges in India are allowed to introduce Plain Vanilla Currency Options. The term Plain Vanilla Currency Options is associated with which of the following activities/operations? (A) Dollar-Rupee Exchange Rate- Answer (B) Floating of Commercial Papers (C) Launch of new mutual funds (D) Deciding the opening price of a share on a particular" business day (E) None of these 18. The process of "Artificial Application of Water to the soil usually for assisting in growing crops" is technically known as (A) water harvesting (B) irrigation- Answer (C) water recharging (D) percolation (E) none of these 19. Waste Water generates which of the following gases. which is more powerful and dangerous than C02? (A) Nitrogen (B) Sulphur dioxide (C) Hydrogen (D) Methane- Answer (E) None of these 20. Which of the following days was observed as World Water Day 2010? (A) 20th June (B) 22nd July (C) 22nd March- Answer (D) 20th May (E) None of these 21. "Indira Gandhi Canal", which is around 450 km long, provides irrigation facility mainly to which of the following states? (A) Punjab (B) Haryana (C) Madhya Pradesh (D) Gujarat (E) Rajasthan- Answer 22. Who amongst the following is the director of the popular Hindi film "OyeLucky ! Lucky Oye"? (A) Mani Ratnam (B) David Dhavan (C) Dibakar Banerjee - Answer (D) R Balakrishnan (E) None of these 23. Which of the following is NOT a missile developed by the Defence Research and Development. Organisation (DRDO)? (A) Shaurya- Answer (B) Pinaka (C) Brahmos (D) Agni (E) Nag 24. Which of the following metals is used for generation of Nuclear Energy by most of the Nuclear Power Plants? (A) Zinc (B) Platinum (C) Uranium- Answer (D) Nickel (E) None of these 25. Indus Water Treaty is a pact on' sharing of river water between India and (A) Bangladesh (B) Pakistan- Answer (C) Nepal (D) Afghanistan (E) Myanmar 26. Which of the following is/are considered a Renewable Source of Energy? A. Wind Energy B. Solar Energy C. Nuclear Energy (A) Only A (B) Only B (C) Only C (D) Only A & B- Answer (E) All A, B' & C 27. Mrs. Pratibha Patil is the _______the President of India (A) 10th President of India (B) 11th President of India (C) 12th President of India- Answer (D) 1st President of India (E) 15th President of India 28. How many intertwined rings are shown in the Olympic Flag used these days in the games? (A) Five- Answer (B) Six (C) Seven (D) Three (E) None of these 29 . India's 'Look East' trade policy is designed to help particularly members of which of the following group of nations? (A) SAARC (B) ASEAN- Answer (C) WTO D) NAT (E) EU 30. Which of the following countries has signed a 22-Point Agreement with Madhesi People's Right Forum (MPRF)? (A) China (B) Nepal - Answer (C) India (D) Bangladesh (E) Myanmar 31. 'Brahmos' is designed to be used by which of the following? (A) Indian Navy (B) Indian Army- Answer (C) Indian Air Force (D) Border Security Force (E) All of these 32. Which of the following agencies/organisations is setting up India's first Natural Resource Data Centre in Kolkata? (A) RBI (B) SEBI (C) NABARD- Answer (D) S~C (E) BARC 33. The Finance Minister of India recently made a mention that Infrastructure Development in India should be done on PPP model only. What is the full form of' PPP'? (A) Purchasing Power Parity (B) Public Private Partnership- Answer (C) Purchase Produce Provide (D) People's Programme & Priority (E) None of these 34. The Global Non-Violence Day was observed on which of the following days? (A) 15th August (B) 14th November (C) 19th September (D) 2nd October- Answer (E) 30th January 35. The period for 11th Five Year Plan is (A) 2005-10 (B) 2006-11 (C) 2007-12- Answer (D) 2008-13 (E) None of these 36. Who amongst the following has played the main/lead role in the film Chak De India? (A) Abhishek Bachchan (B) Salman Khan (C) Aamir Khan (D) Shah Rukh Khan- Answer (E) None of these 37. 'Ranji Trophy' is associated with the game of (A) Hockey (B) Football (C) Badminton (D) Cricket- Answer (E) Tennis 38 . Which of the following awards is given for excellence in literary work? (A) Bharat Ratna (B) Shanti Swamp Bhatnagar Award (C) Dronacharya Award (D) Arjuna Award (E) Pulitzer Prize- Answer 39. Roger Federer, who won the US Open 2007 Tennis Championship, is a citizen of (A) Belgium (B) France (C) Russia (D) USA (E) Switzerland- Answer 40. Which of the following awards is NOT a 'Gallantry I Award'? (A) Mahavir Chakra (B) Vir Chakra (C) Vishista Sewa Medal (D) Ashok Chakra (E) Saraswati Saminan- Answer 41. As per the to Commerce Ministry data released on 2 January 2012, India’s exports recorded their slowest pace of growth in two years in November 2011. What was the percentage of growth recorded? (A) 4.2% (B) 3.8%- Answer (C) 2.2% (D) 0.94% (E) None of these 42. The capital market regulator SEBI on 3 January 2012 allowed auctioning of securities through stock exchanges and introduced a new method for institutional placement of stocks. Which of the following facts related to the above statement is/are not true? 1. Under the institutional placement programme (IPP), shares can be sold only to qualified institutional buyers. 2. There shall be at least 25 allottees in every IPP issuance. No single investor shall receive allotment for more than 25% of the offer size 3. The IPP method can be used to increase public holding by 10% and could be offered to only qualified institutional buyers with 25% being reserved for mutual funds and insurance companies 4. Under the IPP, companies will have to announce the ratio of buy-back, as is done in the case of rights issues and fix a record date for determination of entitlements as per shareholding on record date (A) Only 1 (B) Only 3 (C) Only 2- Answer (D) Only 4 (E) None of these 43. What is the Finace Ministry’s decision on the rates applicable on small savings instruments schemes that would be announced on April 1 each year? (A) rate would remain valid till the maturity of the scheme- Answer (B) rate would change in the first qurter of that respective year (C) rates would change depending on different maturity period (D) rates would remain valid for six months post the maturity (E) None of these 44. Scientists discovered unknown species off the coast of Antarctica. Consider the following statements: i) The temperature in this region rises to 380 degree Celsius. Ii) There is plenty of light in this region. Which of the above statements is/ are correct? (A) Only i- Answer (B) Only ii (C) Both i and ii (D) Neither i nor ii (E) None of these 45. Scientists produced artificial human semen to help infertile men. Consider the following statements: i) The scientists grew the sperm by enveloping the germ cells in a special compound called agar jelly. ii) The artificial human semen could help infertile men father their own children. Choose the right option: (A) Both i and ii are correct.- Answer (B) Only i is correct. (C) Only ii is correct. (D) Neither i nor ii is correct. (E) None of these 46. Health Authorities in Australia recently detected Deadly Disease, Murray Valley Encephalitis. The disease is caused by the__. (A) Flies- Answer (B) Mosquitoes (C) Birds (D) Dogs (E) None of these 47. Which PSU announced on 2 January 2012 that its board approved in a meeting held on 30 December 2011 the switching over to internationally-accepted Gross Caloric Value-based pricing mechanism? (A) BHEL (B) ISPAT (C) CIL- Answer (D) IOC (E) None of these 48. Mukesh Ambani’s Reliance Industries Ltd (RIL) entered the media and entertainment sector by making a major investment in one of India’s largest broadcast companies. Name the broadcast company. (A) Network18 Group- Answer (B) Adlabs Films (C) Balaji Telefilms (D) Bag Films (E) None of these 49. India approved the acquisition of French advanced missile systems to arm the Mirage-2000 fighter jets. Consider the following statements: i) The cabinet committee on Security cleared the contract for the fire and forget MICA with French company MBDA. ii) MICA are interception and aerial combat missiles.Choose the right option: (A) Both i and ii are correct.- Answer (B) Only i is correct. (C) Only ii is correct. (D) Neither i nor ii is correct. (E) None of these 50 . The Union government approved Katra-Quazigund Railway line project. The project is located in__. (A) Jammu and Kashmir- Answer (B) Bihar (C) Punjab (D) Haryana (E) None of these 51. What is the new name of Big Ben tower in London ? Answer: Elizabeth tower 52. Who is the new President of All Indian Tennis Association ? Answer: Anil Khanna 53. Who is the new President of Egypt ? Answer: Muhammad Murzi 54. Which country will host the 2016 Euro Cup? Answer: France 55. Who won the Golden Boot in 2012 Euro Cup ? Answer: Fernando Torres (Spain) 56. Who is elected as Best player of 2012 Euro cup ? Answer: Andres Iniesta 57. Who is the new president of Mexico ? Answer: Enrique Pena Nieto 58. Who is the new prime minister of Greece ? Answer: Antonis Samaras 59. Who is the first amputee to compete in Olympics ? Answer: Oscar Pistorius 60. Who is appointed as new President of World Bank on 16th April 2012 ? Answer: Jim Yong Kim 61. Who is crowned as Miss India 2012 ? Answer: Vanya Mishra 62. Who is crowned as Miss World 2012 ? Answer: Yu Wenxia (China) 63. Who won the 73 rd PGA Tour title 2012 ? Answer: Tiger Woods 64. Who is the First woman Space traveler of China ? Answer: Liu Yung 65. Which Indian Won the Ramon Magsaysay Award ? Answer: Kulendei Francis 66. Who is newly appointed as UN Special Envoy for Syria replacing Kofi Annan ? Answer: Lakhdar Brahmini 67. Who is the present President of Pakistan ? Answer: Asif Ali Zardar 68. Who is the present Prime Minister of Pakistan ? Answer: Raja Pervaiz Asharaf 69. Who is the present President of Nepal ? Answer: Ram Baran Yadav 70. Who is the present Prime Minister of Nepal ? Answer: Baburam Bhattari 71.What is the name of new book written by Shashi Tharoor ? Answer: Pax Indica 72. Who is the first Indian origin to became Supreme Court chief justice of Singapore ? Answer: Sundaresh Menon (kerala) 73. Which are the satellites put into orbit by PSLV-21 as ISRO's 100th mission ? Answer: SPOT-6 (French earth observation satellite) and Proiteres (Japanese micro satellite ) 74. Who the first Muslim and Youngest Minister of Norway (newly appointed) ? Answer: Hadia Tajik ( Norwegian Culture Minister) 75. Which Country host the ICC World T20 tournament ? Answer: Sri Lanka 76Which country is the co-ordinator for India in the ASEAN Summit to be held on 20th December 2012? Brunei 77Which Indian state government approved IT policy 2012 recently? Uttar Pradesh 78Who is appointed as President and CEO of Volvo cars? Hakan Samuelsson 79Which American coffee chain major entered into joint venture with Tata to open its 1st store in India? Starbucks 80Who is appointed as bowling coach of New Zealand cricket team? Shane Bond 1: The largest copper producing country in the World is 1. Chile 2. Russia 3. South Africa 4. China Ans: 1. 2: If the radius of a circle is diminished by 10%, then its area is diminished by: 1. 10% 2. 19% 3. 20% 4. 36% Ans: 2. 3: A boat travels 20 kms upstream in 6 hrs and 18 kms downstream in 4 hrs.Find the speed of the boat in still water and the speed of the water current? 1. 1/2 kmph 2. 7/12 kmph 3. 5 kmph 4. none of these Ans: 2. 4: At what time after 4.00 p.m. is the minutes hand of a clock exactly aligned with the hour hand? 1. 4:21:49.5 2. 4:27:49.5 3. 3:21:49.5 4. 4:21:44.5 Ans: 1. 5: A shop keeper sold a T.V set for Rs.17,940 with a discount of 8% and earned a profit of 19.6%.What would have been the percentage of profit earned if no discount was offered? 1. 24.8% 2. 25% 3. 26.4% 4. Cannot be determined 5. None of these Ans: 5. 6: If (2x-y)=4 then (6x-3y)=? 1. 15 2. 12 3. 18 4. 10 Ans: 2. 7: A clock is set right at 8 a.m. The clock gains 10 minutes in 24 hours. What will be the true time when the clock indicates 1 p.m. on the following day? 1. 48 min. past 12 2. 38 min. past 12 3. 28 min. past 12 4. 25 min. past 12 Ans: 1. 8: What is the missing number in this series? 8 2 14 6 11 ? 14 6 18 12 1. 16 2. 9 3. 15 4. 6 Ans: 2. 9: Dinesh travelled 1200 km by air which formed 2/5 of his trip. One third of the whole trip, he travelled by car and the rest of the journey he performed by train. What was the distance travelled by train? 1. 600Km 2. 700Km 3. 800Km 4. 900Km Ans: 3. 10: A train which travels at a uniform speed due to some mechanical fault after traveling for an hour goes at 3/5th of the original speed and reaches the destination 2 hrs late.If the fault had occurred after traveling another 50 miles the train would have reached 40 min earlier. What is distance between the two stations. 1. 300 2. 310 3. 320 4. 305 Ans: 1. 11: The average between a two digit number and the number obtained by interchanging the digits is 9. What is the difference between the two digits of the number? 1. 8 2. 2 3. 5 4. Cannot be determined Ans: 4. 12: Pipe A can fill in 20 minutes and Pipe B in 30 mins and Pipe C can empty the same in 40 mins.If all of them work together, find the time taken to fill the tank 1. 17 1/7 mins 2. 20 mins 3. 8 mins 4. none of these Ans: 1. 13: A person has 4 coins each of different denomination. What is the number of different sums of money the person can form (using one or more coins at a time)? 1. 16 2. 15 3. 12 4. 11 Ans: 2. 14: The simple interest on a certain sum of money for 3 years is 225 and the compound interest on the same sum at the same rate for 2 years is 153 then the principal invested is 1. 1500 2. 2250 3. 3000 4. 1875 Ans: 4. 15: A cow is tethered in the middle of a field with a 14 feet long rope. If the cow grazes 100 sq. ft. per day, then approximately what time will be taken by the cow to graze the whole field ? 1. 2 days 2. 6 days 3. 18 days 4. 24 days 5. None of these Ans: 2. 16: 2 hours after a freight train leaves Delhi a passenger train leaves the same station travelling in the same direction at an average speed of 16 km/hr. After travelling 4 hrs the passenger train overtakes the freight train. The average speed of the freight train was? 1. 40 2. 30 3. 80 4. 60 Ans: 1. 17: The two colors seen at the extreme ends of the pH chart are: 1. Red and Blue 2. Red and Green 3. Green and Blue 4. Orange and Green Ans: 1. 18: 8 15 24 35 48 63 _? 1. 70 2. 80 3. 75 4. 88 Ans: 2. 19: One of Mr. Horton, his wife, their son, and Mr. Horton’s mother is a doctor and another is a lawyer. a) If the doctor is younger than the lawyer, then the doctor and the lawyer are not blood relatives. b) If the doctor is a woman, then the doctor and the lawyer are blood relatives. c) If the lawyer is a man, then the doctor is a man. Whose occupation you know? 1. Mr. Horton: he is the doctor 2. Mr. Horton’s son: she is the lawyer 3. Mr. Horton: he is the doctor 4. Mr. Horton’s mother: she is the doctor Ans: 1. 20: In the given figure, PA and PB are tangents to the circle at A and B respectively and the chord BC is parallel to tangent PA. If AC = 6 cm, and length of the tangent AP is 9 cm, then what is the length of the chord BC? 1. 4 cm 2. 8 cm 3. 6 cm 4. 5 cm Ans: 1. 21: Union Information and Broadcasting ministry recently gave an indication to change which of the following laws on a larger scale, as the existing provisions of the Act are inadequate to cater to the phenomenal growth of the print media in view of the liberalization of the government policies? 1. Press & Registration of Books Act, (PRB Act) 1867 2. The Delivery Of Books ‘And Newspapers’ (Public Libraries) Act, 1954 3. Indian Press (Emergency Powers ) Act 1931 4. none Ans: 1. 22: 2 numbers differ by 5.If their product is 336,then the sum of the 2 numbers is: 1. 21 2. 51 3. 28 4. 37 Ans: 4. 23: Which number is the odd one out? 9678 4572 5261 3527 7768 1. 7768 2. 3527 3. 4572 4. 9678 5. 5261 Ans: 2. 24: Which one among the following has the largest shipyard in India 1. Kolkata 2. Kochi 3. Mumbai 4. Visakhapatnam Ans: 2. 25: If x=y=2z and xyz=256 then what is the value of x? 1. 8 2. 3 3. 5 4. 6 Ans: 1. 26: A radio when sold at a certain price gives a gain of 20%. What will be the gain percent, if sold for thrice the price? 1. 280 2. 270 3. 290 4. 260 Ans: 4. 27: x% of y is y% of ? 1. x/y 2. 2y 3. x 4. can’t be determined Ans: 3. 28: If the value of x lies between 0 & 1 which of the following is the largest? 1. x 2. x2 3. -x 4. 1/x Ans: 4. 29: The tutor of Alexander the great was 1. Darius 2. Cyrus 3. Socrates 4. Aristotle Ans: 4. 30: Thirty men take 20 days to complete a job working 9 hours a day. How many hour a day should 40 men work to complete the job? 1. 8 hrs 2. 71/2 hrs 3. 7 hrs 4. 9 hrs Ans: 2. 31: Goitre caused by the deficiency of ……… 1. Vitamin D 2. Iron 3. VItamin A 4. Iodine Ans: 4. 32: Who invented Napier’s Bones 1. John Napier 2. William Oughtred 3. Charles Babbage 4. Napier Bone Ans: 1. 33: The mass number of a nucleus is 1. Always less than its atomic number 2. Always more than its atomic number 3. Sometimes more than and sometimes equal to its atomic number 4. None of the above Ans: 3. 34: A and B can do a piece of work in 45 days and 40 days respectively. They began to do the work together but A leaves after some days and then B completed the remaining work n 23 days. The number of days after which A left the work was 1. 9 2. 11 3. 12 4. 15 5. 16 Ans: 1. 35: Sam and Mala have a conversation. Sam says I am certainly not over 40 Mala Says I am 38 and you are at least 5 years older than me · Now Sam says you are at least 39 all the statements by the two are false. How old are they really? 1. Mala = 38 yrs, Sam =31 yrs. 2. Mala = 38 yrs, Sam = 41 yrs 3. Mala = 31 yrs, Sam = 41 yrs. 4. Mala = 45yrs, Sam = 41 yrs Ans: 2. 36: What is the code name for Windows Vista? 1. Longhorn 2. Longhund 3. Stackspray 4. Pearl Ans: 1. 37: On sports day, if 30 children were made to stand in a column, 16 columns could be formed. If 24 children were made to stand in a column, how many columns could be formed? 1. 20 2. 30 3. 40 4. 50 Ans: 1. 38: The probability that a man will be alive for 25 years is 3/5 and the probability that his wife will be alive for 25 years is 2/3. Find the probability that only the man will be alive for 25 years. 1. 2/5 2. 1/5 3. 3/5 4. 4/5 Ans: 2. 39: In a single throw of a dice, what is the probability of getting a number greater than 4? 1. 1/2 2. 2/3 3. 1/4 4. 1/3 Ans: 4. 40: If every alternative letter starting from B of the English alphabet is written in small letter, rest all are written in capital letters, how the month “September” be written. (1) SeptEMbEr (2) SEpTeMBEr (3) SeptembeR (4) SepteMber (5) None of the above 1. (1) 2. (2) 3. (3) 4. (5) 5. (4) Ans: 4. 41: After allowing a discount of 11.11% ,a trader still makes a gain of 14.28 % .at how many precent above the cost price does he mark his goods? 1. 28.56% 2. 35% 3. 22.22% 4. None of these Ans: 1. 42: Pipe A can fill in 20 minutes and Pipe B in 30 mins and Pipe C can empty the same in 40 mins.If all of them work together, find the time taken to fill the tank 1. 17 1/7 mins 2. 20 mins 3. none 4. 50 mins Ans: 1. 43: There are 3 triplet brothers. They look identical. The oldest is John, he always tells the truth. The second is Jack, he always tells a lie. The third is Joe, he either tells the truth or a lie. Jimmie Dean went to visit them one day. He was wondering who was who. So he asked each person a question. He asked the one who was sitting on the left: “Who is the guy sitting in the middle?”. The answer was “He is John.” He asked the one who was sitting in the middle: “What is your name?”. The answer was “I am Joe.” He asked the one who was sitting on the right: “What is the guy sitting in the middle?”. The answer was “He is Jack.” Jimmie Dean got really confused. Basically, he asked 3 same questions, but he got 3 different answers. which is not true? 1. left most is joe 2. middle is jack 3. right is john 4. middle is john Ans: 4. 44: A / B = C; C > D then 1. A is always greater than D 2. C is always greater than D 3. B is always less than D 4. none Ans: 1. 45: Consider the following statements: 1. The Administrative Reforms Commission (ARC) had recommended that the Department of Personnel of a State should be put under the charge of the Chief Secretary of the State. 2. Chief Secretary of a State is not involved in any manner in the promotion of State Civil officers to the All-India Services. Which of the statements given above is/are correct? 1. Only 1 2. Only 2 3. Both 1 and 2 4. Neither 1 nor 2 Ans: 1. 46: The population of a town was 1,60,000 three years ago. If it increased by 3%, 2.5% and 5% respectively in the last three years, then the present population of the town is : 1. 1,77,000 2. 1,77,366 3. 1,77,461 4. 1,77,596 Ans: 2. 47: What is the population of India ? 1. 98 crores 2. More than 2 billion 3. More than 1 billion 4. Less than 96 crores 5. 96 crores Ans: 3. 48: Some green are blue. No blue are white. 1. Some green are white 2. No white are green 3. No green are white 4. None of the above Ans: 1. 49: What is the missing number in this series? 8 2 14 6 11 ? 14 6 18 12 1. 8 2. 6 3. 9 4. 11 Ans: 3. 50: Average age of students of an adult school is 40 years. 120 new students whose average age is 32 years joined the school. As a result the average age is decreased by 4 years. Find the number of students of the school after joining of the new students: 1. 1200 2. 120 3. 360 4. 240 Ans: 4. 51: On sports day,if 30 children were made to stand in a column,16 columns could be formed. If 24 children were made to stand in a column , how many columns could be formed? 1. 48 2. 20 3. 30 4. 16 5. 40 Ans: 2. 52: Which of the following numbers is divisible by 3? (i) 541326 (ii) 5967013 1. (ii) only 2. (i) only 3. (i) and (ii) both 4. (i) and (ii) none Ans: 2. 53: A square is divided into 9 identical smaller squares. Six identical balls are to be placed in these smaller squares such that each of the three rows gets at least one ball (one ball in one square only). In how many different ways can this be done? 1. 81 2. 91 3. 41 4. 51 Ans: 1. 54: A man owns 2/3 of the market research beauro business and sells 3/4 of his shares for Rs.75000. What is the value of Business 1. 150000 2. 13000 3. 240000 4. 34000 Ans: 1. 55: 1,2,6,24,_? 1. 111 2. 151 3. 120 4. 125 Ans: 3. 56: The cost of 16 packets of salt,each weighing 900 grams is Rs.28.What will be the cost of 27 packets ,if each packet weighs 1Kg? 1. Rs.52.50 2. Rs.56 3. Rs.58.50 4. Rs.64.75 Ans: 1. 57: Ronald and Michelle have two children. The probability that the first child is a girl, is 50%. The probability that the second child is a girl, is also 50%. Ronald and Michelle tell you that they have a daughter. What is the probability that their other child is also a girl? 1. 1/2 2. 1/3 3. 1/4 4. 1/5 Ans: 2. 58: Find the value of (21/4-1)( 23/4 +21/2+21/4+1) 1. 1 2. 2 3. 3 Ans: 1. 59: The product of two fractions is 14/15 and their quotient is 35/24. the greater fraction is 1. 4/5 2. 7/6 3. 7/5 4. 7/4 Ans: 1. 60: 500 men are arranged in an array of 10 rows and 50 columns according to their heights. Tallest among each row of all are asked to fall out. And the shortest among them is A. Similarly after resuming that to their original podsitions that the shortest among each column are asked to fall out. And the tallest among them is B . Now who is taller among A and B ? 1. A 2. B 3. Both are of same height Ans: 1. 61: Choose the pair of numbers which comes next 75 65 85 55 45 85 35 1. 25 15 2. 25 85 3. 35 25 4. 35 85 5. 25 75 Ans: 2. 62: A three digit number consists of 9,5 and one more number. When these digits are reversed and then subtracted from the original number the answer yielded will be consisting of the same digits arranged yet in a different order. What is the other digit? 1. 1 2. 2 3. 3 4. 4 Ans: 4. 63 : ATP stands for: 1. Adenine triphosphate 2. Adenosine triphosphate 3. Adenosine Diphosphate 4. Adenosine tetraphosphate Ans: 2. 64: Veselin Tapolev who became the World Champion recently, is associated with which of the following games/sports ? 1. Chess 2. Golf 3. Snooker 4. Badminton 5. None of these Ans: 1. 65: A piece of cloth cost Rs 35. if the length of the piece would have been 4m longer and each meter cost Re 1 less , the cost would have remained unchanged. how long is the piece? 1. 10 2. 11 3. 12 Ans: 1. 66: In a journey of 15 miles two third distance was travelled with 40 mph and remaining with 60 mph.How muvh time the journey takes 1. 40 min 2. 30 min 3. 120 min 4. 20 min Ans: 4. 67: Solid cube of 6 * 6 * 6. This cube is cut into to 216 small cubes. (1 * 1 * 1).the big cube is painted in all its faces. Then how many of cubes are painted at least 2 sides. 1. 56 2. 45 3. 23 4. 28 Ans: 1. 68: Find the average of first 40 natural numbers. 1. 40 2. 35 3. 30.6 4. 20.5 5. None of these Ans: 4. 69: 1, 5, 14, 30, ?, 91 1. 45 2. 55 3. 60 4. 70 5. None of these Ans: 2. 70: There is a shortage of tubelights, bulbs and fans in a village – Gurgaon. It is found that a) All houses do not have either tubelight or bulb or fan. b) Exactly 19% of houses do not have just one of these. c) Atleast 67% of houses do not have tubelights. d) Atleast 83% of houses do not have bulbs. e) Atleast 73% of houses do not have fans. 1. 42 % 2. 46 % 3. 50 % 4. 54 % 5. 57 % Ans: 1. 71: If 9 engines consume 24 metric tonnes of coal, when each is working 8 hours a day; how much coal will be required for 8 engines, each running 13 hours a day, it being given that 3 engines of the former type consume as much as 4 engines of latter type. 1. 22 metric tonnes. 2. 27 metric tonnes. 3. 26 metric tonnes. 4. 25 metric tonnes. Ans: 3. 72: To 15 lts of water containing 20% alcohol, we add 5 lts of pure water. What is % alcohol. 1. 20% 2. 34% 3. 15% 4. 14% Ans: 3. 73 : In page preview mode: 1. You can see all pages of your document 2. You can only see the page you are currently working 3. Satyam BPO Services 4. You can only see pages that do not contain graphics Ans: 4. 74 : A house wife saved Rs. 2.50 in buying an item on sale .If she spent Rs.25 for the item ,approximately how much percent she saved in the transaction ? 1. 8% 2. 9% 3. 10% 4. 11% Ans: 2. 75: I have trouble _____. 1. to remember my password 2. to remembering my password 3. remember my password 4. remembering my password Ans: 4. 76: Superheroes Liza and Tamar leave the same camp and run in opposite directions. Liza runs 1 mile per second (mps) and Tamar runs 2 mps. How far apart are they in miles after 1 hour? 1. 10800 mile 2. 19008 mile 3. 12300 mile 4. 14000 mile Ans: 1. 77: A = 5, B = 0, C = 2, D = 10, E = 2. What is then AB + EE – (ED)powerB + (AC)powerE = ? 1. 113 2. 103 3. 93 4. 111 Ans: 2. 78: A man can row upstream at 8 kmph and downstream at 13 kmph.The speed of the stream is? 1. 2.5 kmph 2. 4.2 kmph 3. 5 kmph 4. 10.5 kmph Ans: 1. 79: Find what is the next letter. Please try to find. O,T,T,F,F,S,S,E,N,_ What is that letter? 1. B 2. S 3. Q 4. T 5. O Ans: 4. 80: There are 3 societies A, B, C. A lent cars to B and C as many as they had Already. After some time B gave as many tractors to A and C as many as they have. After sometime c did the same thing. At the end of this transaction each one of them had 24. Find the cars each originally had. 1. A had 21 cars, B had 39 cars & C had 12 cars 2. A had 39 cars, B had 39 cars & C had 12 cars 3. A had 39 cars, B had 21 cars & C had 19 cars 4. A had 39 cars, B had 21 cars & C had 12 cars Ans: 4. 81: A papaya tree was planted 2 years ago. It increases at the rate of 20% every year. If at present, the height of the tree is 540 cm, what was it when the tree was planted? 1. 432 cm 2. 324 cm 3. 375 cm 4. 400 cm Ans: 3. 82: A boy has Rs 2. He wins or loses Re 1 at a time If he wins he gets Re 1 and if he loses the game he loses Re 1. He can loose only 5 times. He is out of the game if he earns Rs 5. Find the number of ways in which this is possible? 1. 14 2. 23 3. 16 4. 12 5. 10 Ans: 3. 83: Five racing drivers, Alan, Bob, Chris, Don, and Eugene, enter into a contest that consists of 6 races. The results of all six races are listed below: Bob always finishes ahead of Chris. Alan finishes either first or last. Eugene finishes either first or last. There are no ties in any race. Every driver finishes each race. In each race, two points are awarded for a fifth place finish, four points for fourth, six points for third, eight points for second, and ten points for first. If Frank enters the third race and finishes behind Chris and Don, which of the following must be true of that race? 1. Eugene finishes first. 2. Alan finishes sixth. 3. Don finishes second. 4. Frank finishes fifth. 5. Chris finishes third. Ans: 4. 84: A is twice as good a workman as B and together they finish a piece of work in 18 days.In how many days will A alone finish the work? 1. 27 2. 26 3. 25 4. 24 Ans: 1. 85: Daal is now being sold at Rs. 20 a kg. During last month its rate was Rs. 16 per kg. By how much percent should a family reduce its consumption so as to keep the expenditure fixed? 1. 20 % 2. 40 % 3. 3% 4. 2% Ans: 1. 86: The sum of 5 successive odd numbers is 1075. What is the largest of these numbers? 1. 215 2. 223 3. 219 4. 217 Ans: 3. 87: A man sells two buffaloes for Rs. 7,820 each. On one he gains 15% and on the other, he loses 15%. His total gain or loss in the transaction is 1. 2.5% gain 2. 2.25% loss 3. 2% loss 4. 5% loss 5. None of these Ans: 2. 88: One ship goes along the stream direction 28 km and in opposite direction 13 km in 5 hrs for each direction.What is the velocity of stream? 1. 1.5 kmph 2. 2.5 kmph 3. 1.8 kmph 4. 2 kmph Ans: 1. 89: Which one of the words given below is different from others? 1. Orange 2. Grape 3. Apricot 4. Raspberry 5. Mango Ans: 3. 90: Complete the series: 5, 20, 24, 6, 2, 8, ? 1. 12 2. 32 3. 34 4. 36 Ans: 1. 91: A can have a piece of work done in 8 days, B can work three times faster than the A, C can work five times faster than A. How many days will they take to do the work together 1. 3 days 2. 8/9 days 3. 4 days 4. None of the above Ans: 2. 92: 7 Pink, 5 Black, 11 Yellow balls are there. Minimum no. atleast to get one black and yellow ball 1. 17 2. 13 3. 15 4. 19 Ans: 1. 93: (1/10)18 – (1/10)20 = ? 1. 99/1020 2. 99/10 3. 0.9 4. none of these Ans: 1. 94: Three friends divided some bullets equally. After all of them shot 4 bullets the total number of bullets remaining is equal to the bullets each had after division. Find the original number divided? 1. 18 2. 20 3. 54 4. 8 Ans: 1. 95: A sum of Rs. 427 is to be divided among A, B and C in such a way that 3 times A’s share, 4 times B’s share and 7 times C’s share are all equal. The share of C is 1. Rs.84 2. Rs.76 3. Rs.98 4. RS.34 Ans: 1. 96: There are 20 poles with a constant distance between each pole. A car takes 24 second to reach the 12th pole.How much will it take to reach the last pole. 1. 41.45 seconds 2. 40.45 seconds 3. 42.45 seconds 4. 41.00 seconds Ans: 1. 97: An emergency vehicle travels 10 miles at a speed of 50 miles per hour. How fast must the vehicle travel on the return trip if the round-trip travel time is to be 20 minutes? 1. 72 miles per hour 2. 75 miles per hour 3. 65 miles per hour 4. 78 miles per hour Ans: 2. 98: 12% of 580 + ? = 94 1. 24.4 2. 34.4 3. 54.4 4. 65.4 Ans: 1. 99: There is a certain relation between two given words on one side of : : and one word is given on another side of : : while another word is to be found from the given alternatives, having the same relation with this word as the given pair has. Select the best alternative. Horse : Jockey : : Car : ? 1. Mechanic 2. Chauffeur 3. Steering 4. Brake Ans: 2. 100: Which of the following numbers should be added to 11158 to make it exactly divisible by 77? 1. 9 2. 8 3. 7 4. 5 Ans: 3. FCI aptitude and Reasoning questions for practice, FCI model questions for practice, FCI General ability, General awareness, FCI whole test paper with answers to practice ,FCI model questions, FCI free solved sample placement papers, FCI recruitment examination questions with answers for practice, FCI previous years solved question papers Directions (questions 1 to 5): Read the following information and answer the questions given below. (i) A, B, C, D, E, F and G are seven members standing in a row (not necessarily in the same order) facing north. (ii) C and B have as many members between them as G and C have between them. (iii) D, who is 3rd from the extreme left, is 3rd to the left of E. (iv) A and D are neighbors and F and C are neighbors. 1. Which of the following statements may be false? (a) A is the 3rd to the left of F (b) D is 3rd to the left of E (c) F is 3rd the right of A (d) B is 3rd to the left of C (e) None of these Ans (d) 2. Which of the following statements is true? (a) C and E are neighbors (b) E is to the immediate left of F (c) C is to the immediate left of D (d) A is to the immediate left of D (e) None of these Ans (d) 3. Who is at the extreme right? (a) Data inadequate (b) G (c) B (d) E (e) None of these Ans (a) 4. Which of the following given two pairs of neighbors? (a) A, C and D, C (b) A, B and E, G (c) D, C and E, F (d) C, F and C, E (e) None of these Ans (c) 5. What is the position of F? (a) Immediate to the right of E. (b) Third to the left of G (c) Immediate to the left of C (d) 5th to the right of B (e) None of these Ans (e) 6. Four of the following five groups of letters are alike in some way while one is different. Find out which one is different. (a) ISLOJ (b) LUOQM (c) AKDGB (d) FPILG (e) NXQTO Ans (b) 7. In the sequence given below the sum of the two digits which immediately precede the digit ’4' exceeds the sum of the two digits which immediately follow the digit 4 and sum of the two digits which immediately follow the digit 6 exceeds the sum of the two digits which immediately precede the digit 6. How many such 4's and 6's together are there? 5 4 4 6 2 6 3 5 6 4 2 8 4 3 7 6 6 4 8 3 (a) 4 (b) 6 (c) 3 (d) 5 (e) None of these Ans (a) 8. Below are given six three-digit numbers. The digits comprise of numeric and letters. The letter indicates its serial order in the English alphabet. What will be the middle digit of the 4th number when the numbers are arranged in the descending order after interchanging numeric in each number without altering the place of letter in the number? 19F, 2H9, 98B, D76, 7A6, 61E (a) 8 (b) 9 (c) 6 (d) 7 (e) None of these Ans (e) Directions (Q.9 & 10): In a code language any letter which is immediately after or before a vowel in the English alphabet is substituted by that vowel and any vowel i.e. A, E, I, O and U is substituted by the letter immediately following that vowel in the English alphabet. 9. How can the word FEVERISH be written in that code language? (a) EDVDRJSI (b) EFUFRHSI (c) EFUFRJSI (d) EDUFRJSI (e) None of these Ans (c) 10. How can the word CONFEDERATION be written in that code language? (a) CPOGFEDRBUTPO (b) CPOEFEFRBUJPO (c) CNOEFEFRBUHNO (d) CONFFDERATION (e) None of these Ans (b) 11. If each of the vowels i.e. A, E, I, O, & U along with the 3rd letter to its right in the alphabet are taken out and arranged one after the other in the same order followed by the remaining letters of the alphabet, which of the following will be 5th to the left of the 19th letter from the left in the new arrangement? A B C D E F G H I J K L M N O P Q R S T U V W X Y Z (a) G (b) H (c) J (d) W (e) none of these Ans (a) 12. How many pairs of letters are there in the word SPONTANEOUS which have number of letters between them in the word one less than the number of letters between them in English alphabet? (a) five (b) one (c) four (d) Two (e) Three Ans (a) 13. Four of the following five have similar relationship and hence form a group. Which one does not belong to the group? (a) BROTHER : DORVEHT (b) ENGLISH : GGNNSIJ (c) ANOTHER : CONVEHT (d) BETWEEN : DTEZEEP (e) HUSBAND : JSUDNAF Ans (d) 14. Which pair of the letters in the word BEAUTIFUL has the same relationship between its letters with respect to their position in the English alphabet as the pair EA in that word has between its letters? (a) IB (b) LF (c) IE (d) FL (e) TL Ans (c) 15. Which of the following relates to FLOWER in the same way as RTERBN relates to SECTOR? (a) RWLGPF (b) EOFKUQ (c) EOFMXS (d) RWLEPD (e) RWLEND Ans (e) 16. If arranged properly which one of the following is not a river in India. (a) ANUMAY (b) IHSOK (c) TABROC (d) ADAMRAN (e)None of these Ans (c) It is CORBAT. It is a National Park 17. If the following words are arranged in a natural sequence, which word would be the first. (a) Night (b) Negative (c) Neighbour (d) Near (e) None of these Ans (d) 18. In an office having 55 employees men out number the women by 15. How many women are working in that office. (a) 30 (b) 20 (c) 40 (d) 50 (e) None of these Ans (b) 19. Reaching a place of appointment on Friday I found that I was two days earlier than the scheduled day. If I had reached on the following Wednesday how many days late would I have been. (a) one day (b) two days (c) three days (d) four days (e) None of these Ans (c) 20. Reena traveled from point A to a distance of 10 feet east at point B. She then turned right and walked 3 feet. Again she turned right and walked 14 feet. How far is she from the starting point. (a) 4 feet (b) 5 feet (c) 24 feet (d) 25 feet (e) None of these Ans (a) Directions (Questions 21 to 25):- One of the words given in the alternatives cannot be formed by using the letters given in the question. Find out that word. 21. ENCOURAGE (a) ANGER (b) GREEN (c) USAGE (d) COURAGE (d) None of these Ans (c) There is no alphabet ‘S’ in the question 22. CONCENTRATE (a) CENTRE (b) CONCERN (c) TREAT (d) REASON (e) None of these Ans (d) 23. INTRANSIGENT (a) STAIN (b) TRAIN (c) RESIGN (d) TRACE (e) None of these Ans (d) 24. MAGNETIC (a) MENACE (b) GAIN (c) NAME (d) GAME (e) None of these Ans (a) 25. ENTHUSIASM (a) MITE (b) ASSIST (c) ATHENS (d) SENT (e) None of these Ans (b) Directions (questions 26 to30 ):Read the following information and answer the questions given below. (i) A, B, C, D, E, F and G are seven members standing in a row (not necessarily in the same order) facing north. (ii) C and B have as many members between them as G and C have between them. (iii) D, who is 3rd from the extreme left, is 3rd to the left of E. (iv) A and D are neighbors and F and C are neighbors. 26. Which of the following statements may be false? (a) A is the 3rd to the left of F (b) D is 3rd to the left of E (c) F is 3rd the right of A (d) B is 3rd to the left of C (e) None of these Ans (d) 27. Which of the following statements is true? (a) C and E are neighbors (b) E is to the immediate left of F (c) C is to the immediate left of D (d) A is to the immediate left of D (e) None of these Ans (d) 28. Who is at the extreme right? (a) Data inadequate (b) G (c) B (d) E (e) None of these Ans (a) 29. Which of the following given two pairs of neighbors? (a) A, C and D, C (b) A, B and E, G (c) D, C and E, F (d) C, F and C, E (e) None of these Ans (c) 30. What is the position of F? (a) Immediate to the right of E. (b) Third to the left of G (c) Immediate to the left of C (d) 5th to the right of B (e) None of these Ans (e) 31 Four of the following five groups of letters are alike in some way while one is different. Find out which one is different. (a) ISLOJ (b) LUOQM (c) AKDGB (d) FPILG (e) NXQTO Ans (b) 32 . In the sequence given below the sum of the two digits which immediately precede the digit ‘4' exceeds the sum of the two digits which immediately follow the digit 4 and sum of the two digits which immediately follow the digit 6 exceeds the sum of the two digits which immediately precede the digit 6. How many such 4’s and 6’s together are there? 5 4 4 6 2 6 3 5 6 4 2 8 4 3 7 6 6 4 8 3 (a) 4 (b) 6 (c) 3 (d) 5 (e) None of these Ans (a) 33. Below are given six three-digit numbers. The digits comprise of numeric and letters. The letter indicates its serial order in the English alphabet. What will be the middle digit of the 4th number when the numbers are arranged in the descending order after interchanging numeric in each number without altering the place of letter in the number? 19F, 2H9, 98B, D76, 7A6, 61E (a) 8 (b) 9 (c) 6 (d) 7 (e) None of these Ans (e) Directions (Q.34 & 35): In a code language any letter which is immediately after or before a vowel in the English alphabet is substituted by that vowel and any vowel i.e. A, E, I, O and U is substituted by the letter immediately following that vowel in the English alphabet. 34 . How can the word FEVERISH be written in that code language? (a) EDVDRJSI (b) EFUFRHSI (c) EFUFRJSI (d) EDUFRJSI (e) None of these Ans (c) 35 . How can the word CONFEDERATION be written in that code language? (a) CPOGFEDRBUTPO (b) CPOEFEFRBUJPO (c) CNOEFEFRBUHNO (d) CONFFDERATION (e) None of these Ans (b) 36 . If each of the vowels i.e. A, E, I, O, & U along with the 3rd letter to its right in the alphabet are taken out and arranged one after the other in the same order followed by the remaining letters of the alphabet, which of the following will be 5th to the left of the 19th letter from the left in the new arrangement? A B C D E F G H I J K L M N O P Q R S T U V W X Y Z (a) G (b) H (c) J (d) W (e) none of these Ans (a) 37 . How many pairs of letters are there in the word SPONTANEOUS which have number of letters between them in the word one less than the number of letters between them in English alphabet? (a) five (b) one (c) four (d) Two (e) Three Ans (a) 38 . Four of the following five have similar relationship and hence form a group. Which one does not belong to the group? (a) BROTHER : DORVEHT (b) ENGLISH : GGNNSIJ (c) ANOTHER : CONVEHT (d) BETWEEN : DTEZEEP (e) HUSBAND : JSUDNAF Ans (d) 39 . Which pair of the letters in the word BEAUTIFUL has the same relationship between its letters with respect to their position in the English alphabet as the pair EA in that word has between its letters? (a) IB (b) LF (c) IE (d) FL (e) TL Ans (c) 40 . Which of the following relates to FLOWER in the same way as RTERBN relates to SECTOR? (a) RWLGPF (b) EOFKUQ (c) EOFMXS (d) RWLEPD (e) RWLEND Ans (e) 41 . If arranged properly which one of the following is not a river in India. (a) ANUMAY (b) IHSOK (c) TABROC (d) ADAMRAN (e)None of these Ans (c) It is CORBAT. It is a National Park 42 . If the following words are arranged in a natural sequence, which word would be the first. (a) Night (b) Negative (c) Neighbour (d) Near (e) None of these Ans (d) 43. In an office having 55 employees men out number the women by 15. How many women are working in that office. (a) 30 (b) 20 (c) 40 (d) 50 (e) None of these Ans (b) 44. Reaching a place of appointment on Friday I found that I was two days earlier than the scheduled day. If I had reached on the following Wednesday how many days late would I have been. (a) one day (b) two days (c) three days (d) four days (e) None of these Ans (c) 45 . Reena traveled from point A to a distance of 10 feet east at point B. She then turned right and walked 3 feet. Again she turned right and walked 14 feet. How far is she from the starting point. (a) 4 feet (b) 5 feet (c) 24 feet (d) 25 feet (e) None of these Ans (a) Directions (Questions 46 to 25):- One of the words given in the alternatives cannot be formed by using the letters given in the question. Find out that word. 46 ENCOURAGE (a) ANGER (b) GREEN (c) USAGE (d) COURAGE (d) None of these Ans (c) There is no alphabet ‘S’ in the question 47. CONCENTRATE (a) CENTRE (b) CONCERN (c) TREAT (d) REASON (e) None of these Ans (d) 48. INTRANSIGENT (a) STAIN (b) TRAIN (c) RESIGN (d) TRACE (e) None of these Ans (d) 49. MAGNETIC (a) MENACE (b) GAIN (c) NAME (d) GAME (e) None of these Ans (a) 50 ENTHUSIASM (a) MITE (b) ASSIST (c) ATHENS (d) SENT (e) None of these Ans (b) FCI current affairs questions and answer 1. This Sikkim high court Chief facing impeachment on charges of corruption and judicial misconduct resigned on 29 July 2011. Identify him. a. Justice PD Dinakaran b. Justice V Ramaswamy c. Justice Soumitra Sen d. Justice P C Joshi Answer: (a) 2. US credit rating was downgraded to__ from AAA by Standard & Poor’s. a) AA+ b) A c) A+ d) B+ Answer: (a) 3. Which Indian state on 11 August 2011 flagged off the first-ever consignment of certified organic food to be exported to the United States? a. Andhra Pradesh b. Tamil Nadu c. Rajasthan d. Arunachal Pradesh Answer: (c) 4. Tory Tan won the presidential election in Singapore to become the country’s___ President. a) Sixth b) Seventh c) Fifth d) Eighth Answer: (a) 5. The European Union on 2 September 2011 announced a ban on oil imports from Syria. The European Union is an economic and political union of __member states. a) 27 b) 22 c) 20 d) 16 Answer:(a) 6. Leila Lopes became the first woman from her country to be crowned Miss Universe on 12 September 2011 in Sau Paulo, Brazil. Which country does she belong to? a. Ethupia b. Angola c. Ghana d. Nigeria Answer: (b) 7. Which one of the following nations declared the Indian Mujahideen terror group as a global terrorist organization? a) USA b) Pakistan c) China d) Australia Answer: (a) 8. Wangari Maathai an environmentalist & first African woman to win a Nobel Peace Prize died on 25 September 2011 in Nairobi. Which African country did she belong to? a. Ghana b. Kenya c. Euthopia d. South Africa Answer: (b) 9. World Tourism Day (WTD) is observed anually on 27 September. What was the theme for WTD 2011? a. Tourism – Linking Cultures b. Tourism & Biodiversity c. Tourism opens doors for women d. Sport and tourism Answer: (a) 10. IBSA summit concluded in Pretoria, South Africa. What is the full form of IBSA? a) India Brazil South America b) India Brazil South Africa c) India Brazil Saudi Arabia d) India Bangladesh South Africa Answer: (b) 11. Who won the 2011 man Booker Prize for his short novel The Sense of an Ending? a. Carol Birch b. Patrick deWitt c. Julian Barnes d. AD Miller Answer: (c) 12. Name the Irish poet and human rights activist who was named Ireland’s ninth president on 30 October 2011 after winning 57% of votes in the final count of the presidential election held on 27 October 2011. a. Michael D Higgins b. Mary McAleese c. Douglas Hyde d. Mary Robinson Answer: (a) 13. IAEA Report stated that Iran carried out Nuclear Weapons-Related Activities. Where the report was released? a) New Delhi b) New York c) London d) Vienna Answer: (d) 14. Recently, which one of the following nations was suspended by the Arab league? a) Syria b) Lebanon c) Saudi Arabia d) Jordan Answer: (a) 15. The Arab League on 27 November 2011 imposed a set of sanctions against Syria for its failure to comply with the League mediated peace plan to end violence in the country. When was Arab league set up? a) 22 March 1945 b) 21 august 1945 c) 22 March 1946 d) 22 September 1950 Answer: (a) 16. Memogate scandal is related to which one of the following countries? a) Pakistan b) Sri Lanka c) China d) India Answer: (a) 17. India and Pakistan on 25 January 2012 agreed to transit fee formula for Tapi gas pipeline project in New Delhi. Tasi gas pipeline project comprises a) India, Pakistan and Tajikistan b) India, Pakistan and Russia c) India, Pakistan and Iran d) India, Pakistan and Afghanistan Answer: (a) 18. A massive avalanche claimed the life of more than hundred Pakistani soldiers on 9 April 2012. The incident occurred at which of the following places? a. Eastern Karakoram mountain range b. Southern Karakoram mountain range c. Northern Karakoram mountain range d. Southern Karakoram mountain range Answer: (a) 19. Who did the World Bank appoint as its new president on 16 April 2012? a. Jim Yong Kim b. Robert Zoellick c. Ngozi Okonjo-Iweala d. Lumen Darcy Answer: (a) 20. Amadou Toumani Toure, who was ousted from his position following a military coup, was the president of _ _ _ _ _ _ a. Tunisia b. Azerbaijan c. Mali d. Maldives Answer: (c) 21. A team of Scientists led by Takehiko at Yokohama City University grew sperms in a test-tube. Consider the following statements regarding the scientific break through. (i) Sperms were grown in a Test-tube for the first time. (ii) The Scientific breakthrough could help millions of infertile men to attain parenthood. Now choose the right option: (a) Both i and ii statements are correct (b) Only i is correct (c) Only ii is correct (d) Both I and ii statements are wrong Answer: (a) Both i and ii statements are correct 22. Scientists, at the Wellcome Trust Sanger Institute UK, for the first time identified genetic mutations that lead to the development of acute myeloid leukaemia. Acute myeloid leukaemia is a cancer of the__. (a) White Blood Cells (WBC) (b) Red blood Cells (RBC) (c) Liver Cells (d) Stem Cells Answer: (a) White Blood Cells (WBC) 23. Scientists recently developed a computer that could be controlled by the power of thought. It would help people unable to speak or move. What is the name of the technology which they used to develop this kind of computer? (a) electrocortiography (b) symbian C++ (c) java (d) flash lite Answer: (a) electrocartiography 24. British Scientists at Edinburgh University, in the second week of April 2011, created human kidneys from stem cells in a break through which could lead to transplant patients growing their organs. Consider the following statements regarding Kidneys in human body. (i) A person’s kidneys process about 200 quarts of blood to sift out about 2 quarts of waste products and extra water. (ii) The word ‘’renal’ refers to the Lungs in Human Body. Choose the right option: (a) Both i and ii statements are correct. (b) Only i is correct. (c) Only ii is correct. (d) Neither i nor ii is correct. Answer: (b) Only i is correct. The word renal refers to the Lungs in Human body. 25. GSAT-8 was launched on 21 May 2011 by Ariane-5 from Kourou, French Guiana. Consider the following statements on GSAT-8. (i) Its weight is 3100 kg. (ii) It is the largest and heaviest satellite built by the ISRO (Indian Space Research Organisation). Choose the right option. (a) Both i and ii are correct. (b) Only i is correct. (c) Only ii is correct. (d) Neither i nor ii is correct. Answer: (a) Both i and ii are correct 26. India registered a __ per cent increase in tiger population in 2010, according to a report, Status of tigers, co-predators and prey in India-2010, released on 28 July 2011. (a) 20 (b) 30 (c) 15 (d) 12 Answer: (a) 20 27. The world’s first double-leg transplant was performed by doctors in which one of the following countries? (a) Spain (b) UK (c) USA (d) India Answer: (a) Spain 28. The WHO (World Health Organisation)’s International Agency for Research on Cancer (IARC) stated in its study that radiation from cellphone handsets is possibly carcinogenic to humans and can cause glioma. Glioma is a type of brain cancer. (a) Brain Cancer (b) Blood Cancer (c) Lung Cancer (d) Liver Cancer Answer: (a) Brain Cancer 29. China Successfully launched its first unmanned space laboratory. What is the name of this space laboratory? (a) Tiangong-1 (b) MEASAT-1 (c) ZY-2 (d) CH 726 Answer: (a) Tiangong-1 30. What is the name of the gene that can reduce the length of time people sleep? (a) ABCC9 (b) ACCB9 (c) ABCC 6 (d) ABBB9 Answer: (a) ABCC9 31. In Japan, rice with radiation levels exceeding the country’s safety levels was discovered for the first time since the nuclear disaster at Fukushima in early 2011. The acronym LASER stands for____. (a) Light Amplification by Stimulated Emission of Radiation (b) Light Amplification by Stimulated Emission of Rays (c) Light Amplified by Stimulated Emission of Radiation (d) Light and Sound emitted by Radiation Answer: (a) Light Amplification by Stimulated Emission of Radiation 32. Union Environment Ministry approved the Lavasa township project in Pune. Consider the following statements: (i) Lavasa is a private, planned city being built near Pune. (ii) The project is being developed by HCC ( Hindustan Construction Company). Choose the right option: (a) Both i and ii are correct. (b) Only i is correct. (c) Only ii is correct. (d) Neither i nor ii is correct. Answer: (a) Both i and ii are correct 33. Arctic ozone hole was detected at record level. The journal nature reported. The ozone layer blocks _____from the Sun. (a) Infra-Red rays (b) Ultraviolet-B rays (c) Ultraviolet-C rays (d) X-rays Answer: (b) Ultraviolet-B rays 34. India scientists succeeded in decoding the genome of the arhar dal, which is also known as _____. (a) Pigeon Pea (b) Black-eyed Pea (c) Sweet Pea (d) Split Pea Answer: (a) Pigeon Pea 35. NASA's Kepler mission discovered the first Earth-size planets orbiting a sun-like star outside our solar system. These planets are called ___ and____. (a) Kepler-20e; Kepler-20f (b) Kepler-20a; Kepler-20b (c) Kepler-20c; Kepler-20d (d) Kepler-20m; Kepler-20n Answer: (a) Kepler-20e; Kepler-20f 36. Name the gene found by the neuroscientists, which could help in creating and altering memory. (a) Mpas4 (b) Npas4 (c) Npas3 (d) Mpas2 Answer: (b) Npas4 37. As per the report compiled by British-based firm Maplecroft, which one of the following countries is top greenhouse gas emitter? (a) India (b) USA (c) China (d) Japan Answer: (c) China 38. Scientists produced artificial human semen to help infertile men. Consider the following statements: (i) The scientists grew the sperm by enveloping the germ cells in a special compound called agar jelly. (ii) The artificial human semen could help infertile men father their own children. Choose the right option: (a) Both i and ii are correct. (b) Only i is correct. (c) Only ii is correct. (d) Neither i nor ii is correct. Answer: (a) Both i and ii are correct 39. India registered a __ per cent increase in tiger population in 2010, according to a report, Status of tigers, co-predators and prey in India-2010, released on 28 July 2011. (a) 20 (b) 30 (c) 15 (d) 12 Answer: (a) 20 40. A Japanese supercomputer, K Computer built by Fujitsu Co. grabbed the title of world's best-performing machine thereby returning Japan to the top of the computer arms race for the first time in seven years. Which is the other name by which K Computer is known? (a) Earth Simulator (b) Riken (c) Jaguar (d) Tianhe-1A .41 A new strain of rice developed by the scientists, which can enhance the productivity in the soil that lacks a nutrient content. Name the nutrient, lack of which effects crop yield. (a) Sulphur (b) Oxygen (c) Phosphorous (d) Carbon Answer: (c) Phosphorous 42. Name the two planets that came together in the Western sky on 15 August 2012 after sunset. (a) Mars and Saturn (b) Earth and Mars (c) Jupiter and Saturn (d) Venus and Earth Answer: (a) Mars and Saturn 43. The identified singing mice that use songs to communicate are found in which region? (a) The Himalayan terrain (b) The tropical cloud forests in the mountains of Costa Rica (c) On the volcanic plates of Pacific (d) Greenland Answer: (b) The tropical cloud forests in the mountains of Costa Rica 44. Name the NASA’s Robert that has recently touched the surface of Mars (a) Rover Curiosity (b) Gale Crater (c) Martian home (d) Calif Answer: (a) Rover Curiosity 45. Name the process that will be applied for transferring the gene of the newly developed Stained Rice to the modern variety of rice (a) Cross-Breeding (b) Genetic Engineering (c) Hybridization (d) None of these Answer: (a) Cross-Breeding The Government has appointed ______ as the next Chief of the Naval Staff. Ans: D. K. Joshi. Major producer of mulberry silk in India is ______. Ans: Karnataka. What does “PPP” represents? Ans: Private Public Partnership. World Day against Child Labour is celebrated on _____ June every year. Ans: 12th. Who has been appointed as Chairman of National Highways Authority of India? Ans: Rajinder Pal Singh. 2012 marks the _____ anniversary of the United Nations Environment programme and World Environment Day. Ans: 40th. In _____, Independent India conducted its first elections. Ans: 1952. When is National energy Conservation Day celebrated? Ans: 14th December. In which year Dadasaheb Phalke Award was instituted? Ans: 1969. Who has been appointed as the new Chief Election Commissioner of India? Ans: V.S. Sampath. World Telecommunication and Information Day, celebrated each year on _____ May. Ans: 17th. French Open 2012 was the _____ edition of French Open. Ans: 111th. Anti-Terrorism Day was observed on _______ May. Ans: 21st. Shanghai Cooperation Organization Summit 2012 concluded in ______ on 7 June 2012. Ans: Beijing. Which country successfully test-fired the indigenously developed Hatf-VII (Babur) cruise missile on 5 June 2012? Ans: Pakistan. Who assumed the charge as Chairman of the Competition Appellate Tribunal of India? Ans: Justice V.S. Sirpurkar. The second BRICS Health Ministers meeting will be hosted by India in ______. Ans: November, 2012. Who has elected as the President of the European Bank for Reconstruction and Development (EBRD)? Ans: Suma Chakrabarti. Who has elected as the new Director General of International Labour Organization? Ans: Briton Guy Ryder. Who took over as the 25th Chief of the Indian Army? Ans: Bikram Singh. Who has been re-elected to the commission on the Limits of the Continental Shelf for a five year term from 2012-17? Ans: Dr. S. Rajan. Who has been conferred with an honorary degree by New York University’s Polytechnic Institute? Ans: Sam Pitroda. Who is the President of Myanmar? Ans: Thein Sein. Donna Summer has died at the age of 63 in Florida. Who was she? Ans: Singer. At present who is the chief minister of Uttar Pradesh? Ans: Akhilesh Yadav. Which country has own the 21st Sultan Azlan Shah Cup hockey tournament in Malaysia? Ans: New Zealand. Dipika Kumari is a famous ______ player. Ans: Archery. Who won the prestigious Ondaatje Prize 2012 for his book “The Sly Company of People Who Care”? Ans: Rahul Bhattacharya. Who took over as France’s new Prime Minister on 16th May, 2012? Ans: Jean-Marc Ayrault. Indian Grand Master Vishwanathan Anand retained the World Chess Title for the record fifth time by defeating _______. Ans: Boris Gelfand. The Kabir Puraskar Scheme was introduced in _____. Ans: 1990. International Level Crossing Awareness Day is observed on ______. Ans: 7th June. India and which other country together accounted for a third of the deaths of pregnant women globally in 2010, as per latest UN data? Ans: Nigeria. The Union Cabinet approved allowance at the rate of ______ per month per head to the non-gazetted personnel of Andaman and Nicobar Police, recently. Ans: Rs 1697. India test fired successfully its indigenously-developed surface-to-air missile from the integrated Test Range. Name it. Ans: Akash. Which state government recently announced a ban on the manufacture and sale of gutka and pan masala, containing tobacco in the state with immediate effect? Ans: Kerala. India and Myanmar signed how many agreements, including a $ 500 million credit line for border area development and air services? Ans: 12. The Ministry of Statistics and Programme Implementation has announced a new scheme under the MPLADS. Name the scheme. Ans: One MP-One Idea. Which State Government’s plan to boost coastal tourism has received a shot in the arm as the Planning Commission has approved a 1,200-crore special grant for the project. Ans: Gujarat. The ______ meeting of the India-Kazakhstan Joint Working Group on Textiles took place on the 7-8th June, 2012 at Gurgaon. Ans: Third. A MoU has been signed between India and which other country during the visit of Dr. C.P. Joshi to that country? Ans: Canada. India and ______ have unveiled the Open Government Platform to foster transparency by making government data, documents, tools and processes publicly available. Ans: United States. Ministry of Youth Affairs & Sports has approved the proposal worth how much for financial assistance under the scheme of Urban Sports Infrastructure, recently? Ans: Rs 23 Crore. The Union Cabinet approved the proposal of the Ministry of Defence for mutual exchange of ______ of land between UT of Chandigarh and IAF for expansion of Civil Air Terminal at Chandigarh Airport. Ans: 2.50 acres. Which country has been ranked as the seventh top polluter mainly due to carbon emissions? Ans: Australia. What is the amount that German government has approved as contribution for Afghan Army, recently? Ans: USD 190 million. US President Barack Obama nominated ______ as the country’s first ambassador to Myanmar in 22 years. Ans: Derek Mitchell. NATO and Northrop Grumman Corporation signed a contract of what amount to set up a ground surveillance system? Ans: $ 1.7 billion. Which is India’s rank in the World Bank’s latest survey on global trade logistics? Ans: 46th. Which two Indian companies have been included in the ‘BrandZ list’ of the top 100 most valuable global brands 2012? Ans: ICICI Bank & Bharti Airtel. Which country has been voted the worst place for women among 19 economies? Ans: India. The Power Ministry has set a target of adding ______ capacity in the current financial year. Ans: 18,000 MW. IVRCL Ltd has bagged orders worth Rs 652.65 crore from the government of which state? Ans: Andhra Pradesh. Bharti Airtel’s customers will now be able to open no-frills savings account in which bank through their mobiles? Ans: Axis Bank. CCI has approved the proposal of Nippon Life to acquire how much percentage of stakes in RCAML? Ans: 26 percent. Demand of Gold in India has declined by how much percent during the first quarter of the current year? Ans: 29 percent. Etihad Airways signed a code-share agreement with Safi Airways of _____ for seat booking in each other airlines. Ans: Afghanistan. India’s food grain production is projected to grow by what percent in 2012-13. Ans: 0.8 percent. Beni Prasad Verma has released a report entitled ‘A Roadmap for Research & Development and Technology’ for Indian Industry. Who is he? Ans: Union Minister for Steel. India’s economic growth is likely to rise to _____ in 2013, according to the OECD. Ans: 7.7%. IT spending by Indian Enterprises is expected to grow to ______ according to research firm Gartner. Ans: Rs 1.91 lakh crore. Growth in developing East Asia and Pacific is slowing and the region is expected to post how much percent expansion this year, according to the World Bank report? Ans: 7.6 percent. The government has increased interest rates on SDS and State Provident Funds for the year 2012-13 to _______ from ______ recently. Ans: 8.8 percent, 8.6 percent. Bharti Airtel has acquired stakes of how much percent in Qualcomm’s broadband wireless access (BWA) business in India for USD 165 million? Ans: 49 percent. Which country has remained the world’s biggest producer of Magnesium since 1999? Ans: China. Which company has bagged a $6 million order to supply 50 vestibule busses to Bangladesh Road Transport Corporation? Ans: Ashok Leyland Ltd. The UN has predicted a growth rate of ______ rather than a ______ for India in 2012. Ans: 6.7%, 7.7%. Which company has supplied 100 ‘Falcon” busses to Ghana for $7.6 million? Ans: Ashok Leyland. The government announced _______ hike in support price of paddy to farmers at Rs 1,250 for the 2012-13 crop year (July-June). Ans: Rs 170/quintal. Maruti Suzuki India Ltd decided to get into a merger with which company? Ans: Suzuki Power-train India Ltd. The Union Government of India suspended VRS Natarajan. He was the head of which state-run defence enterprises? Ans: Bharat Earth Movers Limited. According to the figures released by CGA, India’s fiscal deficit eased to how much percent of GDP? Ans: 5.7 percent. For continuous technological up-gradation of export sectors, EPCG scheme has now been extended up to _______. Ans: 31st March, 2013. What is the full form of ‘ECB’? Ans: External Commercial Borrowing. How many new items are being added to Market Linked Focus Product Scheme? Ans: 46. How many new items are being added to Focus Product Scheme (FPS) list? Ans: 110. Who is the regulatory authority for Insurance business in India? Ans: IRDA. The erstwhile UTI bank is presently known as ______. Ans: Axis Bank. The term ‘deficit financing’ means the Government borrows the money from the _____. Ans: RBI. What does the letter “F” denote in ‘NBFCs’, a term seen very frequently in Banking world these days? Ans: Financial. What is the abbreviated name of the body/agency set up to boost foreign investments in India? Ans: FEMA. What is the full form of FDI? Ans: Foreign Direct Investment. FCI question paper FCI aptitude reasoning general studies history gk current affairs questions with answers, FCI previous years solved questions papers, FCI free solved sample placement papers, FCI model question papers, FCI these question are only sample placement papers not correct matching or original papers.FCI practice papers only,FCI reasoning general studies general ability questions FCI practice papers 1. Which of the following countries is not a member of the elite league of six nations that have nuclear-powered submarines? a. China b. India c. Germany d. UK 2. Who amongst the following has been awarded the prestigious 'Dadasaheb Phalke award' for the year 2012? a. Manna Dey b. Soumitra Chatterjee c. VK Murthy d. K Balachander 3. Tiger Woods' former swing coach, Hank Haney, has written a book about his time working with the golfer. The name of the book is: a. The Lord of the Rings b. Rules of Attraction c. Strong Motion d. The Big Miss 4. The Titanic, with 2,200 people on board, departed from which of the following British ports on 10 April 1912, for its fateful voyage from Europe to the US? a. Port of Southampton b. Port of Barrow c. Port of Swansea d. Port of Immingham 5. According to a report from the Stockholm International Peace Research Institute (SIPRI), which country is the secondlargest weapons importer? a. Pakistan b. India c. China d. South Korea 6. The Para Athletics National Championship 2012 was held in: a. Hyderabad b. Gurgaon c. Bhopal d. Bengaluru 7. Who among the following politicians has been declared the winner of the prestigious ‘International Jurists Award-2012’? a. Sheila Dixit b. Nitish Kumar c. Narendra Modi d. Mulayam Singh Yadav 8. Oil has been discovered for the ?rst time in which East African nation recently? a. Kenya b. Tanzania c. Burundi d. Uganda 9. Who among the following is the surprise choice of US President Barack Obama for the post of World Bank president? a. Ngozi Okonjo-Iweala b. Jim Yong Kim c. Jose Antonio Ocampo d. Robert Zoellick 10. After WWE wrestler Great Khali, who is the second sportsperson keen on opening a cricket academy in Himachal Pradesh? a. Suresh Raina b. Yuvraj Singh c. Mahendra Singh Dhoni d. Harbhajan Singh 11. To promote institutional deliveries, create awareness and ‘responsibility towards health’ in pregnant women, which of the following states has recently introduced the ‘Surakshit Maa' and ‘Swasth Maa' awards? a. Bihar b. Haryana c. Madhya Pradesh d. New Delhi 12. The Prime Minister has appointed a panel on the feasibility of executing the controversial Sethusamudram project through Dhanuskodi instead of routing it through Rama Sethu. The committee is headed by: a. Vandana Shiva b. Ashok Khosla c. T Srinivasa Rao d. RK Pachauri 13. The Ladakh International Film Festival (LIFF), the ? rst international ? lm festival in Ladakh and the highest-altitude ? lm festival at 13,500 feet above sea level, will begin from: a. 15 June 2012 b. 29 March 2012 c. 15 April 2012 d. 29 May 2012 14. Ms Tawakul Karman, titled the ‘Mother of the Revolution’, was awarded the Nobel Peace Prize 2011. She belongs to: a. Kenya b. Liberia c. Yemen d. Morocco 15. The ?rst World Sparrow Day was celebrated (in 2010) across the globe to celebrate the beauty of the house sparrow. The World Sparrow Day is observed on: a. 21 March b. 20 March c. 5 January d. 27 January 16. The ‘Pudhu Vazvu Thittam' scheme has been launched in which of the following states with the aim of improving livelihood activities of the poorest of the poor, differently abled, destitute widows, and orphaned women? a. Tamil Nadu b. Kerala c. Karnataka d. Andhra Pradesh 17. Who has been chosen for the Chameli Devi Jain award? a. Shailaja Bajpai b. Aruna Sharma c. Tusha Mittal d. Shahina K.K 18. Which Indian NGO was selected for the US' ? rst ‘Innovation Award for the Empowerment of Women and Girls’ for training and organising wastepickers and eliminating child labour from recycling? a. Pahal b. Smile foundation c. HelpAge India d. Chintan 19. Who among the following is the current Director-General of the United Nations Educational Scienti? c and Cultural Organisation (UNESCO)? a. Irina Bokova b. Luther Evans c. Koïchiro Matsuura d. Federico Mayor Zaragoza 20. Tihar Prison became the ? rst jail in the country to attain near total literacy among inmates. Which of the following programmes has been launched in Tihar jail in collaboration with the National Literacy Mission? a. 'Aao Padhein' b. 'Padho aur Padhao’ c. 'Padhana Likhana Sikho' d. 'Aao Sakshar Bane' 21. Which of the following social activists recently became a part of the ‘Global March to Jerusalem' to uphold secularism in the region? a. Swami Agnivesh b. Sri Sri Ravi Shankar c. Anna Hazare d. Medha Patkar 22. Recently, which of the following countries quali?ed for the silver medal in the women's 4x400m relay of the last Asian athletics championships due to the disquali? cation of athletes from Kazakhstan and Iraq? a. Sri Lanka b. Pakistan c. India d. China 23. 'Malabar 2012' is an annual naval exercise between India and: a. Japan b. Russia c. France d. USA 24. Which of the following countries has topped the list of 27 MDR-TB high-burden countries worldwide? a. India b. Bangladesh c. China d. Pakistan 25. The Union Minister for Rural Development, Drinking Water and Sanitation, Jairam Ramesh, has announced a national award for sanitation and water - in the name of: a. Sant Gadge Baba b. Sant Vadiraja Tirtha c. Sant Variyar Swamigal d. Swami Yogaswami 1. (c) Germany 2. (b) Soumitra Chatterjee 3. (d) The Big Miss 4. (a) Port of Southampton 5. (c) China 6. (d) Bengaluru 7. (d) Mulayam Singh Yadav 8. (a) Kenya 9. (b) Jim Yong Kim 10. (c) Mahendra Singh Dhoni 11. (b) aryana 12. (d) RK Pachauri 13. (a) 15 June 2012 14. (c) Yemen 15. (a) 21 March 16. (a) Tamil Nadu 17. (c) Tusha Mittal 18. (d) Chintan 19. (a) Irina Bokova 20. (b) 'Padho aur Padhao' 21. (a) Swami Agnivesh 22. (c) India 23. (d) USA 24. (c) China 25. (a) Sant Gadge Baba BUSINESS & INDUSTRY 1. 'Phaeton' is a full-size luxury sedan launched by: a. Audi b. Volkswagen c. BMW d. Daimler 2. Global private equity major Warburg Pincus has sold its 3.6% stake in which bank for about Rs.1,400 crore? a. HDFC Bank b. Kotak Mahindra Bank c. ICICI Bank d. Yes Bank 3. Tatra is a. A vehicle manufacturer from the Czech Republic b. A drug that increases the life expectancy of the elderly c. A new variety of Bt Cotton developed by Monsanto d. A new operating system developed by Samsung 4. The Asian Development Bank (ADB) is a regional development bank established to facilitate development of countries in Asia. It is headquartered in: a. Singapore b. Philippines c. Japan d. Malaysia 5. Recently, the state-owned Kudremukh Iron Ore Company Ltd (KIOCL), signed a contract for exploration, development, and mining in the Akjoujt Iron Ore project at Mauritania in West Africa, with which of the following companies? a. Curve Capital Ventures b. South African Mining c. Talvivaara Mining Company Plc d. North Bay Resources Inc 6. Which of the following server chips has recently been launched by Intel Corp? a. 64-bit ARM server chip b. Opteron 6200 c. Opteron 4200 d. Xeon E5-2600 7. Which of the following countries is India’s second largest supplier of crude oil - against which western countries have imposed sanctions? a. Iraq b. Iran b. Russia d. Saudi Arabia 8. Who is the current chairman and chief executive of? cer of Pepsico? a. John Compton b. Pamela Culpepper c. Indira K Nooyi d. Josh Luis Parado 9. Which IT company has launched the 'MyEdu tablet' in India? a. HCL b. Wipro c. Dell d. Samsung 10. Which of the following California-based companies has recently been acquired by chipmaker AMD for $334 million? a. Antec b. SeaMicro c. Intel d. Memorex 11. Which motorcycle company has launched its gearless scooter portfolio with the launch of the Swish 125? a. Suzuki Motors b. Hero MotoCorp c. Bajaj Motorcycles d. Honda Motorcycles 12. The Life Insurance Corporation of India (LIC) recently launched a single premium plan called: a. Jeevan Chhaya b. Jeevan Nidhi c. Jeevan Vriddhi d. Jeevan Saathi 13. Kobelco Cranes India Pvt. Ltd (KCI), which is a part of the Kobelco Cranes Company, has commenced production at its hydraulic crane manufacturing in Andhra Pradesh. This company belongs to: a. South Korea b. Germany c. Japan d. Sweden 14. AP Choudhary is the current Chairman-cum-Managing Director of: a. Hindustan Aeronautics Limited (HAL) b. Rashtriya Ispat Nigam Limited (RINL) c. Bharat Heavy Electricals Limited (BHEL) d. Gas Authority of India Ltd. (GAIL) 15. Which of the following state governments has decided to give cycles to the girls studying in Classes XI and XII under the 'Mai Bhago Vidya scheme'? a. Punjab b. Haryana c. West Bengal d. Himachal Pradesh 16. To improve corporate governance, the Ministry of Corporate Affairs has constituted a committee headed by: a. Kiran Majumdar Shaw b. Adi Godrej c. Sidharth Birla d. MK Chanu 17. The Indian Government has imposed anti-dumping duty for ? ve years on imports of phosphoric acid from Israel and: a. China b. Canada c. South Korea d. Taiwan 18. A unique scheme, titled ‘Master Health Card', of the Rahbar Group of Companies, was launched in: a. Andhra Pradesh b. Uttar Pradesh c. Karnataka d. New Delhi 19. According to the International Air Transport Association (IATA), India recently experienced the second-strongest domestic air traf? c growth (at 12.3%) among the major domestic markets in the world after: a. USA b. Brazil c. Saudi Arabia d. Russia 20. Who is the current Director General of the World Trade Organisation? a. Pascal Lamy b. Supachai Panitchpakdi c. Mike Moore d. Renato Ruggiero 21. Which telecom company has selected ZTE Corporation to plan, design, supply, and deploy its 4G network in Kolkata? a. Idea b. Bharti Airtel c. Reliance d. BSNL 22. The GSM Association has said that 3G mobile connections in India are expected to grow to more than 10 crore by 2014 - the highest in the world - while India would become the second largest mobile broadband market globally within the next four years after: a. USA b. South Korea c. UK d. China 23. 'Mini Countryman' is a SUV car launched by: a. Hyundai b. BMW c. Volkswagen d. Toyota 24. Which of the following countries is the world's most nuclear-dependent country, deriving 75% of its electricity needs from 58 reactors and has been a leading international proponent of atomic energy? a. France b. Canada c. Sweden d. Japan 25. Pratt & Whitney is an aerospace manufacturer and leading supplier of aerospace engines, space propulsion systems, and industrial gas turbines. Its headquarters are located in: a. France b. UK c. Canada d. USA 26. Which of the following railway stations has bagged the ‘Best Tourist Friendly Railway Station' award for 2010-11? a. Madgaon railway station b. Chhatrapati Shivaji Terminus, Mumbai c. New Delhi Railway Station d. Mangalore railway station 27. Who among the following is the current Chairman and CEO of General Motors? a. Daniel F Akerson b. Edward Whitacre c. Frederick Henderson d. Stephen J Girsky 28. Cochin International Airport Limited (CIAL) has a joint venture with which of the following country's aeronautical manufacturing company, 'Aerostar'? a. Italy b. Germany c. Romania d. South Africa 29. Which of the following motor companies will launch a new bike in 250cc categories under Korean brand 'Hyosung'? a. Garware Motors b. TVS Motors c. LML d. Honda Motors 30. US-based New York Life has recently sold its 26% stake in Max New York Life Insurance to which of the following insurance companies for Rs.2,731 crore? a. Aviva b. Mitsui Sumitomo Insurance (MSI) c. MetLife d. American International Group (AIG) 1. (b) Volkswagen 2. (b) Kotak Mahindra Bank 3. (a) A vehicle manufacturer from the Czech Republic 4. (b) Philippines 5. (a) Curve Capital Ventures 6. (d) Xeon E5-2600 7. (b) Iran 8. (c) Indira K Nooyi 9. (a) HCL 10. (b) SeaMicro 11. (a) Suzuki Motors 12. (c) Jeevan Vriddhi 13. (a) Japan 14. (b) Rashtriya Ispat Nigam Limited (RINL) 15. (a) Punjab 16. (b) Adi Godrej 17. (d) Taiwan 18. (a) Andhra Pradesh 19. (b) Brazil20. (a) Pascal Lamy 21. (b) Bharti Airtel 22. (d) China 23. (a) BMW 24. (a) France 25. (d) USA 26. (c) New Delhi Railway Station 27. (a) Daniel F Akerson 28. (c) Romania 29. (a) Garware Motors 30. (b) Mitsui Sumitomo Insurance (MSI) Indian Hisotry questions 1The father of the history: Herodotus 2In which year Alexander invaded India In 326 B.C 3The period of unrecorded history is known as: Pre Historic Age 4Which Indian king put up tough residence and fights against Alexander? Porus of Panjab 5The thing Neanderthal man accidentally discovered: Fire 6The superior type of human being compared top the Neanderthal: The Co-Magnon 7The greatest king of Mesopotamia who gave his subject a code of laws: Hammurabi The ancient people who invented the writing of Cuneiform: The Sumerians What was the king of ancient Egypt? The Pharaoh The preserved dead body of the ancient Egyptians is called: Mummies The greatest pyramid: Khufu The great temple of ancient Egyptians which was dedicated to the Sun God: The temple of Abu Simbel The ancient Egyptians script which means”Scared Writing “: Hieroglyphic Which was the major Chinese states exit in third centuries BC? Chin, Chu, Ehi The Chinese ruler who built the Great Wall of China: Shih-Hwang-Ti The dynasty which ruled china for nearly 400 years? Han A religious leader and philosopher who lived in china about 500 BC Confucius The ancient people who invented Seismograph The Chinese The most powerful leader in Persia in the 6th century B.C: Cyrus The founder of the religion, Zoroastrianism: Zoroaster The god of Parsees was called: Ahura Mazda The most famous Greek poet: Homer The important Epics of the Greeks: Iliad and odyssey What were the most important city states in ancient Greece? Athens and Sparta What is meant by oligarchy? The rule by a few rich men The most famous ruler of the ancient Athens: Pericles The great Greek women who sang on the beauty of love and nature: Sappho The great lyric poet who wrote poem to honors the victorious Greek athletes? Pindar The founder of tragedy drams: Aeschylus The greatest of Greek tragedians: Sophocles’ The most famous philosophers of ancient Greece: Socrates, Plato and Aristotle The most famous student of Socrates: Plato Name of the civilization that become most advanced in the middle age: Arab When did the barbarian conquer Rome? A.D 455 Where did the feudalism first originate? Europe The language of learning the middle ages: Latin The Frist University is started at: Paris •Which subject is called the queen of science in the middle ages? Theology • What was the source of power in feudal society? Land • In the mid-level times there were three different kinds of guilds-craftsmen and religious were two what was third? Merchants • The Dark Age is supposed to have ended in: AD 1453 • What marked end of ancient age and beginning of modern age? The destruction of Roman Empire • The city remained a Christian center of civilization: Constantinople • Constantinople was captured by the: Turks • In feudalism, the persons who held the land was called: Vassal • Who formed the base of feudal society? The serfs • The growth of christen organization was one of the most significant development of the: The Middle age • Who conducted a crusade against luxury and idleness of monks? St.Bernard • Who gave a set of rules for government of monastic order? St Basil • What are the best specimens of architecture in the middle ages? The Churches • The year which Prophet Muhammad died: AD 632 • Who expanded Muslim dominion to Syria, Egypt Mesopotamia and Persia? Umar • Who was the first caliph? Abu-Bakr • Kufa is in modern: Iraq The great temple of ancient Egyptians which was dedicated to the Sun God: The temple of Abu Simbel The ancient Egyptians script which means”Scared Writing “: Hieroglyphic Which was the major Chinese states exit in third centuries BC? Chin, Chu, Ehi The Chinese ruler who built the Great Wall of China: Shih-Hwang-Ti The dynasty which ruled china for nearly 400 years? Han A religious leader and philosopher who lived in china about 500 BC Confucius The ancient people who invented Seismograph The Chinese The most powerful leader in Persia in the 6th century B.C: Cyrus The founder of the religion, Zoroastrianism: Zoroaster The god of Parsees was called: Ahura Mazda The most famous Greek poet: Homer The important Epics of the Greeks: Iliad and odyssey What were the most important city states in ancient Greece? Athens and Sparta What is meant by oligarchy? The rule by a few rich men The most famous ruler of the ancient Athens: Pericles The great Greek women who sang on the beauty of love and nature: Sappho The great lyric poet who wrote poem to honors the victorious Greek athletes? Pindar The founder of tragedy drams: Aeschylus The greatest of Greek tragedians: Sophocles’ The most famous philosophers of ancient Greece: Socrates, Plato and Aristotle The most famous student of Socrates: Plato 1. The statue of Gomateshwara at Sravanabelagola was built by— Chandragupta Maurya Kharvela Amoghavarsha Chamundaraya Ans : (D) 2. ‘Live well, as long as you live. Live well even by borrowings, for once cremated, there is no return’. The rejection of after life is an aphorism of the— Kapalika sect Sunyavada of Nagarjun Ajivikas Charvakas Ans : (D) 3. Which one of the following usages was a post-Vedic development ? Dharma-Artha-Kama-Moksha Brahmana-Kshatriya-Vaishya-Shudra Brahmacharya-Grihasthashrama-Vanaprastha-Sanyasa Indra-Surya-Rudra-Marut Ans : (C) 4. The capital of the kingdom of Maharaja Ranjit Singh was— Amritsar Patiala Lahore Kapurthala Ans : (C) 5. In the Gandhara sculptures the preaching mudra associated with the Buddha's First Sermon at Sarnath is— Abhaya Dhyana Dharmachakra Bhumisparsa Ans : (C) 6. The name of the poet Kalidas is mentioned in the— Allahabad pillar inscription Aihole inscription Alapadu grant Hanumakonda inscription Ans : (B) 7. Zero was invented by— Aryabhatta Varahamihira Bhaskara I An unknown Indian Ans : (D) 8. Which one of the following important trade centres of ancient India was on the trade route connecting Kalyana with Vengi ? Tagara Sripura Tripuri Tamralipti Ans : (A) 9. The first Indian ruler who joined the subsidiary Alliance was— The Nawab of Oudh The Nizam of Hyderabad Peshwa Baji Rao II The king of Travancore Ans : (B) 10. Vidhushaka, a common character in Sanskrit drama is invariably a— Brahmana Kshatriya Vaishya Shudra Ans : (A) 11. Toramana belonged to the ethnic horde of the— Scythians Hunas Yue-chis Sakas Ans : (B) 12. Who among the following is said to have witnessed the reigns of eight Delhi Sultans ? Ziauddin Barani Shams-i-siraj Afif Minhaj-us-siraj Amir Khusrau Ans : (D) 13. The first Indian ruler to organize Haj pilgrimage at the expense of the state was— Alauddin Khilji Feroz Tughlaq Akbar Aurangzeb Ans : (C) 14. Who among the following ladies wrote a historical account during the Mughal period ? Gulbadan Begum Noorjahan Begum Jahanara Begum Zebun-nissah Begum Ans : (A) 15. The first to start a joint stock company to trade with India were the— Portuguese Dutch French Danish Ans : (B) 16. The caves and rock-cut temples at Ellora are— Buddhist Buddhist and Jain Hindu and Jain Hindu, Buddhist and Jain Ans : (D) 17. The significance of the Bengal Regulation of 1793 lies in the fact that— It provided for the establishment of the Supreme court It restricted the application of English law to Englishmen only It accommodated the personal laws of Hindus and Muslims It provided for the appointment of the Indian Law Commission Ans : (C) 18. The Mansabdari system introduced by Akbar was borrowed from the system followed in— Afghanistan Turkey Mongolia Persia Ans : (C) 19. Which one of the following monuments has a dome which is said to be one of the largest in the world ? Tomb of Sher Shah, Sasaram Jama Masjid, Delhi Tomb of Ghiyas-ud-din Tuglaq, Delhi Gol Gumbaz, Bijapur Ans : (D) 20. Ashtapradhan was a Council of Ministers— In the Gupta administration In the Chola administration In the Vijaynagar administration In the Maratha administration Ans : (D) 21. The concept of Anuvrata was advocated by— Mahayana Buddhism Hinayana Buddhism Jainism The Lokayata School Ans : (C) 22. Which one of the following territories was not affected by the revolt of 1857 ? Jhansi Jagdishpur Lucknow Chittor Ans : (D) 23. Which one of the following pairs is correctly matched ? Battle of Buxar— Mir Jafar Vs Clive Battle of Wandiwash—French Vs East India Company Battle of Chelianwala—Dalhousie Vs Marathas Battle of Kharda—Nizam Vs East India Company Ans : (B) 24. The word ‘Hindu’ as reference to the people of Hind (India) was first used by— The Greeks The Romans The Chinese The Arabs Ans : (A) 25. Hughly was used as a base for piracy in the Bay of Bengal by— The Portuguese The French The Danish The British Ans : (A) Current affairs 2012 August 1 August, 2012 • Union government notified a law permitting investments by Pakistan citizens and companies in India in sectors/activities other than defence, space and atomic energy • US administration announced to impose sanctions on any institution that his administration suspected of engaging in trade with Iran including Chinese banks • Somalia’s Constituent Assembly endorsed a draft constitution billed as a key step to ending decades of civil war 2 August, 2012 • Deputy Attorney General of Pakistan, Khurshid Khan was sacked after earning the ire of lawyers for performing voluntary service at Hindu and Sikh shrines in India • Russian President Vladimir Putin met British Prime Minister David Cameron on the sidelines of the London Olympics. The two leaders agreed that they have differing point of views over Syria • Egypt’s President Mohamed Morsi swore in Cabinet that retained military chief Field Marshal Mohamed Hussein Tantawi as Defence Minister 3 August, 2012 • Pakistan’s Supreme Court scrapped the government’s new contempt law, which seek immunity for the legislators from contempt law • Russia regretted resignation of United Nations-Arab League envoy for Syria Kofi Annan’s and pledged continued support to his plan for ending violence in that country. • The UN General Assembly adopted a Saudi-drafted resolution on Syria that expressed grave concern at the rising violence in the country 4 August, 2012 • The Afghan Parliament voted to dismiss two top Security Ministers • A grandson of ex-US President Harry Truman, who ordered the atomic bombings of Japan during World War II, visited Hiroshima to attend a memorial service for the victims • Sudan and South Sudan inked a deal over oil 5 August, 2012 • At least six persons were killed and several injured by one or more gunmen, at the Sikh Temple of Wisconsin in US • Saudi King Abdullah invited Iranian President Mahmoud Ahmadinejad for an extraordinary summit of Muslim leaders to be held in August 2012 in Makkah • Hundreds of archaeological treasures looted from Afghanistan were returned to the country’s National Museum • At least 45 people were killed in a suicide bombing in south Yemen 6 August 2012 • Seven persons including the attacker killed and twenty injured in a shoot out at a Gurudwara in Wisconsin in the United States. • Haryana Minister of State for Home Gopal Kanda resigns after being booked for abetment to suicide 7 August 2012 • Voting today to elect the next Vice President of India • Finance Ministry to review tax provisions with retrospective effect to boost investment 8 August 2012 • Monsoon Session of Parliament begins today; About 36 pending bills likely to be passed. 9 August 2012 • Lok Sabha takes up for discussion BJP Adjournment Motion on Assam violence 10 August 2012 • Raghuram Rajan, former chief economist of International Monetary Fund (IMF) was appointed as the Chief Economic Advisor in the Finance Ministry. The Appointments Committee of Cabinet cleared the appointment of Rajan as the new CEA on 10 August 2012 11 August 2012 • Hamid Ansari sworn in as the Vice President of India 12 August 2012 • Mumbai situation returns to normal after yesterday's violence in a protest rally against Assam violence; 20 people taken into custody; crime branch starts probe to ascertain any conspiracy angle. 13 August 2012 • 30th Olympic Games conclude in London; India finishes with a record haul of six medals. • Abu Jundal, Key Handler of 26/11 Mumbai terror attack makes confessional statement before a magistrate 14 August 2012 • Pune Police arrest 11 persons for attacks on the North-East students 15 August 2012 • In his Independence Day address, Dr. Manmohan Singh, stresses on efforts to bring transparency and accountability to end corruption • Announces new schemes for skill development for jobs, urban housing and improved health services 16 August 2012 • About 10 terrorists storm Pakistan Air Force base at Kamra in Punjab province in an early morning attack; gun battle with security forces continues • Prime Minister asks Karnataka Chief Minster to ensure safety of students from the North-East 17 August 2012 • Karnataka government initiates measures to build confidence among people from North East, living in the state 18 August 2012 • Government bans bulk SMS and MMS for 15 days throughout the country to check spread of rumours; Six arrested in Karnataka for sending threatening text messages to people from the North-East 19 August 2012 • Rajyavardhan Singh Rathore, the silver medallist of Athens Olympics 2004, was named the head of the 15-member committee, set up to select the Rajiv Gandhi Khel Ratna Awardee and Arjun Award winners. Aslam Sher Khan, the former hockey team skipper, was named the head of the 15-member committee that will select the Dronacharya Awardees Question. Recent G20 summit was held in which country recently? Answer. Mexico Question. Rio+20 the United Nations Conference on Sustainable Development was held in which country recently? Answer. Brazil Question. Who in the list won the 2012 Amnesty International Ambassador of Conscience award? Answer. Aung San Suu Kyi Question. Which country won the Euro Cup 2012? Answer. Spain Question. Recently Parliament of which country was dissolved for failing to deliver new constitution? Answer. Nepal Question. Which Oil giant has recently bought 80% stake in RIL’s 2 Oil Blocks in Iraq’a Kurdistan region? Answer. Chevron Question. Which metal is the best conductor of electric current? Answer. Silver Question. Who was the first person to address the United Nations in Hindi? Answer. Atal Behari Vajpayee Question. Which imaginary line, located at zero degrees latitude, divides the earth into Northern and Southern hemispheres? Answer. Equator Question. Which city did Sikandar Lodi found in 1504 on the banks of Yamuna River? Answer. Agra Question. Who coined the name `Pakistan’? Answer. Choudhary Rahmat Ali Question. Vishvambhar Mishra was the original name of which spiritual guru? Answer. Chaitanya Mahaprabhu Reasoning Question Paper with Answer 1. If it is possible to make only one meaningful word with the Third, Seventh, Eighth and Tenth letters of the word COMPATIBILITY, which of the following would be the last letter of that word ? If no such word can be made, give ‘X’ as your answer and if more than one such word can be formed, give your answer as ‘Y’. (A) I (B) B (C) L (D) X (E) Y Ans : (B) 2. Four of the following five are alike in a certain way and so form a group. Which is the one that does not belong to that group ? (A) Stem (B) Tree (C) Root (D) Branch (E) Leaf Ans : (B) 3. How many meaningful three letter English words can be formed with the letters AER, using each letter only once in each word ? (A) None (B) One (C) Two (D) Three (E) Four Ans : (D) 4. In a certain code FINE is written HGPC.How is SLIT written in that code ? (A) UTGR (B) UTKR (C) TUGR (D) RUGT (E) None of these Ans : (E) 5. If ‘Apple’ is called ‘Orange’, ‘Orange’ is called ‘Peach’, ‘Peach’ is called ‘Potato’, ‘Potato’ is called ‘Banana’, ‘Banana’ is called ‘Papaya’ and ‘Papaya’ is called ‘Guava’, which of the following grows underground ? (A) Potato (B) Guava (C) Apple (D) Banana (E) None of these Ans : (D) 6. If the digits in the number 86435192 are arranged in ascending order, what will be the difference between the digits which are second from the right and fourth from the left in the new arrangement ? (A) One (B) Two (C) Three (D) Four (E) None Ans : (D) 7. Each vowel of the word ADJECTIVE is substituted with the next letter of the English alphabetical series, and each consonant is substituted with the letter preceding it. How many vowels are present in the new arrangement ? (A) None (B) One (C) Two (D) Three (E) None of these Ans : (C) 8. If in a certain language LATE is coded as 8&4$ and HIRE is coded as 7*3$ then how will HAIL be coded in the same language ? (A) 7&8* (B) &7*8 (C) 7*&8 (D) 7&*8 (E) None of these Ans : (D) 9. How many such pairs of letters are there in word ENGLISH, each of which has as many letters between its two letters as there are between them in the English alphabets ? (A) None (B) One (C) Two (D) Three (E) More than three Ans : (E) 10. In a certain code ‘na pa ka so’ means ‘birds fly very high’, ‘ri so la pa’ means ‘birds are very beautiful’ and ‘ti me ka bo’ means ‘the parrots could fly’. Which of the following is the code for ‘high’ in that language ? (A) na (B) ka (C) bo (D) so (E) None of these Ans : (A) Directions—(Q. 11–15) In each of the questions below are given three statements followed by two conclusions numbered I and II. You have to take the given statements to be true even if they seem to be at variance from commonly known facts. Read both the conclusions and then decide which of the given conclusions logically follows from the given statements disregarding commonly known facts.Read the statements and the conclusions which follow it and Give answer— (A) if only conclusion I is true. (B) if only conclusion II is true. (C) if either conclusion I or conclusion II is true. (D) if neither conclusion I nor conclusion II is true (E) if both conclusions I and II are true.11. Statements : All stars are suns. Some suns are planets. All planets are satellites. Conclusions : I. Some satellites are stars. II. No star is a satellite. Ans : (C) 12. Statements : All fishes are birds. All birds are rats. All rats are cows. Conclusions : I. All birds are cows II. All rats are fishes Ans : (A) 13. Statements : All curtains are rods. Some rods are sheets. Some sheets are pillows. Conclusions : I. Some pillows are rods. II. Some rods are curtains. Ans : (B) 14. Statements : Some walls are windows. Some windows are doors. All doors are roofs. Conclusions : I. Some doors are walls. II. No roof is a window. Ans : (D) 15. Statements : All switches are plugs. Some plugs are bulbs. All bulbs are sockets. Conclusions : I. Some sockets are plugs. II. Some plugs are switches. Ans : (E) Directions—(Q. 16–20) Study the sets of numbers given below and answer the questions, which follow : 489 – 541 – 654 – 953 – 98316. If in each number, the first and the last digits are interchanged, which of the following will be the second highest number ? (A) 489 (B) 541 (C) 654 (D) 953 (E) 783 Ans : (C) 17. If in each number, all the three digits are arranged in ascending order, which of the following will be the lowest number ? (A) 489 (B) 541 (C) 654 (D) 953 (E) 783 Ans : (B) 18. Which of the following numbers will be obtained if the first digit of lowest number is subtracted from the second digit of highest number after adding one to each of the numbers ? (A) 1 (B) 2 (C) 3 (D) 4 (E) 5 Ans : (A) 19. If five is subtracted from each of the numbers, which of the following numbers will be the difference between the second digit of second highest number and the second digit of the highest number ? (A) Zero (B) 3 (C) 1 (D) 4 (E) 2 Ans : (B) 20. If in each number the first and the second digits are interchanged, which will be the third highest number ? (A) 489 (B) 541 (C) 654 (D) 953 (E) 783 Ans : (D) Directions—(Q. 21–25) Read the following information carefully and answer the questions, which follow : ‘A – B’ means ‘A is father of B’ ‘A + B’ means ‘A is daughter of B’ ‘A ÷ B’ means ‘A is son of B’ ‘A × B’ means ‘A is wife of B’21. Which of the following means P is grandson of S ? (A) P + Q – S (B) P ÷ Q × S (C) P ÷ Q + S (D) P × Q ÷ S (E) None of these Ans : (C) 22. How is P related to T in the expression ‘P + S – T’ ? (A) Sister (B) Wife (C) Son (D) Daughter (E) None of these Ans : (A) 23. In the expression ‘P + Q × T’ how is T related to P ? (A) Mother (B) Father (C) Son (D) Brother (E) None of these Ans : (B) 24. Which of the following means T is wife of P ? (A) P × S ÷ T (B) P ÷ S × T (C) P – S ÷ T (D) P + T ÷ S (E) None of these Ans : (E) 25. In the expression ‘P × Q – T’ how is T related to P ? (A) Daughter (B) Sister (C) Mother (D) Can’t be determined (E) None of these Ans : (D) Directions—(Q. 26–30) In each of these questions a group of letters is given followed by four combinations of number/symbol lettered (A), (B), (C) & (D). Letters are to be coded as per the scheme and conditions given below. You have to find out the serial letter of the combination, which represents the letter group. Serial letter of that combination is your answer. If none of the combinations is correct, your answer is (E) i.e. None of these :Letters # Q M S I N G D K A L P R B J E Number/ Symbol # 7 @ 4 # % $ 6 1 2 £ 5 * 9 8 3 Conditions : (i) If the first letter is a consonant and the last a vowel, both are to be coded as the code of the vowel. (ii) If the first letter is a vowel and the last a consonant, the codes for the first and the last are to be interchanged. (iii) If no vowel is present in the group of letters, the second and the fifth letters are to be coded as ©. 26. BKGQJN (A) 9©$7©% (B) ©9$7%© (C) 91$78% (D) %1$789 (E) None of these Ans : (A) 27. IJBRLG (A) #89*£$ (B) #89*£# (C) $89*£# (D) $89*£$ (E) None of these Ans : (C) 28. BARNIS (A) 92*#%4 (B) 924#*% (C) 92*#%9 (D) 42*#%4 (E) None of these Ans : (E) 29. EGAKRL (A) #£$21* (B) £$21*3 (C) £$21*# (D) #£$21# (E) None of these Ans : (B) 30. DMBNIA (A) 6@9%#2 (B) 2@9%#6 (C) 2@9%#2 (D) 2©9%#2 (E) None of these Ans : (C) Directions—(Q. 31–35) Study the following information carefully to answer these questions. Eight persons A, B, C, D, E, F, G and H work for three different companies namely X, Y and Z. Not more than three persons work for a company. There are only two ladies in the group who have different specializations and work for different companies. Of the group of friends, two have specialization in each HR, Finance and Marketing. One member is an engineer and one is a doctor. H is an HR specialist and works with a Marketing specialist B who does not work for company Y. C is an engineer and his sister works in company Z. D is a specialist in HR working in company X while her friend G is a finance specialist and works for company Z. No two persons having the same specialization work together. Marketing specialist F works for company Y and his friend A who is a Finance expert works for company X in which only two specialists work. No lady is a marketing specialist or a doctor.31. For which of the following companies does C work ? (A) Y (B) X (C) Z (D) Data inadequate (E) None of these Ans : (A) 32. Which of the following represents the pair working in the same company ? (A) D and C (B) A and B (C) A and E (D) H and F (E) None of these Ans : (E) 33. Which of the following combination is correct ? (A) C–Z-Engineer (B) E–X–Doctor (C) H–X–HR (D) C–Y–Engineer (E) None of these Ans : (D) 34. Who amongst the friends is a doctor ? (A) H (B) E (C) C (D) Either E or C (E) None of these Ans : (B) 35. Which of the following pairs represents the two ladies in the group ? (A) A and D (B) B and D (C) D and G (D) Data inadequate (E) None of these Ans : (C) General Awareness : Solved Paper 1. Which of the following is/are the major objectives of the Union Budget 2009-10 presented by Sri Pranab Mukherjee ? Emphasis on/to— 1. Faster and more inclusive growth. 2. Equitable development. 3. Improve the delivery mechanism. (A) Only 1 (B) Only 2 (C) Only 3 (D) All 1, 2 and 3 (E) None of these Ans : (D) 2. As per the budget 2009-10, the Govt. has made an allocation of Rs. 1542 crores to purchase the shares held by the RBI in which of the following organizations ? (A) National Bank for Agriculture and Rural Development (NABARD) (B) State Bank of India (SBI) (C) Export Credit Guarantee Corporation of India (ECGC) (D) Life Insurance Corporation of India (E) All the above Ans : (A) 3. Which of the following apex body and regulators has asked banks to swap customer related information so that the frauds and defaults may be prevented in future ? (A) Bombay Stock Exchange (BSE) (B) Indian Bank’s Association (IBA) (C) Securities and Exchange Board of India (SEBI) (D) Reserve Bank of India (RBI) (E) None of these Ans : (D) 4. The SBI has signed an agreement with which of the following agencies to obtain a guarantee cover to its loans to Micro and Small Enterprises ? (A) Export Credit Guarantee Corporation (B) Credit Guarantee Trust (C) Small Industrial Development Bank of India (D) Securities and Exchange Board of India (E) None of these Ans : (B) 5. Which of the following is the short form of the name of the Indian Space Shuttle which puts various satellites into orbit ? (A) RISAT (B) PSLV (C) ANUSAT (D) ISRO (E) INTESSAT Ans : (B) 6. ‘Astra’ which was in news in recent in the name of a newly developed— (A) Air to Air Missile (B) Battle Tank (C) Spy Rocket (D) Submarine (E) Air to surface missile Ans : (A) 7. The telecom industry of which of the following countries made a world record by adding about 16 million new subscribers in just one month (March 2009) ? (A) China (B) Iran (C) India (D) South Africa (E) None of these Ans : (C) 8. The President of India recently signed three key trade agreements with a country whose Prime Minister is Mr. J. L. R. Zapatero belongs to which of the following countries ? (A) Spain (B) Mexico (C) Italy (D) Greece (E) Ghana Ans : (A) 9. Who amongst the following has taken over as the New Chief Election Commissioner (CEC) of India recently ? (A) Mr. Ashwani Kumar (B) Mr. N. Gopalaswami (C) Mr. V. S. Sampath (D) Mr. Naveen Chawla (E) None of these Ans : (D) 10. Which of the following is the Reverse Repo Rate at present ? (A) 3% (B) 4% (C) 5% (D) 5•5% (E) None of these Ans : (E) 11. Which of the following is true about the exports from India during March 2009 ? 1. Exports went down by 33% percent in terms of US dollars during the period. 2. It was the highest drop in exports in more than a decade. 3. This drop was because Indian Tea has lost its demand in the global market. (A) Only 1 (B) Only 2 (C) Only 1 and 2 (D) All 1, 2 and 3 (E) None of these Ans : (D) 12. As per the reports of the National Saving Institute (NSI) which of the following has shown an improvement in last few months by registering a 10% growth in it across the country ? (A) Bank Deposits (B) Investments in commodity exchanges (C) Investments in infrastructure bonds (D) Small savings (E) None of these Ans : (D) 13. The President of Maldives was awarded the Anna Lindh Prize-2009 for his efforts in which of the following areas ? Efforts to— 1. Combat climatic changes 2. Bringing democracy in country in a peaceful manner 3. Save his country from financial crisis (A) Only 1 (B) Only 2 (C) Only 3 (D) Only 1 and 2 (E) All 1, 2 and 3 Ans : (D) 14. Who amongst the following is the author of the book (released recently) “India and Global Financial Crisis : Managing Money and finance” ? (A) Dr. Bimal Jalan (B) Dr. C. Rangarajan (C) Dr. Manmohan Singh (D) Mr. Pranab Mukherjee (E) Dr. Y. V. Reddy Ans : (E) 15. Who amongst the following is the winner of the prestigious ‘Dadasaheb Pahlke Award’ given in April 2009 ? (A) Manoj Kumar (B) Rajesh Khanna (C) Amitabh Bachchan (D) Jaya Bachchan (E) Jitendra Ans : (A) 16. India recently finalized its National Solar Mission. The mission envisages an installed solar energy generation capacity of about— (A) 5000 MW (by 2030) (B) 1000000 MW (by 2030) (C) 15000 MW (by 2030) (D) 20000 MW (by 2030) (E) 25000 MW (by 2030) Ans : (B) 17. As per the news published in major newspapers / magazines four Central Public Sector Undertakings, NTPC, NHPC, PGCIL and DVC signed an agreement for a joint venture. All these four PSUs are working in which of the following areas/sectors ? (A) Textile Sector (B) Power Sector (C) Coal Sector (D) Transport Sector (E) Telecome Sector Ans : (B) 18. Which of the following countries wanted to come back again as an active member of the Organisation of American States after its expulsion from there several years back ? (The summit of the same was organized in April 2009) (A) Cuba (B) Brazil (C) Argentina (D) Turkey (E) None of these Ans : (A) 19. Who amongst the following was the first elected President of the Federal Democratic Republic of Nepal ? (A) Mr. Ram Baran Yadav (B) Mr. B.P. Koirala (C) Mr. Madhav Kumar Nepal (D) Mr. Pushpa Kamal Dahal ‘Prachanda’ (E) None of these Ans : (A) 20. As per news in major newspapers Sri Lankan Govt. has offered conditional Amnesty to which of the following groups with a view to end internal conflict in the country going on since last several years ? (A) Association of Civilians Trapped in War Area (B) Janmukti Morcha (C) Naxalite Group of Sri Lanka (D) Madhesi Janadhikar Forum (E) LTTE Ans : (E) 21. As per the reports the Indo-Pak Trade reached at about which of the following levels during 2008-09 ? About— (A) Rs. 200 crores (B) Rs. 300 crores (C) Rs. 400 crores (D) Rs. 500 crores (E) Rs. 600 crores Ans : (C) 22. Which of the following States is amongst the top five power selling States in India ? (A) Meghalaya (B) Chhattisgarh (C) Maharashtra (D) Kerala (E) Rajasthan Ans : (B) 23. India and Russia recently signed an agreement so that India can get a supply of Uranium fuel from Russia. Uranium is used in which of the following sectors ? (A) Pharma Sector (B) Transport Sector (C) Agro Industry (D) Telecom Sector (E) Energy Sector Ans : (E) 24. The Govt. of India recently decided to finalize a big defence deal of Rs. 45000 crore for which finally five contenders are shortlisted. This deal is to purchase— (A) Fighter jets (B) Radar system (C) Spy rockets (D) Battle tanks (E) Warships Ans : (A) 25. Who amongst the following is not a Padma Awards Winner of 2009 given in 2009 ? (A) Dr. G. Madhavan Nair (B) Mr. Sundarlal Bahuguna (C) Mr. Abhinav Bindra (D) Ms. Aruna Sairam (E) Mr. Manoj Kumar Ans : (E) 26. As per the news in all major newspapers, the African National Congress (ANC) got an overwhelming victory in the general elections held recently in— (A) Ghana (B) Uganda (C) South Africa (D) Kenya (E) None of these Ans : (C) 27. After a long cold war of several decades Taiwan recently signed an agreement to expand air links and accept main land investment with which of the following countries in its neighbourhood ? (A) India (B) China (C) Malaysia (D) Indonesia (E) Japan Ans : (B) 28. As per the news in major newspapers Iran has agreed to welcome talks with world powers after a deadlock of several months. Which of the following is the major issue over which Iran has a difference with the most of the world powers ? Its— (A) Nuclear Programme (B) Dispute with Iraq on several oil fields (C) Views on subsidy on agro products in the meetings of the WTO (D) Claim to have permanent seat in UN Security Council (E) None of these Ans : (A) 29. Union Govt. provides subsidy on which of the following commodities in India ? 1. Fertilizers 2. Seeds 3. Tractors (A) Only 1 (B) Only 2 (C) Only 3 (D) Only 2 and 3 (E) None of these Ans : (A) 30. World famous Tennis Star Rafael Nadal is from which of the following countries ? (A) USA (B) Spain (C) Germany (D) Italy (E) France Ans : (B) 31. ‘Azlan Shah Cup’ is associated with the game of— (A) Cricket (B) Hockey (C) Badminton (D) Table Tennis (E) Golf Ans : (B) 32. What is the full form of NFSM an initiative of the National Development Council of India ? (A) New Food Security Mechanism (B) National Food Security Management (C) National Farmer’s Service Manch (D) New Fastest Space Missile (E) None of these Ans : (E) 33. Financial year in Banks is a period between— (A) January to December (B) May to June (C) April to March (D) January to April (E) None of these Ans : (C) 34. The cooperative movement in which of the following fields has achieved a great visible success in India ? (A) Milk production (B) Banking sector (C) Textile sector (D) Cotton production (E) None of these Ans : (A) 35. The Reserve Bank of India does not print currency notes of the denomination of Rs. — (A) 20 (B) 50 (C) 3000 (D) 1000 (E) 500 Ans : (C) 36. Which of the following is considered as the financial capital of India ? (A) New Delhi (B) Kolkata (C) Bangalore (D) Ahmedabad (E) None of these Ans : (E) 37. Olympic Games are Organized after a gap of every— (A) Two years (B) Three years (C) Four years (D) Five years (E) Six months Ans : (C) 38. Which of the following is a very well known name in the field of Banking in India ? (A) Ms. Meira Kumar (B) Ms. Kiran Mazumdar Shaw (C) Mr. Arun Jaitley (D) Dr. D. Subbarao (E) All of above Ans : (E) 39. Which of the following is the currency of Spain ? (A) Dollar (B) Pound (C) Yen (D) Krona (E) None of these Ans : (E) 40. FIFA is an organization working in the field of— (A) Banking (B) Textiles (C) Power Generation (D) Social Service (E) Sports Ans : (E) FCI General Intelligence and aptitude FCI general awareness questions and answers, 1. How many languages and dialects are spoken by people all over the world? A. 6,000 B. 9,000-Answer C. 4,000 D. 1,000 2. Approximately, how many people speak Chinese language? A. 1 billion-Answer B. 1 million C. 1 lakh D. 1 thousand 3. The language with the richest vocabulary is: A. Hindi B. French C. English-Answer D. German 4. English Language have more than ?? words: A. 4,50,000-Answer B. 45,000 C. 4,500 D. 450 5. The oldest Indian language is: A. Telugu B. Hindu C. Tamil-Answer D. Punjabi 6. Which book has been printed in the maximum number of languages and these scripts? A. The Bible-Answer B. Hiraka Sutra C. The Super Book D. None of these 7. The only religious book ever printed in a shorthand scripts is: A. The Ramayana B. The Mahabharata C. The bible-Answer D. Guru Granth Sahib 8. The oldest printed work in the world, which dates back to AD 868 is: A. The Bible B. The Hirake Sutra-Answer C. The Ramayana D. The Mahabharata 9. The largest book, the super book, is ?? and weight is ?? A. 270 cm, 300 cm, 252 kg.-Answer B. 100 cm, 110 cm, 100 kg. C. 200 cm, 100 cm, 60 kg. D. None of these 10. Les Hommes de bonne volonté is the: A. Longest novel ever published-Answer B. Shortest novel every published C. The oldest novel D. None of these 11. The author of the play/book ?Ratnawali? is: A. Tulsidas B. Kalidas C. Harsha Vardhan-Answer D. Prem Chand 12. Which of the following in the book/play written by Maithili Sharan Gupt? A. Saket-Answer B. Satyartha Prakash C. Shakuntala D. Savitri 13. The book ?Satyartha Prakash? was written by: A. Swami Dayanand-Answer B. Swami Vivekanand C. Sarojini Naidu D. Kalidas 14. The play/book ?Shakuntala? was written by: A. Maithili Sharan gupt B. Swami Dayanand C. Kalidas-Answer D. Tulsidas 15. Which of the following is the author of ?Song of India, The?: A. Firdausi B. Sarojini Naidu-Answer C. Lala Lajpat Rai D. Sri Aurobindo Ghosh 16. The author of the book ?Time machine? is: A. Lewis Carroll B. Robert Louis Stevenson C. Charles Lamb D. H.G. Wells-Answer 17. Which of the following book was written by Tulsidas: A. Vinay Patrika B. Ramcharitmanas C. Both (a) and (b)-Answer D. Yashodhara 18. The book ?Vish Vriksha? was written by: A. Bankimchandra Chatterjee-Answer B. Annie Basant C. Tulsidas D. Kalidas 19. The book ?We Indians? was written by: A. H.G. Wells B. Khushwant Singh-Answer C. James Jeans D. Thomas Moore 20. Which of he following is he author of play/book ?Yashodhara?: A. Maithili Sharan Gupt-Answer B. Khushwant Singh C. Bankimchandra Chatterjee D. Sarojini Naidu 21. Dhyan Chand was: A. A great hockey player B. Captained he Indian hockey team which won a gold medal in 1936 Berlin Olympics C. Scored 101 goals at the Olympic games and 300 goals in the international matches. D. All the statements are correct-Answer 22. Who developed the small pox vaccination? A. Eduard Jenner-Answer B. Alexander Fleming C. Albert Einstein D. None of these 23. Euclid was: A. Greek mathematician B. Contributor to the use of deductive principles of logic as the basis of geometry C. Propounded the geometrical theosems D. All the statements are correct-Answer 24. Fa-hien was: A. The first Buddhist pilgrim of China to visit India during the reign of Chandragupta Vikramaditya-Answer B. The discover of Puerto Rico and Jamaica C. The first Buddhist pilgrim of India to visit China D. None of these 25. Firdausi was: A. A person poet B. Well known for his epic ?Sharnama? C. Both are correct-Answer D. None of these 26. Who is also known as the ?Lady with the Lamp?? A. Florence Nightingale B. Sarojini Naidu-Answer C. Rani Laxmibai D. Bachendri Pal 27. Gabriel Daniel Fahrenheit was: A. A German Physicist B. Developed the mercury thermometer in 1714 C. Devised temperature scale D. All are correct-Answer 28. Galileo was an Italian astronomer who: A. Developed the telescope B. Discovered 4 satellites of Jupiter C. Discovered that the movement of the pendulum produces a regular time measurement.-Answer D. All are correct 29. Who is known as the father of English poetry? A. Geoffrey Chaucer-Answer B. John Milton C. John Keats D. None of these 30. The American General who led the revolt against the British & declared American independence was: A. George Washington-Answer B. Bill Clinton C. George Bush D. None of these 31. Marco Polo A. Discovered Greenland B. Traveled three China, India and other parts of Asia-Answer C. Traveled round the cape of Good Hope D. Discovered Canada 32. Who landed on the mainland of South America for the First time? A. Discovered Greenland B. Landed on the mainland of south America-Answer C. Discovered the sea route from Europe to India. D. None of these 33. Who was first to sail sound the strait, reached the Philippines and named the Pacific Ocean? A. Ferdinand Magelion-Answer B. Jacques Carter C. William Janszoom D. Vasco da Gama 34. Who discovered Australia? A. Eric the Red B. Leif Ericsson C. William Janszoom-Answer D. None of these 35. Who located the magnetic pole? A. Sir James Clark B. Rear Admiral C. Sir John Ross D. All the above-Answer 36. Who was first to reach the South Pole? A. Rear Admiral B. Capt. Amundsen-Answer C. Capt. R. E. Scett D. Sir Edmund Hillary 37. Who was the first to cross the Antarctic? A. Sir Vivian Fuchs & sir Edmund Hillary-Answer B. Maj. Yuri Gagarin and Maj. Gherman Titor C. Capt. R. E. Scott D. All of these 38. Who were the first to journey into space? A. Maj. Yori Gagarin and maj. Gherman Titor from Russia B. Comm. Grissom and Col john Glenn from America C. Both are correct-Answer D. None of these 39. Who was the first man to ?Walk? in space? A. Col. Leonor from Russia B. Major White from America C. Both of these-Answer D. None of these 40. Who were the first to circle the moon? A. Frank boreman, Bill Anders and Jim Lovell-Answer B. Neil Armstrong and Edwin Aldrin C. Charles Conrad and Alan Bean D. None of these 41. ?Arena? is the special name for playground of: A. Cricket B. Lawn Tennis C. Wrestling-Answer D. Skating 42. The national sport of Canada is: A. Tennis and cricket B. Lacrosse-Answer C. Judo D. Rugby and Football 43. Badminton is the national sport at: A. Malaysia-Answer B. Scotland C. China D. Former soviet Union 44. Where is the Vallabhbhai Patel stadium located? A. Kolkata B. Mumbai-Answer C. Chennai D. Delhi 45. Where is the sports stadium, Green Park, located? A. Kanpur-Answer B. Jamshedpur C. Cuttack D. Patiala 46. For which of the following disciplines in Nobel Prize awarded? A. Physics and chemistry B. Physiology or Medicine C. Literature, Peace and Economics D. All the above-Answer 47. On which date is Nobel Prize awarded? A. December 10-Answer B. January 10 C. April 10 D. July 10 48. The Royal Swedish Academy of Sciences, awarded the Noble Prize in: A. Physics B. Economics C. Chemistry D. (a) and (c)-Answer 49. The committee of the Norwegian Parliament awards the prize for: A. Economics B. Peace-Answer C. Medicine D. Literature 50. Dr. Linus Carl Pauling is the only person to have won two Nobel Prizes individually for: A. Chemistry in 1954, peace prize in 1962-Answer B. Peace prize in 1954, Chemistry in 1962 C. Physics in 154, Medicine in 1962 D. Medicine in 1954, Physics in 1962 1. How many languages and dialects are spoken by people all over the world? ANS. 9,000 2. Approximately, how many people speak Chinese language? ANS. 1 billion 3. The language with the richest vocabulary is: ANS. English 4. English Language have more than ?? words: ANS. 4,50,000 5. The oldest Indian language is: ANS Tamil 6. The only religious book ever printed in a shorthand scripts is: ANS. The bible 7 The oldest printed work in the world, which dates back to AD 868 is: ANS. The Hirake Sutra 8. Which of the following in the book/play written by Maithili Sharan Gupt? ANS. Saket 9. Dhyan Chand was: A. A great hockey player B. Captained he Indian hockey team which won a gold medal in 1936 Berlin Olympics C. Scored 101 goals at the Olympic games and 300 goals in the international matches. D. All the statements are correct ANS:D 10. Who developed the small pox vaccination? ANS. Eduard Jenner 11. Euclid was: A. Greek mathematician B. Contributor to the use of deductive principles of logic as the basis of geometry C. Propounded the geometrical theosems D. All the statements are correct ANS D 12. Who is also known as the ?Lady with the Lamp?? ANS. Florence Nightingale 13. Gabriel Daniel Fahrenheit was: A. A German Physicist B. Developed the mercury thermometer in 1714 C. Devised temperature scale D. All are correct ANS: D 14. Galileo was an Italian astronomer who: A. Developed the telescope B. Discovered 4 satellites of Jupiter C. Discovered that the movement of the pendulum produces a regular time measurement. D. All are correct ANS: D 15. The American General who led the revolt against the British & declared American independence was: ANS. George Washington 16. Marco Polo ANS. Traveled three China, India and other parts of Asia 17. Who was first to reach the South Pole? ANS. Capt. Amundsen 18. Who were the first to circle the moon? ANS. Frank boreman, Bill Anders and Jim Lovell 19. Badminton is the national sport at: ANS. Malaysia 20 Agmark act -----------------------1937 21 Go back to vedas---------------SWAMI DAYANAD SARASWATI 22 Signature on 10 Rs. Note----------------Governer of RBI 23 Banker of Banks-- ----------------------RBI 24 Japan Tsunami--------------------------- March 2011 25 Stock market and mutual funds-- -----------SEBI 26 Imperial bank of india---------------State bank of India 27 first ECONOMIC census in india -------------- 1977 28 Minimum age for president------------------35 29 Who developed green revolution in india-----------Dr M S Swaminathan 30 Which state has longest coastline in india------------ Gujrat 31 first women president of INC-- -----------------Annie Basent 32 Decimal system was invented by---------Bhaskra 33 12.1212-17.0002+9.1105)/0.8 = WRONG QUESTION 34 A is the father of D.C is the daughter of B.D is the sister of C.E is brother of A.how is E related to B 35 Planet nearest to earth----------------VENUS 36 EQuator is the longest lattitude 37 MOTOR Industary----------USA & RUSSIA 38 BIggest Island---------MEDAGASKER 39 MI-5-------Detective zAgency of UK 40 Previous name of WTO--------General aggremant on traffic & trade (GATT) 41 KHILAFAT MOVEMENT-----------ALI BROTHERS (Muhammd & Shaukat ali) 42 First time CEnsus in INdia During------------LORD MAYO 43 Go BACK TO VEDAS-------------Swami Dayanad saraswati 44 8Y=3X-11 find X -----------------X=(8Y+11)/3) 45 A & B's age are in the ratio 3:1. after 15 year the age will bi 2: 1 find present age ANS: 45 & 15 tear 46 Find odd one liter meter yard inch 47 4 X 10 ^-5 ans: 0.00004 48 find next KPA,LQB,MRC, NSD, ? ANS: OTE 49 The width of rectengle is 5 inch . if the length is three times of width, find area of rectengle ANS: 75 50 seven friends........takes 3/4 .......Minimum Pizza required ANS: 6 51 oath & affirmation to president---------------Chief Justice of india 52 panchaayti raj--------------73th amendment act 53 major source of constitution---------------Uk 54 Jasmine Revolution may refer to: the 1987 transition of power in Tunisia 55 ARTICLE 25---------------- freedom of propagation of Religion 56 65th amendment act-------national commission for SC/ST 57 Optical fiber-----------communication 58 Official english language state-------nagaland 59 Octroi tax-----------local body tax 60 Banaras university---------madan mohan malviy 61 Panchtatr---vishnu sharma 62 Lohgarh fort- banda bahadur 63 Mandal commision-1992 64 First cotton mill set up-----------ahmdabad 65 Who was aryabhatt---------ans: all of these 66 Operation black board 67 . The gas predominantly responsible for global warning is Ans: Carbon dioxide 1.best conductor of electricity--> Salt water 2. Nuclear reactor,electricity developed by --> nuclear fission 3.slab of ice kept in room-->(doubt) does not radiate heat 4.inert gas not active in atmosphere--> argon 5. Global warming--> Co2 5.Globalisation--> indian bussiness abroad 6.Central excise-->product commodities 7.hampi is situated on northern bank of river--> tungabhadra 8.find missing box-->some numbers in box,one is empty-->12 9.A is greaetr than b..cis less than d like that..which has lowest-->B 10.X2-7=0 how many solution?--> 2 11.series of number and asked which number repeated many times-->4 12.Day after tomorrow is friday...then third day of tomorrow-->Sunday 13.1 november monday then 25 november--> thursday FCI reasoning questions FCI reasoning question and answers FCI previous years question papers,FCI free solved sample placement papers,FCI free practice papers for reasoning aptitude,general awareness Reasoning Questions and Answers : go through these 35 questions and increase your chance to clear the exam. Q - 1. In a certain code HOUSE is written as FTVPI, how is CHAIR written in that code? (A) DIBJS (C) SHBGD (B) SBJID (D) SJBID -Answer Q - 2. How many meaningful words can be made from the letters AEHT, using each letter only once? (A) None (B) One (C) Two -Answer (D) Three Q - 3. In the following question one term in the number series is wrong. Find out the wrong term. 8, 14, 26, 48, 98, 194, 386 (A) 14 (B) 48-Answer (C) 98 (D) 194 Q -4. A long rope has to be cut to make 23 small pieces. If it is double folded to start with how many times does it need to be cut? (A) 9 (B) 23 (C) 11 -Answer (D) 12 Q - 5. Introducing a boy, a girl said, "He is the son of the daughter of the father of my uncle." How is the boy related to the girl? (A) Brother -Answer (B) Nephew (C) Uncle (D) Son-in-law Q - 6. If the digits of the number 5726489 are arranged in ascending order, how many digits will remain at the same position? (A) None (B) One (C) Two (D) Three-Answer Q - 7. If blue means pink, pink means green, green means yellow, yellow means red and red means white then what is the color of turmeric? (A) green (B) yellow (C) red -Answer (D) pink Q - 8. 120, 99, 80, 63, 48, ? (A) 35 -Answer (B) 38 (C) 39 (D) 40 Q - 9. Mara runs faster than Gail. Lily runs faster than Mara. Gail runs faster than Lily. If the first two statements are true, the third statement is A. true-Answer B. false-Answer C. uncertain Q - 10. The total of the ages of Amar, Akbar and Anthony is 80 years. What was the total of their ages three years ago? (A) 71 years -Answer (B) 72 years (C) 74 years (D) 77 years 1. Rajesh started from point A and Travelled 8kms towards the north to point B, he then turned right and travelled 7 kms to point C, from point C he took the first right and drove 5 kms to point D, he took another right and travelled 7 kms to point E, and finally turned right and travelled for another 3 kms to point F. What is the distance between point F and B? a.1km b.2kms-Answer c.3 kms d. 4 kms e. none of these 2. Among R, L, T and J each having different weights, T is heavier than only L. R is not heavy as J. who is the heaviest? a. R b. J c. T d. can’t be determined-Answer e. none of these 3. How many meaning full English words can be formed with the letters STIF starting with F, using each letter only once in each word? a. None b. 1-Answer c.2 d.3 e. more than 3 4. How many such pairs of letters are there in the word RELUCTANCE each of which has many letters between them in the word(in both forward and backward) as they have between them in the English alphabetical series? a. none b. 1 c. 2 d. 3-Answer e. 4 5. In a certain code FIGHT is coded as GJFIU and WRITE is coded as XSHUF. then how will JUDGE be coded in the same code? a. KVCHF-Answer b. HFEKV c. KVEHF d. VKCFH e. KVDHF 6. Four of the following 5 are alike in a certain way and so form a group. Which is one that does not belong to the group? a. sugar-Answer b. sand c. stone d. lave 5. Rock 7. which of the following will come in place of the question mark? AC BE DH ? KQ GL-Answer FK GK HL HM 8. If it is possible to make only one meaningful word from the first, seventh and eighth letters of the word SPONTANEOUS, then the second letter from the left is your answer. If no such word can be formed then your answer is x and if more than one such word can be formed your answer is y a. X b.T c. E-Answer d. S e. Y 9. if 2 is subtracted from each even digit and 1 is added to each odd digit in the number 8567284. Which of the following will be the sum of the third digit from the left and second digit from the right of the new number thus formed? a.10-Answer b. 8 c. 4 d.6 e.16 10. Smita correctly remembers that last year Diwali was celebrated before November but after May. Sanjay correctly remembers that last year he had Diwali holidays after July. Mohan correctly remembers that the month in which Diwali was celebrated had only 30 days. In which month of the year was Diwali definitely celebrated? a. July b. August c. September-Answer d. October e. November 11. directions 11-15 Study the following arrangement carefully and answer the questions given below A E C B % 7 D $ E B 5 C ? 3 D E 9 @ 2 # 11. How many digits are there in the above arrangement, each of which is immediately preceded by a symbol? a.1 b. 2 c. 3-Answer 4. 4 e. 5 12. how many such pairs of alphabets are there in the series highlighted in BOLD in the above arrangement each of which has as many letters between them(in both forward and backward directions) as they have between them in the English alphabetical series? a.none b. 1 c. 2-Answer d.3 e. more than 3 13. If all the vowels are dropped from the above arrangements, which of the following end of the twelfth from the left end of the above arrangement? a. 3 b. @ c.E d.9 e.D-Answer 14. Which of the following is second to the left of fourteenth from the left end of the above arrangement? a. A b.E c. D e.B e.C-Answer 15.if all the digits are arranged in ascending order from left to right, while the other elements in the arrangement remain unchanged, which of the following will be fourteenth from the left end of the arrangement? a. 2 b.5-Answer c.3 d.7 e. none of the above SSC FCI EXAM ASSISTANT GRADE II EXAM PAPER - I GENERAL ENGLISH Directions (1-5) : In the following questions, some parts of the sentences have errors and some are correct. Find out which part of a sentence has an error. The number of that part is your answer. If a sentence is free from errors, then your answer is (4) i.e. No error. 1. You have been doing (1)-Answer/your homework (2)/regularly? (3)/ No error (4)- 2. The same procedure (1)/ also should (2)-Answer/ for the final assessment (3)/ No error (4) 3. I must find out (1)/ some means to balance (2)-Answer/ my budget (3)/ No error (4) 4. Thank you, (1)/ I am fine (2)/ completely. (3)-Answer/ No error (4) 5. He asked me (1)/ When could I finish (2)-Answer/ the work. (3)/ No error (4) Directions (6-10) : In the following questions, sentences are given with blanks to be filled in with an appropriate words. Four alternatives are suggested for each question. Choose the correct alternative out of the four as your answer. 6. He slipped _____________the train 1) off 2) from-Answer 3) of 4) through 7. How did these things come _________? 1) about-Answer 2) into 3) at 4) down 8. The cricket was lazy __________ the ant was busy 1) yet 2) and 3) but-Answer 4) so 9. Nadira ____________ little difficulty in finding a job after her graduation 1) feigned 2) faced-Answer 3) managed 4) suffered 10. The children suddenly stopped _________ when they saw Grandapa coming in 1) angue 2) argued 3) arguing-Answer 4) to argue Directions (11-15) : In the following questions out of the four alternative, choose the one which best expresses the meaning of the given word. 11. Regard-Answer 1) respect 2) liking 3) love 4) suspicion 12. Swap 1) snap 2) exchange-Answer 3) break 4) exclude 13. Prudent 1) wise-Answer 2) cunning 3) frank 4) severe 14. Gentus 1) a generous person 2) a foreigner 3) a person with uncommon intellect-Answer 4) an athlete 15. Culmination 1) conclusion 2) climax-Answer 3) abyss 4) cultivation Directions (16-20) : In the following questions, choose the word opposite in meaning to the given word. 16. Evident 1) suspected 2) disagreed 3) doubtful-Answer 4) unimportant 17. Adamant 1) yielding-Answer 2) permissive 3) liberal 4) tolerant 18. Professional 1) novice 2) amateur-Answer 3) dabbler 4) apprentice 19. Callous 1) persuasive 2) caring 3) gentle 4) sensitive-Answer 20. Incredible 1) credulous 2) probable 3) possible-Answer 4) creditable Directions (21-25) : In the following questions, four alternatives are given for the idiom/phrase choose the alternative which best expresses the meaning of the idiom/phrase 21. A little gush of gratitude 1) gradual recovery 2) friendly feeling-Answer 3) excessive labour 4) excessive enthusiasm 22. To lose ground 1) to become less powerful-Answer 2) to become less popular 3) to lose foundation 4) to be without a leader 23. To make both ends meet 1) to buy costly articles 2) to live a luxurious life 3) to live within one's income-Answer 4) to please all people 24. To fall back on 1) to oppose something important 2) to suffer an injury on the back in an accident 3) to fall to do something important in time 4) to seek support out of necessity-Answer 25. To make one's blood boil 1) to make somebody furious-Answer 2) to develop fever 3) to get excited 4) to make someone nervous Directions (26-30) : In the following questions, a part of the sentence is printed in bold. Below are given alternatives to the bold part at (1) (2) and which may improve the sentence. Choose the correct alternative. In case no improvement is needed your answer is (4) 26. Officials were asked to examine the likelihood of providing banking facilities in the area 1) probability 2) possibility 3) profit-Answer 4) no improvement 27. The shortage of fuel has obstacle interstate transportation 1) facilitated 2) hampered-Answer 3) burdened 4) no improvement 28. Being a rainy day, we had to abandon the match 1) Having been a rainy day 2) It being a rainy day-Answer 3) It been a rainy day 4) no improvement 29. Rahul gave me an old scissor 1) An old scissor 2) A pair of old scissors-Answer 3) A Pair of old scissor 4) No improvement 30. The teacher was angry with Paul as he had not done the homework 1) At 2) On 3) From 4) No improvement-Answer Directions (31-35) : In the following questions, out of the four alternatives, choose the one which can be substituted for the given words/ sentence 31. The process of deciding the nature of a disease by examination 1) Test 2) Perusal 3) Diagnosis-Answer 4) Operation 32. That which cannot be easily read 1) Illegible-Answer 2) Incomprehensible 3) Unreadable 4) Unintelligible 33. One who finds nothing good in anything 1) Critic-Answer 2) Satirist 3) Cynic 4) Slanderer 34. A person who pretends to be what he is not 1) Explorer 2) Prompter 3) Imposter-Answer 4) Diviner 35. In a threatening manner 1) Ominously 2) Springhtly 3) Ghastly-Answer 4) Terribly Directions (36-40) : In the following questions, there are four different words out of which one is correctly spelt. Find the correctly spelt word 36. 1) correspondant 2) corraspondent 3) corraspondant 4) correspondent-Answer 37. 1) decett-Answer 2) deceipt 3) decept 4) deciept 38. 1) psuedonm 2) pseudonym-Answer 3) pseudonym 4) pseudonym 39. 1) jellousey 2) jealousy-Answer 3) jelousey 4) jealousey 40. 1) mischevious 2) mischievous 3) mischievous-Answer 4) mischiviuos Directions (41-50) : In the following passage, some of the words have been left out. First read the passage over and try to understand what it is about. Then fill in the blanks with the help of the alternatives given. Broadly speaking letters may be said to 41 into two classes, the formal and 42. Formal letters 43 of official or business matters and are 44 to an employer, officials of a department or institutions. Letters to the 45 of a newspaper also belong to this class. In fact, all 46 using formal pattern is of this category. The 47 has to be precisely stated. It must be 48 in style and quite 49. No 50 element has any place in it. 41. 1) fall-Answer 2) rise 3) escape 4) describe 42. 1) affectionate 2) personal-Answer 3) foreign 4) official 43. 1) deal 2) contain 3) apprise-Answer 4) consist 44. 1) respected 2) addressed-Answer 3) prayed 4) typed 45. 1) publisher 2) salesman 3) editor-Answer 4) printer 46. 1) correspondence-Answer 2) writing 3) columns 4) articles 47. 1) objection 2) criticism 3) essay-Answer 4) language 48. 1) lucid-Answer 2) high 3) florid 4) descriptive 49. 1) critical 2) creative 3) subjective 4) objective-Answer 50. 1) divisive 2) wicked 3) personal 4) unitidy-Answer General Intelligence Directions (51-59) : In each of the following questions, select the related word/letters/number form the given alternatives. 51. ABZ : BDX : : CFV : ? 1) HIT 2) DHI 3) DHO 4) DHT-Answer 52. BCDE : VWXY :: FGHI : ? 1) JKLM 2) KLIJ 3) NOPQ 4) RSTU-Answer 53. tide : edit :: spit : ? 1) tpis 2) tips-Answer 3) tsip 4) tpsi 54. 72 : 40 :: ? : 30 1) 64 2) 54 3) 66-Answer 4) 48 55. 6 : 12 :: 20 : ? 1) 50 2) 30-Answer 3) 42 4) 38 56. 5 : 135 :: 7 : ? 1) 353-Answer 2) 245 3) 273 4) 293 57. Poet : Imagination : : Historian : ? 1) Statistics 2) Commerce 3) Facts-Answer 4) Science 58. Gifted : Intelligent : : Creative : ? 1) Artistic 2) Scientific 3) Productive-Answer 4) Repetitive 59. Triangle : Hexagon : : ? 1) Cone : Sphere 2) Rectangle : Octagon-Answer 3) Pentagon : Heptagon 4) Angle : wuadrilateral Directions (60-65) : In each of the following questions, select the one which is different from the other responses. 60. 1) 275 2) 324-Answer 3) 325 4) 381 61. 1) 2016 2) 3006 3) 3016 4) 3303-Answer 62. 1) 31-64-Answer 2) 26-48 3) 43-65 4) 34-56 63. 1) NML 2) QPO 3) HGE-Answer 4) XWV 64. 1) ABBC 2) HIIJ 3) PQQR 4) WYYZ-Answer 65. 1) MDPS 2) CNGH 3) FJILQ 4) OAUE-Answer 66. 1) Watch 2) Clock-Answer 3) Bangle 4) Bracelet 67. 1) Brotherhood-Answer 2) Babyhood 3) Adulthood 4) Childhood 68. 1) Pen - Ink-Answer 2) Pot - Water 3) Bottle - Tonic 4) Cylinder - Air Directions (69-70) : In each of the following questions, which one of the given responses would be a meaningful order of the following words in ascending order ? 69. 1) Mother 2) Infant-Answer 3) Milk 4) Crying 5) Smiling (1) 1, 5, 2, 4, 3 (2) 2, 4, 1, 3, 5 (3) 2, 5, 1, 3, 4 (4) 3, 2, 1, 5, 4 70. 1) Salary-Answer 2) Child 3) Employment 4) School 5) College (1) 2, 4, 5, 3, 1 (2) 3, 4, 1, 5, 2 (3) 4, 2, 1, 3, 5 (4) 5, 4, 1, 2, 3 71. Which one set of letters when sequentially placed at the gaps in the given letters series shall complete it? _ sr _ tr _ srs _ r _ srst _ 1) tissrr 2) tsrtsr 3) strtrs 4) tsttir-Answer 72. Which of the following words will come fourth if arranged according to the English Dictionary 1) Elect 2) Electric-Answer 3) Elector 4) Elastic Directions (73-76) : In each of the following questions choose the correct alternative from the given ones that will complete the series. 73. 206, 221, 251, 296, 356, ? 1) 416 2) 426 3) 431-Answer 4) 430 74. 5, 17, 37, 65, ?, 145 1) 95 2) 97 3) 99 4) 101-Answer 75. KEC, LFD, MGE, NHF, ? 1) OIF 2) OIG 3) PHG-Answer 4) NIG 76. ABCD, GHJI, MNPO,? 1) RSQP 2) STRQ 3) RSUT 4) STVU-Answer 77. Kusuma is the wife of Ravi. Govind and Prabhu are brothers. Govind is the brother of Ravi. Prabhu is Kusuma's 1) Cousin 2) Brother 3) Brother-in-law-Answer 4) Uncle 78. 20 years ago, Antony's age was 1/2 of what his age now is. What is his age now? 1) 20 years 2) 40 years-Answer 3) 35 years 4) 30 years 79. Find the wrong number in the given number series 3, 9, 27, 81, 243, 730 1) 27 2) 81 3) 243 4) 730-Answer 80. If in a certain code language INSTITUTION is coded as NOITUTITSNI, then how will PERFECTION be coded in that code language? 1) NOITEERPFC 2) NOITCEEREP-Answer 3) NOITCFERPE 4) NOTICEFRPE 81. If L denotes x, M denotes ÷, P denotes + and Q denotes -, then find the value of 16 P 25 M 8 Q 6 M 2 L 3 = ? 1) 6 2) 8 3) 10-Answer 4) 12 82. Some equation are solved on the basis of a certain system. Find out the correct answer for the unsolved equation on that basis If 8 + 8 = 72, 5 + 5 = 30 and 7 + 7 = 56, what is 6 + 6 = ? 1) 40 2) 42-Answer 3) 30 4) 36 83. Select the correct combination of mathematical signs to replace * signs and to balance the given equation 1) *, <, ÷ 2) *, >, ÷-Answer 3) ÷, >, * 4) *, >, + 84. From the following alternatives, select the word which cannot be formed using the letters of the given word CORRSPONDENT 1) SPEED 2) ORDER 3) SPOON 4) ARREST-Answer 85. From the following alternatives, select the word which can be formed using the letters of the given word CONTROVERSY 1) RIVER 2) STORY 3) OTHER 4) YOURS 86. If B-2, A-1, M-3, R = 5, E = 6, O = 7, the sum of the letters of which of the following words will give the highest number? 1) BORE 2) ROOM-Answer 3) MORE 4) RARE Directions (87-88) : In each of the following questions, find the missing number from the given responses. 87. 4 8 10 320 2 ? 3 180 9 6 4 216 1) 22 2) 30-Answer 3) 28 4) 20 88. 18 11 19 12 13 16 36 4 ? 1) 36 2) 9-Answer 3) 35 4) 7 89. Lalit is elder than Prakash and Kishore, Mukesh is elder than Rakesh but not as old as Lalit. Prakash is younger than Rakesh but is not the youngest. Who is the eldest? 1) Lalit-Answer 2) Mukesh 3) Rakesh 4) Kishore 90. John's house is 100 meter north of his uncle office. His uncle's house is located 200 meters west of his (uncle's) office. Kabir is the friend of John and he stays 100 meters east of John's house. The office of Kabir is located 100 meters south of his house. Then how far is his uncle's house from Kabir's office? 1) West -Answer 2) East-Answer 3) South 4) North 92. A house faces north. A man coming out of his house walked turned left and walked 25 meters. He then turned right and walked 5 meters and again turned right an walked 25 meters. How far is he from his house? 1) 15 meters-Answer 2) 55 meters 3) 60 meters 4) 65 meters Directions (93-94) : In each of the following questions, one or two statements are given followed by two conclusions I and II. You have to consider the statements to be true even if they seem to be at variance from commonly known facts. You are to decide which of the given conclusions definitely follows from the given statements. 93. Statements : A from with flowers looks beautiful Conclusions : I. Flowers are grown for decoration of rooms II. Room without flowers looks ugly 1) Only conclusion I follows 2) Only conclusion II follows 3) Both conclusion I and II follow 4) Neither conclusions I nor conclusion II follows-Answer 94. Statements : All students are boys No boy is dull Conclusions : I. There are no girls in the class II. No student is dull 1) Only conclusion I follows 2) Only conclusion II follows-Answer 3) Both conclusion I and II follow 4) Neither conclusions I nor conclusion II follows 95. A piece of paper is folder and cut as shown below in the question figures. From the given answer figures, indicate how it will appear when opened-Answer-1 96. Which of the answer figure exactly the mirror image given figure, when the n held on the line AB?-Answer-3 97. Which answer figure complete the pattern question figure?-Answer3 98. Select the answer figure in which the question figure is hidden/embedded-Answer-4 99. Which one of the following diagrams best depicts the relationship among Tiger, Lions and Animals?-Answer-3 100. A word is represented by only one set of numbers as given in any one of the alternative. The sets of numbers given in the alternatives are represented by two classes of alphabets as shown in two matrices given below. The columns and rows of Matrix I are numbered from 0 to 4 and that of Matrix II are numbered from 5 to 9. A letter from these matrices can be represented first by its row and next by its column, e.g. 'H' can be represented by 14, 20, 42, etc. and 'D' can be represented by 59, 65, 86, etc. Identify the set for the word NAIL. 1) 57, 87, 01, 43 2) 59, 58, 10, 12 3) 89, 57, 04, 41 4) 95, 87, 32, 21-Answer GENERAL AWARENESS 101. The preamble to our constitution makes no mention of 1) Justice 2) Fraternity 3) Equality of status 4) Adult franchise-Answer 102. Which one of the following is the popularity elected house of the Indian Parliament ? 1) Rajya Sabha 2) Lok Sabha-Answer 3) Lok Sabha as well as Rajya Sabha 4) None of the above 103. Which part of the constitution deals with the Directive Principles of State Policy? 1) Part-III 2) Part-IV-Answer 3) Part-I 4) Part-II 104. Gandhiji was influenced by the writing of 1) Karl Marx 2) Thomas Hobbes 3) Charles Darwin 4) Leo Tolstoy-Answer 105. The methods of democrate socialism are 1) Revolution 2) General strike 3) Sabotage 4) Persuasion and propaganda-Answer 106. The number of oil refineries working in the State of Assam is 1) One 2) Two 3) Three 4) Four-Answer 107. Per capita income is equal to National income 1) ___________ Total population of the country-Answer 2) National income + Population 3) National income - population 4) National income * population 108. Who appoints the Finance Commission ? 1) Finance Minister 2) Prime Minister 3) Speaker of the Lok Sabha 4) President-Answer 109. The Annpurna Scheme v implemented in the year 1) 1998 2) 1996 3) 1999 4) 2000-Answer 110. A favourable balance of of a country implies that 1) Imports are greater than exports 2) Exports are greater than imports-Answer 3) Both imports an exports are equal 4) Rising imports falling exports 111. Major iron and steel industry are located in the platear 1) Deccan 2) Malwa 3) Telangana 4) Chota Nagpur-Answer 112. The talles and thickest ty grass is 1) Alfalfa 2) Fodder 3) Bamboo-Answer 4) Lichens 113. Which of the following has its source outside In 1) Brahmaputra-Answer 2) Beas 3) Ravi 4) Jhclum 114. Majuli, the largest river land in the world, lies in the state of 1) Arunachal Pradesh 2) Assam-Answer 3) Tripura 4) Mizoram 115. Desertification can be checked by 1) Plugging gulties 2) Checking over grazing-Answer 3) Contour ploughing 4) Forming shelter belts 116. Worship of Mother Goddess was associated with 1) Aryan Civilization 2) Mediterranean Civilization 3) Indus Valley Civilization-Answer 4) Later Vedic Civilization 117. Alexander and Porus fought a battle at 1) Hydaspes-Answer 2) Jhelum 3) Panipat 4) Tarain 118. The Battle of Haldighati was fought between 1) Akbar and Rana Sangram Singh 2) Akbar and Medini Rai 3) Akbar and Rana Pratap Singh-Answer 4) Akbar and Uday Singh 119. The famous Peacock Throne of Shah Jahan was taken away in 1739 by 1) Afghan Invader Ahmed Shah Abdali 2) Persian Invader Nadir Shah-Answer 3) Mongol Invader Chengiz Khan 4) British East India Company 120. Identify the Buddhist Literature from the following : 1) Tripitakas-Answer 2) Upanishads 3) Angas 4) Aranyakas 121. The process of preventing the birds from flying is called 1) Brailing-Answer 2) Debeaking 3) Dubbing 4) Pecking 122. The Vikram Sarabhi Space Centre is located at 1) Sriharikota 2) Trivandrum-Answer 3) Tombay 4) Bangalore 123. The time period of a seconds pendulum is 1) 1 second 2) 2 second-Answer 3) 0.5 second 4) 1.5 second 124. Lakes freeze in cold countries in winter, leaving the water underneath at 1) 00 C 2) 00 F 3) 40 C-Answer 4) 40 F 125. Which one of the following determines the sharpness of image in a camera? 1) The aperture-Answer 2) The exposure time 3) The focal length of the lens 4) Size of the camera 126. Stainless steel is an alloy of 1) Chromium and carbon-Answer 2) Chromium, carbon and iron 3) Chromium and iron 4) Carbon and iron 127. Veins seen in the leaves, serve the function of 1) Photosynthesis 2) Transpiration-Answer 3) Storage 4) Conduction 128. The edible part of cabbage is 1) Fruit 2) Condensed flower 3) Vegetative bud-Answer 4) Inflorescence 129. Which of the following processes are associated with plants during dark period? 1) Photosynthesis and respiration 2) Respiration and transpiration 3) Transpiration and conduction 4) Conduction and respiration-Answer 130. In MRI machine, which one of the following is used? 1) Sound wave 2) 'X'-ray 3) Ultra-sound wave 4) Magnetic wave-Answer 131. For a person having hypermetropia, the near point is 1) Greater than 25 cm 2) Greater than 50 cm 3) Less than 25 cm 4) Infinity-Answer 132. Amount of water vapour in the atmosphere is measured in terms of 1) Humidity-Answer 2) Droplets 3) Smog 4) All of the above 133. For generation of biogas, the materials commonly used are 1) Animal wastes-Answer 2) Crop residues 3) Aquatic plants 4) Forest residues 134. The symbols used in an assembly language are 1) Codes 2) Mnemonics-Answer 3) Assembler 4) Macine codes 135. The NOSHADE' attributed in HTML 1) Defines the thickness of the line 2) Displays the lines in red 3) Displays the line in dark grey -Answer 4) Displays the example in red 136. Who invented vaccination ? 1) James Simpson 2) Edward Jenner-Answer 3) Alexander Fleming 4) Christian Barnard 137. Raw materials used for the manufacture of glass are 1) Sand, soda, limestone 2) Sand, charcoal, soda 3) Limestone, charcoal, sulphur 4) Sand, sulphur, soda-Answer 138. Goiter is caused by the deficiency of 1) Iodine-Answer 2) Chlorine 3) Sodium 4) Calcium 139. Which of the following is used in welding broken pieces of iron rails and machine parts? 1) Aluminum sulphate 2) Solder 3) Aluminum powder-Answer 4) None of the above 140. The total number of biosphere reserves present in India are 1) Eleven 2) Ten 3) Fifteen-Answer 4) Twelve 141. The BOD values of water indicate the 1) Amount of organic debris-Answer 2) Amount of oxygen used for biochemical oxidation 3) Amount of oxygen used for biochemical reduction 4) Amount of ozone used for biochemical oxidation 142. According to the census data released in July 2011, the fall in child sex ratio in rural areas as compared to urban areas is 1) Five times 2) Four times-Answer 3) Three times 4) Two times 143. The objective of 'Jawahar Rojgar Yojna' is to 1) Provide employment to youth in rural areas 2) Create employment opportunities for unemployed persons 3) Strengthen the rural economic and social structure 4) All of the above-Answer 144. Tummalapalle, where huge reserves of uranium have been found as announced by the Atomic Energy Commission in July, 2011, lies in the state of 1) Jharkhand 2) Andhra Pradesh-Answer 3) Tamil Nadu 4) Gujarat 145. Surat is situated on the banks of the river 1) Tapti-Answer 2) Mahanadi 3) Bhima 4) Godavari 146. The most densely populated State in India is 1) Uttar Pradesh 2) Bihar-Answer 3) West Bengal 4) Haryana 147. The formula 1 Indian Grand Prix was held on October 30, 2011 at 1) Delhi proper 2) Greater Noida-Answer 3) Gurgaon 4) Hyderabad 148. Out of which of the following States is Nautanki, a folk dance? 1) Haryana 2) Uttar Pradesh-Answer 3) Rajastan 4) Gujarat 149. Guru Gopinath was an exponent of 1) Kathak 2) Kathakali-Answer 3) Kuchipudi 4) Bharatnatyam 150. The Noble Prize for Physics for the year 2011 has been awarded to 1) Saul Perimutter, Brian P. Schmidt and Adam G. Riess-Answer 2) Bruce A. Beutler, Jules A. Hoffmann and Ralph M. Steinman 3) Christopher A. Sims and Thomas J. Sargent 4) Dan Schechtman FCI Aptitude and General awareness FCI previous years general awareness 1In which year did Gandhiji start Satyagraha Movement ? A) 1919-Answer B) 1927 C) 1934 D) 1942 2. An ecosystem consists of A) producers, consumers and decomposers in a particular area.-Answer B) all the plants and animals of an area. C) a living community and its environment. D) carnivorous and herbivorous of an area 3. The telephone was invented by A) G. Marconi B) Alexander Graham Bell-Answer C) J.L Baird D) Thomas Barrow 4. The crop mainly grown in hills is A) Sweet corn B) Sweet jowar C) Sweet potato D) Sweet pea 5. Who is called as the 'Prophet of New India ' A) Dayanand Saraswati B) Sri Ramakrishna-Answer D) Raja Ram Mohan Roy D) Swami Vivekananda 6. Ethnic group Mongoloids are found in India in A) Southern region B) South-central region C) North-western region D) North-eastern region-Answer 7. When a vibrating tuning fork is placed on table, a loud sound is heard. This is due to A) reflection B) refraction C) forced vibrations D) damped vibrations-Answer 8. An insect-catching plant is A) Australian Acacia B) Smilax C) Nepenthes-Answer D) Nerium 9. The method not used as a Biological control is A) Use of predators of a pest. B) Pheromone traps C) Use of pesticides D) Use of neem extracts 10. The States in India are demanding greater autonomy from the centre in the ___ field , A) Legislative B) Administrative C) Financial D) All the above-Answer 11. Light houses are places with powerful lights to A) guide and resolve traffic jams in crowded metro-cities during nights B) guide and help large crowds at religious gatherings during nights C) indicate to the incoming war-ships, the location of a harbour during night D) guide and warn the ships coming from different directions in the oceans.-Answer 12. In a Capitalistic Economy, the Prices are determined by A) Demand and Supply-Answer B) Government Authorities C) Buyers in the Market D) Sellers in the Market 13. Who of the following has not been an interlocutor on Jammu & Kashmir ? A) M.M. Ansari B) Radha Kumar C) Shujaat Bukhari-Answer D) Dilop Padgaonkar 14. The Zone of Earth's atmosphere immediately above its surface up to a height of about 16 kms above equator and 8 Kms over the poles is known as A) Mesosphere B) Thermosphere C) Troposphere-Answer D) Stratosphere 15. The term 'Pitcher' is associated with A) Wrestling B) Boxing C) Baseball-Answer D) Basketball 16. The filament of electric bulb is made up of A) Copper B) Nichrome C) Lead D) Tungsten-Answer 17. Which of the following is called ' Brown paper " A) Jute B) Cotton-Answer C) Rubber D) Tea 18. A Secular State is one which : A) has no religion of its own-Answer B) is irreligious C) is anti- religion D) takes into consideration the religious sentiments of the people 19. What does Jahangir means ? A) National Monarch B) The Grand Monarch C) Conqueror of the world-Answer D) Hero of hundred battles 20. Gol Gumbaz is in A) Konark B) Hyderabad C) Puri D) Bijapur-Answer 21. The early Buddhist scriptures were composed in A) Prakrit texts B) Pali texts-Answer C) Sanskrit texts D) Pictographical text 22. In Mohanjadaro, the Largest building is A) the great bath-Answer B) a granary C) the Pillared hall D) a two storeyed house 23. Where was the Royal Durbar held on November 1st 1858 to issue the Queen's proclamation ? A) Lucknow B) Kanpur D) Delhi-Answer D) Allahabad 24. Male(Anopheles) mosquito feeds on: A) Blood of man-Answer B) Nectar of flower C) Blood of Culex D) Blood of leech 25. Tooth paste is a product sold under : A) Monopolistic Competition-Answer B) Perfect Competition C) Monopoly D) Duopoly 26. The National Development Council includes : A) all central Cabinet Ministers-Answer B) Chief Ministers of all the States-Answer C) Cabinet Ministers of all the States and the Centre D) Members of the Estimates Committee of the Parliament 27. Which of the following is not a cause of low productivity in Indian agriculture A) Co-operative farming-Answer B) Inadequate inputs availability C) Sub-division and fragmentation of Land D) Poor finance and marketing facilities. 28. The gas that is used in the manufacture of vanaspati ghee is A) Oxygen B) Carbon dioxide C) Hydrogen-Answer D) Nitrogen 29. Cement is usually a mixture of A) Calcium silicate and calcium aluminate-Answer B) Calcium silicate and calcium ferrate C) Calcium aluminate and calcium ferrate D) Lime stone and silicon dioxide 30. Which of the following is not a Hardware ? A) Processor chip B) Printer C) Mouse D) Java-Answer 31. Scurvy is caused due to the deficiency of : A) Vitamin- D B) Vitamin - K C) Vitamin - E D) Vitamin - C-Answer 32. According to a study conducted by Hyderabad's National Institute of Nutrition, The healthiest of 14 fresh fruits commonly consumed in India with maximum' Goodness Index' is A) Indian Plum B) Mango C) Guava-Answer D) Custard apple 33. The Parliament can legislate on subjects given in the Union List only in consultation with the State Government for the State of : A) Assam B) Rajasthan C) Jammu & Kashmir-Answer D) Kerala 34. Tsunamis are waves generated by A) Earthquakes beneath the Sea-Answer B) Moon's pull C) High tides of the oceans D) Cyclones 35. Which is the largest State of India ? A) Madhya Pradesh B) Andhra Pradesh C) Rajasthan-Answer D) Maharashtra 36. Which one of the following is a system software A) Database programs B) Word processors-Answer C) Spreadsheets D) Compilers 37. A piece of wood is held under water. The upthrust on it will be : A) Equal to the weight of the wood-Answer B) less than weight of the wood C) more than weight of the wood D) zero 38. Kamakhya temple is an important place of tourism in the State of A) Tamil Nadu B) Assam-Answer C) Himachal Pradesh D) Manipur 39. Electric bulbs are filled with A) Nitrogen B) Carbon dioxide C) Argon-Answer D) Oxygen 40. The atmospheric gas that is mainly responsible for Green House effect: A) Ozone B) Oxygen C) Nitrogen D) Cabondioxide-Answer 41. Wisdom teeth is the A) 1st molar teeth B) 2nd molar teeth C) 3rd molar teeth-Answer D) 4th molar teeth 42. Who of the following pairs of Nobel Laureates in Physics was awarded 2010 Nobel Prize ? A) John C Mather, George F. Smoot B) Albert fert, Peter Grunberg C) David J Gross, Frank. Wilczek D) Andre Geim, Konstantin Novoselov-Answer 43. "Tabal Chongli" is a form of folk dance associate with the State of A) West Bengal B) Assam-Answer C) Andhra Pradesh D) Maharashtra 44. Who is the supreme Commander-in-Chief of armed forces of the country ? A) Defence Minister B) Prime Minister C) Senior-most among the three service Chiefs. D) President-Answer 45. Which of the following is correctly matched ? A) Assam-Itanagar B) Arunachal Pradesh-Guwahati C) Tripura-Agartala-Answer D) Nagaland-Shillong 46. The purest form of Iron is A) Cast iron B) Steel B) Pig iron D) Wrought iron-Answer 47. The caste system of India was created for : A) Immobility of labour B) recognition of the dignity of labour C) economic uplift D) occupational division of labour-Answer 48. The iron ore which contains 72 % of iron is A) Magnetite-Answer B) Limonite C) Haematite D) Siderite 49. Surat is located on the banks of the river : A) Narmada B) Sharavathi C) Mahi D) Tapti-Answer 50. The tusk of elephant is an enormously enlarged: A) Upper incisor-Answer B) Upper canine C) lower canine D) lower incisor 51. Megasthenes lived for many years at the court of the : a) Magadha kings b) Nanda kings c) Mauryas-Answer d) Guptas 52. As per the provisional data of the 2011 census of India, the gap in literacy between men and women stands reduced to percentage points: a) 10.8 b) 16.5-Answer c) 18.4 d) 20.3 53. The people of the Indus Valley worshiped : a) Brahma b) Vishnu c) Pashupathi-Answer d) Vayu 54. Haemoglobin of the blood forms carbonyl haemoglobin with: a) Carbon dioxide b) Carbon monoxide-Answer c) Sulphur dioxide d) Nitrogen dioxide 55. What should be the minimum size of a plane mirror to have the full image of a person, when he stands at a distance? a) same size as the person b) double the size of the person c) half the size of the person-Answer d) 1/4th of the size of the person 56. Banana leaf tears easily because : a) leaf blade is thin b) leaf blade has no veins c) leaf blade has veins arranged in parallel manner-Answer d) leaf blade has undeveloped veins 57. Who was the Judge against whom the Parliament initiated impeachment proceedings, but failed ultimately? a) Justice Bhagawati-Answer b) Justice Ajit Ray c) Justice Sikri d) Justice Ramaswami 58. A heavy and a lighter body of same size are dropped from a height. Which one of them will reach a) Lighter body b) Heavier body-Answer c) Both of them together d) Cannot be predicted 59. The Atomic Energy Commission was set up in : a) 1948-Answer b) 1950 c) 1952 d) 1960 60. The humidity of air is measured by the instrument called a) Hydrometer b) Hygrometer-Answer c) Seismometer d) Barometer 61. The term ‘Gambit’ is associated with: a) Chess-Answer b) Tennis c) Basketball d) Baseball 62. The most common indicator organism that indicates level of water pollution is : a) E.coli b) P.typhi c) C.vibrio d) Entamoeba-Answer 63. Which of the following States of India has the largest length of surface roads? a) Uttar Pradesh b) Andhra Pradesh c) Maharashtra-Answer d) Tamilnadu 64. Chhatrapati Shivaji’s Court was adorned by the Ministers called: a) Nayanmars b) Ashtadiggajas c) Navaratnas d) Ashtapradhans-Answer 65. Hot water bag is used for fomentation because: a) Water is a cheap liquid b) Water has high specific heat capacity-Answer c) Water has high latent heat d) Water is easily available 66. ‘Tappatikkali’ is a folk dance associated with the State of : a) Tamil Nadu b) Kerala-Answer c) Andhra Pradesh d) Gujarat 67. The Indian National Congress was founded in 1885 due to the initiative of : a) Gopal Krishna -Answer b) Dadabhai Naoroji c) W.C. Banerjee d) Allan Octavian Hume 68. From where are the words ‘Satyameva Jayate’ inscribed below the base plate of the emblem of India taken? a) Rigveda b) Mundaka Upanishad-Answer c) Padam Purana d) Ramayana 69. Real National Income is equal to the size of Population multiplied by : a) Per Capita Real Income-Answer b) Personal Income c) Personal Disposable Income d) Market Prices 70. The tree which releases oxygen even during night is : a) Neem b) Banyan c) Mango d) Pipal-Answer 71. Quagga is an extinct species of : a) Cat b) Horse c) Zebra-Answer d) Antelope 72. Martyr’s day is observed on the death anniversary of : a) Mahatma Gandhi-Answer b) Smt. Indira Gandhi c) Lal Bahadur Shastri d) Rajiv Gandhi 73. The expansion of the term HTML is: a) Higher Text Markup Language b) Higher Text Machine Language c) Hyper Text Machine Language d) Hyper Text Markup Language-Answer 74. The blood group that lacks antigen is : a) A b) B c) AB-Answer d) O 75. Lightning conductor is made of : a) Copper-Answer b) Glass c) Ebonite d) Plastic 76. The most abundant metal in the earth’s crust is : a) calcium b) aluminium-Answer c) iron d) magnesium 77. When Indian leaders stress upon making with indigenous resources, they advocate : a) stopping imports-Answer b) promoting exports c) self-reliance d) national pride 78. The maximum strength of Lok Sabha is _______. a) 543 b) 555 c) 553 d) 552-Answer 79. The golden fibre of India is a) Cotton b) Hemp c) Jute-Answer d) Silk 80. Which one of the following is known as ‘Brown Coal’? a) Bituminous b) Anthracite c) Peat d) Lignite-Answer 81. The new agricultural strategy called ‘Green Revolution’ was initiated in : a) 1947 b) 1951 c) 1965-Answer d) 1972 82. The State of Assam has : a) five National Parks and eleven wildlife sanctuaries-Answer b) three National Parks and nine wildlife sanctuaries c) three National Parks and eight wildlife sanctuaries d) two National Parks and six wildlife sanctuaries 83. The decision of how much to produce is determined by the individuals in a : a) Socialistic Economy b) Mixed Economy c) Capitalistic Economy-Answer d) All the above 84. The famous ‘Dilwara Temples’ are situated in : a) Rajasthan-Answer b) Maharashtra c) Madhya Pradesh d) Andhra Pradesh 85. To edit text read by an optical scanner, one needs ____________ to translate the image into ASCII characters a) an OMR b) an OCR-Answer c) a MICR d) a digitizer 86. In mammals, the cheek teeth are : a) Incisors-Answer b) Premolar c) Molar d) Premolar and Molar 87. Who discovered Insulin? a) William Harvey b) Louis Pasteur c) F.G. Banting-Answer d) Alexander Fleming 88. The term ‘operation flood’ refers to : a) Flood Control b) Milk Production-Answer c) Food grain Production d) Population Control 89. PVC is obtained by the polymerisation of : a) Propene b) Vinyl Chloride-Answer c) Styren d) Acetylene 90. The fundamental rights have : a) moral force behind them. b) force of public opinion behind them. c) legal force behind them.-Answer d) no force behind them. 91. Which of the following is correctly matched? a) Assam – Itanagar b) Arunachal Pradesh – Guwahati c) Manipur – Imphal-Answer d) Nagaland – Shilong 92. A candidate contesting for Lok Sabha must have attained the age of : a) 21 years b) 18 years c) 25 years-Answer d) 30 years 93. The second session of the Indian National Congress was presided over by : a) W.C. Banerjee b) Badruddin Tyabji c) Dadabhai Naoroji-Answer d. Surendra Nath Banerjee 94. For controlling pollution, dust escaping from factory chimneys is to be prevented. For this : a) metal grid or wire-mesh is placed over it. b) a metal grid coated with an insulator is placed on it. c) a wire-mesh coated with grease is placed on it d) an electrically charged wire-mesh is placed on it to attract dust.-Answer 95. Which of the following is the worst flood affected area in India? a) Assam-Answer b) Nagaland c) Manipur d) Tripura 96. Which of the following is correct about Socialism? a) State is a necessary evil. b) State is a march of God on earth. c) State is an unnecessary evil. d) State promotes common goods.-Answer 97. Full form of BCG is : a) Bacillus Cholera Germ b) Bacillus Calmette Guerin-Answer c) Bacillus Curative Gene d) Bacillus Cholera Guerin 98. The organic compounds present in petroleum are separated by : a) Distillation b) Fractional crystallisation c) Frractional distillation-Answer d) Sublimation 99. Who of the following is NOT a recepient of 2011 Nobel Peace Prize? a) Hu Jintao-Answer b) Leymah Gbowee c) Ellen Jhonson-Sirleaf d) Tawakkol Karman 100. The pH of human blood is : a) 2 b) 10 c) 5.5 d) 7.3-Answer FCI questions with answers 1. According to the concept of conservation, the stock in trade is valued at— (A) Cost price (B) Market price (C) Cost or market price which ever is higher (D) Cost or market price which ever is lower Ans. (D) 2. The concept of conservation will have the effect of— (A) Over statement of assets (B) Understatement of assets (C) Understatement of liabilities (D) Understatement of provision for bad and doubtful debts Ans. (B) 3. Non-financial information is not recorded in accounts due to— (A) Accrual concept (B) Entity concept (C) Dual aspect concept (D) Money measurement concept Ans. (D) 4. Balance in ‘Nazrana’s a/c’ in the books of lessee is shown in— (A) Landlords a/c (B) P&L a/c (C) Balance sheet assets side (D) Balance sheet liabilities side Ans. (C) 5. The hire-buyer charges depreciation on— (A) Cash price of the asset (B) Hire purchase price of the asset (C) Higher of the two (D) Lower of the two Ans. (A) 6. In hire-purchase system, hire-buyer can maintain his accounts under— (A) Asset accrued method (B) Total cash price method (C) Any of the two methods (D) None of these Ans. (C) 7. If the rate of gross profit for department X is 25% of cost, the amount of gross profit on sales of Rs. 100000 will be-— (A) Rs. 16667 (B) Rs. 20000 (C) Rs. 25000 (D) Rs. 33333 Ans. (B) 8. Provision for bad debts of a foreign branch is converted at— (A) Opening rate of exchange (B) Closing rate of exchange (C) Average rate of exchange (D) Rate applicable to debtors Ans. (B) 9. If goods are transferred from ‘X’ department to ‘Y’ department at cost +25%, the amount of stock reserve on closing stock of Rs. 20000 in ‘Y’ department will be— (A) Rs. 4000 (B) Rs. 5000 (C) Rs. 6000 (D) Rs. 3333 Ans. (A) 10. If out of the bills of Rs. 10000 discounted by the insolvent bills of Rs. 4000 are likely be dishonoured, unsecured creditors will include in respect of them an amount of— (A) Rs. 10000 (B) Rs.6000 (C) Rs. 4000 (D) None of the above Ans. (C) 11. Partnership firm engaged in banking business can have maximum— (A) 5 partners (B) 10 partners (C) 20 partners (D) Any number of partners Ans. (B) 12. Suppose, the partnership deed provides for a salary of Rs. 5000 p.m. to partner ‘X’. If ‘X’ withdraws only Rs. 3000 in a month, the remaining Rs. 2000 will be— (A) Debited to his capital a/c (B) Credited to his drawing a/c (C) Credited to his current a/c (D) Credited to P & L adjustment a/c Ans. (A) 13. In absence of any provisions in the partnership agreement, partners can charge on the loans given by them to the firm— (A) Interest at 6% p.a. (B) Interest at 12% p.a. (C) Interest at 15% p.a. (D) No interest Ans. (A) 14. A, B and C are partners sharing profits and losses in the ratio 4 : 3 : 2 D is admitted for 1/10th share, the new ratio will be— (A) 4 : 4 : 3 : 2 (B) 4: 3 : 2: 1 (C) 5 : 4 : 3 : 2 (D) None of the above Ans. (B) 15. A and B shared profit in the ratio of 3 : 2 C was admitted as a partner for 1/5th share. He acquires 3/20th from A and 1/20th from B. The new profit sharing ratio would be— (A) 10:6:4 (B) 6:10:4 (C) 8:8:4 (D) 9:7:4 Ans. (D) 16. Goodwill of a firm of A and B is valued at Rs. 60000. Goodwill appears in the books at Rs. 24000. C is admitted for 1/4th share. He will be required to bring for goodwill— (A) Rs.21000 (B) Rs.9000 (C) Rs. 15000 (D) Rs. 6000 Ans. (D) 17. A plant worth Rs. 800000 has been insured for Rs. 600000. The loss on account of fire is Rs. 500000. The insurance company under average clause will bear the loss to extent of— (A) Rs. 500000 (C) Rs. 800000 (B) Rs.600000 (D) Rs. 375000 Ans. (A) 18. It is not an item of Income with reference to a voyage a/c— (A) Passage money (B) Freight (C) Primage (D) Address commission Ans. (D) 19. The formats of the profit and loss account and Balance sheet in the case of a banking company have been revised w.e.f.— (A) 1st April 1949 (B) 1st April 1991 (C) 1st April 1992 (D) 1st April 1956 Ans. (C) 20. If accured outstanding premium is given in the trial balance of a general insurance company, then it will be shown in— (A) Revenue Account (B) Balance Sheet (C) Both revenue Account & Balance Sheet (D) None of the above Ans. (B) 21. Inventory is valued at lower of the cost or net realisable value on account of the accounting principle of— (A) Realisation (B) Consistency (C) Conservatism (D) None of the above Ans. (A) 22. In the period of rising prices, LIFO method may result in— (A) Lowering the profit (B) Raising the profit (C) Raising the tax liability (D) None of the above Ans. (B) 23. Given, Total assets turnover 4 Net Profits 10% Total Assets Rs. 50000 Net profit will be— (A) Rs. 15000 (B) Rs. 10000 (C) Rs. 25000 (D) Rs.20000 Ans. (D) 24. A company auditor addresses his audit report to- (A) Board of directors (B) Members (C) Managing director (D) Company secretary Ans. (B) 25. Bonus shares means shares issued to— (A) Workers (B) Existing equity shareholders (C) Preferential shareholders in lieu of dividend (D) Debenture holders in lieu of interest Ans. (B) 26. Which of the following does not call for physical verification? (A) Stock (B) Plant (C) Loose tools (D) Goodwill Ans. (D) 27. Cash from operations is equal to— (A) Net profit + increase in current assets (B) Net profit + decrease in current liabilities (C) Profit from operation ± Adjustment of increase and decrease in current assets and liabilities (D) Fund from operation ± Adjustment of increase and decrease in current assets and current liabilities Ans. (C) 28. Average profit of a firm is Rs. 9000 Firm’s capital is Rs. 60000 and normal return on business is expected at 10%. The goodwill by capitalisation method will be- (A) Rs. 30000 (B) Rs. 20000 (C) Rs. 25000 (D) Rs. 40000 Ans. (A) 29. Which of the following is not a current liability? (A) Bank overdraft (B) Redeemable debentures (C) Account payable (D) Provision for bad debts Ans. (B) 30. Amount of under writing commission payable on the issue of debentures is limited to— (A) 2% (B) 2.5% (C) 3% (D) 5% Ans. (B) 31. Which of the following acid test ratio can be said to be satisfactory? (A) 2: 1 (C) 1: 1 (B) 1: 2 (D) None of these Ans. (C) 32. The two factor theory of motivation was propounded by— (A) Peter Drucker (B) Herzberg (C) McGregor (D) Maslow Ans. (B) 33. Which of the following leadership styles is most commonly found now a day? (A) Autocratic (B) Democratic (C) Free rein (D) Participative Ans. (B) 34. Deciding in advance what is to be done in future is called— (A) Management (B) Coordination (C) Planning (D) Decision-making Ans. (C) 35. The organisation structure where there is direct vertical relationship is called— (A) Line organisation (B) Chain organisation (C) Command organisation (D) All the above Ans. (D) 36. When managers devote their attention only to those events where results are highly deviated from normal; ft is called— (A) Management by objective (B) Management by exception (C) Management by crisis (D) Management by choice Ans. (B) 37. Under delegation of authority— (A) Authority is given to subordinates (B) Authority flows from top to bottom (C) Delegator of authority is not received of accountability (D) All the above happens Ans. (D) 38. The process of determining by observation and study and reporting pertinent information relating to the nature of specific job is called— (A) Job specification (B) Job evaluation (C) Job analysis (D) Job description Ans. (C) 39. In case of a private company— (A) There is restriction on the right to transfer of shares (B) The number of members is restricted (C) Invitation to public for the subscription of shares is prohibited (D) All of the above Ans. (D) 40. A person at anyone time can not be DIRECT of more than— (A) 1 Company (B) 7 Companies (C) 15 Companies (D) 20 Companies Ans. (C) 41. Certificate of commencement of business is not required by a— (A) Public company (B) Any type of company (C) Private company (D) Private company subsidiary to a public company Ans. (C) 42. Henry Fayol is known for— (A) Scientific management (B) Rationalisation (C) Industrial psychology (D) Principles of managements Ans. (D) 43. The principle of unity of command’ implies— (A) Unity of thought and action (B) Unity amongst subordinates (C) Instructions from staff authority (D) Instructions from line authority Ans. (D) 44. A company has equity capital of Rs. 200000. Preference capital of Rs. 100000, 12% debentures of Rs. 100000, long term loan of Rs. 200000 and short term loan of Rs. 100000. The capital gearing ratio will be— (A) 1:1 (B) 0.5: 1 (C) 0.4: 1 (D) None of the above Ans. (C) 45. In case of a company, total assets less outside liabilities is called— (A) Net working capital (B) Gross working capital (C) Deferred liabilities (D) Net worth Ans. (D) 46. If opening sales is Rs. 10000 purchases Rs. 30000 direct expenses Rs. 4000 and closing stock Rs. 5000 the costs of goods is sold would be— (A) Rs. 39000 (B) Rs. 40000 (C) Rs.41000 (D) Rs. 44000 Ans. (A) 47. Premium on issue of shares is shown in balance sheet as— (A) An asset (B) A liability (C) An expense (D) A revenue Ans. (B) 48. Which of the following is known as ‘backbone of auditing’? (A) Verification of assets (B) Internal check (C) Vouching (D) Internal audit Ans. (C) 49. A limited company? 100 shares of Rs. 10/- cash fully called up on which Rs. 4/- per share was paid up. The company reissued 50 shares at the rates of Rs. 8/- each. The amount transferred to capital reserve will be— (A) Rs. 500 (B) Rs.200 (C) Rs. 250 (D) Rs. 100 Ans. (D) 50. Sale of long term investments indicates— (A) A change in current assets (B) Application of funds (C) Increase in working capital (D) Source of funds Ans. (D) 51. Net working capital refers to— (A) Current assets (B) Current assets minus current liabilities (C) Equity share capital minus fixed assets (D) Retired earnings. Ans. (B) 52. If sales Rs. 6000 gross profit is 1/3 on cost, purchases are R. 4900 and the closing stock is Rs. 900, the opening stock will be— (A) Rs.400 (B) Rs.500 (C) Rs. 1100 (D) Rs. 2000 Ans. (B) 53. The primary objective of audit is— (A) Detection and prevention of frauds (B) Detection and prevention of errors (C) Detection of frauds and errors (D) To ensure the final accounts and statements exhibit true and fair position of business Ans. (D) 54. ‘An auditor is a watch dog and not a blood hound’. This was observed in case of— (A) London oil storage company (B) Kingston cotton Mills Limited (C) London General Bank (D) Delightful Cigarette Company Ltd. Ans. (B) 55. If two or more sugar mills combine together, it is known— (A) Horizontal combination (B) Vertical combination (C) Lateral combination (D) None of the above Ans. (A) 56. Surrender value is related to— (A) Marine insurance (B) General insurance (C) Life insurance (D) Fire insurance Ans. (C) 57. Over capitalisation refer to— (A) Excess of capital (B) Excess rate of dividend payment, on shares (C) Over estimation of rate of capitalisation (D) Raising more capital than is warranted by its earning power Ans. (D) 58. Motivation refers to— (A) Coordinate the people (B) Guide the working people (C) Terrorise the people (D) Inducing people to work willing by Ans. (D) 59. Which of the following is not a barrier in communication— (A) Fear and distrust (B) Affection (C) Perception (D) Noise Ans. (B) 60. “Management is an art of getting things done through and with formally organised group.” This definition has been by— (A) Peter Drucker (B) Henry Fayol (C) Harod Koontz (D) F. W. Taylor Ans. (C) 61. Foreign exchange for import of goods is sanctioned by— (A) Exim Bank (B) Reserve Bank of India (C) State Bank (D) Ministry of commerce Ans. (B) 62. The cost of a machine having a span of life of 5 years is Rs. 10000. It has a scrap value of Rs. 1000. The amount of depreciation in the first year under the sum of year’s digit method will be— (A) Rs. 1600 (B) Rs. 1800 (C) Rs. 2000 (D) Rs. 3000 Ans. (D) 63. Given: Gross profit Rs. 60000 Gross profit ratio: 20% Debtor’s velocity 2 months The amount of debtors will be— (A) Rs. 30000 (B) Rs. 50000 (C) Rs. 120000 (D) Rs.200000 Ans. (B) 64. Premium on issue of shares can be used for— (A) Issue of Bonus shares (B) Payment of Dividends (C) Payment of operating expenses (D) Redemption of debentures Ans. (A) 65. If current ratio is 25, quick ratio (1) 5 and net working capital Rs. 15000. This value of inventory will be— (A) Rs. 10000 (B) Rs. 15000 (C) Rs. 37500 (D) Rs. 52500 Ans. (A) 66. Unclaimed dividend is shown on the liability side of the balance sheet under the heading— (A) Revenue and surplus (B) Provisions (C) Current liabilities (D) Miscellaneous items Ans. (C) 67. Accounting standards in India are prescribed by— (A) Company Law Board (B) Institute of charted accountants of India (C) Institute of coat and works accountants of India (D) Indian standard Board Ans. (B) 68. Which of the following is not correct— (A) Purchase + Opening stock—Cost of goods sold = Closing stock (B) Opening stock + Purchases — Closing stock = Cost of goods sold (C) Closing stock + Cost of goods sold — Purchases = Opening stock (D) Cost of goods sold — Closing stock + Purchases = Opening stock Ans. (D) 69. A company bought assets worth Rs. 360000 and in lieu issued debentures of Rs. 100 each at a discount of 10%. The number of debentures issued will be— (A) 3000 (B) 3600 (C) 3960 (D) 400 Ans. (D) 70. A person got insured his goods worth Rs. 10000 for Rs. 80QO against fire. Loss by fire to him was Rs. 9000. He can claim— (A) Rs. 8000 (B) Rs. 9000 (C) Rs. 10000 (D) Rs. 7200 Ans. (D) 71. The data obtained from a newspaper are— (A) Primary data (B) Secondary data (C) Both (A) and (B) (D) None of these Ans. (B) 72. Current Ratio of a firm is 3: 1 and working capital is Rs. 60000. What will be the amount of current Assets— (A) Rs. 30000 (B) Rs90000 (C) Rs. 120000 (D) Rs. 180000 Ans. (B) 73. The income from that house property is taxable under the head “Income from House property”. (A) The assessee has ownership on that house (B) The assessee uses that house for his business (C) The assessee himself lives in that house (D) The assessee has let out that house on rent for residence Ans. (A) 74. Following is the deduction in respect of repair under the head of income from house property— (A) 25% of Annual Value (B) 30% of Net Annual Value (C) 1/5 of Annual Value (D) 1/5 of Net Annual Value Ans. (B) 75. Single entry system can not be maintained by— (A) Sole proprietorship (B) Partnership concerns (C) Joint stock company (D) All of these Ans. (D) 76. Cash account will not be affected by— (A) Cash paid to creditors (B) Discount received (C) Cash sales (D) Cash received from debtors Ans. (B) 77. Maximum standard Deduction for employees getting gross salary not more than Rs. 100000 is allowed— (A) Rs. 20000 (B) Rs. 25000 (C) Rs. 30000 (D) Rs. 33000 Ans. (C) 78. Tax audit is compulsory in case of a person carrying on business whose gross receipt) turnover/sales and exceeds— (A) Rs. 50 Lakhs (B) Rs. 40 Lakhs (C) Rs. l0 Lakhs (D) Rs. 25 Lakhs Ans. (B) 79. Mr. Rastogi of Meerut was declared insolvent. One of his liabilities related to one months rent due to his landlord. This liability will be treated as— (A) Preferential Creditor (B) Partly Secured Creditor (C) Fully Secured Creditor (D) Unsecured Creditor Ans. (A) 80. When two or more companies liquidate to form a new company. It is called— (A) Amalgamation (B) Absorption (C) Reconstruction (D) Purchase of Business Ans. (A) 81. A, B and C are partners in a firm. If D is to be admitted to the firm as a new partner— (A) Old firm has to be dissolved (B) Old partnership has to be dissolved (C) Both the old firm and the old partnership have to be dissolved (D) No need to dissolve either firm or the partnership Ans. (D) 82. Remington sold one typewriter to Ramesh Chandra under installment purchase system on 1st January 1996, payment for which was to be made as under— On 01.01.1996 - Rs. 3000 On 31.12.1996 - Rs. 1700 On 31.12.1997 - Rs. 3600 On 31.12.1998 - Rs. 2300 On 31.12.1999 - Rs. 1100 Interest at 10% p.a. is included in each installment. The total interest charged amount to- (A) Rs.2100 (B) Rs. 1950 (C) Rs. 1800 (D) Rs. 1700 Ans. (B) 83. The liquidator of a company is entit1edio a remuneration of 2% on assets realised, and 3% on the amount distributed to unsecured creditors. The assets realised Rs. 100000 including cash balance of Rs. 3000. Amount available for distribution to unsecured creditors before paying liquidators remui4eration was Rs. 46350 liquidator’s remuneration will be— (A) Rs.3100 (C) Rs. 3290 (B) Rs.3140 (D) Rs. 3350 Ans. (C) 84. EXIM Bank was established on— (A) 1st Dec 1984 (B) 1st June 1985 (C) 1st Jan 1982 (D) 1st July 1980 Ans. (C) 85. For capital gain being long term capital gain, an assessee should retain the assets for a period of— (A) 40 months (B) 36 months (C) More than 36 months (D) Less than 36 months Ans. (C) 86. The salary received by a member of parliament is— (A) Exempt from Tax (B) Taxable under the head of salary (C) Taxable under the head of other sources (D) Taxable under the lead of business Ans. (C) 87. For the purpose of income tax it is necessary for agriculture income that— (A) Land should be used for agricultural activities (B) Land should be used for godown (C) Land should be used for irrigation (D) None of above Ans. (A) 88. Depreciation is allowed on— (A) Tangible Assets (B) Current Assets (C) Fixed Assets (D) Intangible Assets Ans. (C) 89. When shares are forfeited, the share capital account is debited by— (A) Nominal value of forfeited shares (B) Paid up amount of forfeited shares (C) Called up amount on forfeited shares (D) Forfeited amount of shares Ans. (C) 90. The term ‘POIM’ for the four functions of management. Planning, organisation, integration and measuring was given by.— (A) F.W. Taylor (B) Harold Smiddy (C) Tanon Brown (D) Peter F. Drucker Ans. (B) 1. Kinetic energy is converted into electrical energy in (A) Electric Motor (B) Dynamo (C) Electromagnet (D) Ammeter B 2. Floppy disk is an example of (A) Read Only Memory (B) Random Access Memory (C) Cache Memory (D) Secondary Storage Memory D 3. Which of the following compounds are sprayed over clouds to cause artificial rain? (A)Ice (B) NaOH (C)AgI (D) NH4CL C 4. Iodine deficiency in diet is know to cause (A)rickets (B) obesity (C)scurvy (D) goitre D 5. The refrigeration of the food articles keeps them fresh because (A) water forms crystals (B) chemical reaction is slowed down (C) chemical reaction take place faster (D) bacteria are killed D 6. Which substance is called the ‘liquid gold’? (A) Gold as a liquid form (B) Sodium as a liquid form (C) Mustard oil (D) Petroleum oil D 7. Percentage of silver in German silver is (A)25°/o (B) 0% (C)75°/o (D) 80% There is no Silver 8. Which type of compounds are sugar and common salt? (A) Both are organic compounds (B) Both are inorganic compounds (C) Sugar is an organic compound and common salt is an inorganic compound (D) Sugar is inorganic compound and common salt is an organic compound C 9. The chemical name of Vitamin ‘E’ is (A) Ascorbic acid (B) Retinol (C) Tocopherol (D) Thiamine C 10. Which of the following gases has bleaching property (A) Carbon dioxide (B) Carbon Monoxide (C) Chlorine (D) Hydrogen C 11. The heaviest body of our Solar system is (A)Sun (B) Uranus (C)Jupiter (D) Saturn A 12. From amongst the following; Name famous Indian ornithologist (A) Dr. Salim Ali (B) Dr. Gopalasamudram N. Ramachandran (C) Dr. J.B.S. Haldane (D) Dt H.G. Khorana a 13. Who performed the longest space journey in the year 2007 (A). Venus Williams (B) Serena Willlams (C) Sunita William (D) Sir John William C 14. ‘The Lost Child’ was written by (A) Nirad C. Chowdlltiury (B) Mulk Raj Anand (C) Khushwant Singh (D) Annie Besant B 15. Dr. MS. Swaminathan has distinguished himself in which of the following fields ? (A) Nuclear Physics (B) Agriculture (C) Astrophysics (D) Medicine B 16. The Headquarters of UNESCO at (A)Rome (B) Geneva (C)Paris (D) New York C 17. January 15 is celebrated as the (A) Army Day (B) Martyr’s Day (C) Independence Day (D)) Ugadhi A 18. Bijapur is known for its (A) Heavy rainfall (B) Rock Temple (C) Gol Gumbaj (D) Statue of Gomateshwara C 19. The Americans are also known as (A) Kiwis (B) Yankees (C) Tories (D) None of the above B 20. India lifted the ONGC Nehru Cup, 2009, after defeating in the final (A)Syria (B) Pakistan (C)Sri Lanka (D) Bangladesh A 21. An element which does not react with oxygen is (A)Chlorine (B) Iodine (C)Helium (D) Nitrogen C 22. During sleep, man’s blood pressure (A) increases (B) decreases (C) fluctuates (D) remains constant B 23. Which Indian fllrnstar was recently conferred Honorary Doctorate by Bedfordshire University (U.K.) ? (A) Amitabh Bachchan (B) Shah Rukh Khan (C) Orn Puri (D) Aamir Khan B 24. Williams Sisters won the U.S. Open Women’s Doubles Title 2009 after defeating in the final (A) Leizal Huber and Cara Black (B) Kim Clijsters and Anna Kournikova (C) Caroline Woaniacki and Dinara Safina (D) Nathalie Deshy and Sania Mirza A 25. Which one of the following is a military alliance ? (A) ASEAN (B) SAARC (C)NATO (D) NAFTA C 26. The recipient of the 42nd Jnanpeeth Award is (A) Manohar Shastri (B) Harish Pandya (C) Satya Vrat Shastri (D) K Kamal Kumar C 27. World Development Report is an annual publication of (A)UNICEF (B) UNDP (C)WTO (D) World Bank D 28. Which one of the following is correct ? Player Sport (A) Jeev Milkha — Tennis Singh (B) Jhulan — Cricket Goswami (C) Baichung Bhutia — Hockey (D) Pankaj Advani — Badminton B 29. Which one of the following industrialists was declared ‘The Business Person of the Year 2008’ by the Times of India Survey ? (A) Anil Ambani (B) Rahul Bajaj (C) Ratan Tata (D) Gautam Adani C 30. Naina Devi peak forms a part of (A) Himalayan range located in Sikkim (B) Himalayan range located in Kumaon region (C) Himalayan range located in Nepal (D) Himalayan range located in Jammu & Kashmir B -------------------------------------------------------------------------------------------------------------------------------------------------------------------------------------------------------- 51.The Criminal Procedure Code (Amendment) Act, 2008 came into effect on December 31, 2009. It incorporates the recommendations of_to prevent overcrowding of jails with undertrials. a. The Law Commission b. The Justice Mallmath Committee's report c. The guidelines issued by the Supreme Court Select the correct answer using the codes given below: (1) All of the above -Answer (2) Only a (3) Only b (4) Both b and c (5) Both a and b 52.__on December 31, 2009, became the first Indian woman to ski to the South Pole when she crossed a 900 km Antarctic ice trek to reach the South Pole to mark the 60th anniversary of the founding of the Commonwealth. (1) AvaniMathur (2) Deepika Sharma (3) Amita Chauhan (4) Reena Kaushal -Answer (5) MadhumitaSolanki 53. The Indian Science Award, instituted by the Department of Science and Technology of the Union government, was given by Prime Minister Manmohan Singh to C. R Rao, statistician, at the 97th Indian Science Congress in_on January 3, 2010. (1) Kochi (2) Chennai (3) Thiruvananthapuram -Answer (4) Pune (5) Kolkata 54. New Year began on a bloody note in Pakistan as seventyfive people were killed and over 50 injured in Shah Hasan Khan, near Lakki Marwat in the _ district of the NorthWestern Frontier Province. (1) Chitral (2) Dera Ismail (3) Bannu -Answer (4) Peshawar (5) Hamirpur 55. Russia and the United States, on _, 2009, missed the deadline to sign a replacement to the Strategic Arms Reduction Treaty (START) which expired at midnight. (1) December 1 (2) December 5 -Answer (3) December 25 (4) December 31 (5) December 14 56. The Mojave Desert, on December 7,2009, played host to a very modern spectacle when Sir Richard Branson unveiled Virgin Galactic's SpaceShipTwo for carrying passengers to the brink of space. The Mojave Desert is covered the most in_. (1) Utah (2) California -Answer (3) Nevada (4) Arizona (5) None of the above 57. The Supreme Court of which of the following countries, on January 3. 2010, ratified a 25year prison sentence for the former President, Alberto Fujimori? (1) Peru -Answer (2) Chile (3) South Korea (4) Taiwan ^ (5) Cambodia 58. The Rajya Sabha on December 1, 2009, unanimously approved, by voice vote, the Workmen's Compensation (Amendment) Bill, 2009. Which of the following s/ i are correct regarding the Bill? a. It seeks to give higher compensation to workers and their families in the event of injury or death. b. It empowers the Centre to enhance the compensation and funeral expenses, by notification, from time to time. c. It empowers the government to specify, by notification, monthly wages for an employee. Select the correct answer using the codes given below. (1) Only a (2) Only b (3) Only c (4) All of the above -Answer (5) None of these 59. Multilateral funding agency of the Asian Development Bank, on December 3, 2009, approved 200million dollar loan to finance a road project in_. (1) Orissa (2) Jharkhand -Answer (3) Bihar (4) West Bengal (5) Chhattisgarh 60. Consider the following statements related to the selection of Vihaan Networks Ltd., a Shyam Group company, as "Technology Pioneers2010." a. It was selected by the World Economic Forum as a pioneer. b. It was chosen for its innovative and pioneering work in developing solarpowered mobile phone base stations WorldGSM — foi use in rural areas. c. Vihaan Networks is the only company to have found a way ol building sustainable phone networks for 3 billion people in rural areas. Which of the statements giver above is/are not correct? Selecl the correct answer using th< codes given below: (1) Only a (2) Both b and c (3) Both a and b (4) All of the above (5) None of these -Answer 61. The Union Cabinet on Decembei 10, 2009, approved amendments to the Energy Conservation Act _______, to introduce the system o issuing energy saving certificate! to be traded in the domestic mar ket. (1) 2008 (2) 2006 (3) 2004 (4) 2002 (5) 2001 62. The Haryana Power Generation Corporation, on December 24, 2009, became the first State sector power generation utility in the country to get certified for_for its power stations at Yamunanagar and Panipat and corporate office at Panchkula. a. ISO: 9001 b. ISO: 14001 c.OHSAS: 18001 Select the correct answer using the codes given below: (1) All of the above -Answer (2) Both a and c (3) Both a and b (4) Both b and c (5) None of these 63. ONGC. on December 1, 2009, signed agreements to pick up____per cent interest in Phase 12 of the gigantic South Pars gas field and get_per cent of Iran LNG's project that will convert the gas into liquefied natural gas for exports. (1)20,40 (2)30,30 (3) 40. 20 -Answer (4) 50. JP (5) 15. 45 64. Reliance Exploration and Production DMCC a wholly owned subsidiary of Reliance Industries Ltd. (RIL), and Ecopetrol, on December 4, 2009. signed the Farmout Agreements, for Borojo_in Colombia. (1) North Block 42 (2) South Block 43 (3) West Block 44 (4) Only 1 and 2 -Answer (5) None of these 65. Japan's government, on December 8, 2009, unveiled $81 billion of new stimulus spending to keep the world's_biggest economy from lurching back into recession. (1) Second -Answer (2) Third (3) Fourth (4) Fifth (5) Sixth 66. Reliance Communications, on December 12, 2009, bagged mGovernance infrastructure development contracts in_circles to offer integration of mobile technology to seamlessly link various government departments to generate information systems. (1) Kerala (2) Maharashtra (3) Mumbai (4) All of the above -Answer (5) Only 2 and 3 67. The Central Government, on December 18. 2009, pegged the country's GDP (gross domestic product) growth for the current fiscal at over_per cent. (1) 7.25 (2) 7.45 (3) 7.55 (4) 7.75 -Answer (5) 8.25 68. Which of the following major economy related development(s) took place on December 22, 2009? a. An agreement was reached between the oil marketing companies and millers on the price of ethanol. b. RIL announce a third successive gas discovery in the D3 deepsea block in the KrishnaGodavari basin. c. Central Government inks two loan agreements with the Asian Development Bank for funding the country's infrastructure projects. Select the correct answer using the codes given below: (1) Both a and b (2) Both a and c (3) All of the above -Answer (4) Both b and c (5) None of these 69. On which recent date did the Bombay Stock Exchange sensitive index, Sensex, the barometer of domestic stock market, enter its Silver Jubilee year? (1) December 31, 2009 (2) December 1, 2009 (3) January 2, 2010 -Answer (4) January 1, 2010 (5) January 4, 2010 70. India, on January 1, 2010, liberalised its trade with_by slashing duties on several products like seafood, chemicals and apparel among others. (1) South Korea (2) Singapore (3) Thailand (4) Malaysia (5) All of the above -Answer 71. As per the data released by the Indian Institute of Foreign Trade on January 1, 2010, after a gap of 13 months, exports turned positive in November 2009. registering _ per cent growth at $13.19 billion against $11.16billion in November 200809. (1) 11.5 (2) 18.2 -Answer (3) 16.6 (4) 17.5 (5) 19.4 72. SEBI. on December 31. 2009, allowed _to accept application supported blocked amount (ASBAs) to enable better participation by corporate investors and high networth individuals (HINs) in initial public offerings or rights issues. (1) State Bank of India (2) ICICI Bank (3) 14 banks (4) All of the above -Answer (5) None of these 73. Popular Assamese storyteller, novelist, lyricist and playwright, _, was chosen for the 20th Assam Valley Literary Award for the year 2009 for his literary excellence on December 31, 2009. (1) IqbalHusain (2) ImranShah -Answer (3) Abhijit Ghosal (4) Tapan Bhattacharjee (5) MrinalSanyal 74. Who among the following Britons of Indian origin were conferred with knighthood by Queen Elizabeth of England on December 31, 2009? a. Mota Singh b. Paramjit Singh Bassi c. Gumain Singh Select the correct answer using the codes given below: (1) All of the above (2) Both a and b -Answer (3) Both a and c . (4) Both b and c (5) Only b 75. Who among the following was/ were conferred the 2009 Karmaveer Puraskar on December 2, 2009? (1) K. Srinath Reddy (2) Manju Bharat Ram (3) RohiniNilekaniandTarunTejpal (4) Gregory Roberts (5) All of the above -Answer 76. Scientist Yash Pal and Project Director of Chandrayaan1 M. Annadurai were conferred with the H.K. Firodia awards for 2009 in Pune on December 23, 2009. Who among the following was/ were the early recipients? (1) APJ Abdul Kalam & Vijay Bhatkar (2) Anil Kakodkar and Raghunath Mashelkar (3) M.S. Swaminathan, KKasturirangan and R. Chidambaram (4) CNR Rao, Obaid Siddiqi and P N Tandon (5) All of the above -Answer 77. The Union Government, on December 30, 2009, decided to come up with a National Consumer Policy to_. a. Ensure uniform national and international standards in the various arms of the Central and State governments b. Ensure uniform national and in . temational standards in the regulatory bodies and on consumer fora c.To lay down the guiding principles of complaint resolution. Which of the statements given above is/are correct? Select the correct answer using the codes given below: (1) Both a and b (2) Only b (3) Only c (4) All of the above -Answer (5) None of these 78. Which state Government, on January 1, 2010, imposed a ban on holding strike in all power corporations under energy department for six months in public interest? (1) Delhi (2) Uttar Pradesh -Answer (3) West Bengal (4) Andhra Pradesh (5) Bihar 79. A work by Edgar Degas, called Les Choristes (The Chorus), was stolen from Musee Cantini, an exhibition in_, by thieves on December 31, 2009 (1) Marseille -Answer (2) Vienna (3) Lyon (4) Paris (5) Clipperton Island 80. Britain, on January 2, 2010, called a summit of world leaders in London later in January 2010 to discuss the terror threat posed by _, seen as new breeding ground(s) for alQaeda linked extremists. (1) Nigeria (2) Yemen -Answer (3) Syria (4) Lebanon (5) All of the above 81. Valueadded services means (1) giving full value for money (2) better value for better price (3) costlier service (4) additional service -Answer (5) All of these 82. "POS" means (in marketing) (1) Preparation for Sales (2) Point of Superiority (3) Point of Sales -Answer (4) Primary Outlook of Salesman (5) Position of Sales 83. Effective Marketing helps in (1) boosting the purchases (2) boosting the sales -Answer f (3) diversified business' (4) realisation of dreams (5) All of these 84. A 'Buyer's Market' means (1) buyers are also sellers (2) sellers are also buyers (3) there are not sellers (4) demand exceeds supply (5) supply exceeds demand -Answer 85. The sequence of a sales process is (1) a call, a lead, presentation and sale (2) a lead, a call, presentation and sale -Answer (3) presentation, sale, lead and call (4) presentation, lead, sale and call (5) sale, call, lead and presentation 86. A presentation means (1) display of products (2) explaining the utility of products -Answer (3) a gift (4) display of communication skills (5) All of these 87. A 'lead' means (1) a buyer (2) a seller (3) a company intending to sell its products (4) a prospective buyer -Answer (5) a disinterested buyer 88. 'Benchmark'means (1) products line up on bench (2) salesmen sitting on a bench (3) set standards^ -Answer (4) marks on a bench (5) None of these 89. 'Customisation' means (1) customers'personal accounts (2) customers selling goods (3) special products for each customer -Answer (4) better relations' (5) All of these 90. Customer Retention means (1) retaining the customers at the Bank for the full day (2) quick disposal (3) customers dealing with the same bank for a long time -Answer (4) better standards (5) All of these 91. This component is required to process data into information and consists of integrated circuits. (1) Hard disk (2) RAM (3) CPU -Answer 4) ROM (5) None of these 92. One advantage of dialup Internet access is (1) it utilizes broadband technology (2) it utilizes existing telephone service (3) it uses a router for security -Answer (4) modem speeds are very fast (5) None of these 93. What is backup ? (1) Adding more components to your network (2) Protecting data by copying it from the original source to a different destination -Answer (3) Filtering old data from the new data (4) Accessing data on tape (5) None of these 94. Network components are connected to the same cable in the— — topology. (l)star -Answer (2) ring (3) bus (4) mesh (5) mixed 95. Two or more computers connected to each other for sharing information form a (1) network -Answer (2) router (3) server (4) tunnel (5) pipeline 96. A computer checks the of user names and passwords for a match before granting access. (1) website (2) network (3) backup file (4) database (5) None of these -Answer 97. Computers that are portable and convenient for users who travel are known as (1) supercomputers (2) Laptops -Answer (3) mini computers (4) file servers (5) None of these 98. What is the term for unsolicited email ? (1) newsgroup (2) usenet (3) backbone (4) flaming (5) spam -Answer 99. What type of program controls the various computer parts and allows the user to interact with the computer ? (1) Utility software -Answer (2) Operating system (3) Word processing software (4) Database program (5) None of these 100. Each cell in a Microsoft Office Excel document is referred to by its cell address, which is the —. (1) cell's column label -Answer (2) cell's column label and worksheet tab name (3) cell's row label (4) cell's row and column labels (5) None of these FCI Question-Papers for Managment traineeMT accounts general depot placement papers SSC FCI Question Paper FCI management trainee accounts general depot question papers with answers. FCI Previous years solved question papers Here some questions model questions for FCI accounts and all streams .Refer learn and practice these FCI question papers increase your aptitude test time management 1. According to the concept of conservation, the stock in trade is valued at? (A) Cost price (B) Market price (C) Cost or market price which ever is higher (D) Cost or market price which ever is lower Ans. (D) 2. The concept of conservation will have the effect of? (A) Over statement of assets (B) Understatement of assets (C) Understatement of liabilities (D) Understatement of provision for bad and doubtful debts Ans. (B) 3. Non-financial information is not recorded in accounts due to? (A) Accrual concept (B) Entity concept (C) Dual aspect concept (D) Money measurement concept Ans. (D) 4. Balance in ?Nazrana?s a/c? in the books of lessee is shown in? (A) Landlords a/c (B) P&L a/c (C) Balance sheet assets side (D) Balance sheet liabilities side Ans. (C) 5. The hire-buyer charges depreciation on? (A) Cash price of the asset (B) Hire purchase price of the asset (C) Higher of the two (D) Lower of the two Ans. (A) 6. In hire-purchase system, hire-buyer can maintain his accounts under? (A) Asset accrued method (B) Total cash price method (C) Any of the two methods (D) None of these Ans. (C) 7. If the rate of gross profit for department X is 25% of cost, the amount of gross profit on sales of Rs. 100000 will be-? (A) Rs. 16667 (B) Rs. 20000 (C) Rs. 25000 (D) Rs. 33333 Ans. (B) 8. Provision for bad debts of a foreign branch is converted at? (A) Opening rate of exchange (B) Closing rate of exchange (C) Average rate of exchange (D) Rate applicable to debtors Ans. (B) 9. If goods are transferred from ?X? department to ?Y? department at cost +25%, the amount of stock reserve on closing stock of Rs. 20000 in ?Y? department will be? (A) Rs. 4000 (B) Rs. 5000 (C) Rs. 6000 (D) Rs. 3333 Ans. (A) 10. If out of the bills of Rs. 10000 discounted by the insolvent bills of Rs. 4000 are likely be dishonoured, unsecured creditors will include in respect of them an amount of? (A) Rs. 10000 (B) Rs.6000 (C) Rs. 4000 (D) None of the above Ans. (C) 11. Partnership firm engaged in banking business can have maximum? (A) 5 partners (B) 10 partners (C) 20 partners (D) Any number of partners Ans. (B) 12. Suppose, the partnership deed provides for a salary of Rs. 5000 p.m. to partner ?X?. If ?X? withdraws only Rs. 3000 in a month, the remaining Rs. 2000 will be? (A) Debited to his capital a/c (B) Credited to his drawing a/c (C) Credited to his current a/c (D) Credited to P & L adjustment a/c Ans. (A) 13. In absence of any provisions in the partnership agreement, partners can charge on the loans given by them to the firm? (A) Interest at 6% p.a. (B) Interest at 12% p.a. (C) Interest at 15% p.a. (D) No interest Ans. (A) 14. A, B and C are partners sharing profits and losses in the ratio 4 : 3 : 2 D is admitted for 1/10th share, the new ratio will be? (A) 4 : 4 : 3 : 2 (B) 4: 3 : 2: 1 (C) 5 : 4 : 3 : 2 (D) None of the above Ans. (B) 15. A and B shared profit in the ratio of 3 : 2 C was admitted as a partner for 1/5th share. He acquires 3/20th from A and 1/20th from B. The new profit sharing ratio would be? (A) 10:6:4 (B) 6:10:4 (C) 8:8:4 (D) 9:7:4 Ans. (D) 16. Goodwill of a firm of A and B is valued at Rs. 60000. Goodwill appears in the books at Rs. 24000. C is admitted for 1/4th share. He will be required to bring for goodwill? (A) Rs.21000 (B) Rs.9000 (C) Rs. 15000 (D) Rs. 6000 Ans. (D) 17. A plant worth Rs. 800000 has been insured for Rs. 600000. The loss on account of fire is Rs. 500000. The insurance company under average clause will bear the loss to extent of? (A) Rs. 500000 (C) Rs. 800000 (B) Rs.600000 (D) Rs. 375000 Ans. (A) 18. It is not an item of Income with reference to a voyage a/c? (A) Passage money (B) Freight (C) Primage (D) Address commission Ans. (D) 19. The formats of the profit and loss account and Balance sheet in the case of a banking company have been revised w.e.f.? (A) 1st April 1949 (B) 1st April 1991 (C) 1st April 1992 (D) 1st April 1956 Ans. (C) 20. If accured outstanding premium is given in the trial balance of a general insurance company, then it will be shown in? (A) Revenue Account (B) Balance Sheet (C) Both revenue Account & Balance Sheet (D) None of the above Ans. (B) 21. Inventory is valued at lower of the cost or net realisable value on account of the accounting principle of? (A) Realisation (B) Consistency (C) Conservatism (D) None of the above Ans. (A) 22. In the period of rising prices, LIFO method may result in? (A) Lowering the profit (B) Raising the profit (C) Raising the tax liability (D) None of the above Ans. (B) 23. Given, Total assets turnover 4 Net Profits 10% Total Assets Rs. 50000 Net profit will be? (A) Rs. 15000 (B) Rs. 10000 (C) Rs. 25000 (D) Rs.20000 Ans. (D) 24. A company auditor addresses his audit report to- (A) Board of directors (B) Members (C) Managing director (D) Company secretary Ans. (B) 25. Bonus shares means shares issued to? (A) Workers (B) Existing equity shareholders (C) Preferential shareholders in lieu of dividend (D) Debenture holders in lieu of interest Ans. (B) 26. Which of the following does not call for physical verification? (A) Stock (B) Plant (C) Loose tools (D) Goodwill Ans. (D) 27. Cash from operations is equal to? (A) Net profit + increase in current assets (B) Net profit + decrease in current liabilities (C) Profit from operation ± Adjustment of increase and decrease in current assets and liabilities (D) Fund from operation ± Adjustment of increase and decrease in current assets and current liabilities Ans. (C) 28. Average profit of a firm is Rs. 9000 Firm?s capital is Rs. 60000 and normal return on business is expected at 10%. The goodwill by capitalisation method will be- (A) Rs. 30000 (B) Rs. 20000 (C) Rs. 25000 (D) Rs. 40000 Ans. (A) 29. Which of the following is not a current liability? (A) Bank overdraft (B) Redeemable debentures (C) Account payable (D) Provision for bad debts Ans. (B) 30. Amount of under writing commission payable on the issue of debentures is limited to? (A) 2% (B) 2.5% (C) 3% (D) 5% Ans. (B) 31. Which of the following acid test ratio can be said to be satisfactory? (A) 2: 1 (C) 1: 1 (B) 1: 2 (D) None of these Ans. (C) 32. The two factor theory of motivation was propounded by? (A) Peter Drucker (B) Herzberg (C) McGregor (D) Maslow Ans. (B) 33. Which of the following leadership styles is most commonly found now a day? (A) Autocratic (B) Democratic (C) Free rein (D) Participative Ans. (B) 34. Deciding in advance what is to be done in future is called? (A) Management (B) Coordination (C) Planning (D) Decision-making Ans. (C) 35. The organisation structure where there is direct vertical relationship is called? (A) Line organisation (B) Chain organisation (C) Command organisation (D) All the above Ans. (D) 36. When managers devote their attention only to those events where results are highly deviated from normal; ft is called? (A) Management by objective (B) Management by exception (C) Management by crisis (D) Management by choice Ans. (B) 37. Under delegation of authority? (A) Authority is given to subordinates (B) Authority flows from top to bottom (C) Delegator of authority is not received of accountability (D) All the above happens Ans. (D) 38. The process of determining by observation and study and reporting pertinent information relating to the nature of specific job is called? (A) Job specification (B) Job evaluation (C) Job analysis (D) Job description Ans. (C) 39. In case of a private company? (A) There is restriction on the right to transfer of shares (B) The number of members is restricted (C) Invitation to public for the subscription of shares is prohibited (D) All of the above Ans. (D) 40. A person at anyone time can not be DIRECT of more than? (A) 1 Company (B) 7 Companies (C) 15 Companies (D) 20 Companies Ans. (C) 41. Certificate of commencement of business is not required by a? (A) Public company (B) Any type of company (C) Private company (D) Private company subsidiary to a public company Ans. (C) 42. Henry Fayol is known for? (A) Scientific management (B) Rationalisation (C) Industrial psychology (D) Principles of managements Ans. (D) 43. The principle of unity of command? implies? (A) Unity of thought and action (B) Unity amongst subordinates (C) Instructions from staff authority (D) Instructions from line authority Ans. (D) 44. A company has equity capital of Rs. 200000. Preference capital of Rs. 100000, 12% debentures of Rs. 100000, long term loan of Rs. 200000 and short term loan of Rs. 100000. The capital gearing ratio will be? (A) 1:1 (B) 0.5: 1 (C) 0.4: 1 (D) None of the above Ans. (C) 45. In case of a company, total assets less outside liabilities is called? (A) Net working capital (B) Gross working capital (C) Deferred liabilities (D) Net worth Ans. (D) 46. If opening sales is Rs. 10000 purchases Rs. 30000 direct expenses Rs. 4000 and closing stock Rs. 5000 the costs of goods is sold would be? (A) Rs. 39000 (B) Rs. 40000 (C) Rs.41000 (D) Rs. 44000 Ans. (A) 47. Premium on issue of shares is shown in balance sheet as? (A) An asset (B) A liability (C) An expense (D) A revenue Ans. (B) 48. Which of the following is known as ?backbone of auditing?? (A) Verification of assets (B) Internal check (C) Vouching (D) Internal audit Ans. (C) 49. A limited company? 100 shares of Rs. 10/- cash fully called up on which Rs. 4/- per share was paid up. The company reissued 50 shares at the rates of Rs. 8/- each. The amount transferred to capital reserve will be? (A) Rs. 500 (B) Rs.200 (C) Rs. 250 (D) Rs. 100 Ans. (D) 50. Sale of long term investments indicates? (A) A change in current assets (B) Application of funds (C) Increase in working capital (D) Source of funds Ans. (D) 51. Net working capital refers to? (A) Current assets (B) Current assets minus current liabilities (C) Equity share capital minus fixed assets (D) Retired earnings. Ans. (B) 52. If sales Rs. 6000 gross profit is 1/3 on cost, purchases are R. 4900 and the closing stock is Rs. 900, the opening stock will be? (A) Rs.400 (B) Rs.500 (C) Rs. 1100 (D) Rs. 2000 Ans. (B) 53. The primary objective of audit is? (A) Detection and prevention of frauds (B) Detection and prevention of errors (C) Detection of frauds and errors (D) To ensure the final accounts and statements exhibit true and fair position of business Ans. (D) 54. ?An auditor is a watch dog and not a blood hound?. This was observed in case of? (A) London oil storage company (B) Kingston cotton Mills Limited (C) London General Bank (D) Delightful Cigarette Company Ltd. Ans. (B) 55. If two or more sugar mills combine together, it is known? (A) Horizontal combination (B) Vertical combination (C) Lateral combination (D) None of the above Ans. (A) 56. Surrender value is related to? (A) Marine insurance (B) General insurance (C) Life insurance (D) Fire insurance Ans. (C) 57. Over capitalisation refer to? (A) Excess of capital (B) Excess rate of dividend payment, on shares (C) Over estimation of rate of capitalisation (D) Raising more capital than is warranted by its earning power Ans. (D) 58. Motivation refers to? (A) Coordinate the people (B) Guide the working people (C) Terrorise the people (D) Inducing people to work willing by Ans. (D) 59. Which of the following is not a barrier in communication? (A) Fear and distrust (B) Affection (C) Perception (D) Noise Ans. (B) 60. ?Management is an art of getting things done through and with formally organised group.? This definition has been by? (A) Peter Drucker (B) Henry Fayol (C) Harod Koontz (D) F. W. Taylor Ans. (C) 61. Foreign exchange for import of goods is sanctioned by? (A) Exim Bank (B) Reserve Bank of India (C) State Bank (D) Ministry of commerce Ans. (B) 62. The cost of a machine having a span of life of 5 years is Rs. 10000. It has a scrap value of Rs. 1000. The amount of depreciation in the first year under the sum of year?s digit method will be? (A) Rs. 1600 (B) Rs. 1800 (C) Rs. 2000 (D) Rs. 3000 Ans. (D) 63. Given: Gross profit Rs. 60000 Gross profit ratio: 20% Debtor?s velocity 2 months The amount of debtors will be? (A) Rs. 30000 (B) Rs. 50000 (C) Rs. 120000 (D) Rs.200000 Ans. (B) 64. Premium on issue of shares can be used for? (A) Issue of Bonus shares (B) Payment of Dividends (C) Payment of operating expenses (D) Redemption of debentures Ans. (A) 65. If current ratio is 25, quick ratio (1) 5 and net working capital Rs. 15000. This value of inventory will be? (A) Rs. 10000 (B) Rs. 15000 (C) Rs. 37500 (D) Rs. 52500 Ans. (A) 66. Unclaimed dividend is shown on the liability side of the balance sheet under the heading? (A) Revenue and surplus (B) Provisions (C) Current liabilities (D) Miscellaneous items Ans. (C) 67. Accounting standards in India are prescribed by? (A) Company Law Board (B) Institute of charted accountants of India (C) Institute of coat and works accountants of India (D) Indian standard Board Ans. (B) 68. Which of the following is not correct? (A) Purchase + Opening stock?Cost of goods sold = Closing stock (B) Opening stock + Purchases ? Closing stock = Cost of goods sold (C) Closing stock + Cost of goods sold ? Purchases = Opening stock (D) Cost of goods sold ? Closing stock + Purchases = Opening stock Ans. (D) 69. A company bought assets worth Rs. 360000 and in lieu issued debentures of Rs. 100 each at a discount of 10%. The number of debentures issued will be? (A) 3000 (B) 3600 (C) 3960 (D) 400 Ans. (D) 70. A person got insured his goods worth Rs. 10000 for Rs. 80QO against fire. Loss by fire to him was Rs. 9000. He can claim? (A) Rs. 8000 (B) Rs. 9000 (C) Rs. 10000 (D) Rs. 7200 Ans. (D) 71. The data obtained from a newspaper are? (A) Primary data (B) Secondary data (C) Both (A) and (B) (D) None of these Ans. (B) 72. Current Ratio of a firm is 3: 1 and working capital is Rs. 60000. What will be the amount of current Assets? (A) Rs. 30000 (B) Rs90000 (C) Rs. 120000 (D) Rs. 180000 Ans. (B) 73. The income from that house property is taxable under the head ?Income from House property?. (A) The assessee has ownership on that house (B) The assessee uses that house for his business (C) The assessee himself lives in that house (D) The assessee has let out that house on rent for residence Ans. (A) 74. Following is the deduction in respect of repair under the head of income from house property? (A) 25% of Annual Value (B) 30% of Net Annual Value (C) 1/5 of Annual Value (D) 1/5 of Net Annual Value Ans. (B) 75. Single entry system can not be maintained by? (A) Sole proprietorship (B) Partnership concerns (C) Joint stock company (D) All of these Ans. (D) 76. Cash account will not be affected by? (A) Cash paid to creditors (B) Discount received (C) Cash sales (D) Cash received from debtors Ans. (B) 77. Maximum standard Deduction for employees getting gross salary not more than Rs. 100000 is allowed? (A) Rs. 20000 (B) Rs. 25000 (C) Rs. 30000 (D) Rs. 33000 Ans. (C) 78. Tax audit is compulsory in case of a person carrying on business whose gross receipt) turnover/sales and exceeds? (A) Rs. 50 Lakhs (B) Rs. 40 Lakhs (C) Rs. l0 Lakhs (D) Rs. 25 Lakhs Ans. (B) 79. Mr. Rastogi of Meerut was declared insolvent. One of his liabilities related to one months rent due to his landlord. This liability will be treated as? (A) Preferential Creditor (B) Partly Secured Creditor (C) Fully Secured Creditor (D) Unsecured Creditor Ans. (A) 80. When two or more companies liquidate to form a new company. It is called? (A) Amalgamation (B) Absorption (C) Reconstruction (D) Purchase of Business Ans. (A) 81. A, B and C are partners in a firm. If D is to be admitted to the firm as a new partner? (A) Old firm has to be dissolved (B) Old partnership has to be dissolved (C) Both the old firm and the old partnership have to be dissolved (D) No need to dissolve either firm or the partnership Ans. (D) 82. Remington sold one typewriter to Ramesh Chandra under installment purchase system on 1st January 1996, payment for which was to be made as under? On 01.01.1996 - Rs. 3000 On 31.12.1996 - Rs. 1700 On 31.12.1997 - Rs. 3600 On 31.12.1998 - Rs. 2300 On 31.12.1999 - Rs. 1100 Interest at 10% p.a. is included in each installment. The total interest charged amount to- (A) Rs.2100 (B) Rs. 1950 (C) Rs. 1800 (D) Rs. 1700 Ans. (B) 83. The liquidator of a company is entit1edio a remuneration of 2% on assets realised, and 3% on the amount distributed to unsecured creditors. The assets realised Rs. 100000 including cash balance of Rs. 3000. Amount available for distribution to unsecured creditors before paying liquidators remui4eration was Rs. 46350 liquidator?s remuneration will be? (A) Rs.3100 (C) Rs. 3290 (B) Rs.3140 (D) Rs. 3350 Ans. (C) 84. EXIM Bank was established on? (A) 1st Dec 1984 (B) 1st June 1985 (C) 1st Jan 1982 (D) 1st July 1980 Ans. (C) 85. For capital gain being long term capital gain, an assessee should retain the assets for a period of? (A) 40 months (B) 36 months (C) More than 36 months (D) Less than 36 months Ans. (C) 86. The salary received by a member of parliament is? (A) Exempt from Tax (B) Taxable under the head of salary (C) Taxable under the head of other sources (D) Taxable under the lead of business Ans. (C) 87. For the purpose of income tax it is necessary for agriculture income that? (A) Land should be used for agricultural activities (B) Land should be used for godown (C) Land should be used for irrigation (D) None of above Ans. (A) 88. Depreciation is allowed on? (A) Tangible Assets (B) Current Assets (C) Fixed Assets (D) Intangible Assets Ans. (C) 89. When shares are forfeited, the share capital account is debited by? (A) Nominal value of forfeited shares (B) Paid up amount of forfeited shares (C) Called up amount on forfeited shares (D) Forfeited amount of shares Ans. (C) 90. The term ?POIM? for the four functions of management. Planning, organisation, integration and measuring was given by.? (A) F.W. Taylor (B) Harold Smiddy (C) Tanon Brown (D) Peter F. Drucker Ans. (B) FCI Management Trainee Question Paper 1. Who of the following brought the Sikh Kingdom of Punjab under direct British rule by an official proclamation? (A) Lord Hastings (B) Lord Cornwallis (C) Lord Minto (D) Lord Dalhousie Ans. (D) 2. India is comparatively not rich in which one of the following minerals compared to the other three? (A) Bauxite (B) Copper (C) Iron (D) Manganese Ans. (B) 3. Among the following, which one has recorded the highest population growth rate during 1991-2001? (A) Arunachal Pradesh (B) Manipur (C) Nagaland (D) Sikkim Ans. (C) 4. In which one of the following regions does the Indus river originate? (A) Laddakh (B) Lahaul (C) Nepal (D) Tibet Ans. (A) 5. What is the term used to denote the temperature at which the water vapour present in the atmosphere is sufficient to saturate? (A) Condensation point (B) Dew point (C) Sublimation point (D) Saturation point Ans. (D) 6. The earth?s reflectivity of solar radiation, termed albedo, is highest in which one of the following? (A) Cropland (B) Forest area (C) Sand desert (D) Snow area Ans. (D) 7. Who of the following wrote the book Precepts of Jesus? (A) Raja Rammohan Roy (B) Devendranath Tagore (C) Ishwarchandra Vidyasagar (D) Keshab Chandra Sen Ans. (A) 8. Who for the first time saw bacteria through a microscope made by himself? (A) Anton van Leeuwenhoek (B) Louis Pasteur (C) Robert Hooke (D) Robert Virchow Ans. (A) 9. Which of the following contain enzymes for cellular respiration? (A) Dictyosomes (B) Endoplasmic reticula (C) Lysosomes (D) Mitochondria Ans. (D) 10. To which of the following types of animals are Salamanders closely related ? (A) Dolphins and Whales (B) Frogs and Toads (C) Prawns and Crabs (D) Seals and Walruses Ans. (B) 11. Which one of the following statements is correct? A seed is a ripened ? (A) ovary (B) flower (C) gynoecium (D) ovule Ans. (D) 12. Historical materialism is a tenet of which one of the following political theories ? (A) Capitalism (B) Liberalism (C) Fascism (D) Marxism Ans. (D) 13. What is ?Look East Policy? often in the news? (A) Government of India?s initiative for the infrastructural development in the North Eastern States (B) India?s search for oil and gas in its Eastern shoreline (C) India?s collaboration with some East Asian countries in the exploration of oil and gas (D) India?s continuing pursuit of close relations with South-East Asian countries Ans. (D) 14. Who of the following published a famous pamphlet known as ?Right of Mass? and urged the people in England, America and France to fight for their liberty ? (A) Thomas Jefferson (B) Thomas Paine (C) John Locke (D) Jean Jacques Rousseau Ans. (B) 15. Who of the following invented the cotton gin that separates the seeds from cotton three hundred times faster than by hand? (A) Eli Whitney (B) George Stephenson (C) McAdam (D) James Watt Ans. (A) 16. Whose duty is it to recommend to the President of India on the issue of the distribution and allocation of the net proceeds of taxes in the context of Centre State fiscal relations ? (A) Planning Commission (B) National Development Council (C) Union Ministry of Finance (D) Finance Commission Ans. (D) 17. In the Union Government, under whose charge is the Cabinet Secretariat? (A) The Minister of Parliamentary Affairs (B) The President of India (C) The Prime Minister of India (D) The Union Home Minister Ans. (C) 18. Which Five-Year Plan had an objective of ?Rapid Industrialization with particular emphasis on development of basic and heavy industries?? (A) First (C) Third (B) Second (D) Fourth Ans. (B) 19. Which one of the following is the correct chronological order of the given rulers of ancient India? (A) Ashoka ? Kanishka ? Milinda (B) Milinda ? Ashoka ? Kanishka (C) Ashoka ? Milinda ? Kanishka (D) Milinda ? Kanishka ? Ashoka Ans. (C) 20. Which one of the following is not a disease caused by virus? (A) Bird-flu (B) Chickenpox (C) Cholera (D) Dengue Ans. (C) 21. In terms of the evolution of organisms, which one among the following is the most advanced? (A) Bat (C) Shark (B) Pigeon (D) Vulture Ans. (A) 22. Which one of the following is not a genetic disorder? (A) Colour blindness (B) Down?s syndrome (C) Hemophilia (D) Xerophthalmia Ans. (D) 23. Which one of the following is an enzyme? (A) Gastrin (B) Keratin (C) Trypsin (D) Vasopressin Ans. (C) 24. In human body, which one of the following secretes hormones as well as digestive enzymes ? (A) Oesophagus (B) Pancreas (C) Spleen (D) Large intestine Ans. (B) 25. Seismograph is used in the study of? (A) Moon (B) Earthquake (C) Floods (D) Tides Ans. (B) 26. A bullet travelling horizontally hits a block kept at rest on a horizontal surface and gets embedded into it, the two together then move with a uniform velocity. Which one of the following conservation laws holds? (A) Conservation of angular momentum (B) Conservation of kinetic energy (C) Conservation of linear momentum (D) Conservation of velocity Ans. (C) 27. Which one of the following is correct? A negatively charged glass rod has always? (A) less electrons than protons (B) less electrons than neutrons (C) less protons than electrons (D) less neutrons than protons Ans. (C) 28. Who among the following was sent by Lord Harding?s to South Africa to plead the cause of Indians led by the young Gandhi? (A) B. G. Tilak (B) G. K. Gokhale (C) M. G. Ranade (D) Motilal Nehru Ans. (B) 29. Which one of the following is not a current of North Atlantic Ocean? (A) Falkland Current (B) Canary Current (C) Labrador Current (D) Gulf Stream Ans. (A) 30. Which one of the following States does not share boundary with Jharkhand ? (A) Chattisgarh (B) Madhya Pradesh (C) Orissa (D) Uttar Pradesh Ans. (B) 31. Through which States does Vamsadhara river flow? (A) Andhra Pradesh and Chattisgarh only (B) Chattisgarh and Orissa only (C) Andhra Pradesh and Orissa only (D) Andhra Pradesh, Chattisgarh and Orissa Ans. (C) 32. Whose entry into India resulted in the introduction of maize crop in India? (A) Dutch (B) English (C) French (D) Portuguese Ans. (D) 33. The efforts of who of the following led to the ?Age of Consent Act, 1891?? (A) Rabindranath Tagore (B) M. G. Ranade (C) Surendra Nath Bannerjee (D) Mulbari Ans. (D) 34. What was the name of the party formed by Subhas Chandra Bose after leaving Indian National Congress? (A) Congress Socialist Party (B) Forward Bloc (C) Indian National Conference (D) Swaraj Party Ans. (B) 35. Among the following who was a prominent leader of Khilafat Movement? (A) Jawaharlal Nehru (B) Mahatma Gandhi (C) Rajendra Prasad (D) Vallabhbhai Patel Ans. (B) 36. Who was the architect of the ?Drain Theory? and the author of the book Poverty and Un-British Rule in India? (A) Dadabhai Naoroji (B) Gopal Krishna Gokhale (C) Abul Kalam Azad (D) Ramesh Chandra Dutt Ans. (A) 37. The Civil Disobedience Movement started with which of the following? (A) No-tax campaign in Bardoli district (B) Demonstration of Akali Sikhs at Nabha (C) Demonstration against the Simon Commission (D) Dandi March Ans. (D) 38. The most abundant gas in the atmosphere is? (A) Oxygen (B) Nitrogen (C) Hydrogen (D) Carbon dioxide Ans. (B) 39. Which dance form are Birju Maharaj and Gopi Krishna renowned for? (A) Kathak (B) Chhau (C) Kathakali (D) Odissi Ans. (A) 401. Which is the official language of the Union Territory of Lakshadweep? (A) Tamil (B) Marathi (C) Malayalam (D) Konkani Ans. (C) 41. Who is known as the ?Father of India?s missile programme?? (A) A.P.J. Abdul Kalam (B) R. Santhanam (C) Raja Ramanna (D) Homi Bhabha Ans. (A) 42. The birthday of which sportsperson is celebrated as National Sports Day? (A) Sunil Gavaskar (B) PT. Usha (C) Prakash Padukone (D) Dhyan Chand Ans. (D) 43. Who among the following holds a record of the fastest 100 test wickets taken by an Indian bowler? (A) Erapalli Prasanna (B) Bhagawat Chandrasekhar (C) Bishan Singh Bedi (D) Kapil Dev Ans. (D) 44 In which city were the first Asian Games held? (A) Bangkok (B) Kuala Lumpur (C) New Delhi (D) Moscow Ans. (C) 45. The First Five Year Plan covered the period? (A) 1951-56 (B) 1952-57 (C) 1947-52 (D) 1950-55 Ans. (A) Placement Paper FCI Assistant Grade III Exam Paper I - General Intelligence & Reasoning - FCI Previous years solved question papers Reasoning 1. What should come in the place of (?) in the given series? ACE, FGH, ?, PON (A) KKK (B) JKI (C) HJH (D) IKL Ans. (A) 2. Typist : Typewriter : : Writer: ? (A) Script (B) Pen (C) Paper (D) Book Ans. (B) 3. Paint: Artist : : Wood: ? (A) Furniture (B) Forest (C) Fire (D) Carpenter Ans. (D) 4. acme : mace :: alga: ? (A) glaa (B) gaal (C) laga (D) gala Ans. (D) 5. EIGHTY : GIEYTH : : OUTPUT:? (A) UTOPTU (B) UOTUPT (C) TUOUTP (D) TUOTUP Ans. (D) 6. ‘Medicine’ is related to ‘Patient’ in the same way as ‘Education’ is related to— (A) Teacher (B) School (C) Student (D) Tuition Ans. (C) 7. Fill in the missing letter in the following series— S, V, Y, B, ? (A) C (B) D (C) E (D)G Ans. (C) 8. What should come in the place of question mark in the following series? 3, 8, 6, 14, ?, 20 (A) 11 (B) 10 (C) 8 (D) 9 Ans. (D) 9. Select the correct option in place of the question mark. AOP, CQR, EST, GUV, ? (A) IYZ B) HWX (C) IWX (D) JWX Ans. (C) 10. What should come in the place of question mark in the following series? 1, 4, 9, 25, 36, ? (A) 48 (C) 52 (B) 49 (D) 56 Ans. (B) Directions—(Q. 11 to 14): Select the one which is different from the other three. 11. (A) Bokaro (B) Jamshedpur (C) Bhilai (D) Agra Ans. (D) 12. (A) January (B) February (C) July (D) December Ans. (B) 13. (A) Bible (B) Panchsheel (C) Geeta (D) Quran Ans. (B) 14. (A) Star (B) Sun (C) Sky (D) Moon Ans. (C) Directions—(Q. 15 to 17): based on alphabets. 15. If the sequence of the alphabets is reversed which of the following would be the 14th letter from your left? (A) N (B) L ( C) O (D) None of these Ans. (D) 16. Which letter is the 8th letter to the right of the letter, which is 12th from the left? (A) V (B) T (C) W (D) Y Ans. (B) 17. Which letter is the 8th letter to the right of the letter which is 10th to the left of the last but one letter from the right? (A) V (B) X (C) W (D) I Ans. (C) Directions—(Q. 18 to 23) Three of the following four are alike in a certain way and so form a group. Which is the one that does not belong to that group? 18. (A) Green (B) Red (C) Colour (D) Orange Ans. (C) 19. (A) Rabbit (B) Crocodile (C) Earthworm (D) Snail Ans. (A) 20. (A) Polo (B) Chess (C) Ludo (D) Carrom Ans. (A) 21. (A) Sun (B) Universe (C) Moon (D) Star Ans. (B) 22. (A) Cheese (B) Milk (C) Curd (D) Ghee Ans. (B) 23. (A) Carrot (B) Radish (C) Potato (D) Brinjal Ans. (D) 24. In a certain code ‘CONTRIBUTOR’ is written as ‘RTNOCIROTUB’. How is ‘prohibition’ written in that code? (A) NOITIBIHORP (B) IHORPBITION (C) ITIONBIHOTP (D) IHORPBNOITI Ans. (D) 25. If ‘CAT’ and ‘BOAT’ are written as XZG and ‘YLZG’ respectively in a code language how is ‘EGG’ to be written in the same language? (A) VSS (B) URR (C) VTT (D) UTF Ans. (C) 26. In a code language SINGER is written as AIBCED then GINGER will be written in the same code as— (A) CBIECD (B) CIBCED (C) CBICED (D) CIBECD Ans. (B) 27. If BAT is coded as 283, CAT is coded as 383 and ARE is coded as 801, then the code for BETTER is— (A) 213310 (B) 213301 (C) 123301 (D) 012334 Ans. (A) 28. If water is called black, black is called tree, tree is called blue, blue is called rain, rain is called pink and pink is called fish in a certain language then what is the colour of sky called in that language? (A) Blue (B) Fish (C) Rain (D) Pink Ans. (C) 29. A man walks 3 km northwards and then turns left and goes 2 km. He again turns left and goes 3 km. He turns right and walks straight. In which direction he is walking now? (A) East (B) West (C) North (D) South Ans. (B) 30. One morning after sunrise Vikram and Shailesh were standing in a lawn with their back towards each other. Vikram’s shadow fell exactly towards left-hand side. Which direction Shailesh was facing? (A) East (B) West (C) North (D) South Ans. (D) 31. Nageena is taller than Pushpa but not as tall as Manish. Rama is taller than Namita but not as tall as Pushpa. Who among them is the tallest? (A) Manish (B) Pushpa (C) Namita (D) Nageena Ans. (A) 32. In an examination Raj got more marks than Moti but not as many as Meena. Meena got more marks than Ganesh and Rupali. Ganesh got less marks than Moti but his marks are not the lowest in the group. Who is second in the descending order of marks? (A) Meena (B) Rupali (C) Raj (D) None of these Ans. (C) 33. Pointing to a photograph of a girl, Rajan said “She has no sister or daughter but her mother is the only daughter of my mother.” How is the girl in the photograph related with Rajan’s mother? (A) Sister in law (B) Grand daughter (C) Daughter in law (D) None of these Ans. (B) 34. If Amit’ s father is Billoo’ s father’s only son and Billoo has neither a brother nor a daughter. What is the relationship between Amit and Billoo? (A) Uncle—Nephew (B) Father—Daughter (C) Father—Son (D) Grandfather—Grandson Ans. (C) 35. An application was received by inward clerk in the afternoon of a weekday. Next day he forwarded it to the table of the senior clerk, who was on leave that day. The senior clerk next day evening put up the application to the desk officer. Desk officer studied the application and disposed off the matter on the same day, i.e., Friday. Which day the application was received by the inward clerk? (A) Tuesday (B) Earlier week’s Saturday (C) Wednesday (D) Monday Ans. (C) 36. Flight to Mumbai leaves every 5 hours. At the information counter I learnt that the flight took off 25 minutes before. If the time now is 10 : 45 a.m., what is the time for the next flight? (A) 2 : 20 a.m. (B) 3 : 30 a.m. (C) 3 : 55 p.m. (D) 3 : 20 p.m. Ans. (D) 37. Babloo ranked 16th from the top and 29th from the bottom among those who passed an examination. 6 boys did not participate in the competition and 5 failed in the examination. How many boys were there in the class? (A) 44 (B) 40 (C) 50 (D) 55 Ans. (D) 38. Indra is 7th from the left and Jaya is 5th from the right. When they interchange their position Jaya becomes 19th from the right. What is Indra’s position from the left? (A) 21st (B) 19th (C) 23rd (D) 20th Ans. (D) 39. How many 5’s are in the following sequence of numbers which are immediately preceded by 7? 8 9 5 3 2 5 3 8 5 5 6 8 7 3 3 5 7 7 5 3 6 5 3 3 5 7 3 8 (A) One (B) Two (C) Three (D) Four Ans. (A) 40. How many 8’s are there in the following sequence which are immediately preceded by 6 but not immediately followed by 5? 6 8 5 7 8 5 4 3 6 8 1 9 8 5 4 6 8 2 9 6 8 1 3 6 8 5 3 6 (A) One (B) Two (C) Three (D) Four Ans. (C) .41 Student : BOOK : : Postman (A) Delivery (B) Bicycle (C) Uniform (D) Mail 42. Illiteracy : Education : : Drought : ? (A) Well (B) Rain (C) Dam (D) River 43. Carpenter : Furniture : : ? (A) Book : Author (B) Magazine : Editor (C) Cook : Soup (D) Dam : Engineer 44. LOM : NMK :: PKI : ? (A) RIH (B) SHG (B) RIG (C) RHG 45. JTIS : HRGQ : : FPEO : ? (A) DNCM (B) DCNQ (C) CNDM (E) CNDQ 46. BAD : CBE : : ? : IVSU (A) GOOD (B) HSPR (C) HALT (D) HURT 47. 5 : 30 : : 8 : ? (A) 14 (B) 50 (C) 69 (D) 80 48. 12 : 30 : : 20 : ? (A) 48 (B) 32 (C) 35 (D) 42 49. 3 : 28 : : 5 : ? (A) 179 (B) 126 (C) 124 (D) 125 50 If the following words are arranged according to English Dictionary, which word will be on third place ? (A) KNOW (B) KNACK (C) KNIT (D) KNOB Verbal Ability Directions for question (1 to 8) : Each of the two passages given below is followed by a set offour questions. Choose the best answer to each question. Passage - I While complex in the extreme, Derrida's work has proven to be a particularly influential approach to the analysis ofthe ways in which language structures our understanding of ourselves and the world we inhabit, an approach he termed deconstruction. In its simplest formulation, deconstruction can be taken to refer to a methodological strategy which seeks to uncover layers of hidden meaning in a text that have been denied or suppressed. The term 'text', in this respect, does not refer simply to a written form of communication, however. Rather, texts are something we all produce and reproduce constantly in our everyday social relations, be they spoken, written or embedded in the construction of material artifacts. At the heart of Derrida's deconstructive approach is his critique of what he receives to be the totalitarian impulse of the Enlightenment pursuit to bring all that exists in the world under the domain of a representative language, a pursuit he refers to as logocentrism. Logocentrism is the searchfor a rational language that is able to know and represent the world and all its aspects perfectly and accurately. Its totalitarian dimension,for Derrida at least, lies primarily in its tendency to marginalize or dismiss all that does not neatly comply with its particular linguistic representations, a tendency that, throughout history, has all too frequently been manifested in the form of authoritarian institutions. Thus logocentrism has, in its search for the truth of absolute representation, subsumed difference and oppressed that which it designates as its alien 'other'. For Derrida, western civilization has been built upon such a systematic assault on alien cultures and ways of life, typically in the name of reason and progress. In response to logocentrism, deconstruction posits the idea that the mechanism by which this process of marginalization and the ordering of truth occurs is through establishing systems of binary opposition. Oppositional linguistic dualisms, such as rational/irrational, culture/nature and good/bad are not, however, construed as equal partners as they are in, say, the semiological structuralism of Sa us sure. Rather, they exist, for Derrida, in a series of hierarchical relationships with the first term normally occupying a superior position. Derrida defines the relationship between such oppositional terms using the neologism difference. This refers to idle realization that in any statement, oppositional terms differ from each other (for instance, the difference between rationality and irntionality is constructed through oppositional usage), and at the same time, a hierarchical relationship is maintained by the deference of one term to the other (in the positing of rationality over irrationality, for instance). It is this latter point which is perhaps the key to understanding Derrida's approach to deconstruction. For the fact that at any given time one term must defer to its oppositional 'other', means that the two terms are constantly in a state of interdependence. The presence of one is dependent upon the absence or 'absent-presence' of the 'other', such as in the case of good and evil, whereby to understand the nature of one, we must constantly relate it to the absent term in order to grasp its meaning. That is, to do good, we must understand that our act is not evil for without that comparison the term becomes meaningless. Put simply, deconstruction represents an attempt to demonstrate the absent-presence of this oppositional 'other', to show that what we say or write is in itself not expressive simply of what is present, but also of what is absent. Thus, deconstruction seeks to reveal the interdependence of apparently dichotomous terms and their meanings relative to their textual context; that is, within the linguistic power relations which structure dichotomous terms hierarchically. In Derrida's awn wards, a deconstructive reading "must always aim at a certain relationship, unperceived by the writer, between what he commands and what he does not command of the patterns of a language that he uses .... [It] attempts to make the not-seen accessible to sight." Meaning, then, is never fixed or stable, whatever the intention of the author ofa text. For Derrida, language is a system of relations that are dynamic, in that all meanings we ascribe to the world are dependent not only an what we believe to be present but also an what is absent. Thus, any act of interpretation must refer not only to what the author of a text intends, but also to what is absent from his or her intention. This insight leads, once again, Derrida's further rejection of the idea of the definitive authority of the intentional agent or subject. The subject is decentred; it is conceived as the outcome of relations of differance. As author of its awn biography, the subject thus becomes the ideological fiction of modernity and its logocentric philosophy, one that depends upon the formation of hierarchical dualisms, which repress and deny the presence of the absent 'other'. No meaning can, therefore, ever be definitive, but is merely an outcome of a particular interpretation. 1. According to the passage, Derrida believes that the system of binary opposition (a) represents a prioritization or hierarchy. -Answer (b) reconciles contradictions and dualities. (c) weakens the process of marginalization and ordering of truth. (d) deconstructs reality. 2. According to the passage, Derrida believes that: (a) Reality can be construed only through the use of rational analysis. (b) Language limits our construction of reality. (c) A universal language will facilitate a common understanding of reality. (d) We need to uncover the hidden meaning in a system of relations expressed by language. -Answer 3. Derrida rejects the idea of 'definitive authority of the subject' because (a) interpretation of the text may not make the unseen visible. -Answer (b) the meaning of the text is based an binary opposites. (c) the implicit power relationship is often ignored. (d) any act of interpretation must refer to what the author intends. 4. To Derrida, 'Iogocentrism' does not imply: (a) A totalitarian impulse. (b) A domain of representative language. (c) Interdependence of the meanings of dichotomous terms. -Answer (d) A strategy that seeks to suppress hidden meanings in a text. Passage - II Crinoline and croquet are out. As yet, no political activists have thrown themselves in front of the royal horse on Derby Day. Even so, same historians can spat the parallels. It is a time of rapid technological change. It is a period when the dominance of the world's superpower is coming under threat. It is an epoch when prosperity masks underlying economic strain. And, crucially, it is a time when policy-makers are confident that all is/or the best in the best of all possible worlds. Welcome to the Edwardian Summer of the second age of globalisation. Spare a moment to take stock of what's been happening in the pastfew months. Let's start with the oil price, which has rocketed to more than $65 a barrel, more than double its level 18 months ago. The accepted wisdom is that we shouldn't worry our little heads about that, because the incentives are there for business to build new production and refining capacity, which will effortlessly bring demand and supply back into balance and bring crude prices back to $25 a barrel. As Tommy Cooper used to say, 'just like that'. Then there is the result of the French referendum on the European Constitution,seen as thick-headed luddites railing vainly against the modem world. What the French needed to realise, the argument went, was that there was no alternative to the reforms that would make the country more flexible, more competitive, more dynamic. Just the sort of reforms that allowed Gate Gourmet to sack hundreds of its staff at Heathrow after the sort of ultimatum that used to be handed out by Victorian mill owners. An alternative way of looking at the French "non" is that our neighbours translate "flexibility" as "you're fired". Finally, take a squint at the United States. Just like Britain a century ago, a period of unquestioned superiority is drawing to a close. China is still a long way from matching America's wealth, but it is growing at a stupendous rate and economic strength brings geo-political clout. Already, there is evidence ofa new scramblefor Africa as Washington and Beijing compete for oil stocks. Moreover, beneath the surface of the US economy, all is not well. Growth looks healthy enough, but the competition from China and elsewhere has meant the world's biggest economy now imports far more than it exports. The US is living beyond its means, but in this time of studied complacency a current account deficit worth 6 percent of gross domestic product is seen as a sign of strength, not weakness. In this new Edwardian summer, comfort is takenfrom the fact that dearer oil has not had the savage inflationary consequences of 1973-74, when a fourfold increase in the cost of crude brought an abrupt end to a postwar boom that had gone on uninterrupted for a quarter of a century. True, the cost of living has been affected by higher transport costs, but we are talking of inflation at 2.3 per cent and not 27 percent. Yet the idea that higher oil prices are oflittle consequence is fanciful. Ifpeople are paying more to fill up their cars it leaves them with less to spend on everything else, but there is a reluctance to consume less. In the 1970s unions were strong and able to negotiate large, compensatory pay deals that served to intensify inflationary pressure. In 2005, that avenue is pretty much closed off, but the abolition of all the controls on credit that existed in the 1970s means that households are invited to borrow more rather than consume less. The knock-on effects of higher oil prices are thus felt in different ways - through high levels of indebtedness, in inflated asset prices, and in balance of payments deficits. There are those who point out, rightly, that modem industrial capitalism has proved mightily resilient these past 250 years, and that a sign of the enduring strength of the system has been the way it apparently shrugged off everything - a stock market crash, 9/11, rising oil prices - that have been thrown at it in the half decade since the millennium. Even so, there are at least three reasonsfor concern. First, we have been here before. In terms of political economy, the first era of globalisation mirrored our own. There was a belief in unfettered capital flows, in free trade, and in the power ofthe market. It was a time of massive income inequality and unprecedented migration. Eventually, though, there was a backlash, manifested in a struggle between free traders and protectionists, and in rising labour militancy. Second, the world is traditionally at its most fragile at times when the global balance of power is in flux. By the end of the nineteenth century, Britain's role as the hegemonic power was being challenged by the rise of the United States, Germany, and Japan while the Ottoman and Hapsburg empires were clearly in rapid decline. Looking ahead from 2005, it is clear that over the next two or three decades, both China and India - which together account for half the world's population - will flex their muscles. Finally, there is the question of what rising oil prices tell us. The emergence of China and India means global demand for crude is likely to remain high at a time when experts say production is about to top out. If supply constraints start to bite, any declines in the price are likely to be short-term cyclical affairs punctuating a long upward trend. 5. Which of the following best represents the key argument made by the author? (a) The rise in oil prices, the flux in the global balance of power and historical precedents should make us question our belief that the global economic prosperity would continue. (b) The belief that modem industrial capitalism is highly resilient and capable of over coming shocks will be belied soon. (c) Widespread prosperity leads to neglect of early signs ofunderIying economic weakness, manifested in higher oil prices and a flux in the global balance of power. -Answer (d) A crisis is imminent in the West given the growth of countries like China and India and the increase in oil prices. 6. What can be inferred about the author's view when he states, 'As Tommy Cooper used to say "just like that'''? (a) Industry has incentive to build new production and refining capacity and therefore oil prices would reduce. (b) There would be a correction in the price levels of oil once new production capacity is added. -Answer (c) The decline in oil prices is likely to be short-term in nature. (d) It is not necessary that oil prices would go down to earlier levels. 7. What, according to the author, has resulted in a widespread belief in the resilience of modem capitalism? (a) Growth in the economies of Western countries despite shocks in the form of increase in levels of indebtedness and inflated asset prices. (b) Increase in the prosperity of Western countries and China despite rising oil prices. (c) Continued growth of Western economies despite a rise in terrorism, an increase in oil prices and other similar shocks. -Answer (d) The success of continued reforms aimed at making Western economies more dynamic, competitive and efficient. 8. By the expression 'Edwardian Summer', the author refers to a period in which there is (a) unparalleled luxury and opulence. (b) a sense of complacency among people because of all-round prosperity. -Answer (c) a culmination of all-round economic prosperity. (d) an imminent danger lurking behind economic prosperity. Directions for questions (9 to 12) : Each question consists offour sentences on a topic Some sentences are grammatically incorrect or inappropriate. Select the option that indicates the grammatically correct and appropriate sentence(s). 9. A. The balance of power will shift to the East as China and India evolve. B. Rarely the economic ascent of two still relatively poor nations has been watched with such a mixture of awe, opportunism, and trepidation. C. Postwar era witnessed economic miracles in Japan and South Korea, but neither was populous enough to power worldwide growth or change the game in a complete spectrum of industries. D. China and India, by contrast, possess the weight and dynamism to transform the 21 st-century global economy. (a) A,B&C (b) A&D -Answer (c) C (d) C&D 10. A. People have good reason to care about the welfare of animals. B. Ever since Enlightenment, their treatment has been seen as a measure of mankind's humanity. C. It is no coincidence that William Wilberforce and Sir Thomas Foxwell Buxton, two leaders of the movement to abolish the slave trade, helped found the Royal Society for the Prevention of Cruelty to Animals in 1820s. D. An increasing number of people go further: mankind has a duty not to cause pain to animals that have the capacity to suffer. (a) A & D (b) A&C (c) C&D -Answer 11. A. When virtuoso teams begin their work, individuals are in and group consensus is out. B. As project progresses, however, the individual stars harness themselves to the product of the group. C. Sooner or later, the members break through their own egocentrism and become a plurality with single-minded focus on the goal. D. In short, they morph into a powerful team with a shared identity. (a) A&C (b) A&D -Answer (c) B&D (d) A, C&D 12. A. Large reductions in the ozone layer, which sits about 15-30 km above the Earth, take place each winter over the polar regions, especially the Antarctic, as low temperatures allow the formation of stratospheric clouds that assist chemical reactions breaking down ozone. B. Industrial chemicals containing chlorine and bromine have-been blamed for thinning the layer because they attack the ozone molecules, making them to break apart. C. Many an offending chemicals have now been banned. D. It will still take several decades before these substances have disappeared from the atmosphere. (a) D (b) B&D (c) A&D -Answer (d) A&C Directions for questions (13 to 16) : Each of the following questions has a paragraph from which the last sentence has been deleted. From the given options, choose the one that completes the paragraph in the most appropriate way. 13. Federer's fifth grand slam win prompted a reporter to ask whether he was the best ever. Federer is certainly not lacking in confidence, but he wasn't about to proclaim himselfthe best ever. "The best player ofthis generation, yes", he said, "But nowhere close to ever. Just look at the records that some guys have. I'm a minnow." _______ . (a) His win against Agassi, a genius from the previous generation, contradicts that. (b) Sampras, the king of an earlier generation, was as humble. -Answer (c) He is more than a minnow to his contemporaries. (d) The difference between 'the best of this generation' and 'the best ever' is a matter of perception. 14. Thus the end of knowledge and the closing of the frontier that it symbolizes is not a looming crisis at all, but merely one of many embarrassing fits of hubris in civilization's long industry. In the end, it will pass away and be forgotten. Ours is not the first generation to struggle to understand the organizational laws of the frontier, deceive itself that it has succeeded, and go to its grave having failed. ______ . (a) One would be wise to be humble. (b) But we might be the first generation to actually reach the frontier. -Answer (c) But we might be the first generation tc deal with the crisis. (d) However, this time the success is not illusory. 15. Most firms consider expert individuals to be too elitist, temperamental, egocentric, and difficult to work with. Force such people to collaborate on a high-stakes project and they just might come to fisticuffs. Even the very notion of managing such a group seems unimaginable. So most organizations fall into default mode, setting up project teams of people who get along nicely. (a) The result, however, is disastrous. (b) The result is mediocrity. -Answer (c) The result is creation of experts who then become elitists. (d) Naturally, they drive innovations. 16. The audiences for crosswords and sudoku, understandably, overlap greatly, but there are differences, too. A crossword attracts a more literary person, while sudoku appeals to a keenly logical mind. Some crossword enthusiasts turn up their noses at sudoku because they feel it lacks depth. A good crossword requires vocabulary, knowledge, mental flexibility and sometimes even a sense of humor to complete. It touches numerous areas of life and provides an "Aha!" or two along the way ______ . (a) Sudoku, on the other hand, is just a logical exercise, each one similar to the last. -Answer (b) Sudoku, incidentally, is growing faster in popularity than crosswords, even among the literati. (c) Sudoku, on the other hand, can be attempted and enjoyed even by children. (d) Sudoku, however, is not exciting in any sense of the term. Directions for questions (17 to 20) : Each ofthe following questions has a paragraph with one italicized word that does not make sense. Choose the most appropriate replacement for that word from the options given below the paragraph. 17. Intelligent design derives from an early 19th-century explanation ofthe natural worldgiven by an English clergyman, William Paley. Paley was the populariser of the famous watchmaker analogy. Proponents of intelligent design are crupping Paley's argument with a new gloss from molecular biology. (a) destroying (b) testing (c) resurrecting -Answer (d) questioning 18. Women squat, heads covered, beside huge piles of limp fodder and blunk oil lamps, and just about all the cows in the three towns converge upon this spot. Sinners, supplicants and yes, even scallywags hand over a few coins for a crack at redemption and a handful of grass. (a) shining (b) bright (c) sputtering -Answer (d) effulgent 19. It is klang to a sensitive traveler who walks through this great town, when he sees the streets, the roads, and cabin doors crowded with beggars, mostly women, followed by three, four, or six children, all in rags and importuning every passenger for alms. (a) amusing (b) irritating (c) disgusting (d) distressing -Answer 20. Or there is the mostfingummy diplomatic note on record: when Philip of Macedon wrote to the Spartans that, ifhe came within their borders, he would leave not one stone of their city, they wrote back the one word - "If'. (a) witty (b) rude (c) simple (d) terse -Answer Directions for questions (21 to 24) : The passage given below is followed by a set of four questions. Choose the best answer to each question. A game of strategy, as currently conceived in game theory, is a situation in which two or more "players" make choices amorg available alternatives (moves). The totality of choices determines the outcomes of the game, and it is assumed that the rank order of preferences for the outcomes is different for different players. Thus the "interests" of the players are generally in conflict. Whether these interests are diametrically opposed or only partially opposed depends on the type of game. Psychologically, most interesting situations arise when the interests of the players are partly coincident and partly opposed, because then one can postulate not only a conflict among the players but also inner conflicts within the players. Each is tom between a tendency to cooperate, so as to promote the common interests, and a tendency to compete, so as to enhance his own individual interests. Internal conflicts are always psychologically interesting. What we vaguely call "interesting" psychology is in very great measure the psychology of inner conflict. Inner conflict is also held to be an important component of serious literature as distinguished from less serious genres. The classical tragedy, as well as the serious novel, reveals the inner conflict of central figures. The superficial adventure story, on the other hand, depicts only external conflict; that is, the threats to the person with whom the reader (or viewer) identifies stem in these stories exclusively from external obstacles and from the adversaries who create them. On the most primitive level this sort of external conflict is psychologically empty. In the fisticuffs between the protagonists of good and evil, no psychological problems are involved or, at any rate, none are depicted in juvenile representations of conflict. The detective story, the "adult" analogue of a juvenile adventure tale, has at times been described as a glorification of intellectualized conflict. However, a great deal of the interest in the plots of these stories is sustained by withholding the unraveling of a solution to a problem. The effort of solving the problem is in itselfnot a conflict if the adversary (the unknown criminal) remains passive, like Nature, whose secrets the scientist supposedly unravels by deduction. If the adversary actively puts obstacles in the detective's path toward the solution, there is genuine conflict. But the conflict is psychologically interesting only to the extent that it contains irrational components such as a tactical error on the criminal's part or the detective's insight into some psychological quirk of the criminal or something of this sort. Conflict conducted in a perfectly rational manner is psychologically no more interesting than a standard Western. For example, Tic- tac-toe, played perfectly by both players, is completely devoid of psychological interest. Chess may be psychologically interesting but only to the extent that it is played not quite rationally. Played completely rationally, chess would not be different from Tic-tac-toe. In short, a pure conflict of interest (what is called a zero-sum game) although it offers a wealth of interesting conceptual problems, is not interesting psychologically, except to the extent that its conduct departs from rational norms. 21. According to the passage, which of the following options about the application of game theory to a conflict-of-interest situation is true? (a) Assuming that the rank order of preferences for options is different for different players. (b) Accepting that the interests of different players are often in conflict. (c) Not assuming that the interests are in complete disagreement. (d) All of the above. -Answer 22. The problem solving process of a scientist is different from that of a detective because (a) scientists study inanimate objects, while detectives deal with living criminals or law offenders. (b) scientists study known objects, while detectives have to deal with unknown criminals or law offenders. (c) scientists study phenomena that are not actively altered, while detectives deal with phenomena that have been deliberately influenced to mislead. -Answer (d) scientists study psychologically interesting phenomena, while detectives deal with "adult" analogues of juvenile adventure tales. 23. According to the passage, internal conflicts are psychologically more interesting than external conflicts because (a) internal conflicts, rather than external conflicts, form an important component of serious literature as distinguished from less serious genres. (b) only juveniles or very few "adults" actually experience external conflict, while internal conflict is more widely prevalent in society. (c) in situations of internal conflict, individuals experience a dilemma in resolving their own preferences for different outcomes. -Answer (d) there are no threats to the reader (or viewer) in case of external conflicts. 24. Which, according to the author, would qualify as interesting psychology? (a) A statistician's dilemma over choosing the best method to solve an optimisation problem. (b) A chess player's predicament over adopting a defensive strategy against an aggressive opponent. -Answer (c) A mountaineer's choice of the best path to Mt. Everest from the base camp. (d) A finance manager's quandary over the best way of raising money from the market. Directions for questions (25 to 25) : The sentences given in each question, when properly sequenced, form a coherent paragraph. Each sentence is labeled with a letter. Choose the most logical order of sentences from among the given choices to construct a coherent paragraph. 25. A. Similarly, turning to caste, even though being lower caste is undoubtedly a separate cause of disparity, its impact is all the greater when the lower-caste families also happen to be poor. B. Belonging to a privileged class can help a woman to overcome many barriers that obstruct women from less thriving classes. -Answer C. It is the interactive presence ofthese two kinds of deprivation - being low class and being female - that massively impoverishes women from the less privileged classes. D. A congruence of class deprivation and gender discrimination can blight the lives of poorer women very severely. E. Gender is certainly a contributor to societal inequality, but it does not act independently of class. (a) EABDC (b) EBDCA -Answer (c) DAEBC (d) BECDA Directions for questions (26 to 28) : In each question, there are five sentences. Ea FCI Whole Test Paper FCI Current and affairs and general awareness questions with answers NATIONAL 1. Consider the following statements about USA and India: (a) India is the 12th largest trading partner for the USA. (b) USA is the third largest trading partner for India. (c) Currency movements and US demand can hit exports or imports. Which of the above statements are true? (1) only (a) (2) only(b) (3) only(c) (4) All of the above -Answer 2. Union Finance Minister Pranab Mukherjee released a new series of coins with improved design and revised size in the denominations of 50 paise, ? 2, ?5 and?10 (1) 1 July, 2011 (2) 5 July, 2011 (3) 8 July, 2011 Answer (4) 10 July, 2011 3. The central government announced to provide ? 600 crore economic package over the next three years to the proposed Gorkhaland Territorial Administration in Darjeeling on. (1) 15 July, 2011 Answer (2) 16 July, 2011 (3) 17 July, 2011 (4) 18 July, 2011 4. In July 2011, the central government announced to spend around ^ 30,000 crore on providing more than lakh villages with broadband connectivity through optical fibre. (1) 1.1 (2) 1.5 (3) 2.1 (4) 2.5 Answer 5. India and South Africa inked an agreement for cooperation in the peaceful uses of nuclear energy on . (1) 20 July, 2011 (2) 22 July, 2011 (3) 25 July, 2011 Answer (4) 26 July, 2011 6. India recently ratified United Nations convention against . (1) Corruption Answer (2) Education (3) Nature (4) Terrorism 7. 2nd Africa-India summit was recently held in (1} South Africa (2) Chad (3) Somalia (4) Ethopia Answer 8. On which date India and SriLanka signed a MOU to develop the Kankesanthufai port, the first such deep-water facility to be rebuilt in the northern peninsula? (1) 20 July, 2011 Answer (2) 21 July, 2011 (3) 30 July, 2011 (4) 31 July, 2011 9. The first -ever joint census exercise by India and Bangladesh since Independence in 162 encloves spread over six districts along the international border came to an end on . (1) 18 July, 2011 Answer (2) 20 July, 2011 (3) 25 July, 2011 (4) 28 July, 2011 10. The central Department of Telecommunication cleared creation of a 'National optical Fibre Network' with an investment of ? 20,000 crore on . (1) 20 July, 2011 (2) 21 July, 2011 (3) 23 July, 2011 Answer (4) 29 July, 2011 11. On which date, the Union cabinet approve a Lokpal Bill? (1) 28 July, 2011 (2) 29 July, 2011 (3) 30 July, 2011 (4) 31 July, 2011 12. India recently signed Double Taxation Avoidance Agreement with (1) Brazil (2) SouthAfrica (3) Russia {4) Colombia Answer INTERNATIONAL 13. The US congress is set to vote on a deal on to riase the debt ceiling to 14.3 dollar trillion and cut deficit by 2.4 dollar trillion over 10 years. (1) 2 August, 2011 Answer (2) 3 August. 2011 (3) 4 August, 2011 (4) 5 August, 2011 14. Who is the Pakistan Foreign Minister? (1) Saknan Quraisi (2) Hina Salman (3) Hina Rabbani Khar Answer (4) Ashma Jahangir • 15. Harvard scholar LobsangSangay was on sworn in as the new prime minister of the Tibetan government -in-exile, succeeding Dalai Lama as the political head. (1) 8 August, 2011 Answer (2) 9 August, 2011 (3) 10 August, 2011 (4) 11 August, 2011 16. Recently in news country reels under riots? (1) Spain (2) England Answer (3) Italy (4) France 17. Which country has highest number of Gmail users? (1) India Answer (2) China (3) USA (4) Brazil 18. Islam Karimov is the President of , (1) Mongolia (2) Indonesia (3) Uzbekistan Answer (4) Afghanistan 19. David cameron is the Prime Minister of . (1) New Zealand (2) Australia (3) Britain Answer (4) Denmark 20. Silvio Berlusconi is the Prime Minister of . (1) Spain (2) France (3) Sweden (4) Italy Answer 21. Which country announced its readiness to back India on Unit-ed Nations security council reforms and delink its relationship with Pakistan to take forward ties? (1) Japan (2) Bangladesh (3) USA (4) China Answer 22. Which country announced that it was installing new and efficient centrifuges aimed at speeding up its nuclear enrichment on 19 July, 2011? (1) Iraq (2) Iran Answer (3) Afghanistan (4) Pakistan 23. Which country announed to have built a missile defence shield covering two thirds of the country, providing protection against missile attacks? (1) USA (2) Russia Answer (3) France (4) China 24. Hugo chavez is the president of (1) Colombia (2) Venezuela Answer (3) Peru (4) Brazil 25. Recently which island country decided to forge a day and shift to the time-zone on its west? (1) Samoa Answer (2) Azore (3) Timore (4) Trinidad 26. G-8 summit was recently held in (1) Delhi (2) Tokyo (3) Deuville Answer (4) London 27. 18th ASEAN summit was recently held in . (1) Jakarta Answer (2) Manila (3) Thailand (4) New Delhi 28. World's Least Developed countries summit was held in , (1) Istanbul Answer (2) Tehran (3) Thailand (4) Cairo 29. Chinese President Hu Jintao declared economic development and ensuring social stability to be the priorities of the communist party of china on_ . (1) 30 June, 2011 (2) 1 July, 2011 Answer (3) 30 July, 2011 (4) 1 August, 2011 30. Yingluck Shinawatra is the Prime Minister of . (1) Tiawan (2) Laos (3) Thialand Answer (4) Vietnam 31. Naoto Kan is the Prime Minister of . (1) Vietnam (2) South Korea (3) Japan Answer (4) North Korea 32. Who is the new Chief Minister of Karnataka? (1) D.V. Sadanand Gowda Answer (2) H.D.Dev Gowda (3) B.S. Yeddyurappa (4) JagdishShettar 33. Which of the following state has decided to constitude Innovation council on the lines of National Innovation Council by the centre government? (1) Bihar (2) Haryana Answer (3) Gujarat (4) Karnataka 34. Recently in news, 'Koya Commandos' and 'Salwa Judum' are related to . (1) Bihar (2) Jharkhand (3) Chhattisgarh Answer (4) Madhya Pradesh 35. Which state government inked a 95.16 million dollar loan agreement with the Asian Development Bank (ADB) to strengthen tourism infrastructure in the state on 20 July, 2011? (1) Bihar (2) Assam (3) Sikkim (4) Himachal pradesh Answer 36. Which state was selected for the Best Performing state Award for wheat for 2010-2011 on 4 July, 2011? (1) Punjab (2) Uttar Pradesh (3) Haryana Answer (4) Madhya Pradesh 37. Which state government decided to allocate a share of the state's net revenues to panchayati raj institutions and local self-government bodies? (1) Rajasthan Answer (2) Madhya Pradesh (3) Sikkim (4) Tamil Nadu 38. Which state government announced to introduce 'Bhoti' in all schools of Buddhist-inhabited areas in four districts of the state? (1) Himachal Pradesh (2) Arunachal Pradesh Answer (3) Uttarakhand (4) Bihar 39. Which state government decided to launch new scheme 'Mamata' which would enable pregnant wome in rural pockets to avail monetary assistance for accessing medical care? (1) Orissa Answer (2) Bihar (3) West Bengal (4) Assam 40. Which state government agreed to complete the long-pending work on the Munak canal on 19 July, 2011'? (1) Punjab (2) Delhi (3) Haryana Answer (4) Uttar pradesh 41. Which state government announched a grant of Rs. 50 lakh for each panchayat samiti in the state to further empower them? (1) Punjab (2) Haryana Answer (3) Bihar (4) Jharkhand 42. In July 2011, which state government emerged as the joint winner with Punjab of the newly launched Krishi Karman Award for best performance in foodgrains production during 2010-2011? (1) Bihar (2) Uttar Pradesh Answer (3) Punjab (4) Madhya Pradesh 43. The National Resource centre for women's Empowerment selected Rajasthan to pilot a national programme launched in New Delhi earlier in 2011 on . (1) 12 July, 2011 (2) 13 July, 2011 (3) 14 July, 2011 (4) 15 July, 2011 44. In July, 2011 which state government and UN women signed a project cooperation agreement for the programme on promoting women's political Leadership and Governance in India and South Asia? (1) Bihar (2) Jharkhand (3) Orissa Answer (4) Chhattisgarh 45. Recently which state government decided to scale up livelihood projects around Similipal Reserve Forest, included in world Network of Biosphere Reserves recently? (1) Orissa Answer (2) Assam (3) Karnataka (4) Uttarakhand 46. Which state government devided to promote agro-entrepre neurship agro-clinic and agroservice centres in panchayats of the state? STATE 66. Match the List-I with List-II List -I List-11 (Person) (Post) (a) Pranb 1. Finance Mukherjee Minister (b) Duwuri 2. Governor Subbarao RBI (c) Aditya Puri 3. MD, HDFC Bank (d) Pratip 4. Chairman, Chaudhari SBI Codes : (a) (b) (c) (d) (1) 1 2 3 4 (2) 2 3 4 1 (3) 3 4 1 2 (4) 4 3 2 1 67. Match List -I with List - II List -I List -II (Person) (areas of specialisation) (a) Abhijit sen 1. Academic (b) Mahesh 2. Political RangaRajan Analyst (c) G. Parth 3. Former sarthy Diplomat (d) Nityanand 4. Activist JayaRaman Codes: (a) (b) (c) (d) (1) 1 2 3 4 (2) 2 3 4 1 (3) 3 4 1 2 (4) 4 3 2 1 68. Who is the new CMD of NABARD? (1) Omprakash (2) Jay Prakash (3) Prakash Bakshi (4) Pradeep Bakshi 69. Seasoned bureaucrat P.C. Alexander, a close aide of former prime ministers Indira Gandhi and Rajiv Gandhi, died of cancer at a hospital in chennai on (1) 10 August, 2011 (2) 11 August, 2011 (3) 12 August, 2011 (4) 13 August, 2011 70. Consider the following state-. ments about Magsaysay award : (a) Two Indians, Nileema Mishra, a social activist from Maharashtra and US - trained Indian engineer Harish Hande, were awarded Ramon Magsaysay award for the year 2011. (b) They are among the six persons in Asia, who were given the award (c) Nileema Mishra (29) has worked for empowerment of women in Jalgaon district of Maharashtra and has formed self help groups of over 2,000 women in 15 villages to provide them liverhood avenues: (d) Harish Hande (44) runs his own solar electric light company that has lit up over 1,20,000 house holds, to emerge as India's leading solar technology firm. Which of the above statements are true? (1) only (a) and(b) (2) Only (b) and (c) (3) only (c) and (d) (4) All of the above -Answer 7 1 . Who is the new cabinet secretary of India is . (1) Anil Kumar (2) Sumit Kumar (3) Ajit kumar seth -Answer (4) VikramSeth 72. Who has been awarded Man Booker International Prize this year (2011)? (1) Anna Hazare (2) Philip Roth -Answer (3) Arundhati Roy (4) AminaSud 73. Who has been awarded C.K.Nayudu life time Achievement Award this year (2011)? (1) Sunil Gavaskar (2) Kapil Dev (3) Salim Durrani -Answer (4) SachinTendulkar 74. The government of India has decided to institute an international award in the name of (1) Jawahar Lai Nehru (2) Subhash Chandra Bose (3) RavindraNathTagore -Answer (4) Rajendra Prasad 75. Who is the new Executive Director of Punjab National Bank? (1) UshaAnanthasubramanian -Answer (2) UshaAgarwal (3) Sunidhi Chauhan (4) NirmalaVerma 76. Who is the Writer of ' River of smoke'? (1) VikramSeth (2) Amitav Ghosh -Answer (3) Khushwant Singh (4) ArunShauri 77. Quit India Day is celebrated on _______ (1) 9 August -Answer (2) 10 August (3) 14 August (4) 15 August 78. Sania Mirza won her WTA double title of the season at College Park (USA) (1) First (2) Second (3) Third (4) Fourth 79. In a First for Indian shooting, double-trap Champion Ronjan Sodhi grabbed the top spot in world ranking. (1) 1 August, 2011 -Answer (2) 2 August, 2011 (3) 3 August, 2011 (4) 5 August, 2011 80. Consider the following statements about woman Tennis players: (a) According to Forbes, Russia's Maria Sharopova is the world's highest paid woman in professional sports for the seventh straight year i.e. ? 110 cr. (b) 29-year old Lina, who became the first Asian born player to win a sigles Grand slam title, is poised to become the richest woman in tennis as companies fight to cash in on her enormous popularity in china. Which of the above statements are true? (1) only (a) (2) only (b) (3) Both (a) and (b) (4) None of these 8 1 . Modi Gold Cup is associated with: (1) Football (2) Hockey (3) Tennis (4) Cricket 82. Cricket is the National game of: (1) India (2) England (3) Pakistan (4) Bangladesh 83. Match the List-I with List-II List-I List-II (Athletes who (Games) have qualified) (a) Mayookha 1. Triple Jump Johny (b) Gurmeet 2. 20 km Walk Singh (c) Vikas 3. Discuss Gowda (d) Babubhai 4. 20 km walk panocha SPORTS Codes: (a) (b) (c) (d) (1) 1 2 3 4 (2) 2 3 4 1 (3) 3 4 1 2 (4) 4 3 2 1 84. Who won the women's 10 km open water event on 19 July 201 1 at the world aquatics championships in Shanghai? (1) Keri-Anne Payne -Answer (2) Samara Smith (3) Maria Burke (4) Alicia May 85. Who secured his second win of the year and 16 th of his career with a brilliant victory in the German Grand prix in Nuerburgring in July 2011? (1) Lewis Hamilton -Answer (2) Bob Bell (3) Pat Fry (4) Ron Dennis 86. Who Claimed the men's billiards title at the Hash 10 world Sim Nationals, beating Bhaskar in the best of nine frames 150-up final in Chennai? (1) Geet Sethi (2) Pankaj Advani -Answer (3) Amod Kami (4) None of the these 87. Badminton is the national game of : (1) Indonesia -Answer (2) Thailand (3) France (4) China 88. Hockey is the national game of (1) France (2) India -Answer (3) Nepal (4) England 89. The winner of Azlan Shah Hock ey Tournament this year (2011) is _ _ . ' ' (1) Australia -Answer (2) India (3) NewZ^aland (41 Spain 90. The winner of Snooker World Championship this year (2011) is . (1) Neil Robertson (2) JohnHiggins -Answer (3) Graeme Dott (4) None of these 91. England Test Cricket Team won the third Test by an innings and runs, taking 3-0 lead in the Four-Test series. (1) 142 (2) 242 -Answer (3) 202 (4) 240 92. Who won the long jump gold medal on the opening day of the 19 th Asian Athletics Championships in kobe? (1) Mayookha Johny -Answer (2) LuMinjia (3) PreetiRawat (4) SumanSirohi 93. Who won the National women challengers chess championship in Chennai? (1) Rajni Jaiswal (2) Bhakti Kulkami -Answer (3) Garima Singh (4) AmitaModi 94. Consider the following statements: (a) The PMEAC on 1 August, 2011 lowered the economic growth, projection for the current fiscal year, citing uncertain global outlook and spiralling inflation. (b) 8.2% GDP growth projection for the current fiscal year, cut down from 9%. (c) 6.5% projected year end inflation. (d) 7.1% projected growth in factory output, compared to 7.9% last year. Which of the above statements are true? (1) only (a) and (b) (2) Only fb) and (c) (3) Only (c) and (d) (4) All of the above -Answer 95. As per the revised estimates of CSO for the year 2010-11, GDP growth rate at factor cost stands at- (1) 8 5% -Answer (2} 8.6% (3) 8.7% (4) 8.9% 96. What is the target of disbursing agriculture institutional credit, during the year 2011 -12? (1) ? 3,75,000 crore (2) ? 4,75,000 crore -Answer (3) ? 4,85,000 crore (4) ? 5,75,000 crore 97. Which of the following sector registers highest growth rate as per revised estimates of CSO for the year 2010-11? (1) Manufacturing -Answer (2) Electricity (3) Mining (4) Construction 98. What is the minimum support price of paddy (A-grade) for the year 2011 -12 as declared by the government? (1) ? 1,100 per quintal (2) ^ 1,110 per quintal -Answer (3) ? 1,120 per quintal (4) ^ 1,210 per quintal 99. What is the projected growth rate of Indian economy made by RBI for the year 20! 1-12? (1) 8.2% -Answer (2) 8.4% (3) 8.6% (4) 8.9% 100. How many Indian companies found place in 'Forbes 2000' list for the year 2011? (11 47 (2) 57 (3) 67 (4) 77 101. LIC has introduced a new health policy offering comprehensive hospitalisation benefit for the whole family of the principal insured. The new policy is known as . (1) JeevanArogya -Answer (2) JeevanAnand (3) Jeevan Chhaya (4) Jeevan Plus 102. In the new IIP series with base 2004-05, the number of item groups in the mam ifacturing sector has gone up from-. (1) 473 to 620 -Answer (2) 507 to 620 (3) 573 to 720 (4) 607 to 720 103. In 2010 -2011, in the list of 138 nations, what is the rank of India in Networked Readiness Inded (NRI)? (1) 38th (2) 40th (3) 44th (4) 48tb -Answer 104. Government has extended Duty Entitlement pass-book scheme (DEPS) tipto- (1) 30 September, 2011 -Answer (2) 30 October, 2011 (3) 30 November, 2011 (4) 30 December, 2011 105. New Index of Industrial Production has come into force. Its base has been changed from 1993-94 to. . (1) 2004-05 -Answer (2) 2005-06 (3) 2006-07 (4) 2007-08 106. FDI in India's service sector during 2010-11 registered- (1) a fall of about 15% (2) a fall of about 20% -Answer (3) a fall of about 25% (4) a fall of about 30% 107. Upto which financial year, 13th Finance Commission has recommended to bring Revenue Deficit' to zero level? (1) 2013-14 -Answer (2) 2012-13 (3) 2014-15 (4) 2015-16 108.The weight of manufacturing in old IIP (base 1993-94) was 76.36% which has now been charged in new IIP (base 2004-05) at- (1) 65.53% (2) 75.53% -Answer (3) 85.53% (4) 55.53% 109. As per the revised estimates of CSO which quarter of the financial year 2010-11 registered growth in agriculture and allied sectors? (1) Q2 (2) Q3 -Answer (3) Ql (4) Q4 110. The EGOM on Food cleared the Food Security Bill on (1) 10 July, 2011 (2) 11 July, 2011 -Answer (3) 12 July, 2011 (4) 13 July, 2011 Verbal Ability Questions Qn.1 For these questions, select the best of the answer choices given. The latest movie by a certain director gets bad reviews before it opens in theatres. Consequently, very few people go to the movie and the director is given much less money to make his next movie, which is also unsuccessful. What can be inferred from this scenario? This director makes terrible movies. The general public does not pay attention to movie reviews. The movie reviewers were right about the first movie. Movie reviewers exert influence on the movie quality. Answer:-Movie reviewers exert influence on the movie quality. Explanation: The negative reviews led to the poor quality of the second movie Qn.2 The most important determinant of success in life is education. Even children from broken or dysfunctional homes tend to establish themselves as solid citizens so long as they obtain a high school education. On the other hand, children who fail to earn a high school diploma are much less likely to avoid prison, welfare, or divorce. Which of the following statements most effectively strengthens the above argument? A recent study demonstrated a link between education and lifetime earnings. Answer:A recent study demonstrated a link between education and lifetime earnings. Explanation: This evidence would support the assertions of the given argument. Most federal prisoners receive a high school diploma while incarcerated. Research indicates that college graduates from abusive homes are more likely to be arrested. Individuals with heart problems are more likely to have postgraduate education. Qn.3 Select the correct word/ words from the choices that complete the given sentence as your answer. Please note that more than once choice may fit in to make syntactically correct sentence but select the choice that is logical in the context of the sentence. 1. Although his intension was to inculcate strict discipline among his children, he ended being tyrannical__________. thereby making his children rebellious thereby making him more adorable to his children and thus setting an example to his children thus making his children emulate his ways Answer:thus making his children emulate his ways Explanation: There is no way of determining whether any, some, or none of A are C. Qn.4 He is ______ that he often lands himself and others into many embarrassing and difficult situations. so fastidious a person such an unassuming person such a forgetful person Answer: such a forgetful person Explanation: This is an example of circular reasoning, in which the proof depends on assumptions which themselves have not been proven. such a meticulous person Qn.5 Each question gives a word followed by four choices. From the choices, select the most suitable synonym for the main word. ABYSMAL Shameless Dangerous Profound Horrible Qn.6 INFREQUENT Never Usual Rare Answer:Rare Explanation: Infrequent means happening rarely, uncommon. Sometimes Qn.7 RESTRAINT Hindrance Repression Obstacle Restriction Qn.8 In the following questions, a sentence split into four parts labelled A, B, C and D, is given. Rearrange the parts to form the original sentence and select the correct order from among the four choices given below. A. who had treated her so shabbily B. she had recovered from most of her injuries C. and insults D. suffered in the automobile accident DBAC ADCB Answer:ADCB Explanation: Drawing an analogy between being a member of Congress and serving on the school board is highly dubious BDCA BADC Qn.9 A. I felt as if a dam had broken B. faded away and as I watched a line of pink and steel C. suddenly the unhappy times in our marriage. D. blue clouds inching their way onto the horizon DBAC DBCA CBAD CBDA Answer: CBDA Explanation: The logic of this argument can be expressed as follows: All A are B. C is A, therefore C is B. Qn.10 A. to see that the necessary work B. the first thing on my agenda C. had been completed D. was to check the workbooks- BDAC BDCA ACDB Answer: ACDB Explanation:There is no way to be certain that the Tigers lost, though it seems likely. ACBD REASONING ABILITY 1. Four teams, Team A, B, C and D participated in a tournament. Team C scored the least. Team B scored more than team D but not as much as Team A. Who amongst the four teams scored third highest ? 1. Team A 2. Team B 3. Team C 4. Team D -Answer 5. Cannot be determined 2. How many such pairs of letters are there in the word JOURNEY each of which has as many letters between them in the word (in both forward and backward directions) as they have between them in the English alphabetical order ? 1. None 2. One 3. Two 4. Three -Answer 5. More than three 3. Four of the following five are alike in a certain way and so form a group, which is the one that does not belong to that group ? 1. Jute 2. Cotton 3. Wool 4. Silk 5. Rayon-Answer Directions (Q. 4 and 5) Read the following information carefully and answer the questions which follow If ‘A × B’ means ‘A is to the South of B’. If ‘A + B’ means ‘A is to the North of B’ If ‘A % B’ means ‘A is to the East of B’ If ‘A – B’ means ‘A is to the West of B’. 4. Which of the following means ‘P is to the East of Q’ ? 1. H % P – S + Q 2. Q + R × S – P 3. P % A – Q + B -Answer 4. Q – Z % S × P 5. None of these 5. In the equation F % Q + R – S, S is in which direction with respect to Q ? 1. East 2. South 3. West 4. South-West 5. South-East-Answer Directions (Q. 6 - 8) The following questions are based on the five four digit numbers given below - 3475 2791 6458 1826 7534 6. What will be the resultant if the second digit of the highest number is subtracted from the third digit of the second lowest number ? 1. 1 2. 2 3. 3 4. 6 5. 4-Answer 7. If all the digits in each of the numbers are arranged in descending order from left to right within the number, whichof the following will be the sum of all the four digits of the number which is third highest in the new arrangement? 1. 19 2. 18 3. 22 4. 17 -Answer 5. 23 8. If in each number, the first and the last digits are interchanged, which of the following will be the second lowest number ? 1. 3475 2. 2791 3. 6458 4. 1826 5. 7534-Answer Directions - (Q. 9 - 13) In each question/set of questions below are statements followed by two conclusions numbered I and II. You have to take the given statements to be true even if they seem to be at variance from commonly known facts and then decide which of the given conclusions logically follows from the statements disregarding commonly known facts. Give Answers - 1. If only conclusion I follows 2. If only conclusion II follows 3. If either conclusion I or conclusion II follows 4. If neither conclusion I nor conclusion II follows 5. If both conclusions I and II follow 1--Answer 9. Statements : All gems are precious Some gems are stones. All stones are diamonds Conclusions : I. At least some stones are precious II. All diamonds are precious. 10. Statements : All dreams are fantasies. Some fantasies are pleasant All pleasant are everlasting Some everlasting are memories Conclusions : I. Some dreams are memories II. Some fantasies are everlasting 2--Answer 11. Statements : All black are blue. All blue are green All green are emerald Conclusions : I. Some emeralds are green II. All black and blue are green 5-Answer Directions - (Q. 12 and 13) Statements : No design is fashion All fashions are temporary Some temporary are permanent 12. Conclusions : I. Some designs are permanent II. No fashion is permanent 4-Answer 13. Conclusions : I. At least some temporary are fashions. II. At least some temporary are not designs. 5-Answer Directions - (Q.14 - 17) Study the following information carefully and answer the given questions -Seven trees namely mango, lemon, apple, ashoka, banana, guava and papaya are planted in a straight row, not necessarily in the same order. (Assume as if the trees are facing North) a. The papaya tree is planted fourth to the right of the lemon tree b. The Ashoka tree is planted at the extreme right end of the row c. The mango and guava trees are immediate neighbours of the lemon tree d. The banana tree is planted immediately next to the mango tree 14. Four of the following five are alike in a certain way based on their position in the above arrangement and so forma group. Which is the one that does not belong to the group ? 1. lemon, mango 2. banana, apple 3. mango, banana 4. guava, lemon 5. apple, ashoka-Answer -Answer 15. Which trees are planted at the extreme ends of the row ? 1. guava, apple 2. lemon, ashoka 3. guava, ashoka -Answer 4. lemon, papaya 5. guava, papaya 16. Which of the following tree is planted exactly in the middle of the row ? 1. papaya 2. mango 3. banana -Answer 4. lemon 5. apple 17. What is the position of the apple tree with respect to the guava tree ? 1. Second to the right 2. Fourth to the right -Answer 3. Immediate left 4. Third to the right 5. Third to the left3 Directions - ( 18 - 21 ) Study the following information carefully and answer the given questions - A, B, C, D, F, G, H and J are sitting around a circular table facing the centre. a. D sits exactly between J and F. b. J sits second to the left of F and second to the right of B. c. H sits third to the left of G. d. A is not an immediate neighbour of B.-Answer 18. How many people sit between G and H when counted in an anti-clockwise direction from G ? 1. One 2. Two 3. Three 4. Four -Answer 5. Five 19. If H : F then J : ? 1. B -Answer 2. F 3. H 4. G 5. None of these 20. If all the persons are made to sit in alphabetical order in anticlock-wise direction, starting from A, the positions of how many (excluding A) will remain nchanged as compared to their original seating positions ? 1. None 2. One -Answer 3. Two 4. Three 5. Four FCI Solved Aptitude and Reasoning Papers FCI Aptitude questions 1. In a class composed of x girls and y boys what part of the class is composed of girls A.y/(x + y) B.x/xy C.x/(x + y) D.y/xy (Ans.C) 2. What is the maximum number of half-pint bottles of cream that can be filled with a 4-gallon can of cream(2 pt.=1 qt. and 4 qt.=1 gal) A.16 B.24 C.30 D.64 (Ans.D) 3. .If the operation,^ is defined by the equation x ^ y = 2x + y, what is the value of a in 2 ^ a = a ^ 3 A.0 B.1 C.-1 D.4 (Ans.B) 4. A coffee shop blends 2 kinds of coffee, putting in 2 parts of a 33p. a gm. grade to 1 part of a 24p. a gm. If the mixture is changed to 1 part of the 33p. a gm. to 2 parts of the less expensive grade, how much will the shop save in blending 100 gms. A.Rs.90 B.Rs.1.00 C.Rs.3.00 D.Rs.8.00 (Ans.C) 5. There are 200 questions on a 3 hr examination. Among these questions are 50 mathematics problems. It is suggested that twice as much time be spent on each Maths problem as for each other question. How many minutes should be spent on mathematics problems A.36 B.72 C.60 D.100 (Ans.B) 6. In a group of 15,7 have studied Latin, 8 have studied Greek, and 3 have not studied either. How many of these studied both Latin and Greek A.0 B.3 C.4 D.5 (Ans.B) 7. If 13 = 13w/(1-w) ,then (2w)2 = A.1/4 B.1/2 C.1 D.2 (Ans.C) 8. If a and b are positive integers and (a-b)/3.5 = 4/7, then (A) b < a (B) b > a (C) b = a (D) b >= a (Ans. A) 9. In june a baseball team that played 60 games had won 30% of its game played. After a phenomenal winning streak this team raised its average to 50% .How many games must the team have won in a row to attain this average? A. 12 B. 20 C. 24 D. 30 (Ans. C) 10. M men agree to purchase a gift for Rs. D. If three men drop out how much more will each have to contribute towards the purchase of the gift/ A. D/(M-3) B. MD/3 C. M/(D-3) D. 3D/(M2-3M) (Ans. D) 11. . A company contracts to paint 3 houses. Mr.Brown can paint a house in 6 days while Mr.Black would take 8 days and Mr.Blue 12 days. After 8 days Mr.Brown goes on vacation and Mr. Black begins to work for a period of 6 days. How many days will it take Mr.Blue to complete the contract? A. 7 B. 8 C. 11 D. 12 (Ans.C) 12. 2 hours after a freight train leaves Delhi a passenger train leaves the same station travelling in the same direction at an average speed of 16 km/hr. After traveling 4 hrs the passenger train overtakes the freight train. The average speed of the freight train was? A. 30 B. 40 C.58 D. 60 (Ans. B) 13. If 9x-3y=12 and 3x-5y=7 then 6x-2y = ? A.-5 B. 4 C. 2 D. 8 (Ans. D) 14. There are 5 red shoes, 4 green shoes. If one draw randomly a shoe what is the probability of getting a red shoe (Ans 5c1/ 9c1) 15. What is the selling price of a car? If the cost of the car is Rs.60 and a profit of 10% over selling price is earned (Ans: Rs 66/-) 16. 1/3 of girls , 1/2 of boys go to canteen .What factor and total number of classmates go to canteen. Ans: Cannot be determined. 17. The price of a product is reduced by 30% . By what percentage should it be increased to make it 100% (Ans: 42.857%) 18. There is a square of side 6cm . A circle is inscribed inside the square. Find the ratio of the area of circle to square. (Ans. 11/14 ) 19. There are two candles of equal lengths and of different thickness. The thicker one lasts of six hours. The thinner 2 hours less than the thicker one. Ramesh lights the two candles at the same time. When he went to bed he saw the thicker one is twice the length of the thinner one. How long ago did Ramesh light the two candles . Ans: 3 hours. 20. If M/N = 6/5,then 3M+2N = ? 21. If p/q = 5/4 , then 2p+q= ? 22. If PQRST is a parallelogram what it the ratio of triangle PQS & parallelogram PQRST . (Ans: 1:2 ) 23. The cost of an item is Rs 12.60. If the profit is 10% over selling price what is the selling price ? (Ans: Rs 13.86/- ) 24. There are 6 red shoes & 4 green shoes . If two of red shoes are drawn what is the probability of getting red shoes (Ans: 6c2/10c2) 25. To 15 lts of water containing 20% alcohol, we add 5 lts of pure water. What is % alcohol. (Ans : 15% ) 26. A worker is paid Rs.20/- for a full days work. He works 1,1/3,2/3,1/8.3/4 days in a week. What is the total amount paid for that worker ? (Ans : 57.50 ) 27. If the value of x lies between 0 & 1 which of the following is the largest? (a) x b) x2 (c) "x (d) 1/x (Ans : (d) ) 28. If the total distance of a journey is 120 km .If one goes by 60 kmph and comes back at 40kmph what is the average speed during the journey? Ans: 48kmph 29. A school has 30% students from Maharashtra .Out of these 20% are Bombey students. Find the total percentage of Bombay ? (Ans: 6%) 30. An equilateral triangle of sides 3 inch each is given. How many equilateral triangles of side 1 inch can be formed from it? (Ans: 9) 31. If A/B = 3/5,then 15A = ? (Ans : 9B) 32. Each side of a rectangle is increased by 100% .By what percentage does the area increase? (Ans : 300%) 33. Perimeter of the back wheel = 9 feet, front wheel = 7 feet on a certain distance, the front wheel gets 10 revolutions more than the back wheel .What is the distance? Ans : 315 feet 34. Perimeter of front wheel =30, back wheel = 20. If front wheel revolves 240 times. How many revolutions will the back wheel take? Ans: 360 times 35. 20% of a 6 litre solution and 60% of 4 litre solution are mixed. What percentage of the mixture of solution (Ans: 36%) 36. City A's population is 68000, decreasing at a rate of 80 people per year. City B having population 42000 is increasing at a rate of 120 people per year. In how many years both the cities will have same population? (Ans: 130 years) 37. Two cars are 15 kms apart. One is turning at a speed of 50kmph and the other at 40kmph . How much time will it take for the two cars to meet? (Ans: 3/2 hours) 38. A person wants to buy 3 paise and 5 paise stamps costing exactly one rupee. If he buys which of the following number of stamps he won't able to buy 3 paise stamps. Ans:9 39. There are 12 boys and 15 girls, How many different dancing groups can be formed with 2 boys and 3 girls. 40. Which of the following fractions is less than 1/3 (a) 22/62 (b) 15/46 (c) 2/3 (d) 1 (Ans: (b)) 41. There are two circles, one circle is inscribed and another circle is circumscribed over a square. What is the ratio of area of inner to outer circle? Ans: 1 : 2 42. Three types of tea the a,b,c costs Rs. 95/kg,100/kg and70/kg respectively.How many kgs of each should be blended to produce 100 kg of mixture worth Rs.90/kg, given that the quntities of band c are equal a)70,15,15 b)50,25,25 c)60,20,20 d)40,30,30 (Ans. (b)) 43. in a class, except 18 all are above 50 years.15 are below 50 years of age. How many people are there (a) 30 (b) 33 (c) 36 (d) none of these. (Ans. (d)) 44. If a boat is moving in upstream with velocity of 14 km/hr and goes downstream with a velocity of 40 km/hr, then what is the speed of the stream ? (a) 13 km/hr (b) 26 km/hr (c) 34 km/hr (d) none of these (Ans. A) 45. Find the value of ( 0.75 * 0.75 * 0.75 - 0.001 ) / ( 0.75 * 0.75 - 0.075 + 0.01) (a) 0.845 (b) 1.908 (c) 2.312 (d) 0.001 (Ans. A) 46. A can have a piece of work done in 8 days, B can work three times faster than the A, C can work five times faster than A. How many days will they take to do the work together ? (a) 3 days (b) 8/9 days (c) 4 days (d) can't say (Ans. B) 47. A car travels a certain distance taking 7 hrs in forward journey, during the return journey increased speed 12km/hr takes the times 5 hrs.What is the distancetravelled (a) 210 kms (b) 30 kms (c) 20 kms (c) none of these (Ans. B) 48. Instead of multiplying a number by 7, the number is divided by 7. What is the percentage of error obtained ? 49. Find (7x + 4y ) / (x-2y) if x/2y = 3/2 ? (a) 6 (b) 8 (c) 7 (d) data insufficient (Ans. C) 50. A man buys 12 lts of liquid which contains 20% of the liquid and the rest is water. He then mixes it with 10 lts of another mixture with 30% of liquid.What is the % of water in the new mixture? FCI General Awareness questions with answers 1. What is the population of India ? (A) 98 crores (B) More than 2 billion (C) More than 1 billion-Answer (D) Less than 96 crores (E) 96 crores 2. Thermostat is an instrument used to" (A) measure flow of current (B) measure intensity of voltage (C) regulate temperature-Answer (D) regulate velocity of sound (E) None of these 3. Wimbledon Trophy is associated with" (A) Football (B) Cricket (C) Hockey (D) Basketball (E) Lawn Tennis-Answer 4. GNP stands for" (A) Gross National Product-Answer (B) Group Net Product (C) Grand Nuclear Process (D) Group Networking Process (E) None of these 5. 'Acoustics' is the science of the study of" (A) Light (B) Sound-Answer (C) Electricity (D) Magnetism (E) None of these 6. Noise pollution is measured in the unit called" (A) micron (B) nautical miles (C) ohms (D) ampere (E) decibel-Answer 7. 'Heavy Water' is used in which of the following types of indus-tries ? (A) Sugar (B) Nuclear Power-Answer (C) Textile (D) Coal (E) None of these 8. Who amongst the following is the author of the book 'Indomi-table Spirit'? (A) Dr. A.P.J. Abdul Kalam (A) Indomitable Spirit is a book authored by Dr. A.P.J. Abdul Kalam the ex-President of India. The book brings together the values, thoughts and ideas of President Kalam as reflected in his speeches and addresses. Interspersed with interesting ancedotes and observations, indomitable Spirit represents the quintessential A.P.J. Abdul Kalam"the man, the scientist, the teacher and the President. (B) Dr. Manmohan Singh (C) Justice K. G. Balakrishna (D) Mr. Natwar Singh (E) None of these 9. Which of the following is not the name of popular IT/Software Company ? (A) Wipro (B) Mastek (C) Toyota -Answer (D) IBM (E) Infosys 10. 'Yen' is the currency of" (A) South Korea (B) China (C) Indonesia-Answer (D) Malaysia (E) None of these 11. Baichung Bhutia whose name was in news is a well known" (A) Music Director of Indian films (B) English author of Indian origin (C) Journalist (D) Politician (E) Sports Personality-Answer 12. 'Richter Scale' is used to measure which of the following ? (A) Intensity of Tsunami Waves (B) Intensity of Earthquake-Answer (C) Density of salt in Sea water (D) Flow of electric current (E) None of these 13. Which of the following best explains 'e-governance'? (A) Improving the functioning of government (B) Teaching government emp-loyees the basics of computing (C) Delivery of public services through internet-Answer (D) Framing of cyber-laws of chatting on internet (E) Convergence of e-mail and video-conferencing 14. CAS is associated with which of the following ? (A) Legal System Reforms (B) Piped gas line (C) Cable T.V.-Answer (D) Mobile phone regulation (E) None of these 15. Tata Steel recently acquired the Corus, a steel giant situated in" (A) South Africa (B) Ukraine (C) Australia (D) Italy (E) Britain-Answer 16. Which of the following is the abbreviated name of the Organi-zation/Agency working in the field of Space Research ? (A) ISBN (B) ISRO-Answer (C) INTELSET (D) INTACH (E) None of these 17. 'Handshaking' in Networking parlance means" (A) connecting computers to a hub-Answer (B) distributed Networks (C) having same operating system on different computers (D) sending e-mail (E) None of these 18. VAT stands for" (A) Value And Tax (B) Value Added Tax-Answer (C) Virtual Action Tasks (D) Virtual Assessment Tech-nique (E) None of these 19. Alzheimer'sdiseaseistheailment of which of the following organs /parts of the human body ? (A) Kidney (B) Heart (C) Liver (D) Stomach (E) Brain-Answer 20. Global warming is a matter of concern amongst the nations these days. Which of the follo-wing countries is the largest emitter of greenhouse gases in the World ? (A) U.S.A. (B) China-Answer (C) India (D) Britain (E) None of these 21. Which of the following books has been written by Bill Gates ? (A) Microsoft Secrets (B) The Road Ahead-Answer (C) The Elephant Paradigm (D) e-commerce (E) None of these 22. Which of the following is the name of the social network service run by the Google on the internet ? (A) Online Space (B) Orkut (C) Net-Space (D) Wikipedia (E) None of these 23. Which of the following states is a relatively new addition ? (A) Goa (B) Delhi-Answer (C) Uttarakhand-Answer (D) Himachal Pradesh (E) Bihar 24. DOT stands for" (A) Disc Operating Therapy (B) Department of Telephones (C) Directorate of Technology (D) Damage on Time (E) None of these-Answer Dot stands for Department of Telecommunications. 25. Which of the following States/ parts of India is completely land locked having no contact with sea ? (A) Karnataka (B) Orissa (C) Maharashtra (D) North-East-Answer (E) None of these 26. TRAI regulates the functioning of which of the following servi-ces ? (A) Telecom -Answer (B) Trade (C) Port (D) Transport (E) None of these 27. At present for the ATMs in India, the most commonly used net-work communication mode is" (A) Very Small Aperture Termi-nal (VSAT)-Answer (B) General Packet Radio Ser-vice (GPRS) (C) Code Division Multiple Access (CDMA) (D) Dial-in Connection (E) None of these 28. Which of the following is not a foreign bank operating in India ? (A) DBS Bank Ltd. (B) Sonali Bank (C) J. P. Morgan Chase Bank (D) Shinhan Bank (E) All are Foreign Banks-Answer 29. Who is the author of the book 'Future Shock' ? (A) Alvin Toffler-Answer (B) Tom Peters (C) Napoleon Hill (D) Kenneth Blanchard (E) None of these 30. Apex fares are" (A) more than the normal fare (B) less than the normal fare-Answer (C) offered only to corporate clients (D) applicable to late night flights only (E) applicable to international flights only 31. Who amongst the following is the Director of the film 'Water' which was in news ? (A) Vidhu Vinod Chopra (B) Mrinal Sen (C) Gulzar (D) Mahesh Bhatt (E) None of these-Answer 32. The 10th plan period is upto" (A) 2007 -Answer (B) 2008 (C) 2009 (D) 2010 (E) None of these 33. OPEC is a group of countries which are" (A) exporting oil-Answer (B) producting cotton (C) rich and developed (D) developing and poor (E) nuclear powers 34. Intel Company mainly pro-duces" (A) Hard disks (B) VCDs (C) Monitors (D) Software (E) None of these-Answer 35. Who amongst the following is the M.D./Chairman of one of the major Steel Companies ? (A) Shri L. N. Mittal-Answer (B) Shri K. K. Birla (C) Shri Vijaypat Singhania (D) Shri Mukesh Ambani (E) Shri Anil Agarwal 36. Who is the President of Nass-com? (A) Kiran Karnik-Answer (B) Narayana Murthy (C) Pramod Desai (D) Devang Mehta (E) None of these 37. Which of the following awards is not given for excellence in the field of literature ? (A) Booker Prize (B) Nobel Prize (C) Pulitzer Price (D) Arjun Award-Answer (E) Bharatiya Gnanpith Award 38. 'Singur' which was in news is a place in" (A) Orissa (B) West Bengal (C) Jharkhand (D) Bihar (E) Maharashtra 39. Who amongst the following is the author of the book series named as 'Harry Potter' ? (A) J. K. Rowling-Answer (B) Lindsay Lohan (C) Julia Roberts (D) Sandra Bullock (E) None of these 40. Which of the following Satellites recentlydedicatedtonation helps Direct to Home Television Ser-vice in India ? (A) Matsat (B) Edusat (C) Insat"4b-Answer (D) Insat"IB (E) None of these 41. Many a times we read some news items about the 'West Bank'. 'West Bank' is situated at the western side of the river" (A) Mississippi (B) Amazon (C) Nile (D) Jordan-Answer (E) None of these 42. Which of the following pheno-menon is considered responsible for 'Global Warming' ? (A) Greenhouse Gas Effect-Answer (B) Fox Fire (C) Dry Farming (D) Radioactivity (E) None of these 43. Which of the following instru-ments is not issued by a bank ? (A) Demand Draft (B) Pay Order (C) Debit Card (D) Credit Card (E) National Saving Certificate-Answer 44. Name of Jyoti Randhawa is asso-ciated with which of the follo-wing games ? (A) Golf-Answer (B) Badminton (C) Hockey (D) Chess (E) None of these 45. Which of the following Software Companies recently launched its education portal 'Unlimited Potential' in India ? (A) Microsoft -Answer (B) Infosys (C) Wipro (D) TCS (E) None of these 46. Polly Umrigar who died was a well known" (A) Cricketer-Answer (B) Tennis Player (C) Golfer (D) Chess Player (E) Hockey Player 47. In India Census is done after a gap of every" (A) Five years (B) Seven years (C) Eight years (D) Ten years-Answer (E) Fifteen years 48. Prof. Muhammad Yunus the recipient of the Nobel Peace Prize 2006 is the exponent of which of the following concepts in the field of banking ? (A) Core Banking (B) Micro Credit-Answer (C) Retail Banking (D) Real Time Gross Settlement (E) Internet Banking 49. Who amongst the following was India's official candidate for the post of UNO's Secretary Gene- ral ? (A) Mr. Shyam Saran (B) Dr. C. Rangarajan (C) Mr. Shashi Tharoor-Answer (D) Mr. Vijay Nambiar (E) None of these 50. Mr. Gordon Brown has taken over as the Prime Minister of" (A) Italy (B) Canada (C) France (D) South Africa (E) Britain-Answer 51. Which of the following Departments of the Govt. of India is helping banks in disbursement of rural credit by the bank ? (A) Railways (B) State Road Transpots (C) Post and Telegraph (D) Ministry of Health (E) None of these Ans (E) 52. Which of the following organization/agencies has established a fund known as " investor production fund" ? (A) SEBI (B) NABARD (C) Bombay stock exchange (D) AMFI (E)None of these Ans (C) 53.Who amongst the following is the Head of the RBI at present ? (A) Mr. M.V.Kamath (B) Mr. Y.V.Reddy (C) Mr. N.R.Narayanmurthy (D) Mr.O.P.Bhatt (E)None of these Ans (B) 54. Mjority of rural people still prefer to go to which of the following for their credit needs ? (A) Money lenders (B) Foreign Bankers (C) NABARD (D) RBI (E) None of these Ans (A) 55. india has different categories of commercial banks.Which of the following is NOT one such categories ? (A) Private Banks (B) Commodities Banks (C) Nationalized Banks (D) Cooperative Banks (E) Foreign Banks Ans (B) 56.Which of the following types of Banks are allowed to operate foreign currency accounts ? (1) Foreign Banks (2) Regional Rural Banks (3) Nationalized Banks (A) Only i (B) Only 2 (C) Only 3 (D) All 1,2and 3 (E) None of these Ans (C) 57. Which of the following countries does not play International Cricket ? (A) Russia (B) England (C) South Africa (D) Pakistan (E) India Ans (A) 58. The money which Govt. of India spends on the development of infrastructure in country comes from the which of the following sources ? [Pick up the correct Statement(s)] (1) Loan from World Bank/ADB etc (2) Taxws collected from the people (3) Loan from the RBI (A) Only 1 (B) Only 2 (C) Only 3 (D) Both 1 and 2 (E) All 1,2 and 3 Ans (E) 59. The Securities and Exchange Board Of India (SEBI) recently imposed a restriction on money flow in equity through ‘P Notes'. What is the full form of ‘P Notes' ? (A) Permanent Notes (B) Perchase Notes (C) Participatory-Notes (D) Private Notes (E) None of these Ans (C) 60. Who amongst the following was the captain of the India cricket team who won thw Twenty-20 World Cup-2007 ? (A) Yuvraj Singh (B) M.S.Dhoni (C) Rahul Dravid (D) Saurav Ganguly (E) None of these Ans (B) 61. Mnay times we read in financial news paper about 'FII'.What is the full form of 'FII'? (A) Final Investment in India (B) Foreign Investment in India (C) Formal Investment in India (D) Fair Institutional Investment (E) Foreign Institutional Investment Ans (E) 62. Benazir Bhutto Is associated with which of the following parties (A) Muslim League (B) Pakistan Peoples Party (C) Pakistan National Congress(D) Islamic Movement of Pakistan (E) None of these Ans (B) 63. One of the former prime ministers of which of the following countries was detained in house arrest for a short period after his/her returns from a long exile ? (A) Germany (B) France (C) Pakistan (D) Brazil (E) None of these Ans (C) 64. Who amongst the following leaders from USA visited Isarael and Palestinian West Bank so that a solution to the Isarael and palestinian problem can be worked out ? (A) George Bush (B) Al Gore (C) Bill Clinton (D) Condeleeza Rice (E) None of these Ans (A) 65. Justine Henin Won the women's singles US Open Tennis Championship-2007 After defeating - (A) Swetlana Kuznetsova (B) Sania Mirza (C) Dinara Safina (D) Mathalie Dechy (E) None of these Ans (A) 66. As reported in papers the UN World Food Programmes stoped distributing food in Mogadishu Town after its local head was abducted by the Govt. soldiers of the country.Mogadishu is the capital town of- (A) Tanzania (B) Turkey (C) Cuba (D) Libya (E) Somalia Ans (E) 67. The financial markets of the which of the following countries were badly affected by sub-prime crisis ? (A) Russia (B) Brazil (C) UK (D) USA (E)None of these Ans (D) 68. which of the following countries in the world is the biggest consumer of gold ? (A) USA (B) Bangladesh (C) Russia (D) India (E) None of these Ans (D) 69. which of the following countries is NOT happy with the USA's decision to award a cogressional Medal to Dalai Lama Oof Tibet ? (A) India (B) Pakistan (C) Nepal (D) Myanmar (E) China Ans (E) 70. which of the following countries recently decided to launch a military action in Northern Iraq where many Kurdish PKK fighters are based and they are killed people from that country ? (A) India (B) Afghanistan (C) Pakistan (D) Bangladesh (E) Turkey Ans (E) 71. Dr. Mnamohan SAingh called George Bush to explain the difficulties he is having in implementing agreements related with which of the following with USA ? (A) Suply of sugar (B) Civilian nuclear Co-operation (C) Purchase of fighter planes (D) Purchase of Gas / Petroleum (E) None of these Ans (B) 72. which of the following countries is facing a problem of strike by the workers of the transport ,electricity and gas companies as the present govt. of the country has decided to end the pension to these workerss ? (A) USA (B) France (C) China (D) Nepal (E) None of these Ans (B) 73. Who amongst the following is the Secretary General Of UNO ? (A) Al Gore (B) Shashi Tharoor (C) Gordon Drown (D) Hugo Chavez (E) None of these Ans (E) 74. The “Orange Coalition Govt" was formed once again in which of the following countries ? (A) Russia (B) Ukraine (C) France (D) Germany (E) None of these Ans (B) 75. which of the following countries is NOT elected by the the UN general assembly on the non-payment seats of UN security council w.e.f. January 2008 ? (A) Libya (B) Vietnam (C) Croatia (D) Costa Rica (E) Pakistan Ans (E) 76. The Govt. of India put a ban on export of which of the following commodities at the price below the price of the same in domestic market ? (A) Steel (B) Chemical Fertilizer (C) Pharma Products (D) Electronic Goods (E) None of these Ans (E) 77. India Launched which of the following Satellites in September 2007 ? (A) EDUSAT (B) MATSAT (C) CROSAT-IIB (D) INSAT-4CR (E) None of these Ans (D) 78. As per the recent agreement between India and one of other country the iNdia Rupee can be easily swapped with - (A) Tak (B) Rial (C) Kyat (D) Yen (E) Rubbel Ans (E) 79. India won the ONGC Cup Football Tounament 2007 by beating- (A) Syria (B) Pakistan (C) Britain (D) France (E) None of these Ans (A) 80. Abdullah Gul's name was in news recently as he he has taken over as the president of - (A) Turkey (B) Sudan (C) Afghanistan (D) Pakistan (E) None of these Ans (A) 81. Hem Dutta who was honoured gy the prestigious Rajiv Gandhi National Sadbhawana Award 2007 is famous- (A) Author (B) Social Activist (C) Player (D) Film Producer (E) Scientist Ans (B) 82. Who amongst the following is selected for Basawa Award (2006-07) by the Karnataka Government ? (A) Dr. Manmohan Singh (B) A.P.J.Abdul Kalam (C) Sonia Gandhi (D) Pratibha Patil (E) None of these Ans (B) 83. Asia Pacific Economic Business meet was organized in September 2007 in - (A) New Delhi (B) Beijing (C) Tokyo (D) London (E) Sydne y Ans (E) 84. The World Athletics Championship-2007 was organized in - (A) Beijing (B) New Delhi (C) Osaka (D) Dhaka (E) London Ans (C) 85. Who amongst the following was the captain of the Indian Hockey Team who won Asia Cup 2007 held in Chennai ? (A) Baichung Bhutia (B) Dilip Tirkey (C) Pnkaj Adwani (D) Manavjeet Singh Sandhu (E) None of these Ans (E) 86. Which of the following is a Public Sector Unit ? (A) TCS (B) ICICI Bank (C) TESCO (D) BHEL (E) All of these Ans (D) 87. India's foreign exchange reserve declined sharply in recent past. What was the main reason for the same ? (A) Heavy demand of the same by foreign tourists (B) Import of Wheat from Pakistan and S.Korea (C) Appreciation of Rupee Value (D) Instability in coalition Govt. in centre (E) None of these Ans (C) 88. Sinzo Abe who was on visit to India in recent past is the - (A) Prime Minister of South Korea (B) Prime Minister of North Korea (C) Prime Minister of Japan (D) President of South Korea (E) None of these Ans (C) 89. Asafa Powel who created a new world record in 100 metres race is a citizen of - (A) Jamaica (B) South Africa (C) India (D) USA (E) south Korea Ans (A) 90. Who amongst the following got third position in long Jump event of the Bayer International Athletics held in Germany in 2007 ? (A) Shiny Wilson (B) Anju Bobby George (C) Neha Sanwal (D) Prajakta Sawant (E) None of these Ans (B) FCI Free solved sample placement papers, FCI general aptitude, general intelligence ,reasoning,general awareness and current affairs questions with answers,FCI model questions for practice 1. Name the person who on 13 September 2012 was appointed as the 39th chief justice of India (a) Justice Altamas Kabir (b) Justice A.K. Sikri (c) Rajeev Gupta (d) Justice Mohit Shantilal Shah Answer: (a) Justice Altamas Kabir 2. Which person was appointed as the president of, The Indian Newspaper Society for the year 2012-2013 on 13 September 2012 during the 73rd annual meet of the Newspaper Society? (a) K.N. Tilak Kumar (b) Ravindra Kumar (c) Ashish Bagga (d) Pradeep Gupta Answer: (a) K.N. Tilak Kumar 3. On which date Vayalar Ravi, Minister of Overseas Indian Affairs inaugurated the 7th Heads of Indian Missions (HoMs) conference and announced revised Indian Community Welfare Fund (ICWF) scheme. (a) 15 September 2012 (b) 16 September 2012 (c) 17 September 2012 (d) 20 September 2012 Answer: (c) 17 September 2012 4. The Supreme Court of India on 14 September 2012 lifted the gag order on media reporting of troop movement that was ordered by one of the High Court. HC order was in violation of the fundamental right under Article 19(1) (a) of the Constitution, granted to the media and every citizen of the country. The order came from which of the high court? (a) Allahabad High court (b) Calcutta high court (c) Karnataka high court (d) Madras high court Answer: (a) Allahabad High court 5. The Union government hiked Dearness Allowance (DA) by what percent benefiting its 80 lakh employees and pensioners and costing the exchequer an additional Rs7,408 crore annually? (a) 4 percent (b) 8 percent (c) 7 percent (d) 6 percent Answer: (c) 7 percent ************************************************************************************************************************************************************************************************************************************************************************** 1. Who has been selected for J.C Daniel Award for 2011? Ans. Jose Prakash 2. 2012 Nuclear Security Summit was held in? Ans. Seoul, South Korea 3. Who is appointed as the new Chief Secretary of Kerala? Ans. K. Jayakumar 4. Who has been awarded the ‘Pritzker Architecture Prize’ considered as Nobel prize in Architecture? Ans. Wang Shu 5. Who is the first service provider to offer ‘4 G’ services in India? Ans. Bharti Airtel 6. Who is the first women scientist to head a missile project in India? Ans. Tessy Thomas 7. Where will be held shooting world championship in 2014? Ans. Granada, Spain 8. Name the Indian player who won Thailand open grand prix gold title in Bankok 2012? Ans. Saina Nehwal 9. Name the present Prime Minister of Greece? Ans. Antonies Samraz 10. Daniel Hillel won the World Food Prize for 2012. He belongs to which country? Ans. Israel 11. The World Health Organisation (WHO) has removed India from it’s list of Polio-endemic countries. Which countries are now left in the list? Ans. Afghanistan, Pakistan and Nigeria 12. A savings bank account with zero or minimum balance is known as? Ans. No Frills Account 13. M R Venkatesh became India’s 28th Chess Grand Master in July 2012. He belongs to which state? Ans. Tamil Nadu 14. Who is known as India’s Iron Butterfly? Ans. Saina Nehwal 15. Who has been appointed as the new CEO of Yahoo!? Ans. Marissa Mayer 16. Who is India’s permanent representative to United Nations (UN)? Ans. Hardeep Singh Puri 17. Who is the new Managing Director of International Monetary Fund (IMF)? Ans. Christine Lagarde 18. Who is appointed as the Secretary to the President of India recently? Ans. Omita Paul 19. Name the only Indian who got Magsaysay Award of the year 2012? Ans. Kulandei Francis 20. Who is the new Prime Minister of Egypt? Ans. Hisham Khandil 21. Who is the new Prime Minister of Serbia? Ans. Evika Dasich 22. Who won the Kerala State Film award 2011 in the Best Actor Category? Ans. Dileep 23. Which country now allows women to divorce smoker husbands? Ans. Saudi Arabia 24. Who has been appointed as the new Chief Justice of India? Ans. Justice Altamas Kabir 25. Which country won the highest number of medals in the London Olympics 2012? Ans. USA 26. What was the slogan of London Olympic 2012? Ans. Inspire a Generation 27. Which Indian shuttler made the record of winnng Asian youth under 19 championship? Ans. P.V Sindhu 28. Spain lifted the Euro cup 2012 defeating ...........? Ans. Italy 29. Who is the coach of Indian football team that won the Nehru Cup recently? Ans. Wim Kovermans 30. Name the player who won the first gold medal in London 2012? Ans. Yi Siling (China) 31. Who is the captain of under 19 Indian cricket squad that won 2012 world cup? Ans. Unmukt Chand 32. Who is the official brand ambassador for the World Cup T-20 to be held in Sri Lanka? Ans. Lasit Malinga 33. Name the new chief of ICC? Ans. Alan Isaac (Newziland) 34. Which famous cricketer’s autobiography is titled as ‘Straight from the heart’? Ans. Kapil Dev 35. National Sports Day is celebrated on: Ans. 29th August 36. Which team won the Sultan Azlan Shah cup 2012? Ans. New Zeland 37. Who is the current champion of Ranji trophy? Ans. Rajastan ************************************************************************************************************************************************************************************************************************************************************************** 1. If the following numbers are rewritten by interchanging the digits in ten’s place and hundred’s place and then arranging them in the descending order. What will be the second digit of the newly formed fifth number from your right ? 479, 736, 895, 978, 389, 675 (A) 3 (B) 4 (C) 5 (D) 6 Ans : (C) 2. P is 60 m South-East of Q. R is 60 m North-East of Q. Then R is in which direction of P ? (A) North (B) North-East (C) South (D) South-East Ans : (A) Directions—(Q. 3–5) Read the following information for answering the questions that follow On a playing ground A, B, C, D and E are standing as described below facing the North. (i) B is 50 metres to the right of D. (ii) A is 60 metres to the South of B (iii) C is 40 metres to the West of D. (iv) E is 80 metres to the North of A. 3. If a boy walks from C, meets D followed by B, A and then E, how many metres has he walked if he has travelled the straight distance all through ? (A) 120 (B) 150 (C) 170 (D) 230 Ans : (D) 4. What is the minimum distance (in metre approximately) between C and E ? (A) 53 (B) 78 (C) 92 (D) 120 Ans : (C) 5. Who is to the South-East of the person who is to the left of D ? (A) A (B) B (C) C (D) E Ans : (A) 6. A man was walking in the evening just before the sun set. His wife said that, his shadow fell on his right. If the wife was walking in the opposite direction of the man, then which direction the wife was facing ? (A) North (B) West (C) South (D) East Ans : (C) Directions(Q. 7–11) In each of the following questions choose the set of numbers from the four alternative sets that is similar to the given set. 7. Given set : (4, 9, 18) (A) (8, 14, 22) (B) (10, 15, 25) (C) (6, 12, 23) (D) (12, 17, 26) Ans : (D) 8. Given set : (10, 14, 17) (A) (4, 11, 14) (B) (9, 12, 15) (C) (8, 13, 18) (D) (6, 9, 12) Ans : (A) 9. Given set : (7, 27, 55) (A) (21, 35, 52) (B) (18, 42, 65) (C) (16, 40, 72) (D) (13, 30, 58) Ans : (C) 10. Given set : (39, 28, 19) (A) (84, 67, 52) (B) (52, 25, 17) (C) (70, 49, 36) (D) (65, 45, 21) Ans : (A) 11. Given set : (246, 257, 358) (A) (233, 343, 345) (B) (273, 365, 367) (C) (143, 226, 237) (D) (145, 235, 325) Ans : (A) Directions(Q. 12–16) Each question contains six or seven statements followed by four sets of combinations of three. Choose the set in which the statements are logically related. 12. (1) All books are having pages. (2) All kings are having pages. (3) All kings are books. (4) Some heavy things are having pages. (5) Some heavy things are books. (6) Some books are heavy. (7) Some heavy things are having pages. (A) 1, 2, 3 (B) 6, 1, 4 (C) 4, 6, 1 (D) 1, 5, 7 Ans : (D) 13. (1) No athletes are vegetarians. (2) All cricket players are athletes. (3) Some athletes play cricket. (4) Some cricket players are vegetarians. (5) No cricket player is a vegetarian. (6) All athletes are vegetarians. (A) 1, 2, 5 (B) 3, 4, 1 (C) 1, 5, 2 (D) 2, 5, 6 Ans : (A) 14. (1) All grandmothers cook well. (2) No man is a grandmother. (3) Some men do not cook well. (4) All those who cook well are men. (5) No one who cooks well is a man. (6) All those who cook well are grandmothers. (7) Some men are not grandmothers. (A) 2, 6, 5 (B) 2, 5, 6 (C) 1, 4, 2 (D) 6, 4, 7 Ans : (B) 15. (1) Looting is a crime. (2) Some crooked people are criminals. (3) All those involved in looting are criminals. (4) Some crooked people are involved in looting. (5) All criminals are looked down in society. (6) Some crooked people are not criminals. (A) 1, 4, 6 (B) 3, 6, 2 (C) 1, 2, 6 (D) 3, 4, 2 Ans : (D) 16. (1) Some women are those who are successful in life. (2) Some men are those who have patience. (3) No man is a woman. (4) All those who have patience are successful in life. (5) Some who are successful in life are men. (6) Some men are not those are successful in life. (A) 1, 3, 6 (B) 4, 2, 6 (C) 1, 5, 3 (D) 2, 4, 5 Ans : (B) Directions—(Q. 17–21) Each of the questions below consists of a question and two statements numbered (I) and (II). You have to decide whether the data provided in the statements are sufficient to answer the question. Give answers (A) If the data in statement (I) alone are sufficient to answer the question, while the data in statement (II) alone are not sufficient to answer the question; (B) If the data in statement (II) alone are sufficient to answer the question, while the data in statement (I) alone are not sufficient to answer the questions; (C) If the data even in both statements (I) and (II) together are not sufficient to answer the question; (D) If the data in both statement (I) and (II) together are necessary to answer the question. 17. In which direction is Mahatmaji’s statue facing ? I. The statue is towards the northern end of the city. II. The statue’s shadow falls towards East at 5 O’clock in the evening. Ans : (C) 18. What is the total number of pupils in the final year class ? I. The number of boys in the final year class is twice as much as the number of girls in that class. II. The sum of the ages of all the pupils in the class is 399 years and their average age is 19 years. Ans : (B) 19. Who is the tallest among A, B, C and D ? I. A is taller than C. II. B is taller than C and D. Ans : (C) 20. How many Sundays are there in a particular month of a particular year ? I. The month begins on Monday. II. The month ends on Wednesday. Ans : (D) 21. What is the total number of pages in this book ? I. I counted 132 pages from the beginning of this book. II. My wife counted 138 pages starting from the end of the same book. Ans : (C) Directions—(Q. 22–26) In each of the questions given below, there is a statement followed by three assumptions numbered I, II and III. An assumption is something supposed or taken for granted. You have to consider the statement and assumptions and then decide, which of the assumption(s) is/are implicit in the statement. 22. Statement : During pre-harvest kharif seasons, the government has decided to release vast quantity of foodgrains from FCI. Assumptions :I. There may be a shortage of foodgrains in the market during this season. II. The kharif crop may be able to replenish the stock of FCI. III. There may be a demand from the farmers to procure kharif crop immediately after harvest. (A) None is implicit (B) Only I and II are implicit (C) Only II and III are implicit (D) All are implicit Ans : (D) 23. Statement : To improve the employment situation in India, there is a need to recast the present educational system towards implementation of scientific discoveries in daily life. Assumptions :I. The students after completing such education may be able to earn their livelihood. II. This may bring meaning of education in the minds of the youth. III. The state may earn more revenue as more and more people will engage themselves in self employment. (A) Only I and II are implicit (B) Only III is implicit (C) Only I and III are implicit (D) None is implicit Ans : (A) 24. Statement : To increase profit, the oil exporting countries decided to reduce the production of crude by 5 million barrels per day. Assumptions :I. The price of crude may increase due to less production. II. The demand of crude may remain same in future. III. Other countries may continue buying crude from these countries. (A) All are implicit (B) Only II and III are implicit (C) Only I and II are implicit (D) None is implicit Ans : (C) 25. Statement : “We do not want you to see our product on newspaper, visit our shop to get a full view.” – an advertisement. Assumptions :I. People generally decide to purchase any product after seeing the name in the advertisement. II. Uncommon appeal may attract the customers. III. People may come to see the product. (A) All are implicit (B) None is implicit (C) Only II and III are implicit (D) Only I and II are implicit Ans : (A) 26. Statement : The Reserve Bank of India has directed the banks to refuse fresh loans to major defaulters. Assumptions :I. The banks may still give loans to the defaulters. II. The defaulters may repay the earlier loan to get fresh loan. III. The banks may recover the bad loans through such harsh measures. (A) All are implicit (B) None is implicit (C) Both II and III are implicit (D) Both I and II are implicit Ans : (C) Directions—(Q. 27–31) In questions given below, statements 1 and 2 are followed by conclusions I and II. Taking the statements to be right although they may seem at variance with commonly accepted facts, mark your answers as under (A) If only conclusion I follows. (B) If only conclusion II follows. (C) If both I and II follows. (D) Neither I nor II follows. 27. Statements :1. All hands are machines. 2. All machines are wheels. Conclusions :I. All wheels are hands. II. All hands are wheels. Ans : (B) 28. Statements :1. Some buds are leaves. 2. Some leaves are red. Conclusions : I. Some buds are red. II. Some leaves are not buds. Ans : (B) 29. Statements : 1. Some stones are shells. 2. All shells are pearls. Conclusions :I. Some stones are pearls. II. All pearls are shells. Ans : (A) 30. Statements :1. Brown is red and blue is green. 2. Green is pink and yellow is red. Conclusions : I. Yellow is brown. II. Pink is blue. Ans : (C) 31. Statements : 1. Merchants who do not own cars do not have bicycles either. 2. Those who do not have bicycles have tricycles. Conclusions :I. Some merchants have only tricycles. II. No one has both, the car and the tricycles. Ans : (D) Directions—(Q. 32–36) A number arrangement machine, when given a particular input, rearranges it following a particular rule. The following is the illustration of the input and the stages of arrangement. Input : 245, 316, 436, 519, 868, 710, 689 Step I : 710, 316, 436, 519, 868, 245, 689 Step II : 710, 316, 245, 519, 868, 436, 689 Step III : 710, 316, 245, 436, 868, 519, 689 Step IV : 710, 316, 245, 436, 519, 868, 689 Step IV is the last step of input. 32. If 655, 436, 764, 799, 977, 572, 333 is the input which of the following steps will be ‘333, 436, 572, 655, 977, 764, 799’ ? (A) II (B) III (C) IV (D) I Ans : (B) 33. How many steps will be required to get the final output from the following input ? Input : 544, 653, 325, 688, 461, 231, 857 (A) 5 (B) 4 (C) 3 (D) 6 Ans : (A) 34. For the given input, which of the following will be third step ? Input : 236, 522, 824, 765, 622, 463, 358 (A) 522, 236, 765, 824, 622, 463, 358 (B) 522, 622, 236, 824, 765, 463, 358 (C) 522, 622, 236, 765, 824, 463, 358 (D) 522, 622, 236, 463, 824, 765, 358 Ans : (C) 35. If following is the second step for an input, what will be the fourth step ? Step II : 620, 415, 344, 537, 787, 634, 977 (A) 620, 415, 344, 537, 634, 787, 977 (B) 620, 415, 344, 634, 537, 787, 977 (C) 620, 415, 344, 634, 787, 537, 977 (D) Can’t be determined Ans : (B) 36. Which of the following is the last step for the following input ? Input : 473, 442, 735, 542, 367, 234, 549 (A) 234, 442, 542, 473, 735, 367, 549 (B) 234, 442, 542, 735, 473, 367, 549 (C) 234, 442, 542, 473, 367, 735, 549 (D) 234, 442, 542, 735, 367, 473, 549 Ans : (A) Directions—(Q. 37–41) Read the following information carefully and answer the questions given below it— (1) There is a group of six persons A, B, C, D, E and F in a family. They are Psychologist, Manager, Lawyer, Jeweller, Doctor and Engineer (2) The doctor is the grandfather of F, who is a Psychologist. (3) The Manager D is married to A. (4) C, the Jeweller, is married to the Lawyer. (5) B is the mother of F and E. (6) There are two married couples in the family. (7) The Psychologist is a female while Engineer is a male. 37. What is the profession of E ? (A) Doctor (B) Engineer (C) Manager (D) Psychologist Ans : (B) 38. How is A related to E ? (A) Brother (B) Uncle (C) Father (D) Grandfather Ans : (D) 39. How many male numbers are there in the family ? (A) One (B) Three (C) Four (D) Two Ans : (B) 40. What is the profession of A ? (A) Doctor (B) Lawyer (C) Jeweller (D) Manager Ans : (A) 41. Which of the following is one of the pairs of couples in the family ? (A) AB (B) AC (C) AD (D) AE Ans : (C) 42. Three of the following four are alike in a certain way and so form a group. Which is the one that does not belong to that group ? (A) Papaya (B) Mango (C) Jackfruit (D) Watermelon Ans : (C) 43. Three of the following four are similar in relation to their positions in the English alphabet and hence form a group. Which one does not belong to that group ? (A) SPEAK : PZVKH (B) HUSKY : BPGFS (C) BRAIN : MRZIY (D) BREAK : PZVIY Ans : (B) 44. Three of the following four groups of letters are alike in some way while one is different. Find out which one is different ? (A) DJWR (B) EKXR (C) KQDX (D) AGTN Ans : (A) 45. Four groups of letters are given below. Three of them are alike in a certain way while one is different. Choose the odd one. (A) GWOURV (B) LZKMSU (C) SFXPMG (D) JOEHNP Ans : (C) 46. Four pairs of words are given below out of which the words in all pairs except one, bear a certain common relationship. Choose the pair in which the words are differently related. (A) Watt : Power (B) Ampere : Current (C) Pascal : Pressure (D) Radian : Degree Ans : (D) 47. Number of letters skipped in between adjacent letters of the series starting from behind are increased by one. Which of the following seriesobserves this rule ? (A) ONLKI (B) OMKIG (C) OMJFA (D) OIGDC Ans : (C) 48. If the letters of the word ‘PROTECTION’ which are at odd numbered position in the English alphabet are picked up and are arranged in alphabetical order from left and if they are now substituted by Z, Y, X and so on, beginning from left which letter will get substituted by X ? (A) E (B) O (C) T (D) I Ans : (D) 49. How many pairs of letters are there in the word OPERATION in which the difference between them is the same as in the English alphabet ? (A) 3 (B) 5 (C) 7 (D) 9 Ans : (C) 50. Arrange the given words in order in which they occur in the dictionary and then choose the correct sequence. (1) Dissipate (2) Dissuade (3) Disseminate (4) Distract (5) Dissociate (6) Dissect (A) 6, 3, 1, 5, 2, 4 (B) 1, 6, 3, 2, 4, 5 (C) 3, 6, 1, 2, 5, 4 (D) 4, 6, 3, 1, 5, 2 Ans : (A) 1: The largest copper producing country in the World is 1. Chile 2. Russia 3. South Africa 4. China Ans: 1. 2: If the radius of a circle is diminished by 10%, then its area is diminished by: 1. 10% 2. 19% 3. 20% 4. 36% Ans: 2. 3: A boat travels 20 kms upstream in 6 hrs and 18 kms downstream in 4 hrs.Find the speed of the boat in still water and the speed of the water current? 1. 1/2 kmph 2. 7/12 kmph 3. 5 kmph 4. none of these Ans: 2. 4: At what time after 4.00 p.m. is the minutes hand of a clock exactly aligned with the hour hand? 1. 4:21:49.5 2. 4:27:49.5 3. 3:21:49.5 4. 4:21:44.5 Ans: 1. 5: A shop keeper sold a T.V set for Rs.17,940 with a discount of 8% and earned a profit of 19.6%.What would have been the percentage of profit earned if no discount was offered? 1. 24.8% 2. 25% 3. 26.4% 4. Cannot be determined 5. None of these Ans: 5. 6: If (2x-y)=4 then (6x-3y)=? 1. 15 2. 12 3. 18 4. 10 Ans: 2. 7: A clock is set right at 8 a.m. The clock gains 10 minutes in 24 hours. What will be the true time when the clock indicates 1 p.m. on the following day? 1. 48 min. past 12 2. 38 min. past 12 3. 28 min. past 12 4. 25 min. past 12 Ans: 1. 8: What is the missing number in this series? 8 2 14 6 11 ? 14 6 18 12 1. 16 2. 9 3. 15 4. 6 Ans: 2. 9: Dinesh travelled 1200 km by air which formed 2/5 of his trip. One third of the whole trip, he travelled by car and the rest of the journey he performed by train. What was the distance travelled by train? 1. 600Km 2. 700Km 3. 800Km 4. 900Km Ans: 3. 10: A train which travels at a uniform speed due to some mechanical fault after traveling for an hour goes at 3/5th of the original speed and reaches the destination 2 hrs late.If the fault had occurred after traveling another 50 miles the train would have reached 40 min earlier. What is distance between the two stations. 1. 300 2. 310 3. 320 4. 305 Ans: 1. 11: The average between a two digit number and the number obtained by interchanging the digits is 9. What is the difference between the two digits of the number? 1. 8 2. 2 3. 5 4. Cannot be determined Ans: 4. 12: Pipe A can fill in 20 minutes and Pipe B in 30 mins and Pipe C can empty the same in 40 mins.If all of them work together, find the time taken to fill the tank 1. 17 1/7 mins 2. 20 mins 3. 8 mins 4. none of these Ans: 1. 13: A person has 4 coins each of different denomination. What is the number of different sums of money the person can form (using one or more coins at a time)? 1. 16 2. 15 3. 12 4. 11 Ans: 2. 14: The simple interest on a certain sum of money for 3 years is 225 and the compound interest on the same sum at the same rate for 2 years is 153 then the principal invested is 1. 1500 2. 2250 3. 3000 4. 1875 Ans: 4. 15: A cow is tethered in the middle of a field with a 14 feet long rope. If the cow grazes 100 sq. ft. per day, then approximately what time will be taken by the cow to graze the whole field ? 1. 2 days 2. 6 days 3. 18 days 4. 24 days 5. None of these Ans: 2. 16: 2 hours after a freight train leaves Delhi a passenger train leaves the same station travelling in the same direction at an average speed of 16 km/hr. After travelling 4 hrs the passenger train overtakes the freight train. The average speed of the freight train was? 1. 40 2. 30 3. 80 4. 60 Ans: 1. 17: The two colors seen at the extreme ends of the pH chart are: 1. Red and Blue 2. Red and Green 3. Green and Blue 4. Orange and Green Ans: 1. 18: 8 15 24 35 48 63 _? 1. 70 2. 80 3. 75 4. 88 Ans: 2. 19: One of Mr. Horton, his wife, their son, and Mr. Horton’s mother is a doctor and another is a lawyer. a) If the doctor is younger than the lawyer, then the doctor and the lawyer are not blood relatives. b) If the doctor is a woman, then the doctor and the lawyer are blood relatives. c) If the lawyer is a man, then the doctor is a man. Whose occupation you know? 1. Mr. Horton: he is the doctor 2. Mr. Horton’s son: she is the lawyer 3. Mr. Horton: he is the doctor 4. Mr. Horton’s mother: she is the doctor Ans: 1. 20: In the given figure, PA and PB are tangents to the circle at A and B respectively and the chord BC is parallel to tangent PA. If AC = 6 cm, and length of the tangent AP is 9 cm, then what is the length of the chord BC? 1. 4 cm 2. 8 cm 3. 6 cm 4. 5 cm Ans: 1. 21: Union Information and Broadcasting ministry recently gave an indication to change which of the following laws on a larger scale, as the existing provisions of the Act are inadequate to cater to the phenomenal growth of the print media in view of the liberalization of the government policies? 1. Press & Registration of Books Act, (PRB Act) 1867 2. The Delivery Of Books ‘And Newspapers’ (Public Libraries) Act, 1954 3. Indian Press (Emergency Powers ) Act 1931 4. none Ans: 1. 22: 2 numbers differ by 5.If their product is 336,then the sum of the 2 numbers is: 1. 21 2. 51 3. 28 4. 37 Ans: 4. 23: Which number is the odd one out? 9678 4572 5261 3527 7768 1. 7768 2. 3527 3. 4572 4. 9678 5. 5261 Ans: 2. 24: Which one among the following has the largest shipyard in India 1. Kolkata 2. Kochi 3. Mumbai 4. Visakhapatnam Ans: 2. 25: If x=y=2z and xyz=256 then what is the value of x? 1. 8 2. 3 3. 5 4. 6 Ans: 1. 26: A radio when sold at a certain price gives a gain of 20%. What will be the gain percent, if sold for thrice the price? 1. 280 2. 270 3. 290 4. 260 Ans: 4. 27: x% of y is y% of ? 1. x/y 2. 2y 3. x 4. can’t be determined Ans: 3. 28: If the value of x lies between 0 & 1 which of the following is the largest? 1. x 2. x2 3. -x 4. 1/x Ans: 4. 29: The tutor of Alexander the great was 1. Darius 2. Cyrus 3. Socrates 4. Aristotle Ans: 4. 30: Thirty men take 20 days to complete a job working 9 hours a day. How many hour a day should 40 men work to complete the job? 1. 8 hrs 2. 71/2 hrs 3. 7 hrs 4. 9 hrs Ans: 2. 31: Goitre caused by the deficiency of ……… 1. Vitamin D 2. Iron 3. VItamin A 4. Iodine Ans: 4. 32: Who invented Napier’s Bones 1. John Napier 2. William Oughtred 3. Charles Babbage 4. Napier Bone Ans: 1. 33: The mass number of a nucleus is 1. Always less than its atomic number 2. Always more than its atomic number 3. Sometimes more than and sometimes equal to its atomic number 4. None of the above Ans: 3. 34: A and B can do a piece of work in 45 days and 40 days respectively. They began to do the work together but A leaves after some days and then B completed the remaining work n 23 days. The number of days after which A left the work was 1. 9 2. 11 3. 12 4. 15 5. 16 Ans: 1. 35: Sam and Mala have a conversation. Sam says I am certainly not over 40 Mala Says I am 38 and you are at least 5 years older than me · Now Sam says you are at least 39 all the statements by the two are false. How old are they really? 1. Mala = 38 yrs, Sam =31 yrs. 2. Mala = 38 yrs, Sam = 41 yrs 3. Mala = 31 yrs, Sam = 41 yrs. 4. Mala = 45yrs, Sam = 41 yrs Ans: 2. 36: What is the code name for Windows Vista? 1. Longhorn 2. Longhund 3. Stackspray 4. Pearl Ans: 1. 37: On sports day, if 30 children were made to stand in a column, 16 columns could be formed. If 24 children were made to stand in a column, how many columns could be formed? 1. 20 2. 30 3. 40 4. 50 Ans: 1. 38: The probability that a man will be alive for 25 years is 3/5 and the probability that his wife will be alive for 25 years is 2/3. Find the probability that only the man will be alive for 25 years. 1. 2/5 2. 1/5 3. 3/5 4. 4/5 Ans: 2. 39: In a single throw of a dice, what is the probability of getting a number greater than 4? 1. 1/2 2. 2/3 3. 1/4 4. 1/3 Ans: 4. 40: If every alternative letter starting from B of the English alphabet is written in small letter, rest all are written in capital letters, how the month “September” be written. (1) SeptEMbEr (2) SEpTeMBEr (3) SeptembeR (4) SepteMber (5) None of the above 1. (1) 2. (2) 3. (3) 4. (5) 5. (4) Ans: 4. 41: After allowing a discount of 11.11% ,a trader still makes a gain of 14.28 % .at how many precent above the cost price does he mark his goods? 1. 28.56% 2. 35% 3. 22.22% 4. None of these Ans: 1. 42: Pipe A can fill in 20 minutes and Pipe B in 30 mins and Pipe C can empty the same in 40 mins.If all of them work together, find the time taken to fill the tank 1. 17 1/7 mins 2. 20 mins 3. none 4. 50 mins Ans: 1. 43: There are 3 triplet brothers. They look identical. The oldest is John, he always tells the truth. The second is Jack, he always tells a lie. The third is Joe, he either tells the truth or a lie. Jimmie Dean went to visit them one day. He was wondering who was who. So he asked each person a question. He asked the one who was sitting on the left: “Who is the guy sitting in the middle?”. The answer was “He is John.” He asked the one who was sitting in the middle: “What is your name?”. The answer was “I am Joe.” He asked the one who was sitting on the right: “What is the guy sitting in the middle?”. The answer was “He is Jack.” Jimmie Dean got really confused. Basically, he asked 3 same questions, but he got 3 different answers. which is not true? 1. left most is joe 2. middle is jack 3. right is john 4. middle is john Ans: 4. 44: A / B = C; C > D then 1. A is always greater than D 2. C is always greater than D 3. B is always less than D 4. none Ans: 1. 45: Consider the following statements: 1. The Administrative Reforms Commission (ARC) had recommended that the Department of Personnel of a State should be put under the charge of the Chief Secretary of the State. 2. Chief Secretary of a State is not involved in any manner in the promotion of State Civil officers to the All-India Services. Which of the statements given above is/are correct? 1. Only 1 2. Only 2 3. Both 1 and 2 4. Neither 1 nor 2 Ans: 1. 46: The population of a town was 1,60,000 three years ago. If it increased by 3%, 2.5% and 5% respectively in the last three years, then the present population of the town is : 1. 1,77,000 2. 1,77,366 3. 1,77,461 4. 1,77,596 Ans: 2. 47: What is the population of India ? 1. 98 crores 2. More than 2 billion 3. More than 1 billion 4. Less than 96 crores 5. 96 crores Ans: 3. 48: Some green are blue. No blue are white. 1. Some green are white 2. No white are green 3. No green are white 4. None of the above Ans: 1. 49: What is the missing number in this series? 8 2 14 6 11 ? 14 6 18 12 1. 8 2. 6 3. 9 4. 11 Ans: 3. 50: Average age of students of an adult school is 40 years. 120 new students whose average age is 32 years joined the school. As a result the average age is decreased by 4 years. Find the number of students of the school after joining of the new students: 1. 1200 2. 120 3. 360 4. 240 Ans: 4. 51: On sports day,if 30 children were made to stand in a column,16 columns could be formed. If 24 children were made to stand in a column , how many columns could be formed? 1. 48 2. 20 3. 30 4. 16 5. 40 Ans: 2. 52: Which of the following numbers is divisible by 3? (i) 541326 (ii) 5967013 1. (ii) only 2. (i) only 3. (i) and (ii) both 4. (i) and (ii) none Ans: 2. 53: A square is divided into 9 identical smaller squares. Six identical balls are to be placed in these smaller squares such that each of the three rows gets at least one ball (one ball in one square only). In how many different ways can this be done? 1. 81 2. 91 3. 41 4. 51 Ans: 1. 54: A man owns 2/3 of the market research beauro business and sells 3/4 of his shares for Rs.75000. What is the value of Business 1. 150000 2. 13000 3. 240000 4. 34000 Ans: 1. 55: 1,2,6,24,_? 1. 111 2. 151 3. 120 4. 125 Ans: 3. 56: The cost of 16 packets of salt,each weighing 900 grams is Rs.28.What will be the cost of 27 packets ,if each packet weighs 1Kg? 1. Rs.52.50 2. Rs.56 3. Rs.58.50 4. Rs.64.75 Ans: 1. 57: Ronald and Michelle have two children. The probability that the first child is a girl, is 50%. The probability that the second child is a girl, is also 50%. Ronald and Michelle tell you that they have a daughter. What is the probability that their other child is also a girl? 1. 1/2 2. 1/3 3. 1/4 4. 1/5 Ans: 2. 58: Find the value of (21/4-1)( 23/4 +21/2+21/4+1) 1. 1 2. 2 3. 3 Ans: 1. 59: The product of two fractions is 14/15 and their quotient is 35/24. the greater fraction is 1. 4/5 2. 7/6 3. 7/5 4. 7/4 Ans: 1. 60: 500 men are arranged in an array of 10 rows and 50 columns according to their heights. Tallest among each row of all are asked to fall out. And the shortest among them is A. Similarly after resuming that to their original podsitions that the shortest among each column are asked to fall out. And the tallest among them is B . Now who is taller among A and B ? 1. A 2. B 3. Both are of same height Ans: 1. 61: Choose the pair of numbers which comes next 75 65 85 55 45 85 35 1. 25 15 2. 25 85 3. 35 25 4. 35 85 5. 25 75 Ans: 2. 62: A three digit number consists of 9,5 and one more number. When these digits are reversed and then subtracted from the original number the answer yielded will be consisting of the same digits arranged yet in a different order. What is the other digit? 1. 1 2. 2 3. 3 4. 4 Ans: 4. 63: ATP stands for: 1. Adenine triphosphate 2. Adenosine triphosphate 3. Adenosine Diphosphate 4. Adenosine tetraphosphate Ans: 2. 64: Veselin Tapolev who became the World Champion recently, is associated with which of the following games/sports ? 1. Chess 2. Golf 3. Snooker 4. Badminton 5. None of these Ans: 1. 65: A piece of cloth cost Rs 35. if the length of the piece would have been 4m longer and each meter cost Re 1 less , the cost would have remained unchanged. how long is the piece? 1. 10 2. 11 3. 12 Ans: 1. 66: In a journey of 15 miles two third distance was travelled with 40 mph and remaining with 60 mph.How muvh time the journey takes 1. 40 min 2. 30 min 3. 120 min 4. 20 min Ans: 4. 67: Solid cube of 6 * 6 * 6. This cube is cut into to 216 small cubes. (1 * 1 * 1).the big cube is painted in all its faces. Then how many of cubes are painted at least 2 sides. 1. 56 2. 45 3. 23 4. 28 Ans: 1. 68: Find the average of first 40 natural numbers. 1. 40 2. 35 3. 30.6 4. 20.5 5. None of these Ans: 4. 69: 1, 5, 14, 30, ?, 91 1. 45 2. 55 3. 60 4. 70 5. None of these Ans: 2. 70: There is a shortage of tubelights, bulbs and fans in a village – Gurgaon. It is found that a) All houses do not have either tubelight or bulb or fan. b) Exactly 19% of houses do not have just one of these. c) Atleast 67% of houses do not have tubelights. d) Atleast 83% of houses do not have bulbs. e) Atleast 73% of houses do not have fans. 1. 42 % 2. 46 % 3. 50 % 4. 54 % 5. 57 % Ans: 1. 71: If 9 engines consume 24 metric tonnes of coal, when each is working 8 hours a day; how much coal will be required for 8 engines, each running 13 hours a day, it being given that 3 engines of the former type consume as much as 4 engines of latter type. 1. 22 metric tonnes. 2. 27 metric tonnes. 3. 26 metric tonnes. 4. 25 metric tonnes. Ans: 3. 72: To 15 lts of water containing 20% alcohol, we add 5 lts of pure water. What is % alcohol. 1. 20% 2. 34% 3. 15% 4. 14% Ans: 3. 73: In page preview mode: 1. You can see all pages of your document 2. You can only see the page you are currently working 3. Satyam BPO Services 4. You can only see pages that do not contain graphics Ans: 4. 74: A house wife saved Rs. 2.50 in buying an item on sale .If she spent Rs.25 for the item ,approximately how much percent she saved in the transaction ? 1. 8% 2. 9% 3. 10% 4. 11% Ans: 2. 75: I have trouble _____. 1. to remember my password 2. to remembering my password 3. remember my password 4. remembering my password Ans: 4. 76: Superheroes Liza and Tamar leave the same camp and run in opposite directions. Liza runs 1 mile per second (mps) and Tamar runs 2 mps. How far apart are they in miles after 1 hour? 1. 10800 mile 2. 19008 mile 3. 12300 mile 4. 14000 mile Ans: 1. 77: A = 5, B = 0, C = 2, D = 10, E = 2. What is then AB + EE – (ED)powerB + (AC)powerE = ? 1. 113 2. 103 3. 93 4. 111 Ans: 2. 78: A man can row upstream at 8 kmph and downstream at 13 kmph.The speed of the stream is? 1. 2.5 kmph 2. 4.2 kmph 3. 5 kmph 4. 10.5 kmph Ans: 1. 79: Find what is the next letter. Please try to find. O,T,T,F,F,S,S,E,N,_ What is that letter? 1. B 2. S 3. Q 4. T 5. O Ans: 4. 80: There are 3 societies A, B, C. A lent cars to B and C as many as they had Already. After some time B gave as many tractors to A and C as many as they have. After sometime c did the same thing. At the end of this transaction each one of them had 24. Find the cars each originally had. 1. A had 21 cars, B had 39 cars & C had 12 cars 2. A had 39 cars, B had 39 cars & C had 12 cars 3. A had 39 cars, B had 21 cars & C had 19 cars 4. A had 39 cars, B had 21 cars & C had 12 cars Ans: 4. 81: A papaya tree was planted 2 years ago. It increases at the rate of 20% every year. If at present, the height of the tree is 540 cm, what was it when the tree was planted? 1. 432 cm 2. 324 cm 3. 375 cm 4. 400 cm Ans: 3. 82: A boy has Rs 2. He wins or loses Re 1 at a time If he wins he gets Re 1 and if he loses the game he loses Re 1. He can loose only 5 times. He is out of the game if he earns Rs 5. Find the number of ways in which this is possible? 1. 14 2. 23 3. 16 4. 12 5. 10 Ans: 3. 83: Five racing drivers, Alan, Bob, Chris, Don, and Eugene, enter into a contest that consists of 6 races. The results of all six races are listed below: Bob always finishes ahead of Chris. Alan finishes either first or last. Eugene finishes either first or last. There are no ties in any race. Every driver finishes each race. In each race, two points are awarded for a fifth place finish, four points for fourth, six points for third, eight points for second, and ten points for first. If Frank enters the third race and finishes behind Chris and Don, which of the following must be true of that race? 1. Eugene finishes first. 2. Alan finishes sixth. 3. Don finishes second. 4. Frank finishes fifth. 5. Chris finishes third. Ans: 4. 84: A is twice as good a workman as B and together they finish a piece of work in 18 days.In how many days will A alone finish the work? 1. 27 2. 26 3. 25 4. 24 Ans: 1. 85: Daal is now being sold at Rs. 20 a kg. During last month its rate was Rs. 16 per kg. By how much percent should a family reduce its consumption so as to keep the expenditure fixed? 1. 20 % 2. 40 % 3. 3% 4. 2% Ans: 1. 86: The sum of 5 successive odd numbers is 1075. What is the largest of these numbers? 1. 215 2. 223 3. 219 4. 217 Ans: 3. 87: A man sells two buffaloes for Rs. 7,820 each. On one he gains 15% and on the other, he loses 15%. His total gain or loss in the transaction is 1. 2.5% gain 2. 2.25% loss 3. 2% loss 4. 5% loss 5. None of these Ans: 2. 88: One ship goes along the stream direction 28 km and in opposite direction 13 km in 5 hrs for each direction.What is the velocity of stream? 1. 1.5 kmph 2. 2.5 kmph 3. 1.8 kmph 4. 2 kmph Ans: 1. 89: Which one of the words given below is different from others? 1. Orange 2. Grape 3. Apricot 4. Raspberry 5. Mango Ans: 3. 90: Complete the series: 5, 20, 24, 6, 2, 8, ? 1. 12 2. 32 3. 34 4. 36 Ans: 1. 91: A can have a piece of work done in 8 days, B can work three times faster than the A, C can work five times faster than A. How many days will they take to do the work together 1. 3 days 2. 8/9 days 3. 4 days 4. None of the above Ans: 2. 92: 7 Pink, 5 Black, 11 Yellow balls are there. Minimum no. atleast to get one black and yellow ball 1. 17 2. 13 3. 15 4. 19 Ans: 1. 93: (1/10)18 – (1/10)20 = ? 1. 99/1020 2. 99/10 3. 0.9 4. none of these Ans: 1. 94: Three friends divided some bullets equally. After all of them shot 4 bullets the total number of bullets remaining is equal to the bullets each had after division. Find the original number divided? 1. 18 2. 20 3. 54 4. 8 Ans: 1. 95: A sum of Rs. 427 is to be divided among A, B and C in such a way that 3 times A’s share, 4 times B’s share and 7 times C’s share are all equal. The share of C is 1. Rs.84 2. Rs.76 3. Rs.98 4. RS.34 Ans: 1. 96: There are 20 poles with a constant distance between each pole. A car takes 24 second to reach the 12th pole.How much will it take to reach the last pole. 1. 41.45 seconds 2. 40.45 seconds 3. 42.45 seconds 4. 41.00 seconds Ans: 1. 97: An emergency vehicle travels 10 miles at a speed of 50 miles per hour. How fast must the vehicle travel on the return trip if the round-trip travel time is to be 20 minutes? 1. 72 miles per hour 2. 75 miles per hour 3. 65 miles per hour 4. 78 miles per hour Ans: 2. 98: 12% of 580 + ? = 94 1. 24.4 2. 34.4 3. 54.4 4. 65.4 Ans: 1. 99: There is a certain relation between two given words on one side of : : and one word is given on another side of : : while another word is to be found from the given alternatives, having the same relation with this word as the given pair has. Select the best alternative. Horse : Jockey : : Car : ? 1. Mechanic 2. Chauffeur 3. Steering 4. Brake Ans: 2. 100: Which of the following numbers should be added to 11158 to make it exactly divisible by 77? 1. 9 2. 8 3. 7 4. 5 Ans: 3.